不等式への招待 第6章

このエントリーをはてなブックマークに追加
1不等式ヲタ ( ゚∀゚)
ある人は蝶を集め、ある人は切手を収集し、ある人は不等式を集める…
          ___          ----- 参考文献〔3〕 P.65 -----
    |┃三 ./  ≧ \   
    |┃   |::::  \ ./ | 
    |┃ ≡|::::: (● (● |  不等式と聞ゐちゃぁ
____.|ミ\_ヽ::::... .ワ......ノ     黙っちゃゐられねゑ!
    |┃=__    \           ハァハァ…
    |┃ ≡ )  人 \ ガラッ

まとめWiki http://wiki.livedoor.jp/loveinequality/

過去スレ
・不等式スレッド (第1章)http://science3.2ch.net/test/read.cgi/math/1072510082/
・不等式への招待 第2章 http://science6.2ch.net/test/read.cgi/math/1105911616/
・不等式への招待 第3章 http://science6.2ch.net/test/read.cgi/math/1179000000/
・不等式への招待 第4章 http://science6.2ch.net/test/read.cgi/math/1245060000/
・不等式への招待 第5章 http://uni.2ch.net/test/read.cgi/math/1287932216/l50
・過去スレのミラー置き場 http://cid-d357afbb34f5b26f.skydrive.live.com/browse.aspx/.Public/

姉妹サイト(?)
キャスフィ 高校数学板 不等式スレ
http://www.casphy.com/bbs/test/read.cgi/highmath/1169210077/
Yahoo! 掲示板 「出題 不等式」
http://messages.yahoo.co.jp/bbs?.mm=GN&action=l&board=1835554&tid=bdpbja1jiteybc0a1k&sid=1835554&mid=10000
2不等式ヲタ ( ゚∀゚):2012/03/29(木) 00:59:57.34
不等式の本

[1] 不等式,ハーディ・リトルウッド・ポリヤ,シュプリンガー,2003年
   http://amazon.jp/o/ASIN/4431710566
[2] 不等式,大関信雄・青木雅計,槇書店,1967年(絶版)
[3] 不等式への招待,大関信雄・大関清太,近代科学社,1987年(絶版)
   http://direct.ips.co.jp/book/Template/Goods/go_Boo...
[4] 不等式入門(数学のかんどころシリーズ9),大関清太,共立出版,2012年
   http://www.kyoritsu-pub.co.jp/bookdetail/978432001...
[5] 不等式入門,渡部隆一,森北出版,2005年
   http://amazon.jp/o/ASIN/4627010494
[6] 不等式の工学への応用、海津聰、森北出版,2004年
   http://amazon.jp/o/ASIN/4627075812
[7] 不等式(モノグラフ4),染取弘,科学新興新社,1990年
   http://amazon.jp/o/ASIN/4894281740
[8] 数学トレッキングツアー第3章「相加平均≧相乗平均」,東京理科大学数学教育研究所,教育出版,2006年
   http://amazon.jp/o/ASIN/4316801988
[9] 数学オリンピック事典,数学オリンピック財団,朝倉書店,2001年
   http://amazon.jp/o/ASIN/4254110871
[10] 獲得金メダル! 国際数学オリンピック第1章「不等式」,小林一章,朝倉書店,2011年
   http://www.asakura.co.jp/books/isbn/978-4-254-1113...
[11] 三角法の精選103問(シリーズ:数学オリンピックへの道 2),T.アンドレースク・Z.フェン著,朝倉書店,2010年
   http://www.asakura.co.jp/books/isbn/978-4-254-1180...
[12] The Cauchy-Schwarz Master Class: An Introduction to the Art of Mathematical Inequalities,J. M. Steele,Cambridge Univ. Pr.,2004年
   http://amazon.jp/o/ASIN/052154677X
[13] Inequalities: A Mathematical Olympiad Approach,Birkhaeuser Basel,2009年
   http://amazon.jp/dp/3034600496
3不等式ヲタ ( ゚∀゚):2012/03/29(木) 01:01:05.88
不等式の記事

[1] 特集 「現代の不等式」 (数理科学 No.386) ,サイエンス社,1995年8月号(絶版)
[2] 特集 「不等式の世界」 (数学セミナー No.2-569) ,日本評論社,2009年2月号
   http://amazon.jp/o/ASIN/B001O9UIZ8
[3] 連載 「不等式の骨組み」 (大学への数学 vol.53,全12回,各4ページ),栗田哲也,東京出版,2009年4月号-2010年3月号
   http://www.tokyo-s.jp/index.shtml

不等式の埋蔵地

[1] RGMIA http://rgmia.vu.edu.au/
[2] Crux Mathematicorum Synopses http://www.journals.cms.math.ca/CRUX/synopses/
[3] Maths problems http://www.kalva.demon.co.uk/
[4] Mathematical Inequalities & Applications http://www.ele-math.com/
[5] American Mathematical Monthly http://www.maa.org/pubs/monthly.html
[6] Problems in the points contest of KoMaL http://www.komal.hu/verseny/feladatok.e.shtml
[7] IMO リンク集 http://imo.math.ca/
[9] Mathematical Olympiads Correspondence Program http://www.cms.math.ca/Competitions/MOCP/
[10] Mathematical Excalibur http://www.math.ust.hk/excalibur/
[11] MathLinks Contest http://www.mathlinks.ro/Forum/contest.html
[12] MATH PROBLEM SOLVING WEB PAGE http://www.math.northwestern.edu/~mlerma/problem_s... (要自動登録)
[13] Wolfram MathWorld http://mathworld.wolfram.com/
[14] GRA20 Problem Solving Group http://www.mat.uniroma2.it/~tauraso/GRA20/main.htm...
[15] American Mathematical Monthly Problems http://www.mat.uniroma2.it/~tauraso/AMM/amm.html
[16] Journal of Inequalities and Applications http://www.hindawi.com/journals/jia/
4132人目の素数さん:2012/03/29(木) 01:01:22.31
          __ノ)-'´ ̄ ̄`ー- 、_
        , '´  _. -‐'''"二ニニ=-`ヽ、
      /   /:::::; -‐''"        `ーノ
     /   /:::::/           \
     /    /::::::/          | | |  |
     |   |:::::/ /     |  | | | |  |
      |   |::/ / / |  | ||  | | ,ハ .| ,ハ|
      |   |/ / / /| ,ハノ| /|ノレ,ニ|ル' 
     |   |  | / / レ',二、レ′ ,ィイ|゙/   私は只の数ヲタなんかとは付き合わないわ。
.     |   \ ∠イ  ,イイ|    ,`-' |      頭が良くて数学が出来てかっこいい人。それが必要条件よ。
     |     l^,人|  ` `-'     ゝ  |        さらに Ann.of Math に論文書けば十分条件にもなるわよ。
      |      ` -'\       ー'  人          一番嫌いなのは論文数を増やすためにくだらない論文を書いて
    |        /(l     __/  ヽ、           良い論文の出版を遅らせるお馬鹿な人。
     |       (:::::`‐-、__  |::::`、     ヒニニヽ、         あなたの論文が Ann of Math に accept される確率は?
    |      / `‐-、::::::::::`‐-、::::\   /,ニニ、\            それとも最近は Inv. Math. の方が上かしら?
   |      |::::::::::::::::::|` -、:::::::,ヘ ̄|'、  ヒニ二、 \
.   |      /::::::::::::::::::|::::::::\/:::O`、::\   | '、   \
   |      /:::::::::::::::::::/:::::::::::::::::::::::::::::'、::::\ノ  ヽ、  |
  |      |:::::/:::::::::/:::::::::::::::::::::::::::::::::::'、',::::'、  /:\__/‐、
  |      |/:::::::::::/::::::::::::::::::::::::::::::::::O::| '、::| く::::::::::::: ̄|
   |     /_..-'´ ̄`ー-、:::::::::::::::::::::::::::::::::::|/:/`‐'::\;;;;;;;_|
   |    |/::::::::::::::::::::::\:::::::::::::::::::::::::::::|::/::::|::::/:::::::::::/
    |   /:::::::::::::::::::::::::::::::::|:::::::::::::::::::::O::|::|::::::|:::::::::::::::/
5不等式ヲタ ( ゚∀゚):2012/03/29(木) 01:02:59.67
海外不等式ヲタの生息地

[1] Journal of Inequalities in Pure and Applied Mathematics http://jipam.vu.edu.au/
[2] MIA Journal http://www.mia-journal.com/
[3] MathLinks Math Forum http://www.mathlinks.ro/Forum/forum-55.html


     凵@    ○   ∇ 、___,、´`゙;~、  ';冫 ☆
           ┏  ━ゝヽ''/  ≧ \━〆A!゚━━┓。
 ╋┓"〓┃  < ゝ\',冫。' |::::  \ ./ |゛△│´'´,.ゝ'┃.      ●┃ ┃┃
 ┃┃_.━┛ヤ━━━━━━|::::: (● (● |━━━━━━━━━  ━┛ ・ ・
        ∇  ┠─Σ-  ヽ::::... .ワ.....ノ  冫 そ',´; ┨'゚,。
           .。冫▽ <   ⊂     ./⊃     乙  ≧   ▽
         。 ┃   Σ   (⌒ゞ ,l, 、''  │   て く
           ┠─ム┼   ゝ,,ノ ノゝ. 、,, .┼ ァ Ζ┨ ミo''`
         。、゚`。、   i/   レ' o。了 、'' ×  个o
        ○  ┃   `、,~´+√ ▽   ',!ヽ.◇    o┃
            ┗〆━┷ Z,.' /┷━''o ヾo┷+\━┛,゛;
       ヾ   凵@              '、´    ∇

我々は不等式ヲタだ!
お前達を同化する。抵抗は無意味だ!


6132人目の素数さん:2012/03/29(木) 03:24:39.86
まとめwiki、もっと充実するといいね。
俺もだけれど、暇な時でも見つけてスレ内の面白かった不等式(とその証明)のっけていければいいのだけれど
7132人目の素数さん:2012/03/29(木) 04:47:49.66
不等式は人気者だな
8132人目の素数さん:2012/03/29(木) 05:34:24.82
>>2のurl何かおかしくね?
9132人目の素数さん:2012/03/29(木) 06:01:41.40
>>8
具体的に、どの本?
10不等式ヲタ ( ゚∀゚):2012/03/29(木) 06:10:08.45
>>2 の修正
すまなんだ
まとめWikiからコピペしたら、長いURLは ...... と略されるのを忘れていました

不等式の本
[1] 不等式,ハーディ・リトルウッド・ポリヤ,シュプリンガー,2003年
   http://amazon.jp/o/ASIN/4431710566
[2] 不等式,大関信雄・青木雅計,槇書店,1967年(絶版)
[3] 不等式への招待,大関信雄・大関清太,近代科学社,1987年(絶版)
   http://direct.ips.co.jp/book/Template/Goods/go_BookstempKindai.cfm?GM_ID=KD1006&CM_ID=004300504&SPM_ID=1115&HN_NO=00430&PM_No=&PM_Class=
[4] 不等式入門(数学のかんどころシリーズ9),大関清太,共立出版,2012年
   http://www.kyoritsu-pub.co.jp/bookdetail/9784320019898
[5] 不等式入門,渡部隆一,森北出版,2005年
   http://amazon.jp/o/ASIN/4627010494
[6] 不等式の工学への応用、海津聰、森北出版,2004年
   http://amazon.jp/o/ASIN/4627075812
[7] 不等式(モノグラフ4),染取弘,科学新興新社,1990年
   http://amazon.jp/o/ASIN/4894281740
[8] 数学トレッキングツアー第3章「相加平均≧相乗平均」,東京理科大学数学教育研究所,教育出版,2006年
   http://amazon.jp/o/ASIN/4316801988
[9] 数学オリンピック事典,数学オリンピック財団,朝倉書店,2001年
   http://amazon.jp/o/ASIN/4254110871
[10] 獲得金メダル! 国際数学オリンピック第1章「不等式」,小林一章,朝倉書店,2011年
   http://www.asakura.co.jp/books/isbn/978-4-254-11132-3/
[11] 三角法の精選103問(シリーズ:数学オリンピックへの道 2),T.アンドレースク・Z.フェン著,朝倉書店,2010年
   http://www.asakura.co.jp/books/isbn/978-4-254-11808-7/
[12] The Cauchy-Schwarz Master Class: An Introduction to the Art of Mathematical Inequalities,J. M. Steele,Cambridge Univ. Pr.,2004年
   http://amazon.jp/o/ASIN/052154677X
[13] Inequalities: A Mathematical Olympiad Approach,Birkhaeuser Basel,2009年
   http://amazon.jp/dp/3034600496

11132人目の素数さん:2012/04/02(月) 01:54:22.46
        人
       (__)
      (__)  / ̄ ̄ ̄ ̄ ̄ ̄ ̄ ̄ ̄ ̄ ̄ ̄ ̄ ̄ ̄ ̄ ̄ ̄ ̄
      (∩・∀・)< もしもし? このスレに不等式1人前お願いします
 □……(つ   ) \___________________
 ̄ ̄ ̄ ̄ ̄ ̄ ̄ ̄|



( ゚∀゚) マイドー、税込1050円デス

x、y、z > 0、xyz=1 のとき、(x/(1+x))^2 + (y/(1+y))^2 + (z/(1+z))^2 ≧ 3/4
12132人目の素数さん:2012/04/02(月) 01:55:54.75
不等式の本が出たってのに、なぜかスルーですな
13132人目の素数さん:2012/04/02(月) 06:22:54.06

x、y、z > 0、xyz=1 のとき、(x^2/(1+x))^2 + (y^2/(1+y))^2 + (z^2/(1+z))^2 ≧ 3/4
14132人目の素数さん:2012/04/02(月) 20:45:12.54
        人
       (__)    / ̄ ̄ ̄ ̄ ̄ ̄ ̄ ̄ ̄ ̄ ̄ ̄ ̄ ̄ ̄
      (__)  / あ〜もしもし、追加注文お願いします
      (∩・∀・)<  うんち特盛り…じゃなくて、類題1人前
 □……(つ   ) \________________
 ̄ ̄ ̄ ̄ ̄ ̄ ̄ ̄|



      ( ゚∀゚) マイドー、税込850円デス
      ( O┬O
  ≡ ◎-ヽJ┴◎  キコキコ

x、y、z > 0、xyz=1 のとき、x/(1+y) + y/(1+z) + z/(1+x) ≧ 3/2
15132人目の素数さん:2012/04/04(水) 06:35:52.58
>>11-14
不等式の左辺の分母の1を(abc)^(1/3)に置き換えたり、
あんなことやこんなことをするんだろうと思うけど、できぬ・・・
16132人目の素数さん:2012/04/08(日) 01:02:39.63
三角形の辺の長さ a、b、c に対して、(a^2)/(b^2 +ac) のとりうる値の範囲を求めよ
17132人目の素数さん:2012/04/14(土) 00:07:17.24
>>16
分母分子a^2で割って、b/a、c/aを置き換えて三角形の成立条件とか使うん蟹?
18132人目の素数さん:2012/04/14(土) 14:40:05.35
これってエレガントというか、きれいな解法はないだろうか?
そんなに各種不等式に詳しくないので、ここで聞きます。

abc=1を満たす正の実数a,b,cにたいして、
a^3+b^3+c^3+6≧(a+b+c)^2
19132人目の素数さん:2012/04/14(土) 23:44:24.72
>>18
コレクションにあるかな〜と思って探したけど持ってなかった
とりあえず ( ゚∀゚)グッジョブ!
考えてみ松
20132人目の素数さん:2012/04/18(水) 17:08:14.50
589:132人目の素数さん[]
2012/04/13(金) 17:58:32.59

表の出る確率がp(1/2≦p≦1)のコインを奇数枚投げたとき
表の出たコインの枚数が,裏の出たコインの枚数より多い確率は
p以上であることを示せ

★東大入試作問者になったつもりのスレ★ 第二十問
http://uni.2ch.net/test/read.cgi/math/1325575627/
21132人目の素数さん:2012/04/18(水) 22:24:05.39
グラフを描いてみると成り立ってそうだけど証明法が解らずにいる不等式

x erfc(x) < 1/4.

22132人目の素数さん:2012/04/19(木) 15:39:59.97
http://www.cs.berkeley.edu/~wkahan/MathH90/Putnam99.pdf

Problem B4: Let ? be a real function with a continuous third derivative such that ?(x) ,
?'(x) , ?"(x) and ?'"(x) are positive for all x . Suppose that ?'"(x) £ ?(x) for all x . Show
that ?'(x) < 2?(x) for all x .
Solution B4: For any x and X , Taylor’s formula for ?(X) with an integral remainder is
?(X) = ?(x) + (X?x)?'(x) + (X?x)2?"(x)/2 + ox
X (X?t)2?'"(t)dt/2 ; and if X < x we find, since
?'"(t) > 0 , that ?(X) < ?(x) + (X?x)?'(x) + (X?x)2?"(x)/2 . Set X := x ? ?'(x)/?"(x) < x to
infer first 0 < ?(X) < ?(x) ? ?'(x)2/?"(x) + (?'(x)2/?"(x))/2 and then ?'(x)2 < 2?(x)?"(x) .
Similarly ?'(X) = ?'(x) + (X?x)?"(x) + ox
X (X?t)?'"(t)dt ; now, since ?'" £ ? and X?t has
the same sign as X?x and also dt , we infer ?'(X) £ ?'(x) + (X?x)?"(x) + ox
X (X?t)?(t)dt .
Integration by parts turns this last inequality into
?'(X) £ ?'(x) + (X?x)?"(x) + (X?x)2?(x)/2 + ox
X (X?t)2?'(t)dt/2 .
Again, if X < x we find, since ?'(t) > 0 , that ?'(X) £ ?'(x) + (X?x)?"(x) + (X?x)2?(x)/2 ;
this time set X := x ? ?"(x)/?(x) < x to infer from 0 < ?'(X) that ?"(x)2 < 2?'(x)?(x) . This
combines with the inequality ?'(x)2 < 2?(x)?"(x) inferred above to prove ?'(x) < 2?(x) , QED.
( An example of such a function ?(x) is s + exp(m・x) for any constants s 3 0 < m £ 1 . The
example s + (mx + O(m2x2+1))n for n 3 2 , s 3 0 and m £ 3O((5/2)5/2/(n(n+1)5/2(n+2))) is
less obvious.)
23132人目の素数さん:2012/04/19(木) 16:44:12.25
>> 18
Cirtoajeのプレプリのアイデアを応用すれば, x,y,z が正のとき,
x^9+y^9+z^9 + 6x^3 y^3 z^3 ≧ xyz((x^6+y^6+z^6)+(x^3y^3+y^3z^3+z^3x^3))
が成り立つことは証明できますが, 泥臭くて長い計算(A4版TeX印刷で2ページくらい)になります。
p=x+y+z, q=xy+yz+zx, r=xyz とおいて, 不等式の (左辺)-(右辺) をp,q,rで書き直したとき, rの2次式になるのがポイントです。
鮮やかな簡明な証明は思いつきません。
ところで, 次の問題のほうがずっと難しいと思いますよ(私の証明でA4版10ページくらい)。
x, y, z が正のとき
x^5+y^5+z^5 + (5 \root{4}\of{5}/4 - 1)xyz(x^2+y^2+z^2)
≧ (5 \root{4}\of{5}/4)(x^4y+y^4z+z^4x)

斉次対称不等式はいろいろな一般論が利用できますが, 巡回不等式のほうは難しいのです。
24132人目の素数さん:2012/04/19(木) 19:14:59.91
5/4^(4/5).
25132人目の素数さん:2012/04/19(木) 21:27:26.13
>>23
不等式の本を早く出してください、ワクワク…
26132人目の素数さん:2012/04/20(金) 14:21:33.95
0≦x[i]≦1,x[1]+x[2]+…+x[n]=1のとき
x[1]+x[1]x[2]+x[1]x[2]x[3]+…+x[1]x[2]x[3]…x[n]
の最大値って出せます?
27132人目の素数さん:2012/04/20(金) 14:35:40.04
>>26
ラグランジュの未定乗数法は試した?
28132人目の素数さん:2012/04/20(金) 14:40:01.88
1。
2926:2012/04/20(金) 19:20:41.75
解決しました
最大値はx[1]=1のときですね
30132人目の素数さん:2012/04/20(金) 20:52:45.54
          __ノ)-'´ ̄ ̄`ー- 、_
        , '´  _. -‐'''"二ニニ=-`ヽ、
      /   /:::::; -‐''"        `ーノ
     /   /:::::/           \
     /    /::::::/          | | |  |
     |   |:::::/ /     |  | | | |  |
      |   |::/ / / |  | ||  | | ,ハ .| ,ハ|
      |   |/ / / /| ,ハノ| /|ノレ,ニ|ル' 
     |   |  | / / レ',二、レ′ ,ィイ|゙/   私は只の数ヲタなんかとは付き合わないわ。
.     |   \ ∠イ  ,イイ|    ,`-' |      頭が良くて数学が出来てかっこいい人。それが必要条件よ。
     |     l^,人|  ` `-'     ゝ  |        さらに Ann.of Math に論文書けば十分条件にもなるわよ。
      |      ` -'\       ー'  人          一番嫌いなのは論文数を増やすためにくだらない論文を書いて
    |        /(l     __/  ヽ、           良い論文の出版を遅らせるお馬鹿な人。
     |       (:::::`‐-、__  |::::`、     ヒニニヽ、         あなたの論文が Ann of Math に accept される確率は?
    |      / `‐-、::::::::::`‐-、::::\   /,ニニ、\            それとも最近は Inv. Math. の方が上かしら?
   |      |::::::::::::::::::|` -、:::::::,ヘ ̄|'、  ヒニ二、 \
.   |      /::::::::::::::::::|::::::::\/:::O`、::\   | '、   \
   |      /:::::::::::::::::::/:::::::::::::::::::::::::::::'、::::\ノ  ヽ、  |
  |      |:::::/:::::::::/:::::::::::::::::::::::::::::::::::'、',::::'、  /:\__/‐、
  |      |/:::::::::::/::::::::::::::::::::::::::::::::::O::| '、::| く::::::::::::: ̄|
   |     /_..-'´ ̄`ー-、:::::::::::::::::::::::::::::::::::|/:/`‐'::\;;;;;;;_|
   |    |/::::::::::::::::::::::\:::::::::::::::::::::::::::::|::/::::|::::/:::::::::::/
    |   /:::::::::::::::::::::::::::::::::|:::::::::::::::::::::O::|::|::::::|:::::::::::::::/
31132人目の素数さん:2012/04/21(土) 14:20:47.71
          __ノ)-'´ ̄ ̄`ー- 、_
        , '´  _. -‐'''"二ニニ=-`ヽ、
      /   /:::::; -‐''"        `ーノ
     /   /:::::/           \
     /    /::::::/          | | |  |
     |   |:::::/ /     |  | | | |  |
      |   |::/ / / |  | ||  | | ,ハ .| ,ハ|
      |   |/ / / /| ,ハノ| /|ノレ,ニ|ル' 
     |   |  | / / レ',二、レ′ ,ィイ|゙/   私は只の数ヲタなんかとは付き合わないわ。
.     |   \ ∠イ  ,イイ|    ,`-' |      頭が良くて数学が出来てかっこいい人。それが必要条件よ。
     |     l^,人|  ` `-'     ゝ  |        さらに Ann.of Math に論文書けば十分条件にもなるわよ。
      |      ` -'\       ー'  人          一番嫌いなのは論文数を増やすためにくだらない論文を書いて
    |        /(l     __/  ヽ、           良い論文の出版を遅らせるお馬鹿な人。
     |       (:::::`‐-、__  |::::`、     ヒニニヽ、         あなたの論文が Ann of Math に accept される確率は?
    |      / `‐-、::::::::::`‐-、::::\   /,ニニ、\            それとも最近は Inv. Math. の方が上かしら?
   |      |::::::::::::::::::|` -、:::::::,ヘ ̄|'、  ヒニ二、 \
.   |      /::::::::::::::::::|::::::::\/:::O`、::\   | '、   \
   |      /:::::::::::::::::::/:::::::::::::::::::::::::::::'、::::\ノ  ヽ、  |
  |      |:::::/:::::::::/:::::::::::::::::::::::::::::::::::'、',::::'、  /:\__/‐、
  |      |/:::::::::::/::::::::::::::::::::::::::::::::::O::| '、::| く::::::::::::: ̄|
   |     /_..-'´ ̄`ー-、:::::::::::::::::::::::::::::::::::|/:/`‐'::\;;;;;;;_|
   |    |/::::::::::::::::::::::\:::::::::::::::::::::::::::::|::/::::|::::/:::::::::::/
    |   /:::::::::::::::::::::::::::::::::|:::::::::::::::::::::O::|::|::::::|:::::::::::::::/
32132人目の素数さん:2012/04/21(土) 18:27:35.41
          __ノ)-'´ ̄ ̄`ー- 、_
        , '´  _. -‐'''"二ニニ=-`ヽ、
      /   /:::::; -‐''"        `ーノ
     /   /:::::/           \
     /    /::::::/          | | |  |
     |   |:::::/ /     |  | | | |  |
      |   |::/ / / |  | ||  | | ,ハ .| ,ハ|
      |   |/ / / /| ,ハノ| /|ノレ,ニ|ル' 
     |   |  | / / レ',二、レ′ ,ィイ|゙/   私は只の数ヲタなんかとは付き合わないわ。
.     |   \ ∠イ  ,イイ|    ,`-' |      頭が良くて数学が出来てかっこいい人。それが必要条件よ。
     |     l^,人|  ` `-'     ゝ  |        さらに Ann.of Math に論文書けば十分条件にもなるわよ。
      |      ` -'\       ー'  人          一番嫌いなのは論文数を増やすためにくだらない論文を書いて
    |        /(l     __/  ヽ、           良い論文の出版を遅らせるお馬鹿な人。
     |       (:::::`‐-、__  |::::`、     ヒニニヽ、         あなたの論文が Ann of Math に accept される確率は?
    |      / `‐-、::::::::::`‐-、::::\   /,ニニ、\            それとも最近は Inv. Math. の方が上かしら?
   |      |::::::::::::::::::|` -、:::::::,ヘ ̄|'、  ヒニ二、 \
.   |      /::::::::::::::::::|::::::::\/:::O`、::\   | '、   \
   |      /:::::::::::::::::::/:::::::::::::::::::::::::::::'、::::\ノ  ヽ、  |
  |      |:::::/:::::::::/:::::::::::::::::::::::::::::::::::'、',::::'、  /:\__/‐、
  |      |/:::::::::::/::::::::::::::::::::::::::::::::::O::| '、::| く::::::::::::: ̄|
   |     /_..-'´ ̄`ー-、:::::::::::::::::::::::::::::::::::|/:/`‐'::\;;;;;;;_|
   |    |/::::::::::::::::::::::\:::::::::::::::::::::::::::::|::/::::|::::/:::::::::::/
    |   /:::::::::::::::::::::::::::::::::|:::::::::::::::::::::O::|::|::::::|:::::::::::::::/
33132人目の素数さん:2012/04/21(土) 22:31:50.93
          __ノ)-'´ ̄ ̄`ー- 、_
        , '´  _. -‐'''"二ニニ=-`ヽ、
      /   /:::::; -‐''"        `ーノ
     /   /:::::/           \
     /    /::::::/          | | |  |
     |   |:::::/ /     |  | | | |  |
      |   |::/ / / |  | ||  | | ,ハ .| ,ハ|
      |   |/ / / /| ,ハノ| /|ノレ,ニ|ル' 
     |   |  | / / レ',二、レ′ ,ィイ|゙/   私は只の数ヲタなんかとは付き合わないわ。
.     |   \ ∠イ  ,イイ|    ,`-' |      頭が良くて数学が出来てかっこいい人。それが必要条件よ。
     |     l^,人|  ` `-'     ゝ  |        さらに Ann.of Math に論文書けば十分条件にもなるわよ。
      |      ` -'\       ー'  人          一番嫌いなのは論文数を増やすためにくだらない論文を書いて
    |        /(l     __/  ヽ、           良い論文の出版を遅らせるお馬鹿な人。
     |       (:::::`‐-、__  |::::`、     ヒニニヽ、         あなたの論文が Ann of Math に accept される確率は?
    |      / `‐-、::::::::::`‐-、::::\   /,ニニ、\            それとも最近は Inv. Math. の方が上かしら?
   |      |::::::::::::::::::|` -、:::::::,ヘ ̄|'、  ヒニ二、 \
.   |      /::::::::::::::::::|::::::::\/:::O`、::\   | '、   \
   |      /:::::::::::::::::::/:::::::::::::::::::::::::::::'、::::\ノ  ヽ、  |
  |      |:::::/:::::::::/:::::::::::::::::::::::::::::::::::'、',::::'、  /:\__/‐、
  |      |/:::::::::::/::::::::::::::::::::::::::::::::::O::| '、::| く::::::::::::: ̄|
   |     /_..-'´ ̄`ー-、:::::::::::::::::::::::::::::::::::|/:/`‐'::\;;;;;;;_|
   |    |/::::::::::::::::::::::\:::::::::::::::::::::::::::::|::/::::|::::/:::::::::::/
    |   /:::::::::::::::::::::::::::::::::|:::::::::::::::::::::O::|::|::::::|:::::::::::::::/
34132人目の素数さん:2012/04/22(日) 01:10:24.15
自演バレバレだぞ prime_132
35132人目の素数さん:2012/04/22(日) 01:38:31.77
さいきん数学板のあちこちに糞AAを貼りまくっている奴の正体がprime_132なのか
検索すると、あちこちに出没しているようだな
36132人目の素数さん:2012/04/22(日) 06:07:17.58
          __ノ)-'´ ̄ ̄`ー- 、_
        , '´  _. -‐'''"二ニニ=-`ヽ、
      /   /:::::; -‐''"        `ーノ
     /   /:::::/           \
     /    /::::::/          | | |  |
     |   |:::::/ /     |  | | | |  |
      |   |::/ / / |  | ||  | | ,ハ .| ,ハ|
      |   |/ / / /| ,ハノ| /|ノレ,ニ|ル' 
     |   |  | / / レ',二、レ′ ,ィイ|゙/   私は只の数ヲタなんかとは付き合わないわ。
.     |   \ ∠イ  ,イイ|    ,`-' |      頭が良くて数学が出来てかっこいい人。それが必要条件よ。
     |     l^,人|  ` `-'     ゝ  |        さらに Ann.of Math に論文書けば十分条件にもなるわよ。
      |      ` -'\       ー'  人          一番嫌いなのは論文数を増やすためにくだらない論文を書いて
    |        /(l     __/  ヽ、           良い論文の出版を遅らせるお馬鹿な人。
     |       (:::::`‐-、__  |::::`、     ヒニニヽ、         あなたの論文が Ann of Math に accept される確率は?
    |      / `‐-、::::::::::`‐-、::::\   /,ニニ、\            それとも最近は Inv. Math. の方が上かしら?
   |      |::::::::::::::::::|` -、:::::::,ヘ ̄|'、  ヒニ二、 \
.   |      /::::::::::::::::::|::::::::\/:::O`、::\   | '、   \
   |      /:::::::::::::::::::/:::::::::::::::::::::::::::::'、::::\ノ  ヽ、  |
  |      |:::::/:::::::::/:::::::::::::::::::::::::::::::::::'、',::::'、  /:\__/‐、
  |      |/:::::::::::/::::::::::::::::::::::::::::::::::O::| '、::| く::::::::::::: ̄|
   |     /_..-'´ ̄`ー-、:::::::::::::::::::::::::::::::::::|/:/`‐'::\;;;;;;;_|
   |    |/::::::::::::::::::::::\:::::::::::::::::::::::::::::|::/::::|::::/:::::::::::/
    |   /:::::::::::::::::::::::::::::::::|:::::::::::::::::::::O::|::|::::::|:::::::::::::::/
37132人目の素数さん:2012/04/22(日) 08:43:25.86
          __ノ)-'´ ̄ ̄`ー- 、_
        , '´  _. -‐'''"二ニニ=-`ヽ、
      /   /:::::; -‐''"        `ーノ
     /   /:::::/           \
     /    /::::::/          | | |  |
     |   |:::::/ /     |  | | | |  |
      |   |::/ / / |  | ||  | | ,ハ .| ,ハ|
      |   |/ / / /| ,ハノ| /|ノレ,ニ|ル' 
     |   |  | / / レ',二、レ′ ,ィイ|゙/   私は只の数ヲタなんかとは付き合わないわ。
.     |   \ ∠イ  ,イイ|    ,`-' |      頭が良くて数学が出来てかっこいい人。それが必要条件よ。
     |     l^,人|  ` `-'     ゝ  |        さらに Ann.of Math に論文書けば十分条件にもなるわよ。
      |      ` -'\       ー'  人          一番嫌いなのは論文数を増やすためにくだらない論文を書いて
    |        /(l     __/  ヽ、           良い論文の出版を遅らせるお馬鹿な人。
     |       (:::::`‐-、__  |::::`、     ヒニニヽ、         あなたの論文が Ann of Math に accept される確率は?
    |      / `‐-、::::::::::`‐-、::::\   /,ニニ、\            それとも最近は Inv. Math. の方が上かしら?
   |      |::::::::::::::::::|` -、:::::::,ヘ ̄|'、  ヒニ二、 \
.   |      /::::::::::::::::::|::::::::\/:::O`、::\   | '、   \
   |      /:::::::::::::::::::/:::::::::::::::::::::::::::::'、::::\ノ  ヽ、  |
  |      |:::::/:::::::::/:::::::::::::::::::::::::::::::::::'、',::::'、  /:\__/‐、
  |      |/:::::::::::/::::::::::::::::::::::::::::::::::O::| '、::| く::::::::::::: ̄|
   |     /_..-'´ ̄`ー-、:::::::::::::::::::::::::::::::::::|/:/`‐'::\;;;;;;;_|
   |    |/::::::::::::::::::::::\:::::::::::::::::::::::::::::|::/::::|::::/:::::::::::/
    |   /:::::::::::::::::::::::::::::::::|:::::::::::::::::::::O::|::|::::::|:::::::::::::::/
38132人目の素数さん:2012/04/22(日) 11:36:02.18
調べてみたら自分より有能な人の存在を認めたくない有象無象の1人にしか見えんのだが。
こんなあわれな子供が荒らしに時間を費やしているとしたら世も末だなぁ。
39132人目の素数さん:2012/04/22(日) 19:07:13.39
          __ノ)-'´ ̄ ̄`ー- 、_
        , '´  _. -‐'''"二ニニ=-`ヽ、
      /   /:::::; -‐''"        `ーノ
     /   /:::::/           \
     /    /::::::/          | | |  |
     |   |:::::/ /     |  | | | |  |
      |   |::/ / / |  | ||  | | ,ハ .| ,ハ|
      |   |/ / / /| ,ハノ| /|ノレ,ニ|ル' 
     |   |  | / / レ',二、レ′ ,ィイ|゙/   私は只の数ヲタなんかとは付き合わないわ。
.     |   \ ∠イ  ,イイ|    ,`-' |      頭が良くて数学が出来てかっこいい人。それが必要条件よ。
     |     l^,人|  ` `-'     ゝ  |        さらに Ann.of Math に論文書けば十分条件にもなるわよ。
      |      ` -'\       ー'  人          一番嫌いなのは論文数を増やすためにくだらない論文を書いて
    |        /(l     __/  ヽ、           良い論文の出版を遅らせるお馬鹿な人。
     |       (:::::`‐-、__  |::::`、     ヒニニヽ、         あなたの論文が Ann of Math に accept される確率は?
    |      / `‐-、::::::::::`‐-、::::\   /,ニニ、\            それとも最近は Inv. Math. の方が上かしら?
   |      |::::::::::::::::::|` -、:::::::,ヘ ̄|'、  ヒニ二、 \
.   |      /::::::::::::::::::|::::::::\/:::O`、::\   | '、   \
   |      /:::::::::::::::::::/:::::::::::::::::::::::::::::'、::::\ノ  ヽ、  |
  |      |:::::/:::::::::/:::::::::::::::::::::::::::::::::::'、',::::'、  /:\__/‐、
  |      |/:::::::::::/::::::::::::::::::::::::::::::::::O::| '、::| く::::::::::::: ̄|
   |     /_..-'´ ̄`ー-、:::::::::::::::::::::::::::::::::::|/:/`‐'::\;;;;;;;_|
   |    |/::::::::::::::::::::::\:::::::::::::::::::::::::::::|::/::::|::::/:::::::::::/
    |   /:::::::::::::::::::::::::::::::::|:::::::::::::::::::::O::|::|::::::|:::::::::::::::/
40132人目の素数さん:2012/04/24(火) 20:03:03.05
ところで, 日本では, 不等式の研究集会とかシンポジウム, って聞きませんね。
誰か開く予定があったら教えて下さい。
41132人目の素数さん:2012/04/24(火) 23:24:55.97
          __ノ)-'´ ̄ ̄`ー- 、_
        , '´  _. -‐'''"二ニニ=-`ヽ、
      /   /:::::; -‐''"        `ーノ
     /   /:::::/           \
     /    /::::::/          | | |  |
     |   |:::::/ /     |  | | | |  |
      |   |::/ / / |  | ||  | | ,ハ .| ,ハ|
      |   |/ / / /| ,ハノ| /|ノレ,ニ|ル' 
     |   |  | / / レ',二、レ′ ,ィイ|゙/   私は只の数ヲタなんかとは付き合わないわ。
.     |   \ ∠イ  ,イイ|    ,`-' |      頭が良くて数学が出来てかっこいい人。それが必要条件よ。
     |     l^,人|  ` `-'     ゝ  |        さらに Ann.of Math に論文書けば十分条件にもなるわよ。
      |      ` -'\       ー'  人          一番嫌いなのは論文数を増やすためにくだらない論文を書いて
    |        /(l     __/  ヽ、           良い論文の出版を遅らせるお馬鹿な人。
     |       (:::::`‐-、__  |::::`、     ヒニニヽ、         あなたの論文が Ann of Math に accept される確率は?
    |      / `‐-、::::::::::`‐-、::::\   /,ニニ、\            それとも最近は Inv. Math. の方が上かしら?
   |      |::::::::::::::::::|` -、:::::::,ヘ ̄|'、  ヒニ二、 \
.   |      /::::::::::::::::::|::::::::\/:::O`、::\   | '、   \
   |      /:::::::::::::::::::/:::::::::::::::::::::::::::::'、::::\ノ  ヽ、  |
  |      |:::::/:::::::::/:::::::::::::::::::::::::::::::::::'、',::::'、  /:\__/‐、
  |      |/:::::::::::/::::::::::::::::::::::::::::::::::O::| '、::| く::::::::::::: ̄|
   |     /_..-'´ ̄`ー-、:::::::::::::::::::::::::::::::::::|/:/`‐'::\;;;;;;;_|
   |    |/::::::::::::::::::::::\:::::::::::::::::::::::::::::|::/::::|::::/:::::::::::/
    |   /:::::::::::::::::::::::::::::::::|:::::::::::::::::::::O::|::|::::::|:::::::::::::::/
42132人目の素数さん:2012/04/25(水) 22:31:37.97
18 abc=1を満たす正の実数a,b,cにたいして、
a^3+b^3+c^3+6≧(a+b+c)^2>>

Schur's Inequality
43132人目の素数さん:2012/04/25(水) 22:38:14.04
x, y, z>0⇒(x^3+3y^3)/(5x+y)+(y^3+3z^3)/(5y+z)+(z^3+3x^3)/(5z+x)≧(2/3)(x^2+y^2+z^2)
44132人目の素数さん:2012/04/26(木) 01:55:04.80
          __ノ)-'´ ̄ ̄`ー- 、_
        , '´  _. -‐'''"二ニニ=-`ヽ、
      /   /:::::; -‐''"        `ーノ
     /   /:::::/           \
     /    /::::::/          | | |  |
     |   |:::::/ /     |  | | | |  |
      |   |::/ / / |  | ||  | | ,ハ .| ,ハ|
      |   |/ / / /| ,ハノ| /|ノレ,ニ|ル' 
     |   |  | / / レ',二、レ′ ,ィイ|゙/   私は只の数ヲタなんかとは付き合わないわ。
.     |   \ ∠イ  ,イイ|    ,`-' |      頭が良くて数学が出来てかっこいい人。それが必要条件よ。
     |     l^,人|  ` `-'     ゝ  |        さらに Ann.of Math に論文書けば十分条件にもなるわよ。
      |      ` -'\       ー'  人          一番嫌いなのは論文数を増やすためにくだらない論文を書いて
    |        /(l     __/  ヽ、           良い論文の出版を遅らせるお馬鹿な人。
     |       (:::::`‐-、__  |::::`、     ヒニニヽ、         あなたの論文が Ann of Math に accept される確率は?
    |      / `‐-、::::::::::`‐-、::::\   /,ニニ、\            それとも最近は Inv. Math. の方が上かしら?
   |      |::::::::::::::::::|` -、:::::::,ヘ ̄|'、  ヒニ二、 \
.   |      /::::::::::::::::::|::::::::\/:::O`、::\   | '、   \
   |      /:::::::::::::::::::/:::::::::::::::::::::::::::::'、::::\ノ  ヽ、  |
  |      |:::::/:::::::::/:::::::::::::::::::::::::::::::::::'、',::::'、  /:\__/‐、
  |      |/:::::::::::/::::::::::::::::::::::::::::::::::O::| '、::| く::::::::::::: ̄|
   |     /_..-'´ ̄`ー-、:::::::::::::::::::::::::::::::::::|/:/`‐'::\;;;;;;;_|
   |    |/::::::::::::::::::::::\:::::::::::::::::::::::::::::|::/::::|::::/:::::::::::/
    |   /:::::::::::::::::::::::::::::::::|:::::::::::::::::::::O::|::|::::::|:::::::::::::::/
45132人目の素数さん:2012/04/27(金) 01:16:09.28
876   ななし [2012/04/26(木) 01:52:05] sage
>>872
コーシーで
 {x(5x+y)+3y(5x+y)+y(5y+z)+3z(5y+z)+z(5z+x)+3x(5z+x)}{左辺}
  ≧ (x^2 +3y^2 +y^2 +3z^2 +z^2 +3x^2)^2
  = 16(x^2 +y^2 +z^2)^2,
ここで
 x(5x+y) +3y(5x+y) +y(5y+z) +3z(5y+z) +z(5z+x) +3x(5z+x)
  = 8(x+y+z)^2
  = 24(x^2 +y^2 +z^2) -8(x-y)^2 -8(y-z)^2 -8(z-x)^2
  ≦ 24(x^2 +y^2 +z^2),
より、
 {左辺} ≧ (2/3)(x^2 +y^2 +z^2),

877   prime_132 [2012/04/26(木) 01:57:59] sage
>>873-876
 正解です!
いろんな方法があるんですね。
46132人目の素数さん:2012/04/28(土) 17:52:30.44
          __ノ)-'´ ̄ ̄`ー- 、_
        , '´  _. -‐'''"二ニニ=-`ヽ、
      /   /:::::; -‐''"        `ーノ
     /   /:::::/           \
     /    /::::::/          | | |  |
     |   |:::::/ /     |  | | | |  |
      |   |::/ / / |  | ||  | | ,ハ .| ,ハ|
      |   |/ / / /| ,ハノ| /|ノレ,ニ|ル' 
     |   |  | / / レ',二、レ′ ,ィイ|゙/   私は只の数ヲタなんかとは付き合わないわ。
.     |   \ ∠イ  ,イイ|    ,`-' |      頭が良くて数学が出来てかっこいい人。それが必要条件よ。
     |     l^,人|  ` `-'     ゝ  |        さらに Ann.of Math に論文書けば十分条件にもなるわよ。
      |      ` -'\       ー'  人          一番嫌いなのは論文数を増やすためにくだらない論文を書いて
    |        /(l     __/  ヽ、           良い論文の出版を遅らせるお馬鹿な人。
     |       (:::::`‐-、__  |::::`、     ヒニニヽ、         あなたの論文が Ann of Math に accept される確率は?
    |      / `‐-、::::::::::`‐-、::::\   /,ニニ、\            それとも最近は Inv. Math. の方が上かしら?
   |      |::::::::::::::::::|` -、:::::::,ヘ ̄|'、  ヒニ二、 \
.   |      /::::::::::::::::::|::::::::\/:::O`、::\   | '、   \
   |      /:::::::::::::::::::/:::::::::::::::::::::::::::::'、::::\ノ  ヽ、  |
  |      |:::::/:::::::::/:::::::::::::::::::::::::::::::::::'、',::::'、  /:\__/‐、
  |      |/:::::::::::/::::::::::::::::::::::::::::::::::O::| '、::| く::::::::::::: ̄|
   |     /_..-'´ ̄`ー-、:::::::::::::::::::::::::::::::::::|/:/`‐'::\;;;;;;;_|
   |    |/::::::::::::::::::::::\:::::::::::::::::::::::::::::|::/::::|::::/:::::::::::/
    |   /:::::::::::::::::::::::::::::::::|:::::::::::::::::::::O::|::|::::::|:::::::::::::::/
47132人目の素数さん:2012/04/29(日) 06:09:34.91
実数 a、b、c が a^2 + b^2 + c^2 = 1 をみたすとき、ab+bc+ca のとりうる値の範囲を求めよ
48132人目の素数さん:2012/04/30(月) 05:00:04.63
>>2
[4]を買ったので、GWに読んでみます。
[2][3]を図書館で借りておいた方がいいですかね?
49132人目の素数さん:2012/05/01(火) 17:07:28.84
>>48ですが、[2][3]を確保しました
50132人目の素数さん:2012/05/02(水) 02:19:16.40
こんな不等式あった
http://s3.gazo.cc/up/s3_7474.png
51132人目の素数さん:2012/05/02(水) 10:03:42.29
  ┏┓     ┏┓           ┏━┓ ┏━┓
┏┛┗┓ ∧∧ ┗━┓        ┃  ┃ ┃  ┃
┗┓┏( つ ゚Д゚).┏┓┃┏━━━┓ ┃  ┃ ┃  ┃
┏┛┗ \ y⊂ ) ┛┃┗━━━┛ ┗━┛ ┗━┛
┗┓┏ /    \┓┃        ┏━┓ ┏━┓
  ┗┛∪ ̄ ̄ ̄\) ┛        ┗━┛ ┗━┛
52132人目の素数さん:2012/05/02(水) 19:41:33.88
>>50
どうやって証明するのでしょう?
53132人目の素数さん:2012/05/02(水) 23:45:34.16
54132人目の素数さん:2012/05/03(木) 00:05:54.70
綺麗すぎワロタ
右辺がすっきりしてるのにも理由があるのか
55132人目の素数さん:2012/05/03(木) 04:13:36.87
>>53
ふつくしい…
56132人目の素数さん:2012/05/03(木) 04:21:50.65
>>53
あなたが神か…
57132人目の素数さん:2012/05/03(木) 07:46:23.82
          __ノ)-'´ ̄ ̄`ー- 、_
        , '´  _. -‐'''"二ニニ=-`ヽ、
      /   /:::::; -‐''"        `ーノ
     /   /:::::/           \
     /    /::::::/          | | |  |
     |   |:::::/ /     |  | | | |  |
      |   |::/ / / |  | ||  | | ,ハ .| ,ハ|
      |   |/ / / /| ,ハノ| /|ノレ,ニ|ル' 
     |   |  | / / レ',二、レ′ ,ィイ|゙/   私は只の数ヲタなんかとは付き合わないわ。
.     |   \ ∠イ  ,イイ|    ,`-' |      頭が良くて数学が出来てかっこいい人。それが必要条件よ。
     |     l^,人|  ` `-'     ゝ  |        さらに Ann.of Math に論文書けば十分条件にもなるわよ。
      |      ` -'\       ー'  人          一番嫌いなのは論文数を増やすためにくだらない論文を書いて
    |        /(l     __/  ヽ、           良い論文の出版を遅らせるお馬鹿な人。
     |       (:::::`‐-、__  |::::`、     ヒニニヽ、         あなたの論文が Ann of Math に accept される確率は?
    |      / `‐-、::::::::::`‐-、::::\   /,ニニ、\            それとも最近は Inv. Math. の方が上かしら?
   |      |::::::::::::::::::|` -、:::::::,ヘ ̄|'、  ヒニ二、 \
.   |      /::::::::::::::::::|::::::::\/:::O`、::\   | '、   \
   |      /:::::::::::::::::::/:::::::::::::::::::::::::::::'、::::\ノ  ヽ、  |
  |      |:::::/:::::::::/:::::::::::::::::::::::::::::::::::'、',::::'、  /:\__/‐、
  |      |/:::::::::::/::::::::::::::::::::::::::::::::::O::| '、::| く::::::::::::: ̄|
   |     /_..-'´ ̄`ー-、:::::::::::::::::::::::::::::::::::|/:/`‐'::\;;;;;;;_|
   |    |/::::::::::::::::::::::\:::::::::::::::::::::::::::::|::/::::|::::/:::::::::::/
    |   /:::::::::::::::::::::::::::::::::|:::::::::::::::::::::O::|::|::::::|:::::::::::::::/
58132人目の素数さん:2012/05/03(木) 09:10:59.07
専ブラで糞AAをあぼーんしている俺に死角はない
59132人目の素数さん:2012/05/03(木) 10:16:07.80
>>53
きたか…!!

  ( ゚д゚ ) ガタッ
  .r   ヾ
__|_| / ̄ ̄ ̄/_
  \/    /
60132人目の素数さん:2012/05/04(金) 23:20:00.95
>>47
-1/2 ≦ ab+bc+ca ≦ 1
61132人目の素数さん:2012/05/05(土) 03:18:22.49
こんなのあった
正数x,y,zがx+y+z=1 をみたすとき、x^3+y^3+z^3+3xyz≧2/9
62132人目の素数さん:2012/05/06(日) 23:08:45.54
あ…ありのまま 今 起こった事を話すぜ!
GWが始まって不等式にハァハァしていたはずが気がついたらGWが終わっていた…
何を言ってるのか分からねーと思うが、俺も何をされたのか分からなかった…
頭がどうにかなりそうだった…
催眠術だとか超スピードだとか、そんなチャチなもんじゃあ 断じてねえ
もっと恐ろしいものの片鱗を味わったぜ…
63132人目の素数さん:2012/05/08(火) 15:54:59.64
非負実数 a、b、c が a+b+c=5 をみたすとき、(a^4)b + (b^4)c + (c^4)a ≦ 256
64132人目の素数さん:2012/05/08(火) 16:00:02.76
>>47
(a^2 + b^2 + c^2) + 2(ab+bc+ca) = (a+b+c)^2 ≧0
(a^2 + b^2 + c^2) - (ab+bc+ca) = (1/2){(a-b)^2 + (b-c)^2 + (c-a)^2} ≧0

>>61
わかりませんぬ
65132人目の素数さん:2012/05/08(火) 19:39:48.45
>>53
え、どっかのサイト?
66132人目の素数さん:2012/05/08(火) 21:39:53.64
>>65
聞く前に自分で確認しろや糞蟲
67132人目の素数さん:2012/05/08(火) 22:41:35.63
実数 a、b、c が a+b+c=0、a≧1 をみたすとき、a^4 + b^4 + c^4 -3abc ≧3/8

( ゚∀゚)プケラッチョ!
68132人目の素数さん:2012/05/09(水) 00:10:26.03
>>67
なんか見たことあるなあ
69132人目の素数さん:2012/05/09(水) 00:12:42.25
>>66
糞蟲野郎
70132人目の素数さん:2012/05/09(水) 21:07:19.55
>>63
問題間違ってないか?
71132人目の素数さん:2012/05/09(水) 21:16:19.31
>>70
間違ってないはずだけど、反例がありましたか?
72132人目の素数さん:2012/05/09(水) 21:16:27.75
          __ノ)-'´ ̄ ̄`ー- 、_
        , '´  _. -‐'''"二ニニ=-`ヽ、
      /   /:::::; -‐''"        `ーノ
     /   /:::::/           \
     /    /::::::/          | | |  |
     |   |:::::/ /     |  | | | |  |
      |   |::/ / / |  | ||  | | ,ハ .| ,ハ|
      |   |/ / / /| ,ハノ| /|ノレ,ニ|ル' 
     |   |  | / / レ',二、レ′ ,ィイ|゙/   私は只の数ヲタなんかとは付き合わないわ。
.     |   \ ∠イ  ,イイ|    ,`-' |      頭が良くて数学が出来てかっこいい人。それが必要条件よ。
     |     l^,人|  ` `-'     ゝ  |        さらに Ann.of Math に論文書けば十分条件にもなるわよ。
      |      ` -'\       ー'  人          一番嫌いなのは論文数を増やすためにくだらない論文を書いて
    |        /(l     __/  ヽ、           良い論文の出版を遅らせるお馬鹿な人。
     |       (:::::`‐-、__  |::::`、     ヒニニヽ、         あなたの論文が Ann of Math に accept される確率は?
    |      / `‐-、::::::::::`‐-、::::\   /,ニニ、\            それとも最近は Inv. Math. の方が上かしら?
   |      |::::::::::::::::::|` -、:::::::,ヘ ̄|'、  ヒニ二、 \
.   |      /::::::::::::::::::|::::::::\/:::O`、::\   | '、   \
   |      /:::::::::::::::::::/:::::::::::::::::::::::::::::'、::::\ノ  ヽ、  |
  |      |:::::/:::::::::/:::::::::::::::::::::::::::::::::::'、',::::'、  /:\__/‐、
  |      |/:::::::::::/::::::::::::::::::::::::::::::::::O::| '、::| く::::::::::::: ̄|
   |     /_..-'´ ̄`ー-、:::::::::::::::::::::::::::::::::::|/:/`‐'::\;;;;;;;_|
   |    |/::::::::::::::::::::::\:::::::::::::::::::::::::::::|::/::::|::::/:::::::::::/
    |   /:::::::::::::::::::::::::::::::::|:::::::::::::::::::::O::|::|::::::|:::::::::::::::/
73132人目の素数さん:2012/05/09(水) 21:40:02.28
>>71
ごめん。いいかも。
74132人目の素数さん:2012/05/10(木) 01:03:07.50
>>61どうやるの?おしえて不等式の神様
75132人目の素数さん:2012/05/10(木) 01:07:12.28
俺洩れも知りたい
76132人目の素数さん:2012/05/10(木) 01:13:07.02
次数が3ってことはあれだな
77132人目の素数さん:2012/05/12(土) 13:22:33.41
> 61 は 9(x^3+y^3+z^3 + 3xyz) - 2(x+y+3)^3 ≧ 0 をSchurの不等式で証明するだけ.
非負実数に関する3変数3次斉次対称不等式はSchurの不等式とMuirheadの不等式だけから証明できる、という定理があります.
> 63も5次斉次式にしてみれば,全部に項が正なので自明.
ところで,次は本当に2日前に(非初等的に)証明できた超難問です.

p = 1.48941118... は次の5次方程式解とする.
7 p^5 + 17 p^4 + 16 p^3 + 16 p^2 - 64 p - 128 = 0
このとき,任意の実数(負数も含む) x, y, z, に対して
x^4+y^4+z^4 + p(x^3y+y^3z+z^3x) ≧ 0
非自明な等号成立条件もあります.
78132人目の素数さん:2012/05/12(土) 19:58:52.59
ワクワクしますね
79132人目の素数さん:2012/05/15(火) 02:23:44.16
77さんがここのno1なの?
80132人目の素数さん:2012/05/15(火) 03:00:08.71
それはどうか知らんが、ここ一、二ヵ月ほどは、
どんな不等式でもたちどころに証明してしまう不等式神を見かけないので寂しい
81132人目の素数さん:2012/05/15(火) 16:36:38.39
どなたか >>21 お願いします (._.)
82132人目の素数さん:2012/05/15(火) 20:36:36.34
>>79
どうやら、不等式ヲタに同化されたのは最近のようだな
不等式ヲタとは何たるかを過去ログの書込で説明しよう!


学問・理系 [数学] “不等式への招待 第2章”

217 名前:132人目の素数さん[sage] 投稿日:2005/03/24(木) 22:03:28
>>216
不等式ヲタは群生体だから、無限の知識と無尽蔵の体力を持ってるんだよ ( ゚∀゚) テヘッ
不等式ヲタは1人2人じゃなく、1ヲタ2ヲタと数えるんだよ ( ゚∀゚) テヘッ
不等式ヲタは三角関数や nCr でも ハァハァ しちゃうんだよ ( ゚∀゚) テヘッ

不等式ヲタはデルタ宇宙域からやってきたよ ( ゚∀゚) テヘッ
不等式ヲタは世界中のあらゆるところから不等式を鬼集してるよ ( ゚∀゚) テヘッ
不等式ヲタは集めた不等式を同化し改良するから、抵抗は無意味だよ ( ゚∀゚) テヘッ

不等式ヲタはちょっとキチガイ入っているよ ( ゚∀゚) テヘッ
不等式ヲタは他スレを荒らさないから安心してね ( ゚∀゚) テヘッ
不等式ヲタのレスの半分は自作自演、残りはなりすましでできてるよ ( ゚∀゚) テヘッ

不等式を見て興奮した君は、すでに不等式ヲタに同化されているよ ( ゚∀゚) テヘッ
今日から君も不等式ヲタだ ( ゚∀゚)9 テヘッ
83132人目の素数さん:2012/05/16(水) 02:15:32.20
$a>0,\ b>0,\ a^2+b^2<1$ のとき,$\frac{ab(1-a^2-b^2)^{1/2}}{a(1-a^2)^{1/2}+b(1-b^2)^{1/2}}$の最大値を求めよ。
84132人目の素数さん:2012/05/16(水) 08:00:22.96
>>83
ヒントくれ!
85132人目の素数さん:2012/05/16(水) 11:47:14.63
甘えるな!
86132人目の素数さん:2012/05/16(水) 12:45:23.80
なんだとー
87132人目の素数さん:2012/05/16(水) 12:59:09.25
これむずいな
88132人目の素数さん:2012/05/16(水) 13:33:44.78
彡        ミ{、{v、⌒ヽ、       __________
  xイ            _イヽ、 ヽ.     |
 〃             川  ヽ.ヾヾ.    .|
               リ    ヽ.v|}    .|
             彡イ__   rェ'v'      | ↑ おねえちゃん いいからだしとるやんけ
           彡彡〃二二、_>'卞》,   |
          ,xイ ,.x≦《tッ= 〕f‐〔テ.} 》  | おれのもちものはでっかいぜ
        _,,x≦三ニ≡《__》"  ヽrく    \
    __xチ'<,        ̄ ̄ f⌒ ,,.. }:. ,   // ̄ ̄ ̄ ̄ ̄ ̄ ̄ ̄
   l´⌒>’`ゝ:;;ゝ          f ゝ-'´`く:.:. i   /'
   {  仆i             , :. ,xェュ,: |
 .  ∧  ゝム              ',:. r''ニ二え |
 ∨  ゝ、_ >,          `  :::   、.|
  \  {    :                、.::.:.:.:.|
    }川     :        :.:. .  ー'',r'
     l「 ̄ ̄≧x、,_        :::::::::::/
89132人目の素数さん:2012/05/16(水) 21:23:32.87
考えるな 感じるんだ
90132人目の素数さん:2012/05/18(金) 01:11:56.98
         \     不等式と言えば?         / Schurムズ… Jensen最強  Lehmusって
          \        ∧_∧ ∩AM-GMだろ / ∧_∧     ∧_∧      ∧_∧
Markovの不等式 \      ( ・∀・)ノ______  /  ( ;・∀・)    (; ´Д`)    (´Д`; )
の証明おしえて ∧ ∧\    (入   ⌒\つ  /|. /  ⊂   ⊂ )    ( つ ⊂ )    ( ⊃   ⊃
         (゚Д゚ )_\    ヾヽ  /\⌒)/  |/     〉 〉\\   〉 〉 く く   //( (
     / ̄ ̄∪ ∪ /| \  || ⌒| ̄ ̄ ̄|    /     (__) (_)  (_.)(_)  (_) (__)
   /∧_∧Polyaを読め \    ∧∧∧∧ /           
  / (;´∀` )_/       \  < 不    > レスの半分は自作自演、残りはなりすましでできてるよ
 || ̄(     つ ||/         \< 等 ま >  集めた不等式を同化し改良するから、抵抗は無意味だよ
 || (_○___)  ||            < 式    > 群生体だから無限の知識と無尽蔵の体力を持ってるよ
――――――――――――――― .< ヲ た >―――――――――――――――――――――
         ∧_∧ いつもながら < タ   >    ∧_∧テヘッ∧_∧   / ̄ ̄ ̄ ̄ ̄ ̄ ̄
         ( ;´∀`) 見事じゃのぉ <か  > \   ( ´∀`)  (´∀` )<不等式への招待
    _____(つ_ と)___       ./∨∨∨ 不\ (    )__(    )  \__復刊マダァ?
 . / \        ___ \キタァ  /  ∧_∧等 \∧ ∧   ∧ ∧  ̄ ̄ ̄/.//| ̄ ̄ ̄
 .<\※ \____.|i\___ヽ.ウヒョ ./γ(⌒)・∀・ ) 式 \   ;) (     ;)   / ┃| |
  ヽ\ ※ ※ ※|i i|.====B|i.ヽ  /(YYて)ノ   ノ  ヲ  \↑ ̄ ̄↑\)_/     |__|/
    \`ー──-.|\.|___|__◎_|_.i‐>/ \  ̄ ̄ ̄ ̄\  タ   \数ヲタ  | | ┃
      ̄ ̄ ̄ ̄|. | ̄ ̄ ̄ ̄| / ||ヽ|| ̄ ̄ ̄ ̄||  め    \   .|_)
91132人目の素数さん:2012/05/18(金) 09:46:49.98
>>83

$\frac{ab(1-a^2-b^2)^{1/2}}{a(1-a^2)^{1/2}+b(1-b^2)^{1/2}}=\frac{(1-a^2-b^2)^{1/2}}{\frac{(1-a^2)^{1/2}}{b}+\frac{(1-b^2)^{1/2}}{a}}$

$a^2+b^2=k^2$と固定し,分母$\frac{(1-a^2)^{1/2}}{b}+\frac{(1-b^2)^{1/2}}{a}$が最小になるときを求める。

$a^2+b^2=k^2$から$\frac{(b^2+1-k^2)^{1/2}}{b}+\frac{(a^2+1-k^2)^{1/2}}{a}=\sqrt{1+\frac{1}{\frac{b^2}{1-k^2}}}+\sqrt{1+\frac{1}{\frac{a^2}{1-k^2}}}$…★となる。

$f(x)=\sqrt{1+\frac{1}{x}}$は,$f''(x)>0$より下に凸なので,$\frac{f(s)+f(t)}{2}\geq f\left(\frac{s+t}{2}\right)$

$s,\,t$にそれぞれ$\frac{a^2}{1-k^2},\,\frac{b^2}{1-k^2}$を代入することにより,★は$a=b$のとき最小とわかる。

すると1変数になるので,あとはどうとでもやれ。$a=b=\frac{1}{\sqrt{2}}$のときに最大値$\frac12\left(\sqrt2-1\right)$をとる。
9291:2012/05/18(金) 10:17:11.98
最後の行まちがった。
$a=b=\sqrt{1-\frac{1}{\sqrt{2}}}$のときだ。
93132人目の素数さん:2012/05/19(土) 11:31:55.01
          __ノ)-'´ ̄ ̄`ー- 、_
        , '´  _. -‐'''"二ニニ=-`ヽ、
      /   /:::::; -‐''"        `ーノ
     /   /:::::/           \
     /    /::::::/          | | |  |
     |   |:::::/ /     |  | | | |  |
      |   |::/ / / |  | ||  | | ,ハ .| ,ハ|
      |   |/ / / /| ,ハノ| /|ノレ,ニ|ル' 
     |   |  | / / レ',二、レ′ ,ィイ|゙/   私は只の数ヲタなんかとは付き合わないわ。
.     |   \ ∠イ  ,イイ|    ,`-' |      頭が良くて数学が出来てかっこいい人。それが必要条件よ。
     |     l^,人|  ` `-'     ゝ  |        さらに Ann.of Math に論文書けば十分条件にもなるわよ。
      |      ` -'\       ー'  人          一番嫌いなのは論文数を増やすためにくだらない論文を書いて
    |        /(l     __/  ヽ、           良い論文の出版を遅らせるお馬鹿な人。
     |       (:::::`‐-、__  |::::`、     ヒニニヽ、         あなたの論文が Ann of Math に accept される確率は?
    |      / `‐-、::::::::::`‐-、::::\   /,ニニ、\            それとも最近は Inv. Math. の方が上かしら?
   |      |::::::::::::::::::|` -、:::::::,ヘ ̄|'、  ヒニ二、 \
.   |      /::::::::::::::::::|::::::::\/:::O`、::\   | '、   \
   |      /:::::::::::::::::::/:::::::::::::::::::::::::::::'、::::\ノ  ヽ、  |
  |      |:::::/:::::::::/:::::::::::::::::::::::::::::::::::'、',::::'、  /:\__/‐、
  |      |/:::::::::::/::::::::::::::::::::::::::::::::::O::| '、::| く::::::::::::: ̄|
   |     /_..-'´ ̄`ー-、:::::::::::::::::::::::::::::::::::|/:/`‐'::\;;;;;;;_|
   |    |/::::::::::::::::::::::\:::::::::::::::::::::::::::::|::/::::|::::/:::::::::::/
    |   /:::::::::::::::::::::::::::::::::|:::::::::::::::::::::O::|::|::::::|:::::::::::::::/
94132人目の素数さん:2012/05/19(土) 17:52:36.26
          __ノ)-'´ ̄ ̄`ー- 、_
        , '´  _. -‐'''"二ニニ=-`ヽ、
      /   /:::::; -‐''"        `ーノ
     /   /:::::/           \
     /    /::::::/          | | |  |
     |   |:::::/ /     |  | | | |  |
      |   |::/ / / |  | ||  | | ,ハ .| ,ハ|
      |   |/ / / /| ,ハノ| /|ノレ,ニ|ル' 
     |   |  | / / レ',二、レ′ ,ィイ|゙/   私は只の数ヲタなんかとは付き合わないわ。
.     |   \ ∠イ  ,イイ|    ,`-' |      頭が良くて数学が出来てかっこいい人。それが必要条件よ。
     |     l^,人|  ` `-'     ゝ  |        さらに Ann.of Math に論文書けば十分条件にもなるわよ。
      |      ` -'\       ー'  人          一番嫌いなのは論文数を増やすためにくだらない論文を書いて
    |        /(l     __/  ヽ、           良い論文の出版を遅らせるお馬鹿な人。
     |       (:::::`‐-、__  |::::`、     ヒニニヽ、         あなたの論文が Ann of Math に accept される確率は?
    |      / `‐-、::::::::::`‐-、::::\   /,ニニ、\            それとも最近は Inv. Math. の方が上かしら?
   |      |::::::::::::::::::|` -、:::::::,ヘ ̄|'、  ヒニ二、 \
.   |      /::::::::::::::::::|::::::::\/:::O`、::\   | '、   \
   |      /:::::::::::::::::::/:::::::::::::::::::::::::::::'、::::\ノ  ヽ、  |
  |      |:::::/:::::::::/:::::::::::::::::::::::::::::::::::'、',::::'、  /:\__/‐、
  |      |/:::::::::::/::::::::::::::::::::::::::::::::::O::| '、::| く::::::::::::: ̄|
   |     /_..-'´ ̄`ー-、:::::::::::::::::::::::::::::::::::|/:/`‐'::\;;;;;;;_|
   |    |/::::::::::::::::::::::\:::::::::::::::::::::::::::::|::/::::|::::/:::::::::::/
    |   /:::::::::::::::::::::::::::::::::|:::::::::::::::::::::O::|::|::::::|:::::::::::::::/
95132人目の素数さん:2012/05/19(土) 17:53:01.51
          __ノ)-'´ ̄ ̄`ー- 、_
        , '´  _. -‐'''"二ニニ=-`ヽ、
      /   /:::::; -‐''"        `ーノ
     /   /:::::/           \
     /    /::::::/          | | |  |
     |   |:::::/ /     |  | | | |  |
      |   |::/ / / |  | ||  | | ,ハ .| ,ハ|
      |   |/ / / /| ,ハノ| /|ノレ,ニ|ル' 
     |   |  | / / レ',二、レ′ ,ィイ|゙/   私は只の数ヲタなんかとは付き合わないわ。
.     |   \ ∠イ  ,イイ|    ,`-' |      頭が良くて数学が出来てかっこいい人。それが必要条件よ。
     |     l^,人|  ` `-'     ゝ  |        さらに Ann.of Math に論文書けば十分条件にもなるわよ。
      |      ` -'\       ー'  人          一番嫌いなのは論文数を増やすためにくだらない論文を書いて
    |        /(l     __/  ヽ、           良い論文の出版を遅らせるお馬鹿な人。
     |       (:::::`‐-、__  |::::`、     ヒニニヽ、         あなたの論文が Ann of Math に accept される確率は?
    |      / `‐-、::::::::::`‐-、::::\   /,ニニ、\            それとも最近は Inv. Math. の方が上かしら?
   |      |::::::::::::::::::|` -、:::::::,ヘ ̄|'、  ヒニ二、 \
.   |      /::::::::::::::::::|::::::::\/:::O`、::\   | '、   \
   |      /:::::::::::::::::::/:::::::::::::::::::::::::::::'、::::\ノ  ヽ、  |
  |      |:::::/:::::::::/:::::::::::::::::::::::::::::::::::'、',::::'、  /:\__/‐、
  |      |/:::::::::::/::::::::::::::::::::::::::::::::::O::| '、::| く::::::::::::: ̄|
   |     /_..-'´ ̄`ー-、:::::::::::::::::::::::::::::::::::|/:/`‐'::\;;;;;;;_|
   |    |/::::::::::::::::::::::\:::::::::::::::::::::::::::::|::/::::|::::/:::::::::::/
    |   /:::::::::::::::::::::::::::::::::|:::::::::::::::::::::O::|::|::::::|:::::::::::::::/
96132人目の素数さん:2012/05/19(土) 20:59:41.30
>>67

|a| ≧ 1 より
(左辺) - (右辺)
  ≧ (3/8)a^4 + b^4 + c^4 - 3aabc
  ≧ (5/8)a^4 + (1/2)(bb+cc)^2 -3aabc
  ≧ (5/8)a^4 + (1/8)(b+c)^4 -3aabc
  = (3/8){a^4 + (b+c)^4} - 3aabc (← |a|≧|b+c|)
  = (3/4){a(b+c)}^2 - 3aabc
  = (3/4){a(b-c)}^2
  ≧ 0,
97132人目の素数さん:2012/05/19(土) 21:39:03.97
>>21 >>81

・x>1 のとき、
 -t^2 ≦ -x^2 -2x(t-x),  (t=x での接線)
 erfc(x) < (2/√π)exp(-x^2)∫[x,∞) exp(-2x(t-x)) dt
  = (2/√π)exp(-x^2)∫[0,∞) exp(-2x・t') dt'
  = (2/√π)exp(-x^2) [ -(1/2x)exp(-2x・t') ]_0,^∞
  = 1/(x√π)exp(-x^2)
  < 1/(e(√π)x)       (← x>1)
  = 1/(4.8180x),

・x<1 のとき
 exp(-t^2) ≧ 1 - t^2,  (マクローリン)
 erfc(x) = 1 - (2/√π)∫[0,x] exp(-t^2) dt
  < 1 - (2/√π)∫[0,x] (1-t^2)dt
  = 1 - (2/√π)[t -(1/3)t^3 ]_0,^x
  = 1 - (2/√π){x -(1/3)x^3}
  < 1 -(9/8){x -(1/3)x^3} (← π<(4/3)^4)
  = 1/(4x) - {2 -8x +9x^2 -3x^4}/(8x)
  = 1/(4x) - {(1-x^2)(1-2x)^2 + (1-x)^4}/(8x)
  < 1/(4x),
98132人目の素数さん:2012/05/20(日) 22:38:37.27
>>21 >>81

 x・erfc(x) ≦ 0.240375586831011
 等号は x = 0.531596885149393
99132人目の素数さん:2012/05/21(月) 21:41:29.21
pを正の定数とする。
周の長さがpである三角形ABCの三辺の長さをa,b,cとする。
このとき、L=1/(a+b-c)+1/(b+c-a)+1/(c+a-b)のとり得る値の最小値をpを用いて表せ。
100132人目の素数さん:2012/05/21(月) 22:41:39.23
>>63 (念のため記録)

〔問題〕
非負実数 a,b,c と 自然数n≧2 について、
 (a^n)b + (b^n)c + (c^n)a ≦ K_n (a+b+c)^(n+1),
ここに K_n = (n^n)/(n+1)^(n+1),

(略解)
 G(a,b,c) = K_n (a+b+c)^n - (a^n)b -(b^n)c -(c^n)a,
とおいて偏微分すると、
 G_a = (n+1)K_n (a+b+c)^n - na^(n-1)・b - c^n,
 G_b = (n+1)K_n (a+b+c)^n - a^n - nb^(n-1)・c,
 G_c = (n+1)K_n (a+b+c)^n - b^n - nc^(n-1)・a,
辺々たす。
 G_a + G_b + G_c ≧ 3(n+1)K_n (a+b+c)^n - (a+b+c)^n
   = {3(n+1)K_n - 1}(a+b+c)^n
   = {3[n/(n+1)]^n - 1}(a+b+c)^n
   > (3/e - 1)(a+b+c)^n  {← (1 + 1/n)^n < e}
   > 0,
∴ {a,b,c} の差を固定して一斉に増加するとき、Gも増加する。
たとえば a,b≧c≧0 のとき、
 G(a,b,c) ≧ G(a-c,b-c,0)
      = G(a',b',0)
      = K_n (a'+b')^(n+1) - (a')^n・b'
      = (1/n){(a'+a'+・・・・・+a'+nb')/(n+1)}^(n+1) - (a')^n・b'
             n個
      ≧ 0,        (← 相加・相乗平均)
 等号成立は a'=nb' すなわち、{a,b,c}={n,1,0} とその rotation
101132人目の素数さん:2012/05/24(木) 06:44:54.95
A.563+B.4461
http://www.komal.hu/verseny/feladat.cgi?a=honap&h=201205&t=mat&l=en

A.561、B.4449、C.1124
http://www.komal.hu/verseny/feladat.cgi?a=honap&h=201204&t=mat&l=en

B.4433、B.4436
http://www.komal.hu/verseny/feladat.cgi?a=honap&h=201203&t=mat&l=en

A.556、B.4427、B.4431、C.1114、K.329
http://www.komal.hu/verseny/feladat.cgi?a=honap&h=201202&t=mat&l=en

A.552、B.4417
http://www.komal.hu/verseny/feladat.cgi?a=honap&h=201201&t=mat&l=en

   ∧_∧ やあ
   (´・ω・`)       /          ようこそ、不等式スレへ。
  /∇y:::::::\   [ ̄ ̄]         この不等式はサービスだから、まず飲んで落ち着いて欲しい。
  |:::⊃:|:::::::::::::|   |──|
 ̄ ̄ ̄ ̄ ̄ ̄ ̄ ̄ ̄ ̄ ̄ ̄ ̄ ̄ ̄ ̄ ̄| うん、「また」なんだ。済まない。
 ̄ ̄ ̄ ̄ ̄ ̄ ̄ ̄ ̄ ̄ ̄ ̄ ̄ ̄ ̄| ̄ ̄ 仏の顔もって言うしね、謝って許してもらおうとも思っていない。
 ̄ ̄ ̄ ̄ ̄ ̄ ̄ ̄ ̄ ̄ ̄ ̄ ̄ ̄/|
    ∇ ∇ ∇ ∇      /./|   でも、このスレタイを見たとき、君は、きっと言葉では言い表せない
    ┴ ┴ ┴ ┴     / / .|   「ときめき」みたいなものを感じてくれたと思う。
 ̄ ̄ ̄ ̄ ̄ ̄ ̄ ̄ ̄ ̄ ̄ ̄|/   |   殺伐とした世の中で、そういう気持ちを忘れないで欲しい
 ̄ ̄ ̄ ̄ ̄ ̄ ̄ ̄ ̄ ̄ ̄ ̄     |   そう思って、このスレを立てたんだ。
   (⊆⊇) (⊆⊇) (⊆⊇)      |
     ||    ||    .||       |   じゃあ、不等式を証明しようか。
   ./|\  /|\  /|\
102132人目の素数さん:2012/05/24(木) 23:36:23.72
          __ノ)-'´ ̄ ̄`ー- 、_
        , '´  _. -‐'''"二ニニ=-`ヽ、
      /   /:::::; -‐''"        `ーノ
     /   /:::::/           \
     /    /::::::/          | | |  |
     |   |:::::/ /     |  | | | |  |
      |   |::/ / / |  | ||  | | ,ハ .| ,ハ|
      |   |/ / / /| ,ハノ| /|ノレ,ニ|ル' 
     |   |  | / / レ',二、レ′ ,ィイ|゙/   私は只の数ヲタなんかとは付き合わないわ。
.     |   \ ∠イ  ,イイ|    ,`-' |      頭が良くて数学が出来てかっこいい人。それが必要条件よ。
     |     l^,人|  ` `-'     ゝ  |        さらに Ann.of Math に論文書けば十分条件にもなるわよ。
      |      ` -'\       ー'  人          一番嫌いなのは論文数を増やすためにくだらない論文を書いて
    |        /(l     __/  ヽ、           良い論文の出版を遅らせるお馬鹿な人。
     |       (:::::`‐-、__  |::::`、     ヒニニヽ、         あなたの論文が Ann of Math に accept される確率は?
    |      / `‐-、::::::::::`‐-、::::\   /,ニニ、\            それとも最近は Inv. Math. の方が上かしら?
   |      |::::::::::::::::::|` -、:::::::,ヘ ̄|'、  ヒニ二、 \
.   |      /::::::::::::::::::|::::::::\/:::O`、::\   | '、   \
   |      /:::::::::::::::::::/:::::::::::::::::::::::::::::'、::::\ノ  ヽ、  |
  |      |:::::/:::::::::/:::::::::::::::::::::::::::::::::::'、',::::'、  /:\__/‐、
  |      |/:::::::::::/::::::::::::::::::::::::::::::::::O::| '、::| く::::::::::::: ̄|
   |     /_..-'´ ̄`ー-、:::::::::::::::::::::::::::::::::::|/:/`‐'::\;;;;;;;_|
   |    |/::::::::::::::::::::::\:::::::::::::::::::::::::::::|::/::::|::::/:::::::::::/
    |   /:::::::::::::::::::::::::::::::::|:::::::::::::::::::::O::|::|::::::|:::::::::::::::/
103132人目の素数さん:2012/05/25(金) 02:08:33.48
>>101

A.556. (s)
 Prove that for arbitrary real numbers a_1, a_2,・・・・・, a_n there exist a real t such that
  Σ[i=1,n] |sin(t-a_i)| ≦ 1/tan(π/2n).

A.561. (s)
 Show that
  (a^3)b/(3a+b)^p + (b^3)c/(3b+c)^p + (c^3)a/(3c+a)^p
     ≧ (a^2)bc/(2a+b+c)^p + (b^2)ca/(2b+c+a)^p + (c^2)ab/(2c+a+b)^p
 holds for all positive numbers a,b,c,p.

A.563. + B.4461.
 Let 1≦p<2 be a real number. Prove that
 (x+y)^p + (X+Y)^p + (x+Y)^p + (X+y)^p ≦ x^p + y^p + Y^p + X^p + (x+y+Y+X)^p,
 for all non-negative real numbers x,y,Y & X.

B.4417. (s)
 Solve the equation sin(x) + (1/2)cos(x) = {sin(x+45゚)}^2

B.4427. (s)
 Show that if α,β and γ are the angles of a triangle, then
  (sinα + sinβ + sinγ)^2 > 9(sinα)(sinβ)(sinγ).

B.4433.
 Solve the equation (1+x)^8 + (1+x^2)^4 = 82x^4.

(s) は解あり。
104132人目の素数さん:2012/05/25(金) 02:09:32.69
>>101

C.1114.
 Solve the equation
 log_2{log_3(x)} = log_3{log_2(x)}.

C.1124.
 Solve the following simultaneous equations:
 x^(x+y) = y^3,
 y^(x+y) = x^12,

K.329.
 Given that x is a positive real number such that x^2 + 1/(x^2) = 7,
 determine the value of x^5 + 1/(x^5) without finding the value of x.

(s) は解あり。
105132人目の素数さん:2012/05/25(金) 02:47:55.76
>>103

A.561.
 f(x) = (x^2)/(2x+k)^p (k>0) は下に凸だから、
 (a^2)/(2a+k)^p ≧ (1/2)(a+c)^2/(a+c+k)^p - (c^2)/(2c+k)^p
         ≧ 2ac/(a+c+k)^p - (c^2)/(2c+k)^p, (k=a+b)
∴ (a^3)b/(3a+b)^p ≧ 2(a^2)bc/(2a+b+c)^p - ab(c^2)/(a+b+2c)^p,
 循環的にたす。

B.4417.
 (右辺) = {sin(x+45゚)}^2 = (1/2)[1−cos(2x+90゚)] = (1/2)[1+sin(2x)] = (1/2) + sin(x)cos(x),
∴ {sin(x)-1/2}{cos(x)-1} = 0,

B.4433.
 (左辺) = (1+2x+x^2)^4 + (1+x^2)^4
    = (x^4){(1/x +2 +x)^4 + (1/x + x)^4}
    = (x^4){(t+1)^4 + (t-1)^4}
    = 2(x^4)(t^4 +6t^2 +1),
∴ t^4 +6t^2 +1 = 41,
  (t^2 +10)(t+2)(t-2) = 0,
・t=-2 のとき、
 t = 1/x +1 +x = -2,
 x = -φ^2, -1/φ^2,
 φ = (1+√5)/2 = 1.618034:黄金比
・t=2 のとき、解なし。
106132人目の素数さん:2012/05/25(金) 03:52:44.94
>>104

C.1114.
 ln{ln(x)/ln(3)} / ln(2) = ln{ln(x)/ln(2)} / ln(3),
 ln(3/2)ln{ln(x)} = ln(3)・ln[ln(3)] - ln(2)・ln[ln(2)],
 ln{ln(x)} = {ln(3)・ln[ln(3)] - ln(2)・ln[ln(2)]}/ln(3/2) = 0.881381626
 ln(x) = 2.414232972
 x = 11.18119078

C.1124.
 (x^2)^(x+y) = y^6,
 y^(x+y) = (x^2)^6,
より
 {y/(x^2)}^(x+y) = {(x^2)/y}^6
∴ y = ±x^2,
また、
 {(x^2)y}^(x+y) = {(x^2)y}^6,
 y = ±1/x^2 または y = 6-x,
 (x,y) = (±1,1)  (x,y) = (2,4) (-3,9)

K.329.
 (x + 1/x)^2 = 7+2 = 9,
 T = x + 1/x = ±3,
 x^5 + 1/x^5 = (x + 1/x){(x^4 +1/x^4) -(x^2 +1/x^2) +1}
       = T{(T^4 -4T^2 +2) -(T^2 -2) +1}
       = T(T^4 -5T^2 +5)
       = ±123.
 (x は ±φ^4, ±1/φ^4 らしい....)
107132人目の素数さん:2012/05/25(金) 17:49:16.25
>>106

C.1114.
対数の底をすべて3/2に変換して解くと、x = (3/2)^( 3^(log(log3)) / 2^(log(log2)) } でOK?(logの底は3/2)
108132人目の素数さん:2012/05/26(土) 04:49:44.30
>>103
B.4427.
 0≦a,b,c≦1 ゆえ、相加・相乗平均から
 (a+b+c)^2 ≧ 9(abc)^(2/3) > 9abc,

>>104
K.329.
 x + 1/x = c のとき
 x^n + 1/x^n = 2T_n(c/2),
 T_n は第1種のチェビシェフ多項式

>>105
A.561. (訂正)
 相加・相乗平均から
 (a^2)/(2a+k)^p ≧ 2ac/{(2a+k)(2c+k)}^(p/2) - (c^2)/(2c+k)^p
         ≧ 2ac/(a+c+k)^p - (c^2)/(2c+k)^p, (k=a+b)

>>107
 たぶんOK
109132人目の素数さん:2012/05/27(日) 08:16:40.66
          __ノ)-'´ ̄ ̄`ー- 、_
        , '´  _. -‐'''"二ニニ=-`ヽ、
      /   /:::::; -‐''"        `ーノ
     /   /:::::/           \
     /    /::::::/          | | |  |
     |   |:::::/ /     |  | | | |  |
      |   |::/ / / |  | ||  | | ,ハ .| ,ハ|
      |   |/ / / /| ,ハノ| /|ノレ,ニ|ル' 
     |   |  | / / レ',二、レ′ ,ィイ|゙/   私は只の数ヲタなんかとは付き合わないわ。
.     |   \ ∠イ  ,イイ|    ,`-' |      頭が良くて数学が出来てかっこいい人。それが必要条件よ。
     |     l^,人|  ` `-'     ゝ  |        さらに Ann.of Math に論文書けば十分条件にもなるわよ。
      |      ` -'\       ー'  人          一番嫌いなのは論文数を増やすためにくだらない論文を書いて
    |        /(l     __/  ヽ、           良い論文の出版を遅らせるお馬鹿な人。
     |       (:::::`‐-、__  |::::`、     ヒニニヽ、         あなたの論文が Ann of Math に accept される確率は?
    |      / `‐-、::::::::::`‐-、::::\   /,ニニ、\            それとも最近は Inv. Math. の方が上かしら?
   |      |::::::::::::::::::|` -、:::::::,ヘ ̄|'、  ヒニ二、 \
.   |      /::::::::::::::::::|::::::::\/:::O`、::\   | '、   \
   |      /:::::::::::::::::::/:::::::::::::::::::::::::::::'、::::\ノ  ヽ、  |
  |      |:::::/:::::::::/:::::::::::::::::::::::::::::::::::'、',::::'、  /:\__/‐、
  |      |/:::::::::::/::::::::::::::::::::::::::::::::::O::| '、::| く::::::::::::: ̄|
   |     /_..-'´ ̄`ー-、:::::::::::::::::::::::::::::::::::|/:/`‐'::\;;;;;;;_|
   |    |/::::::::::::::::::::::\:::::::::::::::::::::::::::::|::/::::|::::/:::::::::::/
    |   /:::::::::::::::::::::::::::::::::|:::::::::::::::::::::O::|::|::::::|:::::::::::::::/
110132人目の素数さん:2012/05/28(月) 09:24:08.03
>>97
うおおおおおおおお! ありがとうございます!
111132人目の素数さん:2012/05/28(月) 13:36:58.54
今まで Schurの不等式を、シューアの不等式と読んでいた… ('A`)ヴォエァ!
http://ja.wikipedia.org/wiki/%E3%82%A4%E3%82%B5%E3%82%A4%E3%83%BB%E3%82%B7%E3%83%A5%E3%83%BC%E3%83%AB
112132人目の素数さん:2012/05/29(火) 12:30:24.10
          __ノ)-'´ ̄ ̄`ー- 、_
        , '´  _. -‐'''"二ニニ=-`ヽ、
      /   /:::::; -‐''"        `ーノ
     /   /:::::/           \
     /    /::::::/          | | |  |
     |   |:::::/ /     |  | | | |  |
      |   |::/ / / |  | ||  | | ,ハ .| ,ハ|
      |   |/ / / /| ,ハノ| /|ノレ,ニ|ル' 
     |   |  | / / レ',二、レ′ ,ィイ|゙/   私は只の数ヲタなんかとは付き合わないわ。
.     |   \ ∠イ  ,イイ|    ,`-' |      頭が良くて数学が出来てかっこいい人。それが必要条件よ。
     |     l^,人|  ` `-'     ゝ  |        さらに Ann.of Math に論文書けば十分条件にもなるわよ。
      |      ` -'\       ー'  人          一番嫌いなのは論文数を増やすためにくだらない論文を書いて
    |        /(l     __/  ヽ、           良い論文の出版を遅らせるお馬鹿な人。
     |       (:::::`‐-、__  |::::`、     ヒニニヽ、         あなたの論文が Ann of Math に accept される確率は?
    |      / `‐-、::::::::::`‐-、::::\   /,ニニ、\            それとも最近は Inv. Math. の方が上かしら?
   |      |::::::::::::::::::|` -、:::::::,ヘ ̄|'、  ヒニ二、 \
.   |      /::::::::::::::::::|::::::::\/:::O`、::\   | '、   \
   |      /:::::::::::::::::::/:::::::::::::::::::::::::::::'、::::\ノ  ヽ、  |
  |      |:::::/:::::::::/:::::::::::::::::::::::::::::::::::'、',::::'、  /:\__/‐、
  |      |/:::::::::::/::::::::::::::::::::::::::::::::::O::| '、::| く::::::::::::: ̄|
   |     /_..-'´ ̄`ー-、:::::::::::::::::::::::::::::::::::|/:/`‐'::\;;;;;;;_|
   |    |/::::::::::::::::::::::\:::::::::::::::::::::::::::::|::/::::|::::/:::::::::::/
    |   /:::::::::::::::::::::::::::::::::|:::::::::::::::::::::O::|::|::::::|:::::::::::::::/
113132人目の素数さん:2012/05/29(火) 15:54:28.70
>>106
C.1124.
 {y/(x^2)}^(x+y-6) = 1
 {(x^2)y}^(x+y+6) = 1

(1) x+y=6 のとき、(x,y) = (2,4) (-3,9)
(2) x+y=-6 のとき、
(3) x+y≠±6 のとき、(x,y) = (±1,1)

(2)のとき、x^3+x^2+1=0 の実数解1つも解じゃないの?
114132人目の素数さん:2012/05/29(火) 16:01:06.53
>>113
(×) x^3+x^2+1=0
(○) x^3+6x^2+1=0

書き間違えました ('A`)ヴォエァ!
115132人目の素数さん:2012/05/30(水) 01:08:20.39
>>113-114
 そうだった......orz

 {y/(x^2)}^(x+y+6) = 1
 {(x^2)y}^(x+y-6) = 1

 x^3 +6x^2 +1 = (x+2)^3 -12(x+2) +17,
より実数解は
 X = -2 -{(17-√33)/2}^(1/3) -{(17+√33)/2}^(1/3)
  = -6.02752466284293
 Y = -6−X = 1/X^2,

>>111
 またまたシュールな読みを....
116132人目の素数さん:2012/05/30(水) 03:06:55.20
C.1121.
 a,b,c are non-zero integers satisfying a+b=c, a≠b.
Prove that if n is a natural number, then the value of the sum
  (a^n)/{b(b-a)} - (b^n)/{a(b-a)} + (c^n)/(ab)
is an integer.


>>111
シュアーな読み方は...
 数セミ増刊「100人の数学者」p.183 日本評論社 (1989)
117132人目の素数さん:2012/06/01(金) 11:00:33.63
ヘルマン・ヴァイル
118132人目の素数さん:2012/06/04(月) 23:42:18.98
          __ノ)-'´ ̄ ̄`ー- 、_
        , '´  _. -‐'''"二ニニ=-`ヽ、
      /   /:::::; -‐''"        `ーノ
     /   /:::::/           \
     /    /::::::/          | | |  |
     |   |:::::/ /     |  | | | |  |
      |   |::/ / / |  | ||  | | ,ハ .| ,ハ|
      |   |/ / / /| ,ハノ| /|ノレ,ニ|ル' 
     |   |  | / / レ',二、レ′ ,ィイ|゙/   私は只の数ヲタなんかとは付き合わないわ。
.     |   \ ∠イ  ,イイ|    ,`-' |      頭が良くて数学が出来てかっこいい人。それが必要条件よ。
     |     l^,人|  ` `-'     ゝ  |        さらに Ann.of Math に論文書けば十分条件にもなるわよ。
      |      ` -'\       ー'  人          一番嫌いなのは論文数を増やすためにくだらない論文を書いて
    |        /(l     __/  ヽ、           良い論文の出版を遅らせるお馬鹿な人。
     |       (:::::`‐-、__  |::::`、     ヒニニヽ、         あなたの論文が Ann of Math に accept される確率は?
    |      / `‐-、::::::::::`‐-、::::\   /,ニニ、\            それとも最近は Inv. Math. の方が上かしら?
   |      |::::::::::::::::::|` -、:::::::,ヘ ̄|'、  ヒニ二、 \
.   |      /::::::::::::::::::|::::::::\/:::O`、::\   | '、   \
   |      /:::::::::::::::::::/:::::::::::::::::::::::::::::'、::::\ノ  ヽ、  |
  |      |:::::/:::::::::/:::::::::::::::::::::::::::::::::::'、',::::'、  /:\__/‐、
  |      |/:::::::::::/::::::::::::::::::::::::::::::::::O::| '、::| く::::::::::::: ̄|
   |     /_..-'´ ̄`ー-、:::::::::::::::::::::::::::::::::::|/:/`‐'::\;;;;;;;_|
   |    |/::::::::::::::::::::::\:::::::::::::::::::::::::::::|::/::::|::::/:::::::::::/
119132人目の素数さん:2012/06/05(火) 21:58:08.94
B.4461.
 Let p≧2 be a real number. Prove that
 (x+y)^p + (X+Y)^p + (x+Y)^p + (X+y)^p ≦ x^p + y^p + X^p + Y^p + (x+y+X+Y)^p,
 for all non-negative real numbers x,y,X & Y.


 f(z) = z^(p-1) は下に凸 → Jensenで
120132人目の素数さん:2012/06/06(水) 15:45:29.03
>>119
まったく分かりませんぬ
121132人目の素数さん:2012/06/06(水) 22:28:20.20
(x+y)^p + (X+Y)^p ≦ (x+y+X+Y)^p
(x+Y)^p + (X+y)^p ≦ (x+y+X+Y)^p

これは分かるが、あとのゴミは何だ?
122132人目の素数さん:2012/06/08(金) 23:28:34.07
>>50,53

The inequality is my own. I proposed first the inequality in 2004.
123132人目の素数さん:2012/06/08(金) 23:35:22.28
kunnyキタ━━━(゚∀゚)━( ゚∀)━(  ゚)━(  )━(゚  )━(∀゚ )━(゚∀゚)━━━!?!?
124132人目の素数さん:2012/06/08(金) 23:45:23.46
          __ノ)-'´ ̄ ̄`ー- 、_
        , '´  _. -‐'''"二ニニ=-`ヽ、
      /   /:::::; -‐''"        `ーノ
     /   /:::::/           \
     /    /::::::/          | | |  |
     |   |:::::/ /     |  | | | |  |
      |   |::/ / / |  | ||  | | ,ハ .| ,ハ|
      |   |/ / / /| ,ハノ| /|ノレ,ニ|ル' 
     |   |  | / / レ',二、レ′ ,ィイ|゙/   私は只の数ヲタなんかとは付き合わないわ。
.     |   \ ∠イ  ,イイ|    ,`-' |      頭が良くて数学が出来てかっこいい人。それが必要条件よ。
     |     l^,人|  ` `-'     ゝ  |        さらに Ann.of Math に論文書けば十分条件にもなるわよ。
      |      ` -'\       ー'  人          一番嫌いなのは論文数を増やすためにくだらない論文を書いて
    |        /(l     __/  ヽ、           良い論文の出版を遅らせるお馬鹿な人。
     |       (:::::`‐-、__  |::::`、     ヒニニヽ、         あなたの論文が Ann of Math に accept される確率は?
    |      / `‐-、::::::::::`‐-、::::\   /,ニニ、\            それとも最近は Inv. Math. の方が上かしら?
   |      |::::::::::::::::::|` -、:::::::,ヘ ̄|'、  ヒニ二、 \
.   |      /::::::::::::::::::|::::::::\/:::O`、::\   | '、   \
   |      /:::::::::::::::::::/:::::::::::::::::::::::::::::'、::::\ノ  ヽ、  |
  |      |:::::/:::::::::/:::::::::::::::::::::::::::::::::::'、',::::'、  /:\__/‐、
  |      |/:::::::::::/::::::::::::::::::::::::::::::::::O::| '、::| く::::::::::::: ̄|
   |     /_..-'´ ̄`ー-、:::::::::::::::::::::::::::::::::::|/:/`‐'::\;;;;;;;_|
   |    |/::::::::::::::::::::::\:::::::::::::::::::::::::::::|::/::::|::::/:::::::::::/
    |   /:::::::::::::::::::::::::::::::::|:::::::::::::::::::::O::|::|::::::|:::::::::::::::/
125prime_132:2012/06/09(土) 00:52:17.15
>>121
 不正解です。

 z^p = z・f(z) です。
126132人目の素数さん:2012/06/09(土) 01:23:32.45
>>123
一体どうしたんだ?
127132人目の素数さん:2012/06/09(土) 02:02:16.26
>>119
f(x+y) ≦ {xf(x) + yf(y)}/(x+y) より、 (x+y)^p ≦ x^p + y^p
f(y+z) ≦ {yf(y) + zf(z)}/(y+z) より、 (y+z)^p ≦ y^p + z^p
f(z+w) ≦ {zf(z) + wf(w)}/(z+w) より、(z+w)^p ≦ z^p + w^p
f(w+x) ≦ {wf(w) + xf(x)}/(w+x) より、(w+x)^p ≦ w^p + x^p

∴ (x+y)^p + (y+z)^p + (z+w)^p + (w+x)^p ≦ 2(x^p + y^p + z^p + w^p)

    ウワァァ!!
    (>'A`)>
    ( ヘヘ
128prime_132:2012/06/09(土) 11:43:43.03
>>119

 (x+y)f(x+y) + (X+Y)f(X+Y) + (x+Y)f(x+Y) + (X+y)f(X+y) ≦ xf(x) + yf(y) + Xf(X) + Yf(Y) + (x+y+X+Y)f(x+y+X+Y)

そこで x,y,X,Y の係数を取り出すと・・・・
129132人目の素数さん:2012/06/09(土) 17:40:29.00
>>128
右辺の f は5つなのに、左辺は4つあるから使い方が分からん
130132人目の素数さん:2012/06/09(土) 18:01:41.11

 ..::::::,、_,、::: ::::: ::: : 
  /ヨミ゙ヽ)-、. :: ::::
─(ノ─ヽ.ソ┴─
131prime_132:2012/06/09(土) 18:31:41.32
>>119

xの係数:  f(x+y) + f(x+Y) ≦ f(x) + f(x+y+X+Y),

一方、Jensenから
 f(x+y) + f(x+Y) ≦ f(x) + f(x+y+Y), etc.
132132人目の素数さん:2012/06/10(日) 00:35:32.95
>>131
> 一方、Jensenから
>  f(x+y) + f(x+Y) ≦ f(x) + f(x+y+Y), etc.

どうやったらJensenの不等式から出てくるのか分かりませんぬ

    |         
    |  ('A`)    
   / ̄ノ( ヘヘ ̄ ̄       
133prime_132:2012/06/10(日) 12:06:19.20
>>132
 fは下に凸、y>0, Y>0 より

 f(x+y) ≦ {Y・f(x) + y・f(x+y+Y)}/(y+Y),
 f(x+Y) ≦ {y・f(x) + Y・f(x+y+Y)}/(y+Y),
辺々たす。
134132人目の素数さん:2012/06/12(火) 08:58:54.11
          __ノ)-'´ ̄ ̄`ー- 、_
        , '´  _. -‐'''"二ニニ=-`ヽ、
      /   /:::::; -‐''"        `ーノ
     /   /:::::/           \
     /    /::::::/          | | |  |
     |   |:::::/ /     |  | | | |  |
      |   |::/ / / |  | ||  | | ,ハ .| ,ハ|
      |   |/ / / /| ,ハノ| /|ノレ,ニ|ル' 
     |   |  | / / レ',二、レ′ ,ィイ|゙/   私は只の数ヲタなんかとは付き合わないわ。
.     |   \ ∠イ  ,イイ|    ,`-' |      頭が良くて数学が出来てかっこいい人。それが必要条件よ。
     |     l^,人|  ` `-'     ゝ  |        さらに Ann.of Math に論文書けば十分条件にもなるわよ。
      |      ` -'\       ー'  人          一番嫌いなのは論文数を増やすためにくだらない論文を書いて
    |        /(l     __/  ヽ、           良い論文の出版を遅らせるお馬鹿な人。
     |       (:::::`‐-、__  |::::`、     ヒニニヽ、         あなたの論文が Ann of Math に accept される確率は?
    |      / `‐-、::::::::::`‐-、::::\   /,ニニ、\            それとも最近は Inv. Math. の方が上かしら?
   |      |::::::::::::::::::|` -、:::::::,ヘ ̄|'、  ヒニ二、 \
.   |      /::::::::::::::::::|::::::::\/:::O`、::\   | '、   \
   |      /:::::::::::::::::::/:::::::::::::::::::::::::::::'、::::\ノ  ヽ、  |
  |      |:::::/:::::::::/:::::::::::::::::::::::::::::::::::'、',::::'、  /:\__/‐、
  |      |/:::::::::::/::::::::::::::::::::::::::::::::::O::| '、::| く::::::::::::: ̄|
   |     /_..-'´ ̄`ー-、:::::::::::::::::::::::::::::::::::|/:/`‐'::\;;;;;;;_|
   |    |/::::::::::::::::::::::\:::::::::::::::::::::::::::::|::/::::|::::/:::::::::::/
    |   /:::::::::::::::::::::::::::::::::|:::::::::::::::::::::O::|::|::::::|:::::::::::::::/
135132人目の素数さん:2012/06/13(水) 13:46:49.67
>>133
スゴ杉! 自力では絶対に気づかない罠
x、y、z、wで計算していたので、直すのが面倒なので、そのまま書くと

f(w+x) + f(x+y) ≦ f(x) + f(w+x+y) …@
f(x+y) + f(y+z) ≦ f(y) + f(x+y+z) …A
f(y+z) + f(z+w) ≦ f(z) + f(y+z+w) …B
f(z+w) + f(w+x) ≦ f(w) + f(z+w+x) …C

@×x + A×y + B×z + C×w より、

(x+y)f(x+y) + (y+z)f(y+z) + (z+w)f(z+w) + (w+x)f(w+x)
 ≦ xf(x) + yf(y) +zf(z) +wf(w) + xf(w+x+y) + yf(x+y+z) + zf(y+z+w) + wf(z+w+x)

示したい右辺は
 ≦ xf(x) + yf(y) +zf(z) +wf(w) + (x+y+z+w)f(x+y+z+w)

なので、次式が成り立てば解決なのか…
xf(w+x+y) + yf(x+y+z) + zf(y+z+w) + wf(z+w+x) ≦ (x+y+z+w)f(x+y+z+w)

ぬ〜ん…

    /⌒ヽ
  /⌒  ・ >
  E ̄U) ε | 
  E ̄∩) ・ >
゛゛゛゛゛゛゛゛゛゛゛゛゛゛゛゛゛゛゛゛゛゛
136132人目の素数さん:2012/06/13(水) 22:09:17.84
a^2+b^2+c^2+d^2=4 (a,b,c,d>0)→(a+b+c+d-2)(1/a+1/b+1/c+1/d +1/2)>=9
137132人目の素数さん:2012/06/13(水) 22:16:56.48
>>136
あやしい
138132人目の素数さん:2012/06/14(木) 00:03:53.21
          __ノ)-'´ ̄ ̄`ー- 、_
        , '´  _. -‐'''"二ニニ=-`ヽ、
      /   /:::::; -‐''"        `ーノ
     /   /:::::/           \
     /    /::::::/          | | |  |
     |   |:::::/ /     |  | | | |  |
      |   |::/ / / |  | ||  | | ,ハ .| ,ハ|
      |   |/ / / /| ,ハノ| /|ノレ,ニ|ル' 
     |   |  | / / レ',二、レ′ ,ィイ|゙/   私は只の数ヲタなんかとは付き合わないわ。
.     |   \ ∠イ  ,イイ|    ,`-' |      頭が良くて数学が出来てかっこいい人。それが必要条件よ。
     |     l^,人|  ` `-'     ゝ  |        さらに Ann.of Math に論文書けば十分条件にもなるわよ。
      |      ` -'\       ー'  人          一番嫌いなのは論文数を増やすためにくだらない論文を書いて
    |        /(l     __/  ヽ、           良い論文の出版を遅らせるお馬鹿な人。
     |       (:::::`‐-、__  |::::`、     ヒニニヽ、         あなたの論文が Ann of Math に accept される確率は?
    |      / `‐-、::::::::::`‐-、::::\   /,ニニ、\            それとも最近は Inv. Math. の方が上かしら?
   |      |::::::::::::::::::|` -、:::::::,ヘ ̄|'、  ヒニ二、 \
.   |      /::::::::::::::::::|::::::::\/:::O`、::\   | '、   \
   |      /:::::::::::::::::::/:::::::::::::::::::::::::::::'、::::\ノ  ヽ、  |
  |      |:::::/:::::::::/:::::::::::::::::::::::::::::::::::'、',::::'、  /:\__/‐、
  |      |/:::::::::::/::::::::::::::::::::::::::::::::::O::| '、::| く::::::::::::: ̄|
   |     /_..-'´ ̄`ー-、:::::::::::::::::::::::::::::::::::|/:/`‐'::\;;;;;;;_|
   |    |/::::::::::::::::::::::\:::::::::::::::::::::::::::::|::/::::|::::/:::::::::::/
    |   /:::::::::::::::::::::::::::::::::|:::::::::::::::::::::O::|::|::::::|:::::::::::::::/
139132人目の素数さん:2012/06/16(土) 05:30:59.85
>>133
降参でござる
140132人目の素数さん:2012/06/16(土) 05:35:04.95
           , /    ,
        ,   / /   ,   /  ,
          / '^メ-' ─/- 、   / ,
       ∠r  _,゛_ /  , ヽ/__/ モウ ダメダ…
        ''ヽ'_・.ノ` ' r/、 ヘ /‐’
       ./ " j 厂゙j | レ_`> j__ /
        '  .:‘::'ニ‘.:‐'´─゙.:´一’
      .:.::::::.:  .:.::.:   .:.:::::.:
      .:.:::::::::::.: .:.:::::::.: .:.:::::::::.:
141prime_132:2012/06/16(土) 12:10:49.03
>>135

fが下に凸だから、線分
 (x, f(x)) ー (x+y+Y, f(x+y+Y))
は線分
 (x+y, f(x+y)) ー (x+Y, f(x+Y))
の上側にある。
線分の中点のx座標はいずれも x+(y+Y)/2 なので、y座標を比べる。



 定義から、fは単調増加....
142prime_132:2012/06/16(土) 13:04:36.48
>>141 訂正

線分の中点のヨコ座標はいずれも x + (y+Y)/2 なので、タテ座標を比べる。
143132人目の素数さん:2012/06/16(土) 15:44:24.68
>>142
それを比べて出てくるのは、{f(x+y)+f(x+y)}/2 < {f(x)+f(x+y+Y)}/2 より
 f(x+y) + f(x+y) < f(x) + f(x+y+Y)

>>135より何も進展しない
144prime_132:2012/06/16(土) 21:27:12.94
>>143

 定義から、fは単調増加....

 f(x+y+Y) < f(x+y+X+Y),  
145132人目の素数さん:2012/06/16(土) 23:51:33.05
>>144
なるほど、そうですね
最終的に ≦ じゃなくて < になるけど、元の不等式に等号成立条件はないのですか?
146prime_132:2012/06/17(日) 02:23:35.20
>>145

 xX = yY = 0,


ところで本題、A.563. (1≦p<2) の方は.....
147132人目の素数さん:2012/06/17(日) 10:13:48.04
三角形ABCにおいて(sinA)^(sinB)+(sinB)^(sinC)+(sinC)^(sinA) > e^(-1/e)+1
148132人目の素数さん:2012/06/17(日) 23:10:46.41
>>146
お手上げ侍

>>147
       ヽ|/
     / ̄ ̄ ̄`ヽ、
    /   ≧     ヽ
   /  \,, ,,/    |
   | (●) (●)|||  |
   |  / ̄⌒ ̄ヽ U.|   ・・・・・・・・ゴクリ。
   |  | .l~ ̄~ヽ |   |
   |U ヽ  ̄~ ̄ ノ   |
   |    ̄ ̄ ̄    |
149132人目の素数さん:2012/06/20(水) 22:21:44.81
136 これ、どうやって解くんだ
難しくねえ?
150132人目の素数さん:2012/06/20(水) 22:23:17.51
          __ノ)-'´ ̄ ̄`ー- 、_
        , '´  _. -‐'''"二ニニ=-`ヽ、
      /   /:::::; -‐''"        `ーノ
     /   /:::::/           \
     /    /::::::/          | | |  |
     |   |:::::/ /     |  | | | |  |
      |   |::/ / / |  | ||  | | ,ハ .| ,ハ|
      |   |/ / / /| ,ハノ| /|ノレ,ニ|ル' 
     |   |  | / / レ',二、レ′ ,ィイ|゙/   私は只の数ヲタなんかとは付き合わないわ。
.     |   \ ∠イ  ,イイ|    ,`-' |      頭が良くて数学が出来てかっこいい人。それが必要条件よ。
     |     l^,人|  ` `-'     ゝ  |        さらに Ann.of Math に論文書けば十分条件にもなるわよ。
      |      ` -'\       ー'  人          一番嫌いなのは論文数を増やすためにくだらない論文を書いて
    |        /(l     __/  ヽ、           良い論文の出版を遅らせるお馬鹿な人。
     |       (:::::`‐-、__  |::::`、     ヒニニヽ、         あなたの論文が Ann of Math に accept される確率は?
    |      / `‐-、::::::::::`‐-、::::\   /,ニニ、\            それとも最近は Inv. Math. の方が上かしら?
   |      |::::::::::::::::::|` -、:::::::,ヘ ̄|'、  ヒニ二、 \
.   |      /::::::::::::::::::|::::::::\/:::O`、::\   | '、   \
   |      /:::::::::::::::::::/:::::::::::::::::::::::::::::'、::::\ノ  ヽ、  |
  |      |:::::/:::::::::/:::::::::::::::::::::::::::::::::::'、',::::'、  /:\__/‐、
  |      |/:::::::::::/::::::::::::::::::::::::::::::::::O::| '、::| く::::::::::::: ̄|
   |     /_..-'´ ̄`ー-、:::::::::::::::::::::::::::::::::::|/:/`‐'::\;;;;;;;_|
   |    |/::::::::::::::::::::::\:::::::::::::::::::::::::::::|::/::::|::::/:::::::::::/
    |   /:::::::::::::::::::::::::::::::::|:::::::::::::::::::::O::|::|::::::|:::::::::::::::/
151132人目の素数さん:2012/06/23(土) 16:41:14.49
正の数 x、y、… に対して、相加平均を A(x、y、…)、総乗平均を G(x、y、…)であらわすことにする

正の数 a、b、c、d に対して
A(a、b、c、d)
 ≧ G(A(a、b、c)、A(b、c、d)、A(c、d、a)、A(d、a、b))
 ≧ G(A(a、b)、A(a、c)、A(a、d)、A(b、c)、A(b、d)、A(c、d))
 ≧ G(a、b、c、d)
152132人目の素数さん:2012/06/23(土) 16:44:53.16
>>151

  ∧_∧
  ( ;´∀`) < おっきしました
  人 Y /
 ( ヽ し
 (_)_)
153132人目の素数さん:2012/06/23(土) 18:32:42.02
>>151
類題が過去スレにあったようなハロゲンガス
ちょっと探してみる

┌──────────────────────―─┐
│                                    |
│                                    |
│                ∧_∧                 ...|
|                ( ;´∀`)               |
|                人 Y /                  .|
|               ( ヽ し                  |
|               (_)_)                 |
|                                             .|
│             Now Bokkiing. ...               |
│                                    |
│                                    |
│       しばらくちんちん勃ててお待ちください。      ..|
│                                    |
└───────────────────────―┘
154132人目の素数さん:2012/06/23(土) 19:08:50.74
>>151

A(a,b,c,d)
 = A(A(a,b,c)、A(b,c,d)、A(c,d,a)、A(d,a,b))
 ≧ G(A(a,b,c)、A(b,c,d)、A(c,d,a)、A(d,a,b))
 = G(A(A(a,b),A(b,c),A(a,c)) 、A(A(b,c),A(b,d),A(b,d))、A(A(a,c),A(a,d),A(c,d))、A(A(a,b),A(a,d),A(b,d)))
 ≧ G(G(A(a,b),A(b,c),A(a,c)) 、G(A(b,c),A(b,d),A(b,d))、G(A(a,c),A(a,d),A(c,d))、G(A(a,b),A(a,d),A(b,d)))
 = G(A(a,b)、A(a,c)、A(a,d)、A(b,c)、A(b,d)、A(c,d))
 ≧ G(G(a,b)、G(a,c)、G(a,d)、G(b,c)、G(b,d)、G(c,d))
 = G(a,b,c,d)
155132人目の素数さん:2012/06/23(土) 20:07:51.38
やぁ、待たせたね
この不等式はサービスだから、まずは証明して落ち着いてほしい

\sqrt{A(a^2、b^2、c^2、d^2)}
 ≧ A(a、b、c、d)
 ≧ ★
 ≧ G(a、b、c、d)

ここで ★には、次の (1) または (2) が入るが、(1) と (2) の間に大小は定まらなかったと思う

(1) \sqrt{A(ab、bc、cd、da)}
(2) \sqrt{A(ab、ac、ad、bc、bd、cd)} ≧ \sqrt[3]{A(abc、bcd、cda、dab)}

>>151は、★に (3) が入ったものだが、(1)、(2)、(3) の式のどれかに大小はつくのだろうか?

(3) G(A(a、b、c)、A(b、c、d)、A(c、d、a)、A(d、a、b)) ≧ G(A(a、b)、A(a、c)、A(a、d)、A(b、c)、A(b、d)、A(c、d))



ヽ(`Д´)ノ   ボッキアゲスパイラル!
ヽ`Д´)
 (ヽ`Д)
 ( ヽ`)
(  ヽ
ヽ(   )ノ
ヽ   )
(ヽ  )
 (´ヽ )
(Д´ヽ
ヽ(`Д´)ノ
156132人目の素数さん:2012/06/23(土) 20:18:56.70
>>154
Now Bokkiing している間に片付けられてしまうとは…
157132人目の素数さん:2012/06/23(土) 23:29:51.75
>>155
お待ちしておりますた。。。??

 A(a,b,c,d) = A(A(a,c)、A(b,d))
       ≧ G(A(a,c)、A(b,d)) = (1)
       ≧ G(G(a,c)、G(b,d))
       = G(a,b,c,d)
158132人目の素数さん:2012/06/23(土) 23:51:12.78
>>151
今月の学コンの問題じゃん
159132人目の素数さん:2012/06/24(日) 00:07:29.59
入試問題?JKに解けるん?
160132人目の素数さん:2012/06/27(水) 21:25:03.16
正の数 a、b、c が a+b+c=1 をみたすとき、(a^2 + b^2 + c^2)^2 ・(1/a + 1/b + 1/c) ≧ 1


!     |     |         |  !  |    |       |     |    !    |
 |    |     !   |    |      |    !       _ヘ、____|_____!_
 |    |          |    |      |         / _/ _/ _/ |_/ _/ _/ _/ _/ _/
 |    !         |     !      |   |     / _/ _/ _/ _ ヘ、_/ _/ _/ _/ _/
 |       |      |        !  |    / _/ _/ _/ _/ _/ _/ _/ _/ _/ _/
 ! |    |      !      |    |    | ~~~|~~~~~~~~~~~~~~~~~~~~~~~
   |     |   |        |    |     |    |    |  _________
   |     !   |        |    |     !    |    |   ||     | |//  ||
   |          |    |  |    !   |      |    |   ||     | |/    ||
|  !         |    |  !         |      |    |   ||,_,ハ,  .| |      ||
|      |     !    |    |       |      !    |   ||ω・ '. .| |      ||
!      |           !    |       !       |   ̄ ̄ ̄ ̄ ̄ ̄ ̄ ̄
彡      |      |         |       |      |
;;;;;彡   !      |         |       |      |___________
;;; 彡,@ソ         | ...........     |       |     """""""""""""""""""""""
彡;;; ~;;彡   .......   !       ヘ、 ............   | .......    ..........    ..........
;;;彡;;;彡彡    .......       .........       !   .....       ..........     .....
''''''''''''''''''' ...............    ............        ..............    .....       ........
161132人目の素数さん:2012/06/27(水) 22:38:04.33
136の不等式、難しいね。上手い方法、誰か、ありますか?

これに比べて、160は簡単です。
162132人目の素数さん:2012/06/27(水) 23:55:46.68
161=136乙
163132人目の素数さん:2012/06/28(木) 01:05:27.44
          __ノ)-'´ ̄ ̄`ー- 、_
        , '´  _. -‐'''"二ニニ=-`ヽ、
      /   /:::::; -‐''"        `ーノ
     /   /:::::/           \
     /    /::::::/          | | |  |
     |   |:::::/ /     |  | | | |  |
      |   |::/ / / |  | ||  | | ,ハ .| ,ハ|
      |   |/ / / /| ,ハノ| /|ノレ,ニ|ル' 
     |   |  | / / レ',二、レ′ ,ィイ|゙/   私は只の数ヲタなんかとは付き合わないわ。
.     |   \ ∠イ  ,イイ|    ,`-' |      頭が良くて数学が出来てかっこいい人。それが必要条件よ。
     |     l^,人|  ` `-'     ゝ  |        さらに Ann.of Math に論文書けば十分条件にもなるわよ。
      |      ` -'\       ー'  人          一番嫌いなのは論文数を増やすためにくだらない論文を書いて
    |        /(l     __/  ヽ、           良い論文の出版を遅らせるお馬鹿な人。
     |       (:::::`‐-、__  |::::`、     ヒニニヽ、         あなたの論文が Ann of Math に accept される確率は?
    |      / `‐-、::::::::::`‐-、::::\   /,ニニ、\            それとも最近は Inv. Math. の方が上かしら?
   |      |::::::::::::::::::|` -、:::::::,ヘ ̄|'、  ヒニ二、 \
.   |      /::::::::::::::::::|::::::::\/:::O`、::\   | '、   \
   |      /:::::::::::::::::::/:::::::::::::::::::::::::::::'、::::\ノ  ヽ、  |
  |      |:::::/:::::::::/:::::::::::::::::::::::::::::::::::'、',::::'、  /:\__/‐、
  |      |/:::::::::::/::::::::::::::::::::::::::::::::::O::| '、::| く::::::::::::: ̄|
   |     /_..-'´ ̄`ー-、:::::::::::::::::::::::::::::::::::|/:/`‐'::\;;;;;;;_|
   |    |/::::::::::::::::::::::\:::::::::::::::::::::::::::::|::/::::|::::/:::::::::::/
    |   /:::::::::::::::::::::::::::::::::|:::::::::::::::::::::O::|::|::::::|:::::::::::::::/
164132人目の素数さん:2012/06/28(木) 01:17:54.87
AAのおっ、っていうのかわいいよね。ついつい、保存してしまう。
165132人目の素数さん:2012/06/28(木) 01:23:08.33
>>164
おっ?
どんなやつですか?
166132人目の素数さん:2012/06/28(木) 01:27:11.79
(´・ω・`)こんな感じ
167132人目の素数さん:2012/06/28(木) 01:28:13.65
168132人目の素数さん:2012/06/28(木) 01:38:21.19
名前があったんですな
169132人目の素数さん:2012/06/28(木) 03:24:12.52
名前はないというか聞いたことないけど
名前ないのかな(´・ω・`)しょぼーんかな。
170132人目の素数さん:2012/06/28(木) 03:26:05.02
>>160
 f(x) = x^r とおくと 
 f "(x) = r(r-1)x^(r-2),

r<0 または r>1 のとき y=f(x) は下に凸
 a^r + b^r + c^r ≧ 3{(a+b+c)/3}^r,

0<r<1 のとき y=f(x) は上に凸
 a^r + b^r + c^r ≦ 3{(a+b+c)/3}^r,

等号成立は a=b=c

>>161 成程。。。
171132人目の素数さん:2012/06/28(木) 16:28:26.46
Jensenの不等式って、万能包丁かYO! (・ε・)YO!YO!
172132人目の素数さん:2012/06/29(金) 21:06:35.95
          __ノ)-'´ ̄ ̄`ー- 、_
        , '´  _. -‐'''"二ニニ=-`ヽ、
      /   /:::::; -‐''"        `ーノ
     /   /:::::/           \
     /    /::::::/          | | |  |
     |   |:::::/ /     |  | | | |  |
      |   |::/ / / |  | ||  | | ,ハ .| ,ハ|
      |   |/ / / /| ,ハノ| /|ノレ,ニ|ル' 
     |   |  | / / レ',二、レ′ ,ィイ|゙/   私は只の数ヲタなんかとは付き合わないわ。
.     |   \ ∠イ  ,イイ|    ,`-' |      頭が良くて数学が出来てかっこいい人。それが必要条件よ。
     |     l^,人|  ` `-'     ゝ  |        さらに Ann.of Math に論文書けば十分条件にもなるわよ。
      |      ` -'\       ー'  人          一番嫌いなのは論文数を増やすためにくだらない論文を書いて
    |        /(l     __/  ヽ、           良い論文の出版を遅らせるお馬鹿な人。
     |       (:::::`‐-、__  |::::`、     ヒニニヽ、         あなたの論文が Ann of Math に accept される確率は?
    |      / `‐-、::::::::::`‐-、::::\   /,ニニ、\            それとも最近は Inv. Math. の方が上かしら?
   |      |::::::::::::::::::|` -、:::::::,ヘ ̄|'、  ヒニ二、 \
.   |      /::::::::::::::::::|::::::::\/:::O`、::\   | '、   \
   |      /:::::::::::::::::::/:::::::::::::::::::::::::::::'、::::\ノ  ヽ、  |
  |      |:::::/:::::::::/:::::::::::::::::::::::::::::::::::'、',::::'、  /:\__/‐、
  |      |/:::::::::::/::::::::::::::::::::::::::::::::::O::| '、::| く::::::::::::: ̄|
   |     /_..-'´ ̄`ー-、:::::::::::::::::::::::::::::::::::|/:/`‐'::\;;;;;;;_|
   |    |/::::::::::::::::::::::\:::::::::::::::::::::::::::::|::/::::|::::/:::::::::::/
    |   /:::::::::::::::::::::::::::::::::|:::::::::::::::::::::O::|::|::::::|:::::::::::::::/
173132人目の素数さん:2012/06/29(金) 21:43:56.65
数学を知ってれば簡単に作れます。

「ウ☆ディ☆タ」とは? 
・完全無料のゲーム作成ツールです。
・ツクールでは物足りないけどプログラミングは苦手という方にお勧め。
・作成したゲームは自由に配布したり、コンテストに投稿することも可能。
・他人が作成した「コモンイベント」を利用すれば、自分では開発が難しい
 システムも容易に実現できます。
174132人目の素数さん:2012/06/29(金) 22:20:57.11
難易度 : ミジンコ
問  題 : 0 < a、b、c < 1 のとき、a + b + c < abc + 2 を証明せよ
175132人目の素数さん:2012/06/29(金) 22:30:06.05
          __ノ)-'´ ̄ ̄`ー- 、_
        , '´  _. -‐'''"二ニニ=-`ヽ、
      /   /:::::; -‐''"        `ーノ
     /   /:::::/           \
     /    /::::::/          | | |  |
     |   |:::::/ /     |  | | | |  |
      |   |::/ / / |  | ||  | | ,ハ .| ,ハ|
      |   |/ / / /| ,ハノ| /|ノレ,ニ|ル' 
     |   |  | / / レ',二、レ′ ,ィイ|゙/   私は只の数ヲタなんかとは付き合わないわ。
.     |   \ ∠イ  ,イイ|    ,`-' |      頭が良くて数学が出来てかっこいい人。それが必要条件よ。
     |     l^,人|  ` `-'     ゝ  |        さらに Ann.of Math に論文書けば十分条件にもなるわよ。
      |      ` -'\       ー'  人          一番嫌いなのは論文数を増やすためにくだらない論文を書いて
    |        /(l     __/  ヽ、           良い論文の出版を遅らせるお馬鹿な人。
     |       (:::::`‐-、__  |::::`、     ヒニニヽ、         あなたの論文が Ann of Math に accept される確率は?
    |      / `‐-、::::::::::`‐-、::::\   /,ニニ、\            それとも最近は Inv. Math. の方が上かしら?
   |      |::::::::::::::::::|` -、:::::::,ヘ ̄|'、  ヒニ二、 \
.   |      /::::::::::::::::::|::::::::\/:::O`、::\   | '、   \
   |      /:::::::::::::::::::/:::::::::::::::::::::::::::::'、::::\ノ  ヽ、  |
  |      |:::::/:::::::::/:::::::::::::::::::::::::::::::::::'、',::::'、  /:\__/‐、
  |      |/:::::::::::/::::::::::::::::::::::::::::::::::O::| '、::| く::::::::::::: ̄|
   |     /_..-'´ ̄`ー-、:::::::::::::::::::::::::::::::::::|/:/`‐'::\;;;;;;;_|
   |    |/::::::::::::::::::::::\:::::::::::::::::::::::::::::|::/::::|::::/:::::::::::/
    |   /:::::::::::::::::::::::::::::::::|:::::::::::::::::::::O::|::|::::::|:::::::::::::::/
176132人目の素数さん:2012/06/30(土) 01:28:56.96
>>174

 2-a-b-c+abc = 2(1-a-b-c+ab+bc+ca-abc) + a(1-b-c+bc) + b(1-c-a+ca) + c(1-a-b+ab)
       = 2(1-a)(1-b)(1-c) + a(1-b)(1-c) + (1-a)b(1-c) + (1-a)(1-b)c,
177132人目の素数さん:2012/07/01(日) 05:22:05.64
難易度 : ミジンコのツノ
問  題 : 正の数 a、b、c、d が a+b+c+d=1 をみたすとき、m の最大値は?
       1/(a+2b+3c) + 1/(b+2c+3d) + 1/(c+2d+3a) + 1/(d+2a+3b) ≧ m
178132人目の素数さん:2012/07/01(日) 05:25:54.71
a ← ミジンコ

ミジンコを魚(捕食者)といっしょに同じ水槽に入れると 頭がだんだんとんがってくるとか…
179132人目の素数さん:2012/07/01(日) 14:13:17.11
8/3
180132人目の素数さん:2012/07/01(日) 14:17:18.59
a, b, c, d>0, abcd=1

1/a+1/b+1/c+1/d+9/a+b+c+d>= 25/4
181132人目の素数さん:2012/07/01(日) 15:09:01.87
          __ノ)-'´ ̄ ̄`ー- 、_
        , '´  _. -‐'''"二ニニ=-`ヽ、
      /   /:::::; -‐''"        `ーノ
     /   /:::::/           \
     /    /::::::/          | | |  |
     |   |:::::/ /     |  | | | |  |
      |   |::/ / / |  | ||  | | ,ハ .| ,ハ|
      |   |/ / / /| ,ハノ| /|ノレ,ニ|ル' 
     |   |  | / / レ',二、レ′ ,ィイ|゙/   私は只の数ヲタなんかとは付き合わないわ。
.     |   \ ∠イ  ,イイ|    ,`-' |      頭が良くて数学が出来てかっこいい人。それが必要条件よ。
     |     l^,人|  ` `-'     ゝ  |        さらに Ann.of Math に論文書けば十分条件にもなるわよ。
      |      ` -'\       ー'  人          一番嫌いなのは論文数を増やすためにくだらない論文を書いて
    |        /(l     __/  ヽ、           良い論文の出版を遅らせるお馬鹿な人。
     |       (:::::`‐-、__  |::::`、     ヒニニヽ、         あなたの論文が Ann of Math に accept される確率は?
    |      / `‐-、::::::::::`‐-、::::\   /,ニニ、\            それとも最近は Inv. Math. の方が上かしら?
   |      |::::::::::::::::::|` -、:::::::,ヘ ̄|'、  ヒニ二、 \
.   |      /::::::::::::::::::|::::::::\/:::O`、::\   | '、   \
   |      /:::::::::::::::::::/:::::::::::::::::::::::::::::'、::::\ノ  ヽ、  |
  |      |:::::/:::::::::/:::::::::::::::::::::::::::::::::::'、',::::'、  /:\__/‐、
  |      |/:::::::::::/::::::::::::::::::::::::::::::::::O::| '、::| く::::::::::::: ̄|
   |     /_..-'´ ̄`ー-、:::::::::::::::::::::::::::::::::::|/:/`‐'::\;;;;;;;_|
   |    |/::::::::::::::::::::::\:::::::::::::::::::::::::::::|::/::::|::::/:::::::::::/
    |   /:::::::::::::::::::::::::::::::::|:::::::::::::::::::::O::|::|::::::|:::::::::::::::/
182132人目の素数さん:2012/07/01(日) 18:18:42.54
>>177

相加・調和平均 or コーシーで
 (左辺) ≧ (4^2)/{(a+2b+3c)+(b+2c+3d)+(c+2d+3a)+(d+2a+3b)}
     = (4^2)/{6(a+b+c+d)},

 b d q ← ミジンコの寒風干し
183132人目の素数さん:2012/07/01(日) 18:21:57.07
>>180
 中国女子数学オリンピック(CGMO)の問題 第10回(2011)

 f(a, b, c, d) = 1/a + 1/b + 1/c + 1/d + 9/(a+b+c+d),
とおく。
 ab<2 ⇒ f(a, b, c, d) ≧ f(√{ab}, √{ab}, c, d)
これから結局、
 F(A) = f(A^3, 1/A, 1/A, 1/A) ≧ 25/4,   (A≧1)
に帰着する。
 A^4 + 3 = (1/3)A^4 + (1/3)A^4 + (1/3)A^4 + 3
 ≧ (4/√3)A^3    (← 相加・相乗平均)
 > (9/4)A^3,

F(A) - 25/4 = {(1/A^3) +3A -4} + (9/4){4A/(A^4 +3) -1}
  = (A-1)^2・(3A^2 +2A+1)/A^3 - (9/4)(A-1)^2・(A^2 +2A+3)/(A^4 +3)
  > (A-1)^2・{(3A^2 +2A+1) - (A^2 +2A+3)}/A^3
  = (A-1)^2・2(A^2 -1)/A^3
  ≧ 0,           (A≧1)

(参考)
  [前スレ.498] [前スレ.508,511-513]
  casphy - 高校数学 - 不等式 - 673
  http://www.imojp.org/
  http://www.imojp.org/challenge/index.html 過去の問題集
184132人目の素数さん:2012/07/01(日) 18:55:35.39
Aren't there any solutions without using mixing variables?

f(a, b, c, d) ≧ f(√{ab}, √{ab}, c, d)
185132人目の素数さん:2012/07/01(日) 18:56:17.36
          __ノ)-'´ ̄ ̄`ー- 、_
        , '´  _. -‐'''"二ニニ=-`ヽ、
      /   /:::::; -‐''"        `ーノ
     /   /:::::/           \
     /    /::::::/          | | |  |
     |   |:::::/ /     |  | | | |  |
      |   |::/ / / |  | ||  | | ,ハ .| ,ハ|
      |   |/ / / /| ,ハノ| /|ノレ,ニ|ル' 
     |   |  | / / レ',二、レ′ ,ィイ|゙/   私は只の数ヲタなんかとは付き合わないわ。
.     |   \ ∠イ  ,イイ|    ,`-' |      頭が良くて数学が出来てかっこいい人。それが必要条件よ。
     |     l^,人|  ` `-'     ゝ  |        さらに Ann.of Math に論文書けば十分条件にもなるわよ。
      |      ` -'\       ー'  人          一番嫌いなのは論文数を増やすためにくだらない論文を書いて
    |        /(l     __/  ヽ、           良い論文の出版を遅らせるお馬鹿な人。
     |       (:::::`‐-、__  |::::`、     ヒニニヽ、         あなたの論文が Ann of Math に accept される確率は?
    |      / `‐-、::::::::::`‐-、::::\   /,ニニ、\            それとも最近は Inv. Math. の方が上かしら?
   |      |::::::::::::::::::|` -、:::::::,ヘ ̄|'、  ヒニ二、 \
.   |      /::::::::::::::::::|::::::::\/:::O`、::\   | '、   \
   |      /:::::::::::::::::::/:::::::::::::::::::::::::::::'、::::\ノ  ヽ、  |
  |      |:::::/:::::::::/:::::::::::::::::::::::::::::::::::'、',::::'、  /:\__/‐、
  |      |/:::::::::::/::::::::::::::::::::::::::::::::::O::| '、::| く::::::::::::: ̄|
   |     /_..-'´ ̄`ー-、:::::::::::::::::::::::::::::::::::|/:/`‐'::\;;;;;;;_|
   |    |/::::::::::::::::::::::\:::::::::::::::::::::::::::::|::/::::|::::/:::::::::::/
    |   /:::::::::::::::::::::::::::::::::|:::::::::::::::::::::O::|::|::::::|:::::::::::::::/
186132人目の素数さん:2012/07/01(日) 19:06:02.04
最近は数オリに出て来る様な不等式ばかりになって、このスレも堕ちたもんだな
187132人目の素数さん:2012/07/02(月) 10:28:16.90
>>186
ROMの分際で抜かすな
188132人目の素数さん:2012/07/02(月) 23:04:25.40
m,nは正の整数で、m<nとする。
0<x<1のとき、{1+(x/m^2)}^m , {1+(x/n^2)}^nの大小を定めよ。
189132人目の素数さん:2012/07/03(火) 02:29:57.08
1977 東工大の過去問

a, b, c>0, a+b+c=1のとき, sum_{cyc} sqrt{(1/a)+45-46a} の最小値
190132人目の素数さん:2012/07/03(火) 09:22:43.65
          __ノ)-'´ ̄ ̄`ー- 、_
        , '´  _. -‐'''"二ニニ=-`ヽ、
      /   /:::::; -‐''"        `ーノ
     /   /:::::/           \
     /    /::::::/          | | |  |
     |   |:::::/ /     |  | | | |  |
      |   |::/ / / |  | ||  | | ,ハ .| ,ハ|
      |   |/ / / /| ,ハノ| /|ノレ,ニ|ル' 
     |   |  | / / レ',二、レ′ ,ィイ|゙/   私は只の数ヲタなんかとは付き合わないわ。
.     |   \ ∠イ  ,イイ|    ,`-' |      頭が良くて数学が出来てかっこいい人。それが必要条件よ。
     |     l^,人|  ` `-'     ゝ  |        さらに Ann.of Math に論文書けば十分条件にもなるわよ。
      |      ` -'\       ー'  人          一番嫌いなのは論文数を増やすためにくだらない論文を書いて
    |        /(l     __/  ヽ、           良い論文の出版を遅らせるお馬鹿な人。
     |       (:::::`‐-、__  |::::`、     ヒニニヽ、         あなたの論文が Ann of Math に accept される確率は?
    |      / `‐-、::::::::::`‐-、::::\   /,ニニ、\            それとも最近は Inv. Math. の方が上かしら?
   |      |::::::::::::::::::|` -、:::::::,ヘ ̄|'、  ヒニ二、 \
.   |      /::::::::::::::::::|::::::::\/:::O`、::\   | '、   \
   |      /:::::::::::::::::::/:::::::::::::::::::::::::::::'、::::\ノ  ヽ、  |
  |      |:::::/:::::::::/:::::::::::::::::::::::::::::::::::'、',::::'、  /:\__/‐、
  |      |/:::::::::::/::::::::::::::::::::::::::::::::::O::| '、::| く::::::::::::: ̄|
   |     /_..-'´ ̄`ー-、:::::::::::::::::::::::::::::::::::|/:/`‐'::\;;;;;;;_|
   |    |/::::::::::::::::::::::\:::::::::::::::::::::::::::::|::/::::|::::/:::::::::::/
    |   /:::::::::::::::::::::::::::::::::|:::::::::::::::::::::O::|::|::::::|:::::::::::::::/
191132人目の素数さん:2012/07/03(火) 12:43:38.08
x, y, z≧0⇒(x^2+2yz)^{7}+(y^2+2zx)^{7}+(z^2+2xy)^{7}≦(x^2+y^2+z^2)^{7}+2(xy+yz+zx)^{7}
192132人目の素数さん:2012/07/03(火) 22:18:22.99
          __ノ)-'´ ̄ ̄`ー- 、_
        , '´  _. -‐'''"二ニニ=-`ヽ、
      /   /:::::; -‐''"        `ーノ
     /   /:::::/           \
     /    /::::::/          | | |  |
     |   |:::::/ /     |  | | | |  |
      |   |::/ / / |  | ||  | | ,ハ .| ,ハ|
      |   |/ / / /| ,ハノ| /|ノレ,ニ|ル' 
     |   |  | / / レ',二、レ′ ,ィイ|゙/   私は只の数ヲタなんかとは付き合わないわ。
.     |   \ ∠イ  ,イイ|    ,`-' |      頭が良くて数学が出来てかっこいい人。それが必要条件よ。
     |     l^,人|  ` `-'     ゝ  |        さらに Ann.of Math に論文書けば十分条件にもなるわよ。
      |      ` -'\       ー'  人          一番嫌いなのは論文数を増やすためにくだらない論文を書いて
    |        /(l     __/  ヽ、           良い論文の出版を遅らせるお馬鹿な人。
     |       (:::::`‐-、__  |::::`、     ヒニニヽ、         あなたの論文が Ann of Math に accept される確率は?
    |      / `‐-、::::::::::`‐-、::::\   /,ニニ、\            それとも最近は Inv. Math. の方が上かしら?
   |      |::::::::::::::::::|` -、:::::::,ヘ ̄|'、  ヒニ二、 \
.   |      /::::::::::::::::::|::::::::\/:::O`、::\   | '、   \
   |      /:::::::::::::::::::/:::::::::::::::::::::::::::::'、::::\ノ  ヽ、  |
  |      |:::::/:::::::::/:::::::::::::::::::::::::::::::::::'、',::::'、  /:\__/‐、
  |      |/:::::::::::/::::::::::::::::::::::::::::::::::O::| '、::| く::::::::::::: ̄|
   |     /_..-'´ ̄`ー-、:::::::::::::::::::::::::::::::::::|/:/`‐'::\;;;;;;;_|
   |    |/::::::::::::::::::::::\:::::::::::::::::::::::::::::|::/::::|::::/:::::::::::/
    |   /:::::::::::::::::::::::::::::::::|:::::::::::::::::::::O::|::|::::::|:::::::::::::::/
193132人目の素数さん:2012/07/04(水) 11:08:05.68
だれか, こんな幼稚の不等式のスレッドではなくて, 大人いうか, 学問的な不等式の
sレッドをたててくれませんか?見ていて不愉快です。
194132人目の素数さん:2012/07/04(水) 11:17:21.20
          __ノ)-'´ ̄ ̄`ー- 、_
        , '´  _. -‐'''"二ニニ=-`ヽ、
      /   /:::::; -‐''"        `ーノ
     /   /:::::/           \
     /    /::::::/          | | |  |
     |   |:::::/ /     |  | | | |  |
      |   |::/ / / |  | ||  | | ,ハ .| ,ハ|
      |   |/ / / /| ,ハノ| /|ノレ,ニ|ル' 
     |   |  | / / レ',二、レ′ ,ィイ|゙/   私は只の数ヲタなんかとは付き合わないわ。
.     |   \ ∠イ  ,イイ|    ,`-' |      頭が良くて数学が出来てかっこいい人。それが必要条件よ。
     |     l^,人|  ` `-'     ゝ  |        さらに Ann.of Math に論文書けば十分条件にもなるわよ。
      |      ` -'\       ー'  人          一番嫌いなのは論文数を増やすためにくだらない論文を書いて
    |        /(l     __/  ヽ、           良い論文の出版を遅らせるお馬鹿な人。
     |       (:::::`‐-、__  |::::`、     ヒニニヽ、         あなたの論文が Ann of Math に accept される確率は?
    |      / `‐-、::::::::::`‐-、::::\   /,ニニ、\            それとも最近は Inv. Math. の方が上かしら?
   |      |::::::::::::::::::|` -、:::::::,ヘ ̄|'、  ヒニ二、 \
.   |      /::::::::::::::::::|::::::::\/:::O`、::\   | '、   \
   |      /:::::::::::::::::::/:::::::::::::::::::::::::::::'、::::\ノ  ヽ、  |
  |      |:::::/:::::::::/:::::::::::::::::::::::::::::::::::'、',::::'、  /:\__/‐、
  |      |/:::::::::::/::::::::::::::::::::::::::::::::::O::| '、::| く::::::::::::: ̄|
   |     /_..-'´ ̄`ー-、:::::::::::::::::::::::::::::::::::|/:/`‐'::\;;;;;;;_|
   |    |/::::::::::::::::::::::\:::::::::::::::::::::::::::::|::/::::|::::/:::::::::::/
    |   /:::::::::::::::::::::::::::::::::|:::::::::::::::::::::O::|::|::::::|:::::::::::::::/
195132人目の素数さん:2012/07/04(水) 21:06:31.52
質問です
何処かの大学の過去問です
ネットで探しても答えは出てきませんでした

(1)
1以上の実数aと0以上の実数x,cに対して
不等式(x+c)^a>=x^a+c^aを示せ
(2)
1以上の実数aと0以上の実数x1x2x3.....xnに対して
不等式(x1+x2+......xn)^a>=x1^a+x2^a....xn^aを示せ
(3)
正の整数pqがp>=qを満たす時0以上の実数x1x2x3.....xnに対して
(x1^q+x2^q.....xn^q)^p>=(x1^p+x2^p.......)^q
を示せ

お願いします
196132人目の素数さん:2012/07/04(水) 22:45:09.51
>>195
(1)
 f(x) = x^a は下に凸だから、
  {c・f(0) + x・f(x+c)}/(x+c) ≧ f(x),
  {x・f(0) + c・f(x+c)}/(x+c) ≧ f(c),
辺々たすと
 f(0) + f(x+c) ≧ f(x) + f(c),
197132人目の素数さん:2012/07/04(水) 23:09:19.48
>>196
ありがとうございます
(1)使えば2出来るのはわかりました
(3)がさっぱりです
198132人目の素数さん:2012/07/04(水) 23:28:48.90
>>197
 p/q = a, xk^q = yk とおく。
199132人目の素数さん:2012/07/06(金) 03:27:18.12
>>189

最小値:17.1412604723691

 a = 0.19140424903813
 b = 0.19140424903813
 c = 0.61719150192374
のとき。
200132人目の素数さん:2012/07/06(金) 03:56:30.26
>>199
恐れながら大佐、凡人にも理解できるように経過を辿って解説をお願いしたいと思います
201132人目の素数さん:2012/07/06(金) 22:44:25.08
199 手計算でせんか!コンピュータを使うせこいまねなんてすんな、どあほ!!
202132人目の素数さん:2012/07/07(土) 04:30:29.70
>>11

xyz=1 だから、いつものように x=b/c, y=c/a, z=a/b (a,b,c>0)とおく。

 D = (a+b)(b+c)(c+a) = (a+b+c)(ab+bc+ca) - abc ≧ 8abc,

 {(左辺) - (3/4)}・D^2
  = {(a+b)b(c+a)}^2 + {(a+b)(b+c)c}^2 + {a(b+c)(c+a)}^2 - (3/4)D^2
  = (1/4)D(D-8abc) + (AAB+BBC+CCA-3ABC)
  ≧ 0,

ここに、A=a^2, B=b^2, C=c^2, D=(a+b)(b+c)(c+a)
203132人目の素数さん:2012/07/07(土) 04:56:34.45
>>13

 {x/(1+x)}^2 = X, {y/(1+y)}^2 = Y, {z/(1+z)}^2 = Z とおく。
これらは x, y, z と同順序である。

r≧0 のとき、チェビシェフにより
 (x^r)X + (y^r)Y + (z^r)Z ≧ (1/3)(x^r + y^r + z^r)(X+Y+Z)
 ≧ (xyz)^(r/3)・(X+Y+Z) (← 相加・相乗平均)
 = X+Y+Z,    (← xyz=1)
 = (>>11 の左辺)
204132人目の素数さん:2012/07/07(土) 06:25:13.10

      (^⌒⌒^)
       | i i i i i|    不等式、証明するよ!
       | i i i i i|    
      (;`・ω・)っ-O・゚・⌒) 
      /  つ━ゝ,.__゚____.,ノ))
           _l从从从从l_
     | ̄ ̄ ̄ ̄ ̄ ̄ ̄ ̄ ̄ ̄|

パフヌーティー・リヴォーヴィッチ・チェビシェフ
 [Pafnuty Lvovich Chebyshev]
     (1821〜1894 露)
205132人目の素数さん:2012/07/07(土) 06:38:18.37
>>14
 xyz=1 だから、例によって x=b/c, y=c/a, z=a/b とおく。

 D ’= (a+b)(b+c)(c+a)abc = {(a+b+c)(ab+bc+ca)-abc}abc,

 {(左辺) - (3/2)}・D '
  = (ab+bc+ca)^3 -(7/2)(a+b+c)(ab+bc+ca)abc +(9/2)(abc)^2 +(ABB+BCC+CAA-3ABC)
  = (1/2)(ab+bc+ca)abc・F_{-1}(a,b,c) + (1/2)(abc)^2・F_{-2}(a,b,c) + (ABB+BCC+CAA-3ABC)
  ≧ 0,

ここで Schur's を使った。
 F_{-1}(a,b,c) = {(ab+bc+ca)^2 -3(a+b+c)abc}/(abc) ≧ 0,
 F_{-2}(a,b,c) = {(ab+bc+ca)^3 -4(a+b+c)(ab+bc+ca)abc +9(abc)^2}/(abc)^2 ≧ 0,
206132人目の素数さん:2012/07/07(土) 18:11:00.79
>>188
 n-m=1 の場合

 {1+(x/m^2)} / {1+(x/n^2)} = 1 + (1/m^2 - 1/n^2)x/[1+(x/n^2)],

 [{1+(x/m^2)}/{1+(x/n^2)}]^m > 1 + m(1/m^2 -1/n^2)/[1+(x/n^2)] > 1 + (x/n^2),

ところで n-m=1, x<n^2 から
 m(1/m^2 -1/n^2)/[1+(x/n^2)] > 1/n^2,

 [{1+(x/m^2)}/{1+(x/n^2)}]^m > 1 + m(1/m^2 -1/n^2)/[1+(x/n^2)] > 1 + (x/n^2),

よって、{1+(x/m^2)}^m > {1+(x/n^2)}^n, (n-m=1)
207132人目の素数さん:2012/07/07(土) 18:29:06.29
>>205
不等式素人だから教えてほしいんだけれど、
「xyz=1 だから、例によって x=b/c, y=c/a, z=a/b とおく。」ていうのって一つのセオリーみたいなものなの?
208205:2012/07/07(土) 19:27:00.02
>>207
小生も不等式は素人なので分かりませんが、「小手技」はあるみたいです。
http://izumi-math.jp/kotewaza/index_j.html
209132人目の素数さん:2012/07/12(木) 13:03:29.14
For positive real numbers a_2, a_3,..., a_n with a_2a_3・・・a_n=1.

Prove that :

(a_2+1)^2(a_3+1)^3・・・(a_n+1)^n≧ n^{n}.
210132人目の素数さん:2012/07/13(金) 06:45:49.30
>>209

 {x^k, 1, 1, ・・・, 1} の相加・相乗平均は
     k-1 個
 x^k -kx +(k-1) = (x-1){x^(k-1) +x^(k-2) + ・・・ +x -(k-1)}
   = Σ[L=1,k-1] (x-1)(x^L - 1)
   ≧ 0,   (x>-1)

 x = {(k-1)/k}(a_k + 1) とおくと、
 (a_k + 1)^k ≧ {(k^k)/(k-1)^(k-1)}a_k,
 k=2〜n について掛ける。

  ∧_∧
  ( ;´∀`)
  人 Y /
 ( ヽ し
 (_)_)
211132人目の素数さん:2012/07/14(土) 02:20:31.81
( ゚∀゚)つ a_k>0 に対して、1 + a_1 + … + a_n < (1+a_1)(1+a_2)…(1+a_n) < e^(a_1 + … + a_n)
212132人目の素数さん:2012/07/14(土) 04:49:50.49
>>209 の改良

ラグランジュの未定乗数法で極小をさがす。
 a_k = λ/(k-λ),    (k=2,・・・,n)
 (与式) = Π[k=2,n] {k/(k-λ)}^k,

・n=3 のとき、
 λ = 6/5, a_2 = 3/2, a_3 = 2/3,
 (与式) ≧ 3125/108 = 28.935185・・・・・ > 27 = 3^3

・n=4 のとき
 a_k = λ/(k-λ),
 λ = {9 + [6*SQRT(11901)-81]^(1/3) - [6*SQRT(11901)+81]^(1/3)}/6
   = 1.3802745863984086
 (与式) ≧ 359.6812734317 > 256 = 4^4
213132人目の素数さん:2012/07/14(土) 22:09:42.38
>>195
(3) r=p/q≧1, y1=x1^q, y2=x2^q,…≧0 として
(y1+y2+…)^r≧y1^r+y2^r+… を示す。
y1+y2+…=1 の制約条件で y1^r+y2^r+… の最大値を求める問題にすると
ラグランジュの未定乗数法で
L=y1^r+y2^r+…−λ(y1+y2+…−1) の最大は
∂L/∂y1=r y1^(r−1)−λ=0 ∴ y1^(r−1)=y2^(r−1)=…=λ/r
∴ y1=y2=…=1/n
y1^r+y2^r+…=n(1/n)^r=1/n^(r−1)≦1=1^r=(y1+y2+…)^r
214132人目の素数さん:2012/07/15(日) 07:27:49.85
>>209 の改良
 L = Σ[k=2,n] {k・log(a_k + 1) - λ・log(a_k)},
 ∂L/∂a_k = k/(a_k + 1) - λ/(a_k) = 0  より
 a_k = λ/(k-λ),
 (2-λ)(3-λ)・・・・・(n-λ) = λ^(n-1), ・・・・ 決定方程式

・n=5 のとき
 (2-λ)(3-λ)(4-λ)(5-λ) = λ^4,
 λ = {71 + [(42√1749261)-13411]^(1/3) - [(42√1749261)+13411]^(1/3)}/42
   = 1.54263254839049
 (左辺) = 7407.43642129488 > 3125 = 5^5,


n→∞ のとき、λ→2
215132人目の素数さん:2012/07/15(日) 09:52:13.07
>>212 >>214

決定方程式(特性方程式)

・n=3 のとき
 λ^2 -(λ-2)(λ-3) = 5λ - 6,

・n=4 のとき
 λ^3 +(λ-2)(λ-3)(λ-4)
 = 2λ^3 -9λ^2 +26λ -24
 = {(2λ-3)^3 + 25(2λ-3) + 6}/4,

・n=5 のとき
 λ^4 - (λ-2)(λ-3)(λ-4)(λ-5)
 = 14λ^3 -71λ^2 +154λ -120
 = {(42λ-71)^3 + 4281(42λ-71) + 26822} / 5292,
216馬鹿を焼く描写 ◆ghclfYsc82 :2012/07/16(月) 20:08:35.85
勉強や努力が足りなくて優秀になれない奴が惨めな思いをするのは当然
なんだよ。それを自分で何もせずに優秀な人間の足を引っ張るとは言語
道断である。他人を貶めるだけで自分は楽をする奴は恥を知れ。今後も
そういう馬鹿者を発見次第、即刻攻撃を掛けて当該スレを焼け野が原に
するので、覚悟をする様に願いたい。こういう考え方が国家を滅ぼす。
無能な馬鹿は自滅するに任せ、優秀な人材こそを選択的に抽出し、それ
を国家が意図して保護しなければならない。そうする事が国家が生き残
る唯一の道である。繰り返す。何の努力もしない馬鹿を無条件に保護す
れば、その結果として誰も努力しなくなるだけである。だから馬鹿を保
護しては絶対にならない。



>みんなで優秀な人間の足を引っ張って沈もうよ。
>そうすれば自分だけが馬鹿で惨めな思いをしなくて
>すむから楽チン。
>一億総白痴可で横並びになれば怖くは無い
>
217132人目の素数さん:2012/07/16(月) 23:08:14.39
>>199-200

a,b は次の根。
1557376x^7 -3114752x^6 +2369920x^5 -838304x^4 +132066x^3 -7273x^2 +466x -94 = 0
218132人目の素数さん:2012/07/17(火) 07:14:06.11
a, b, c > 0, abc=1 のとき、(a^2+b^2)/(a+b+c^2) + (b^2+c^2)/(b+c+a^2) + (c^2+a^2)/(c+a+b^2) ≧ 2




_| ::|_
 ̄| ::|/|           ┌──┐
  | ::|  |     .┌──┐| ∧_∧  いいな、俺たちの誰かが殉職したら・・
/|_|  |┌──┐| ∧_∧|(・ω・` )
  |文|  | | ∧_∧(    )⊂   )
  | ̄|  | | (    )⊂   ) (_Ο Ο :::
  | ::|  | | ⊂   ) (_Ο Ο わかってる、生き延びた奴が
  | ::|/ .|_ (_Ο Ο ::::::::: :::::: 不等式を収集し、証明する !
  | ::| :::::::::::::::::::::::::::::::: 俺たちゃ死んでも仲間だぜ !!
219132人目の素数さん:2012/07/18(水) 07:24:50.64
>>218
 a,b,c>0, a+b+c=s のとき、

 (a+b)^2/{(a+b)s/3+c^2} + (b+c)^2/{(b+c)s/3+a^2} + (c+a)^2/{(c+a)s/3+b^2} ≧ 2*2,

(略証)
  {(a+b)s/3 + c^2} + {(b+c)s/3 + a^2} + {(c+a)s/3 + b^2}
 = {(s-c)s/3 + c^2} + {(s-a)s/3 + a^2} + {(s-b)s/3 + b^2}
 = s^2 + {(a-b)^2 + (b-c)^2 + (c-a)^2}/2
 ≧ s^2,
これとコーシーで・・・・
220132人目の素数さん:2012/07/18(水) 17:32:45.07
うむ
221219:2012/07/19(木) 03:59:41.48
>>218 (続き)

・・・・では上手く行かない .... orz

そこで、方針を変えて、
 a+b+cc ≦ (a+b)(a+b+c)/3 + cc ≦ (5aa+5bb+8cc)/6 = (5S+3cc)/6,

 (aa+bb)/(a+b+cc) ≧ 6(S-cc)/(5S+3cc)
    = (10/9) -(4/3S)cc + 4(cc -S/3)^2/{S(5S+3cc)}
    ≧ (10/9) -(4/3S)cc,
ここに S = aa + bb + cc.
循環的にたすと、
 (左辺) ≧ (10/3) - (4/3) = 2,
222219:2012/07/19(木) 04:07:25.29
>>218 (続き)

つまり、6(1-x)/(5+3x) = f(x) は 下に凸なので

 (左辺) ≧ f(aa/S) + f(bb/S) + f(cc/S)
   ≧ 3f((aa+bb+cc)/3S)
   = 3f(1/3)
   = 2,
223132人目の素数さん:2012/07/22(日) 08:03:19.04
>>219

(左辺)
 = (a+b)^2/{(a+b)s/3 + c^2} + (b+c)^2/{(b+c)s/3 + a^2} + (c+a)^2/{(c+a)s/3 + b^2}
 = 3(s-c)^2/(ss-cs+3cc) + 3(s-a)^2/(ss-as+3aa) + 3(s-b)^2/(ss-bs+3bb)
 = g(c/s) + g(a/s) + g(b/s),

ここに、g(x) = 3(1-x)^2/(1-x+3xx) とおいた。
 g(1/3) = 4/3, g(1/2) = 0.6

x=1/3 における接線:
 g(x) - g(1/3) + (16/3)(x - 1/3) = (16x-1)(x - 1/3)^2/(1-x+3xx),
 g(x) > (4/3) - (16/3)(x - 1/3),  (x>1/16)

x=1/2 における接線:
 g(x) - g(1/2) + 3.36(x - 1/2) = 1.44(7x+2)(x - 1/2)^2 > 0,
 g(x) > 0.6 - 3.36(x - 1/2),

・a, b < s/16 < c のとき、
 (左辺) > g(a/s) + g(b/s) > 2g(1/16) = 2(25/9) = 50/9,

・a < s/16 < b,c のとき、
 g(a/s) > g(0) - (32/9)a/s = 3 - (32/9)a/s,  (0<a/s<1/16)
 g(b/s) + g(c/s) > 2g(1/2) - 3.36(b+c-s)/s = 1.2 + 3.36a/s
 (左辺) > 4.2 - 0.2a/s > 4,

・s/16 < a,b,c のとき、
 x=1/3 での接線の上側にあるから、
 (左辺) = g(a/s) + g(b/s) + g(c/s) > 3g(1/3) = 4,
224132人目の素数さん:2012/07/22(日) 17:46:02.52
>>218

 abc=1 だから、s=a+b+c≧3,
 これと〔補題〕 >>219 から簡単に・・・
225132人目の素数さん:2012/07/22(日) 20:51:27.14
神あらわるあらわる
226132人目の素数さん:2012/07/22(日) 20:55:19.15
Σ[k=1 to n] 2(4k^2+6k+1)/(2k+2)! < 1 を証明せよ

簡単すぎてズコー
        ∧∧
       ヽ(・ω・)/   
      \(.\ ノ
    、ハ,,、  ̄
     ̄
227132人目の素数さん:2012/07/23(月) 07:00:48.54
>>226

 2(4k^2 +6k +1)/(2k+2)! = 2{(2k+2)(2k+1) - 1}/(2k+2)! = 2/(2k)! - 2/(2k+2)!
k=1 to n についてたす。

  ∧_∧
  ( ;´∀`)
  人 Y /
 ( ヽ し
 (_)_)
228132人目の素数さん:2012/07/23(月) 21:35:33.21
>>227
この程度でボッキするなら、以下の問題ならフルボッキが静まらないだろうな ( ゚∀゚)プケラッチョ!

(1) Σ[k=1 to n] k・k!
(2) Σ[k=0 to n-1] 2^k・tan(2^k・x)
229132人目の素数さん:2012/07/25(水) 05:28:32.80
>>228

簡単すぎる!
230描者の品格 ◆ghclfYsc82 :2012/07/25(水) 07:19:17.81


>367 :匿名希望:2012/07/23(月) 17:23:38.69
> >>365
> 頭悪いのはお前の方だろ?
> 「猫」という字を「描」に間違えやがってwww
> 小学生並みの頭の悪さだぞ、お前!
> 焼かれるのはお前の方だろ?
> 外国では猫の丸焼きというゲテモノ料理もあるらしいぜwww
>
231132人目の素数さん:2012/07/25(水) 07:26:56.29
>>228

(1) k・k! = (k+1)! - k!,

(2) tanθ = cotθ - 2cot(2θ),

  2^k・tan(2^k・x) = 2^k・cot(2^k・x) - 2^(k+1)・cot{2^(k+1)・x},
232描者の品格 ◆ghclfYsc82 :2012/07/25(水) 07:28:21.70


>367 :匿名希望:2012/07/23(月) 17:23:38.69
> >>365
> 頭悪いのはお前の方だろ?
> 「猫」という字を「描」に間違えやがってwww
> 小学生並みの頭の悪さだぞ、お前!
> 焼かれるのはお前の方だろ?
> 外国では猫の丸焼きというゲテモノ料理もあるらしいぜwww
>
233132人目の素数さん:2012/07/25(水) 21:20:35.79
>>228 の〔系〕

Σ[k=1 to n] (1/2^k)tan{x'/(2^k)} = (1/2^n)cot{x'/(2^n)} - cot(x'),
234132人目の素数さん:2012/07/25(水) 21:34:55.76
>>233
  ∧_∧
  ( ;´∀`) < してやられたわい
  人 Y /
 ( ヽ し
 (_)_)
235132人目の素数さん:2012/07/27(金) 01:55:51.45
( ゚∀゚)つ Σ[k=1 to ∞] arctan{1/(n^2+n+1)}
236132人目の素数さん:2012/07/27(金) 01:58:34.58
>>235
ミスった ( 'A`)つ Σ[n=1 to ∞] arctan{1/(n^2+n+1)}
237132人目の素数さん:2012/07/27(金) 02:01:47.71
ティンコ臭いスレはここでつか?
238132人目の素数さん:2012/07/27(金) 07:40:06.13
>>236

 1/(n^2 +n+1) = {(1/n) - 1/(n+1)}/{1 +(1/n)(1/(n+1))},
 arctan{1/(n^2 +n+1)} = arctan(1/n) - arctan{1/(n+1)},
あるいは
 1/(n^2 +n+1) = {(n+1)-n}/{1+n(n+1)},
 arctan{1/(n^2 +n+1)} = arctan(n+1) - arctan(n),
239真描vs偽描 ◆ghclfYsc82 :2012/07/27(金) 15:36:38.23


>462 名前:132人目の素数さん :2012/07/26(木) 23:54:17.40
> >>461
> 専門学校生が
> 「あらやだイケメンに触られて気持ちいい」
> って思ってたら通報されなかっただろうに
> 気持ち悪いおじさんになるために努力を積み重ねてきた結果
> 「キモ顔のおじさんが、気持ち悪く触ってきて超キモい」
> って思わせることに成功し逮捕されたんだよね
> 努力を実らせた立派な人だと思う
>
>
> 努力して痴漢で逮捕される夢を叶えた描者さんはただ者じゃないと思います
> すばらしい
>
240132人目の素数さん:2012/07/27(金) 15:37:15.81
p≦a, q≦b, r≦c のとき、
p+q+r + √{(p+q+r)^2 -3(pq+qr+rp)} ≦ a+b+c + √{(a+b+c)^2 -3(ab+bc+ca)}

は成り立つですか?
241真描vs偽描 ◆ghclfYsc82 :2012/07/27(金) 18:08:21.12
勉強や努力が足りなくて優秀になれない奴が惨めな思いをするのは当然
なんだよ。それを自分で何もせずに優秀な人間の足を引っ張るとは言語
道断である。他人を貶めるだけで自分は楽をする奴は恥を知れ。今後も
そういう馬鹿者を発見次第、即刻攻撃を掛けて当該スレを焼け野が原に
するので、覚悟をする様に願いたい。こういう考え方が国家を滅ぼす。
無能な馬鹿は自滅するに任せ、優秀な人材こそを選択的に抽出し、それ
を国家が意図して保護しなければならない。そうする事が国家が生き残
る唯一の道である。繰り返す。何の努力もしない馬鹿を無条件に保護す
れば、その結果として誰も努力しなくなるだけである。だから馬鹿を保
護しては絶対にならない。



>みんなで優秀な人間の足を引っ張って沈もうよ。
>そうすれば自分だけが馬鹿で惨めな思いをしなくて
>すむから楽チン。
>一億総白痴可で横並びになれば怖くは無い
>
242132人目の素数さん:2012/07/28(土) 10:47:41.16
>>240

p≦a, q=b, r=c のとき、

 (右辺) - (左辺) = a +√{(a+b+c)^2 -3(ab+bc+ca)} -p -√{(p+b+c)^2 -3(pb+bc+cp)}
   = (a-p) + (a-p){a+p-(b+c)}/[ √{(a+b+c)^2 -3(ab+bc+ca)} + √{(p+b+c)^2 -3(pb+bc+cp)} ]
   = (a-p)(1+δ)
   ≧ 0,

∵ |δ| = |a+p-(b+c)| / [ √{(a+b+c)^2 -3(ab+bc+ca)} + √{(p+b+c)^2 -3(pb+bc+cp)} ]
    ≦ |a+p-(b+c)| / [ |a-(b+c)/2| + |p-(b+c)/2| ]
    ≦ 1,

* (a+b+c)^2 - 3(ab+bc+ca) = |a-(b+c)/2|^2 + (3/4)(b-c)^2 ≧ |a-(b+c)/2|^2,
 (p+b+c)^2 - 3(pb+bc+cp) = |p-(b+c)/2|^2 + (3/4)(b-c)^2 ≧ |p-(b+c)/2|^2,
243描は偽猫 ◆ghclfYsc82 :2012/07/30(月) 11:54:37.96
馬鹿が嫉妬してそういう事を延々と続けてるから:
★★★『この国からはマトモな人が誰も出て来ない。国家が傾くだけ。』★★★
ですよね。その結果としてこの国は馬鹿で溢れ返ってしまうんですよね。
たとえ有能な者であっても、自分が努力をして上昇スルよりも、サボっ
て自分が馬鹿になって他人の足を引っ張った方が周囲の理解も得られ易
いし、加えてその方が楽ですからね。だから誰も向上心を持つ事に価値
を見出せない。でもその結果があの国会ですよ、あの国会ね。まあ:
★★★『向上心を持って努力する人を皆で潰す国家に明るい未来なんて有り得ない』★★★
なんですけどね。こうやって国家が滅んで行くんですワ。そして『ソレ
で良し』とするのがその考え方でしょ。まあ沈むのは貴方なんだから、
まあ勝手にしたらエエんだけどサ。こんな嫉妬を正当化スル考え方しか
出来ないから、例えばジョッブスみたいな人材が出ないんですよね、こ
の国からはサ。あの天才ジョッブスでさえ、周囲の馬鹿ゾンビから足を
引っ張られたら、あんな凄い歴史的な実績は残せなかったでしょうナ。
まあ日本にはイノベーションは必要無いから、ソレこそ『同じ穴の狢』
という言い方で、皆で協力して国家を崩壊に導いてるんですよね。

そう、貴方が国家を潰してるんだよ。でもどうぞお好きに潰しなさいな。
私にはもう関係が無いのでね。馬鹿の国だよ、この国は。どうぞ貴方が
国会議員にでもなって、この国の息の根を止めて下さいな。サッサと国
が崩壊したら、皆が一瞬で楽になるヨ。



>有能であれば、その卑劣な行為を回避すればいい
>足を引っ張られるのであれば、それは要領がない証拠
>それは有能ではなく無能者である
>
>同じ穴の狢
>
244132人目の素数さん:2012/07/31(火) 21:56:57.49
x≧0、y≧0、z≧0、x+y+z≦1の解の集合Xを、凸多面体としてパラメータ表示せよ

誰か教えてください!おねがいします
245描は無能 ◆ghclfYsc82 :2012/07/31(火) 21:58:09.94
ソレは絶対にお断りや。こんな有害無益な馬鹿板なんてワシが最後まで
徹底的に焼き払ったるヨ。そやし思いっきり苦しんで耐え忍べや。まあ
『アンタ等は自業自得』っちゅう事やろうナ。執拗な妨害行為が今後も
何年にも亘って延々と続くんを覚悟をスルっちゅう事やろうナ。

ワシはやナ、オマエ等みたいなド馬鹿に謝って欲しいんでも反省して欲
しいんでも何でもナイのや。唯単に崩壊して消えて欲しいだけなんだヨ。
そやからこうやって徹底抗戦をしてや、アンタ等みたいな馬鹿を傷め付
けてるだけなんやワ。そやし早よ諦めろや。

因みにもし「優秀な人の足を引っ張っても良い」のであれば:
★★★『馬鹿の足を思いっきり引っ張っても、ソレは当然の事ながら許される。』★★★
という事にナリマスわナ。



>664 名前:132人目の素数さん :2012/07/30(月) 22:05:57.48
> 猫頼むから消えてくれ
>
246132人目の素数さん:2012/08/01(水) 00:20:53.85
>>191
 f は3回微分可能で f ''' >0 とする。

 f(S) - f(S-A) - f(S-B) + f(S-A-B) = g(S) - g(S-B)
   = B・g '(S-Bθ)     (←平均値の定理)
   = B{f '(S-Bθ) - f '(S-A-Bθ)}
   = AB f "(S-Aφ-Bθ)   (←平均値の定理)
   = AB f "(ξ),           (*)
ここに min{S,S-A,S-B,S-A-B} ≦ ξ ≦ MAX{S,S-A,S-B,S-A-B}

 (右辺) - (左辺) = f(x^2 +y^2 +z^2) + 2f(xy+yz+zx) - f(x^2 +2yz) - f(y^2 +2zx) - f(z^2 +2xy)
   = f(x^2 +y^2 +z^2) - f(x^2 +2yz) - f(z^2 +2xy) + f((2x-y+2z)y)
   - {f((2x-y+2z)y) - 2f(xy+yz+zx) + f(y^2 +2zx)}
   = (δ^2)f "(ξ1) - (δ^2)f "(ξ2)    (← *)
   = (δ^2)(ξ1 - ξ2)f '''(η)  (← 平均値の定理)
   ≧ 0,        (← ξ1-ξ2≧0, f '''≧0)

ここで、δ=(x-y)(y-z), ξ1 ≧ (2x-y+2z)y,
y は xとzの中間にあるとしても一般性を失なわないから、ξ2 ≦ (2x-y+2z)y,
∴ ξ1-ξ2 ≧ 0,
247描もマルチ ◆ghclfYsc82 :2012/08/01(水) 04:09:35.41
ソレは絶対にお断りや。こんな有害無益な馬鹿板なんてワシが最後まで
徹底的に焼き払ったるヨ。そやし思いっきり苦しんで耐え忍べや。まあ
『アンタ等は自業自得』っちゅう事やろうナ。執拗な妨害行為が今後も
何年にも亘って延々と続くんを覚悟をスルっちゅう事やろうナ。

ワシはやナ、オマエ等みたいなド馬鹿に謝って欲しいんでも反省して欲
しいんでも何でもナイのや。唯単に崩壊して消えて欲しいだけなんだヨ。
そやからこうやって徹底抗戦をしてや、アンタ等みたいな馬鹿を傷め付
けてるだけなんやワ。そやし早よ諦めろや。

因みにもし「優秀な人の足を引っ張っても良い」のであれば:
★★★『馬鹿の足を思いっきり引っ張っても、ソレは当然の事ながら許される。』★★★
という事にナリマスわナ。



>664 名前:132人目の素数さん :2012/07/30(月) 22:05:57.48
> 猫頼むから消えてくれ
>
248132人目の素数さん:2012/08/01(水) 22:46:28.93
>>246

 fは2回微分可能で f " は単調増加
でもおk.
249132人目の素数さん:2012/08/02(木) 12:20:03.96
正の実数 x, y ,z が x^2+y^2+z^2=3 をみたすとき
1/(x^3+2) + 1/(y^3+2) + 1/(z^3+2) ≧ 1
が成り立つことを示せ

お願いします
250132人目の素数さん:2012/08/03(金) 05:03:28.55
>>249

 1/(x^3 +2) = (3-x^2)/6 + (1/6)(x+2){x(x-1)}^2 /(x^3 +2) ≧ (3-x^2)/6,
循環的にたす。
251132人目の素数さん:2012/08/04(土) 01:44:13.12
>>249の一般化

n 個の正の実数 x_1, x_2, ,, x_n が (x_1)^{n-1} + (x_2)^{n-1} + …+(x_n)^{n-1} = n を満たすとき、
 1/( (x)^n+ n-1) + 1/((x_2)^n +n-1) + … + 1/( (x_n)^n + n-1) ≧ 1

が成り立つことを示せ。
252132人目の素数さん:2012/08/04(土) 06:13:20.07
>>250 の一般化

 n(n-1) - {n-x^(n-1)}(x^n +n -1) = x^(n-1) (x^n +n -1 -nx)
   = x^(n-1) (x-1)^2 Σ[j=0,n-2] (n-1-j)x^j
   ≧ 0,   (相加・相乗平均)

∴ 1/(x^n +n -1) ≧ {n-x^(n-1)} / {n(n-1)},

∴ (左辺) ≧ {n^2 - Σ[k=1,n] (x_k)^(n-1)} / {n(n-1)},
253132人目の素数さん:2012/08/05(日) 06:21:11.68
>>18
〔問題〕
正の実数a,b,cに対して、
 a^3 + b^3 + c^3 + 6abc ≧ (a+b+c)^2・G,
 ここに G = (abc)^(1/3),


(キャスフィーの解答)
基本対称式を
 a+b+c = s,
 ab+bc+ca = t,
 abc = G^3,
とおく。
 (左辺) = s(s^2 -3t) + 9G^3,
 (左辺) - (右辺) = s^3 -3st + 9G^3 - Gs^2
   = (3/4)(s^3 -4st +9G^3) + (1/4)(s^3 -4Gs^2 +9G^3)
   = (3/4)F_1 + (1/4)(s-3G)(s^2 -Gs -3G^2)
   ≧ 0,
>>42 に従って Schur's (n=1) を使った。
 F_1 = s^3 -4st +9G^3 ≧0,
また
 s^2 -Gs -3G^2 = (s-3G)^2 + 5G(s-3G) + 3G^2 ≧ 0,

全然エレガントぢゃねぇ...
254132人目の素数さん:2012/08/05(日) 16:34:29.63
>>136の改題

a,b,c,d >0 のとき

 (a+b+c+d+2)(1/a+1/b+1/c+1/d +1/2) ≧ 9

を示せ。
255132人目の素数さん:2012/08/05(日) 19:26:37.47
>>253
そんな小細工するより、普通に c=1/ab を代入して 左辺ー右辺≧0 を示すのが一番簡単で早い。
256132人目の素数さん:2012/08/05(日) 20:21:06.13
>>254
 右辺は25だな。

>>255
 ではドゾー
257132人目の素数さん:2012/08/05(日) 22:04:41.49
>>254
シコって賢者タイムに入ってから考え直せ!

  (*^д゚) シコッ☆
 _(ヽηノ_ 
    ヽヽ 
258132人目の素数さん:2012/08/05(日) 22:49:42.88
>>256>>257
なら >>136の解答を教えてください。
考えているができないのです。
259132人目の素数さん:2012/08/05(日) 22:54:49.58
>>258
出典を述べよ
260132人目の素数さん:2012/08/05(日) 22:57:19.23
>>259
出典って言ってても、 >>136じゃないから分からんよ。
261132人目の素数さん:2012/08/05(日) 23:21:33.66
うせろかす
262132人目の素数さん:2012/08/05(日) 23:32:11.56
>>261
分からないんなら、そう言えばいいのに
263132人目の素数さん:2012/08/06(月) 00:09:04.64
>>136 名前:132人目の素数さん[] 投稿日:2012/06/13(水) 22:09:17.84
> a^2+b^2+c^2+d^2=4 (a,b,c,d>0)→(a+b+c+d-2)(1/a+1/b+1/c+1/d +1/2) >=9

x= a/2, y =b/2, z= c/2, d=w/2 とおくと、
 x^2 + y^2 + z^2 + x^2= 1 のとき、
  (x+y+z+w-1)(1/x +1/y +1/z +1/x +1) ≧ 9
を示せばよい。
264描は猫ですかね? ◆ghclfYsc82 :2012/08/07(火) 19:22:28.20


>14 名前:132人目の素数さん :2012/08/07(火) 17:39:00.96
> >>13
> 旧コテ猫あらため描つまりお前自身の事だろ、増田哲也に限り無く近い人間。
> 筑波大学で痴漢と言えば増田哲也だから連続性も明らかになってるから
> わざわざ限り無く近い人間なんて呼び方しなくていいんだけどな
>
265132人目の素数さん:2012/08/07(火) 19:23:18.30
          __ノ)-'´ ̄ ̄`ー- 、_
        , '´  _. -‐'''"二ニニ=-`ヽ、
      /   /:::::; -‐''"        `ーノ
     /   /:::::/           \
     /    /::::::/          | | |  |
     |   |:::::/ /     |  | | | |  |
      |   |::/ / / |  | ||  | | ,ハ .| ,ハ|
      |   |/ / / /| ,ハノ| /|ノレ,ニ|ル' 
     |   |  | / / レ',二、レ′ ,ィイ|゙/   私は只の数ヲタなんかとは付き合わないわ。
.     |   \ ∠イ  ,イイ|    ,`-' |      頭が良くて数学が出来てかっこいい人。それが必要条件よ。
     |     l^,人|  ` `-'     ゝ  |        さらに Ann.of Math に論文書けば十分条件にもなるわよ。
      |      ` -'\       ー'  人          一番嫌いなのは論文数を増やすためにくだらない論文を書いて
    |        /(l     __/  ヽ、           良い論文の出版を遅らせるお馬鹿な人。
     |       (:::::`‐-、__  |::::`、     ヒニニヽ、         あなたの論文が Ann of Math に accept される確率は?
    |      / `‐-、::::::::::`‐-、::::\   /,ニニ、\            それとも最近は Inv. Math. の方が上かしら?
   |      |::::::::::::::::::|` -、:::::::,ヘ ̄|'、  ヒニ二、 \
.   |      /::::::::::::::::::|::::::::\/:::O`、::\   | '、   \
   |      /:::::::::::::::::::/:::::::::::::::::::::::::::::'、::::\ノ  ヽ、  |
  |      |:::::/:::::::::/:::::::::::::::::::::::::::::::::::'、',::::'、  /:\__/‐、
  |      |/:::::::::::/::::::::::::::::::::::::::::::::::O::| '、::| く::::::::::::: ̄|
   |     /_..-'´ ̄`ー-、:::::::::::::::::::::::::::::::::::|/:/`‐'::\;;;;;;;_|
   |    |/::::::::::::::::::::::\:::::::::::::::::::::::::::::|::/::::|::::/:::::::::::/
    |   /:::::::::::::::::::::::::::::::::|:::::::::::::::::::::O::|::|::::::|:::::::::::::::/
266132人目の素数さん:2012/08/07(火) 22:11:58.07
>>263
分からないんなら、そう言えばいいのニダ
267baka描 ◆ghclfYsc82 :2012/08/08(水) 04:55:25.42


>14 名前:132人目の素数さん :2012/08/07(火) 17:39:00.96
> >>13
> 旧コテ猫あらため描つまりお前自身の事だろ、増田哲也に限り無く近い人間。
> 筑波大学で痴漢と言えば増田哲也だから連続性も明らかになってるから
> わざわざ限り無く近い人間なんて呼び方しなくていいんだけどな
>
268255:2012/08/10(金) 01:52:30.38
>>256
それでは・・・

 f(a,b,c) = a^3 + b^3 +c^3 - (a+b+c)^2 とおく。
 ab≧1≧c としても一般性を失わない。
 √(ab) = γ とおくと、相加・相乗により a+b ≧ 2γ ≧ 2,

∴ {f(a, b, c) - f(γ, γ, c)}/(√a - √b)^2
 = (a+b+γ)^2 -(√a + √b)^2 -2c
 = (a+b-2)(a+b+2γ+1) +ab +2γ +2(1-c)
 ≧ 0,
となるので、a=b の場合を考えればよい。
  f(a,a,1/a^2) = 2a^3 + 1/a^6 +6 -(2a + 1/a^2)^2
    = (2a^9 -4a^8 +6a^6 -4a^5 -a^2 +1)/(a^6)
    = (a-1)^2 (2a^7 -2a^5 +2a^4 +2a^3 +2a^2 +2a+1)/(a^6)
    ≧ 0,

エレガントでない事ぢゃあ引けをとらねぇ....
269255:2012/08/10(金) 01:59:30.60
>>268 の訂正を....orz

 f(a,b,c) = a^3 + b^3 + c^3 +6 -(a+b+c)^2 とおいた。
270132人目の素数さん:2012/08/10(金) 02:18:16.57
(a^3 + b^3 + c^3)/3 ≧ 3√(abc) =1 より a^3 + b^3 + c^3 ≧ 3
271132人目の素数さん:2012/08/10(金) 02:20:25.17
a^3 + b^3 + c^3 -3 = a^3 + b^3 + c^3 -3abc = (a+b+c)(a^2 + b^2 + c^2 -ab -bc- ca)
272132人目の素数さん:2012/08/10(金) 22:14:11.36
>>253
 華麗なるキャスフィー、偉大なキャスフィー、
273132人目の素数さん:2012/08/12(日) 23:30:21.46
>>136, >>263
[一般化]
 n 個の実数 x_k >0 が 農[k:1->n] (x_k)^2 = 1 を満たすとき、

 ( 農[k:1->n] x_k - 1)( 農[k:1->n] 1/x_k + 1) ≧ n^2 -n √n -1

が成立することを示せ。
(n=4 のときが >>136
274baka描 ◆ghclfYsc82 :2012/08/12(日) 23:31:42.41


>14 名前:132人目の素数さん :2012/08/07(火) 17:39:00.96
> >>13
> 旧コテ猫あらため描つまりお前自身の事だろ、増田哲也に限り無く近い人間。
> 筑波大学で痴漢と言えば増田哲也だから連続性も明らかになってるから
> わざわざ限り無く近い人間なんて呼び方しなくていいんだけどな
>
275132人目の素数さん:2012/08/14(火) 22:44:33.49
>>273

n=2 のときは
 (x+y-1)(1/x + 1/y +1) = x+y + xy/{(1+x)(1+y)} + {1 - 1/[(1+x)(1+y)]}(xx+yy-1)/(xy)
  = √(1+2xy) + xy/{(1+x)(1+y)}   (←題意)
  > 1
で簡単だが....

〔例〕
x_1 = √{1-(n-1)ε^2}, x_2 = ・・・ = x_n = ε,
ε→0 とすると (左辺) → (n-1)^2.
276132人目の素数さん:2012/08/15(水) 18:52:01.24
>>275
右辺を以下の様にするとどうでしょう?
n=4の時は >>136と一致します。

【改題】
[一般化]
 n 個の実数 x_k >0 が 農[k:1->n] (x_k)^2 = 1 を満たすとき、

 ( 農[k:1->n] x_k - 1)( 農[k:1->n] 1/x_k + 1) ≧ (n-1)^2

を示せ。
277132人目の素数さん:2012/08/16(木) 02:02:41.44
>>276

n≧4 では >>273
 ただし右辺は n^2 -(n-1)√n -1, 等号成立は x_k = 1/√n

n≦4 では >>276

かな?
278132人目の素数さん:2012/08/16(木) 02:45:44.53
>>277
証明が出来ないorz

是非、解答をお願いします。
(n の値で不等式が変わるのは面白いですね)
279132人目の素数さん:2012/08/18(土) 00:54:14.61
>>278

ぬ〜ん…

    /⌒ヽ
  /⌒  ・ >
  E ̄U) ε | 
  E ̄∩) ・ >
゛゛゛゛゛゛゛゛゛゛゛゛゛゛゛゛゛゛゛゛゛゛
280132人目の素数さん:2012/08/19(日) 04:23:18.17
>>278
           , /    ,
        ,   / /   ,   /  ,
          / '^メ-' ─/- 、   / ,
       ∠r  _,゛_ /  , ヽ/__/ モウ ダメダ…
        ''ヽ'_・.ノ` ' r/、 ヘ /‐’
       ./ " j 厂゙j | レ_`> j__ /
        '  .:‘::'ニ‘.:‐'´─゙.:´一’
281132人目の素数さん:2012/08/21(火) 15:39:09.64
   /                i丶   \ 
   /    /   ∧.      l  \   ヽ
  ,' /    !.  / ',     l    ヽ   ',             /
  ! l    l.  /   ',      lヽ、___',.  }            /   \
  | l    | / __ノヘ   /リ!      l  |             ̄ ̄ ̄ヽ
  | |     l ̄      ', //   _  l  |
  j l    l    _  V   〃 ̄ ` !〉 l/             │  │
.  V 〉、   !  /´ ̄`         l∧!             │  │
    !{ _\ l                  j                  ノ
   l ゝ __ ヽl       ___    /l               __
    !   >.      丶__ノ    イ l                /__/ ヽヽ
    l      `> 、  __ . <   |                  /
    |       リ丶____r‐'</)_j_                 /
    |     _/_: : : : : :: >' /`ー 、\
    |  >'´ ..ィ: \_: :/-、/}>ァ'´  \',            ─┼─
    r<_. <: : : : : :./ ヽ  Y  i /   \              │
    l: :\: : : : : : :/  ヽ \〉l´V /     \           ノ
282【大審議中】:2012/08/26(日) 22:44:06.51
     ∧,,∧  ∧,,∧         ∧,,∧  ∧,,∧    ∧,,∧  ∧,,∧         ∧,,∧  ∧,,∧
  ∧ (´・ω・) (・ω・`) ∧∧  ∧ (´・ω・) (・ω・`) ∧ (´・ω・) (・ω・`) ∧∧  ∧ (´・ω・) (・ω・`) ∧∧
 ( ´・ω) U) ( つと ノ(ω・` )( ´・ω) U) ( つと ノ( ´・ω) U) ( つと ノ(ω・` )( ´・ω) U) ( つと ノ(ω・` )
 | U (  ´・) (・`  ) と ノ | U (  ´・) (・` | U (  ´・) (・`  ) と ノ | U (  ´・) (・`  ) と ノ
  u-u (l     ∧,,∧  ∧,,∧ u-u (l    ) (∧,,∧u-u (l     ∧,,∧  ∧,,∧ u-u (l    ) (∧,,∧   /⌒ヽ
      `u-∧ (´・ω・) (・ω・`) ∧∧`u-∧ (´・ω・) (・ω`u-∧ (´・ω・) (・ω・`) ∧∧`u-∧ (´・ω・) (・ω・`) ∧∧
       ( ´・ω) U) ( つと ノ(ω・` )( ´・ω) U) ( つと ノ( ´・ω) U) ( つと ノ(ω・` )( ´・ω) U) ( つと ノ(ω・` )
       | U (  ´・) (・`  ) と ノ| U (  ´・) (・`  | U (  ´・) (・`  ) と ノ| U (  ´・) (・`  )と ノ
    ∧,,∧u-∧,,∧   ) (   ノu ∧,,∧-u∧,,∧  ) (∧,,∧u-∧,,∧   ) (   ノu ∧,,∧-u∧,,∧  ) (   ノu ∧,,∧  ∧,,∧
  ∧ (´・ω・) (・ω・`) ∧∧  ∧ (´・ω・) (・ω・`) ∧ (´・ω・) (・ω・`) ∧∧  ∧ (´・ω・) (・ω・`) ∧∧-u'∧ (´・ω・) (・ω・`) ∧∧
 ( ´・ω) U) ( つと ノ(ω・` )( ´・ω) U) ( つと ノ( ´・ω) U) ( つと ノ(ω・` )( ´・ω) U) ( つと ノ(ω・` ) ( ´・ω) U) ( つと ノ(ω・` )
 | U (  ´・) (・`  ) と ノ | U (  ´・) (・` | U (  ´・) (・`  ) と ノ | U (  ´・) (・`  ) と ノ | U (  ´・) (・`  )と ノ
  u-u (l     ∧,,∧  ∧,,∧ u-u (l    ) (∩,,∩u-u (l     ∧,,∧  ∧,,∧ u-u (l    ) (∩,,∩   /⌒ヽ -u (l    ) (   ノu-u
      `u-∧ (´・ω・) (・ω・`) ∧∧`u-∧ (´・ x ・) (・ω`u-∧ (´・ω・) (・ω・`) ∧∧`u-∧ (´・ x ・) (・ω・`) ∧∧`u-u' `u-u'
283132人目の素数さん:2012/08/31(金) 00:28:41.64
f''(x)≧0 (x≧0) のとき、x≧0に対して ∫[0, x] {f(2t)-f(t)} dt ≦ x{f(2x)-f(x)}/2 を示せ。

('A` ) プウ
ノヽノ) =3'A`)ノ ヒャー
  くく へヘノ
284132人目の素数さん:2012/08/31(金) 23:26:50.37
   /           /           ′  `ヽ
   ′         / ´ ̄`ヽ       /     、ヽ
    !        /´ .,. -、 ,ノ     , '    、   ヽ
    l       _ ´/ .,´ ./ {     /  l.    l   l ヽ
       _ - ´  、  > .{  /._  /   '´ 「` 、 !   |  \
    ヾ         `´   `´(_,ノィ´   .ハ. l .l  ,|   l  、 ヽ、    , -─- 、
      ` - _         ,ィ´ |l    l |' /-+、./   ハ  ヽ、 ヽ、 〈      `ヽ
          ` ┬--r‐ 7´ハ. ト l 、  ,' レ´l::::レ、ヽ  / l l ヽ `ヽ、 ヽ、` -―- 、  ヽ
         |  l!     ハ「ヽト、 、./ ' lー'::::::::}/l〉/.  ′|  、  `ー `-    }   .l
           ′ l |.、   〈| し::::::::l ´    、::::::;ノ .レ'  ′ |!  l    、⌒ ー-‐'   ./
           / ィ  l ヽ  トヽ;;:::::ソ __' -,、 `´ "イ  /  | ! .|   , - 、、, - 、_ /
.         / /l  lヽ `ヽ_ゝ""   V   } ./|  /.   l l l  ノ、 }
      /- ´ l  .| ヽ  l `,‐- _ゝ __,ノィ´ ,イ /,ヽ_ ./.  l l //`´
      /´     |  |! ヽ.  l l` 、_  `ヽ_,/_..∠ | /( .( 〉 r┐ //
.             l  | 、  ヽ l人   `>'ヨ_ト、 `l ′`´.`_o」 './
              、 | 、  >、l-'O _,>、/ | | 、`'` ̄「 ̄ノス,.<
               `  \ 〉,、,_ -_>、. / .| |  ゝ   .〉- ' ノ \
               /`.┬=t'/ `ー' ー´   / 、 /    \
                /  ` - _>  __   _,. ´  `  、    ヽ
                 /   ,. '´     ̄ ̄        `  、   ヽ
                  /  /                     ヽ   ヽ
285132人目の素数さん:2012/08/31(金) 23:30:53.00
       |: : : : : : : :,イ /'7 /: : : : : : :/ \ \
       |: : : : : :/: { ゝ::/ 7: : : : /    \ ヽ
       |:__; ;- ´: : : ` ´: :`:´○::/  ',     ヽ ,
       'ヽ: : : : : : : : : : : : : :.iイ   i |ヽ __|  i i, ',`、
        ` - : :_:_:_:_: : イ」_|_|,   ト,イ´, || | | .| ,ヽ
            / |'/´|V,,ゝト,  イ レ  }ノ |.| i|i  , \
             /   |  { O::::`, ヽノ  /⌒'7 ノイ)   ,  \
       (⌒ヽ/ / \ |ヽヾニソ ,      // レ   i\ \ _,,,--,,,_
         `{ニ;(´\ | ヽト.    _ノ   ノ|  /   | | ` ー|    \
       /{_フ7  } |  | > ,,__,, < / /   | |    `´⌒ヽ  ヽ
       /´ ヽ フ  ノ  | |'\_:_::ゝ/´: : :V / |  || |       }   }
          7i二二it ナ\| [:ニ[|\-'-//  / /'| ,|   く´⌒`ー  ノ
          | |  | ロ  ヽ /:/ |;ト′ | ̄''ヽ/__./ |/  r ヽヽ_, ‐-‐‐´
           | |   ー-ッ--く∠__|:》___ ー´ 〈〈`i  ...:::ヽノ
            | i|   /      ´  [二>,,,○|...::::''''
          /: :ー ´:ヽ        {: :[ニ/ヽ:::'''、
        /: : : : : : / ̄`ー ___ノ/   /: : :\
286132人目の素数さん:2012/08/31(金) 23:34:24.88
                 _,_. . : : : : . 、
               _,. :'´: : : : : : : : : :`ヽ
            _,.:'´: : : : : : : : : : : : : : : :ハ
          ∠,ヽ: : : : : : : : : : : : : : : : : : :i
        , '´ ̄¨´‐.、 `y : : : : : : : : : : : : : : {     ,.、
      /  .   : :  Y }  ヽ: : : : : : : : : : : |  _,.f´__,\
      .′ :   :    :}v  ,,ゝ: : : : : : : : : : |―¬;.:.:.:.:_:.:.:\
.     l .  :     :. .:!|  ̄ ! ヽ : : : : : : : : ヽj_,.-亠'¨⌒ヽ:_ノ
      ! {  :  :.  |:.:,!r' , 、〉   \∧_: : : :\
     }.:}.  :.l  :.: .!:.i T~^.|        ̄=-'¨
   (_,ノ_;{:...:.:.:!: .:.:. .ハ:{_.`|  i
      f彡Y:.:.}:|:.!:.:!:{:イ'` |  !         / ̄ ̄ ̄ ̄ヽ\
 /\ ⌒j,ィ:.{:!:ハ:|:!{{|   j  !        /          / |
/   l   , '´ ̄,ト! }川,,ー/,.  {,,,.........,,,,,.,,,,,,,,,,.ト .... _ _ - ´   | ,,,,,,
l   |  /   __」 、__ノ―-}n.n r}ー===-=-=- l 、 /_,-、 )    ノ-=''
.、  ` ´  /        ´ ゙           >、_'ー'o_ - ´ 
 ヽ、  /
.   ` ´
はてなようせい
http://www6.atwiki.jp/moetext/pages/14.html
287132人目の素数さん:2012/09/01(土) 05:21:39.70
>>283
微分のことは微分でするのもいいが...

 (左辺) = ∫[0,x] ∫[0,t] f '(t+u) du dt
     = ∬[0≦u≦t≦x] f '(t+u) du dt 
     = (1/2)∫[0,x] ∫[0,x] f '(t+u) du dt (←対称性 t & u)

 (右辺) = (1/2)∫[0,x] du ∫[0,x] f '(t+x) dt
     = (1/2)∫[0,x] ∫[0,x] f '(t+x) dt du
題意より、
 u≦x ⇒ f '(t+u) ≦ f '(t+x)
288132人目の素数さん:2012/09/01(土) 16:15:37.24
【問題】
実数 a, b, c >0 に対して、次の不等式を示せ。

a/(b+c) + b/(a+c) + c/(a+b) ≧ 3/2
289132人目の素数さん:2012/09/01(土) 16:18:32.35
【問題】
実数 a_1,,, a_n >0 に対して、次の不等式を示せ。

a_1/a_n + a_2/a_1 + a_3/a_2 + ・・・+ a_n/a_{n-1} ≧ n
290132人目の素数さん:2012/09/01(土) 18:16:41.02
>>288

コーシーにより
 2(ab+bc+ca){左辺}
 = {a(b+c) + b(c+a) + c(a+b)} {a/(b+c) + b/(c+a) + c/(a+b)}
 ≧ (a+b+c)^2
 ≧ 3(ab+bc+ca),
また題意より、ab+bc+ca >0,

参考文献[3], p.28-30 (1987) 例題4
H.S.Shapiro: Amer. Math. Monthly, 61, p.571 (1954) "Problem 4603"
L.J.Mordell: Abh. Math. Sem. Univ. Hamburg, 22, p.229-240 (1958)
A.Zulauf: Abh. Math. Sem. Univ. Hamburg, 22, p.240-241 (1958)
291132人目の素数さん:2012/09/01(土) 18:24:53.98
参考文献[3] は神! 日本人の書いた本で、これを超える不等式本はないよな。
292132人目の素数さん:2012/09/01(土) 18:48:57.98
                  __________________
         /      ∩|不等式への |
        /      / /|     招待 |
           /  / / ..|_________________|
              / /     | |
  .        / / /∧   ./ /
            / / ´_ゝ`)/  復刊キボンヌ!
          / |      /
            |    /
            |   /⌒l
             ヽ   | /
           / | ゙ー'| L
        /     |  /(_  ヽ
         / / ノ
       /  / /
     /   ( ヽ  
293132人目の素数さん:2012/09/01(土) 18:51:46.88
小中学校のときに、夏休みの宿題の読書感想文に課題図書が指定されてたよな
そこでだ!
高校生の夏休みの宿題に「不等式への招待」を課題図書として指定しようではないか
294132人目の素数さん:2012/09/01(土) 18:55:15.33
>>288

コーシーまたはチェビシェフ(*)により
 (左辺) = (a+b+c){1/(b+c) + 1/(c+a) + 1/(a+b)} - 3
     = (1/2){(b+c) + (c+a) + (a+b)} {1/(b+c) + 1/(c+a) + 1/(a+b)} - 3
     ≧ 9/2 - 3
     = 3/2,

*) Σ 乱順序積 ≧ Σ 逆順序積
295132人目の素数さん:2012/09/01(土) 19:18:30.22
>>294 に1行追加...

 = (1/2){(b+c) + (c+a) + (a+b)} {1/(b+c) + 1/(c+a) + 1/(a+b)} -3
 = (1/2){9 + α(b-c)^2 + β(c-a)^2 + γ(a-b)^2} -3
 ≧ 9/2 - 3

ここに、α = 1/[(c+a)(a+b)], β = 1/[(a+b)(b+c)], γ = 1/[(b+c)(c+a)],
296132人目の素数さん:2012/09/01(土) 19:55:53.68
>>289
nについての帰納法による。

・n=2 のとき
 (左辺) = a_1/a_2 + a_2/a_1 = 2 + (a_1-a_2)^2 /(a_1・a_2) ≧ 2 (←相加・相乗平均)

・n>2 のとき
 左辺を S_n とおく。
 a_n を最大(or 最小)の要素としても一般性を失わない。
 S_n - S_(n-1) - 1 = a_1/a_n + a_n/a_{n-1} - a_1/a_{n-1} -1
          = (a_n - a_1)[1/a_{n-1} - 1/a_n]
          ≧ 0,
297132人目の素数さん:2012/09/01(土) 22:00:37.95
ここの住人って>>288-289のような数列の不等式に強いな。

逆に、>>283 のような微積分の不等式には弱い気がする。
Jensenの不等式や積分の平均値の定理とかで出る。
298287:2012/09/01(土) 22:28:55.61
>>297
 積分だけで出ますが何か?
299287:2012/09/02(日) 00:30:57.45
>>283 >>297

部分積分で
 (左辺) = x{f(2x) - f(x)} - ∫[0,x] t{2f '(2t) - f '(t)} dt
     = (右辺)*2 - ∫[0,x] t{2f '(2t) - f '(t)} dt,
これより
 (左辺) - (右辺) = (1/2)∫[0,x] {f(2t) - f(t) - 2t・f '(2t) + t・f '(t)} dt
  = (1/2)∫[0,x] t{f '(t) - f '(2t)} dt
  + (1/2)∫[0,x] {f(2t) - f(t) - t・f '(2t)} dt,
ところで、題意より
  f '(t) - f '(2t) ≦ 0,  (0<t)
  f(2t) - f(t) - t・f '(2t) = ∫[0,t] {f '(t+u) - f '(2t)} du ≦ 0, (t<u)
とか。
300132人目の素数さん:2012/09/02(日) 02:17:02.65
>>291
今買える本の中では、渡部隆一の本がわりとしっかり書いてある。

[5] 不等式入門,渡部隆一,森北出版,2005年
   http://amazon.jp/o/ASIN/4627010494
301132人目の素数さん:2012/09/02(日) 02:33:45.97
>>297
チッチッチッ、弱いんじゃないんだな〜、これが!
積分使うのは一本道で簡単だから、それ以外の方法を紹介してくれているんだyo
その程度のことは、初代スレからの常連達は暗黙の了解の上なんだyo yo
蒐集して出題した者達も、想定外のどんな解答を見せてくれるかワクテカしながら見ているんだyo yo yo
302132人目の素数さん:2012/09/02(日) 02:36:24.20
言い忘れたが、出題された不等式を改造するのは暗黙の約束
元よりも良い評価をしたり、変数を増やしたり、類題を作ったりするのは日常茶番劇なのだyo
恐れ入ったか、この野郎w
303132人目の素数さん:2012/09/02(日) 02:38:45.25
>>300
表紙の色が変わっているぅ!
俺のはピンク色のフニャフニャ表紙なんだぜ!
304132人目の素数さん:2012/09/02(日) 02:53:58.53
>>302
ヘンタイのすくつなんですね
305132人目の素数さん:2012/09/02(日) 03:28:42.99
>>302 >>304
そう言われると改造しない訳には・・・・


>>289 の系
 (b_1・b_2・・・・b_n)^(1/n) = G, (相乗平均)
 a_1 = b_1/G,
 a_k = a_{k-1}・b_k/G  (1<k<n)
 a_n = 1,
とおくと、
 (b_1+b_2+・・・・+b_n)/G ≧ n,
すなわち
 (b_1+b_2+・・・・+b_n)/n ≧ G,
306132人目の素数さん:2012/09/02(日) 03:31:08.07
>>304
コニャロ、ティクショウメ!最高の褒め言葉ダゼィ!

>>305
神キタ━━━┌(_Д_┌ )┐━━━!!
307132人目の素数さん:2012/09/02(日) 04:38:35.41
>>301
>>294-296は標準的な解答で、とても想定外とは思えんのだが

積分が一本道だとか、積分の不等式の威力を分かっとらんな。
アプリオリ評価とか面白いのが沢山ある。
308132人目の素数さん:2012/09/02(日) 06:16:43.13
>>307
口だけなら何とでも言える
309132人目の素数さん:2012/09/02(日) 06:20:43.87
盛り上がってまいりました
310132人目の素数さん:2012/09/02(日) 08:18:09.17
               __)__)_〉
                _ ‐≦つC≧‐.._    _|_○
            i/´_ ニ  _ニ‐、`ヽヽ.   / | ヽ
              !  { ・ ) { ・ `!   `y
          {i ミ `ニ立` - 'ミ u _j   ‐/‐,
          /    ゝ=、(  `  ∫}   / ./
            i    `ーij' `     ーイ
          '、   、__,      /     |
         , ‐ゝ .. _    __ -‐く        |
         /      ̄      ヽ
          / y              !  i    ヽ  /
          / / o      o   }  l   _/
         ,' i                l  |
311132人目の素数さん:2012/09/02(日) 09:48:09.00
【283】の類題
f '''(x)≧0 のとき、x≧0に対して
 ∫[0,x] {f(3t)-2f(2t)+f(t)} dt ≦ x{f(3x)-2f(2x)+f(x)}/3,
でしょうか?

312132人目の素数さん:2012/09/02(日) 11:22:36.38
>>308
では n ≧4のとき、>>276 を証明してください。
313132人目の素数さん:2012/09/02(日) 13:20:01.34
>>312

n>4 のとき √n > 2,

 a_k = 1/√n,  [k:1→n] とおくと

 (左辺) = (√n - 1)(n√n +1)
    = n^2 -(n-1)√n - 1
    < n^2 -2(n-1) -1
    = (n-1)^2,
にて >>276 は不成立....orz
314132人目の素数さん:2012/09/02(日) 13:29:37.75
【283】の類題
f ""(x)≧0 のとき、x≧0に対して
 ∫[0,x] {f(4t)-3f(3t)+3f(2t)-f(t)} dt ≦ x{f(4x)-3f(3x)+3f(2x)-f(x)}/4,
でしょうか?


一般に、x≧0 で f^(n) ≧0 のときは、
 g(x) = Σ[k=1,n] (-1)^(n-k) C(n-1,k-1) f(kx)
だろうなぁ...
315132人目の素数さん:2012/09/02(日) 15:42:03.25
>>305
この解法は上手いね


>>313
右辺を変えると成り立ちますか?

【改題】
[一般化]
 n 個 (n ≧4) の実数 x_k >0 が 農[k:1->n] (x_k)^2 = 1 を満たすとき、

 ( 農[k:1->n] x_k - 1)( 農[k:1->n] 1/x_k + 1) ≧ n^2 -(n-1)√n - 1

を示せ。
316132人目の素数さん:2012/09/02(日) 18:57:29.91
【問題】
実数 a_1,,, a_n >0 に対して、次の不等式を示せ。

a_1/a_n + a_2/a_1 + a_3/a_2 + ・・・+ a_n/a_{n-1} ≧ n

x/y+y/x=x/y+1+(y-x)/x=1+(xx+yy-xy)/xy=1+1+(y-x)(y-x)/xy
x/z+y/y+z/x=1+x/z+z/x=3+(z-x)(z-x)/xz
w/z+x/y+y/x+z/w=4+(y-x)(y-x)/xy+(z-w)(z-w)/wz
...

317あのこうちやんは始皇帝だった:2012/09/02(日) 19:21:23.39

 20代と60代の、ニート・無職の虫けらども!!!!!!!!!!!!!!

 早く定職に就け!!!!!!!!!!!!!!!!!!!!!


318132人目の素数さん:2012/09/02(日) 22:04:16.59
df/dai=1/(ai-1)-(ai+1)/ai^2=0
ai=((ai+1)(a1-1))^.5
ai=a
f(a)=n

319132人目の素数さん:2012/09/02(日) 23:05:19.60
>>275-277

n=2 のときは
 (x+y-1)(1/x + 1/y) = (x+y-1)(x+y)/xy
  = (x^2 +y^2 +2xy -x-y)/xy
= (2xy-x-y+1)/xy    (←題意)
  = {xy + (1-x)(1-y)}/xy
  = 1 + {(1-x)/x}{(1-y)/y}
  > 1,
で簡単だが....

※別法
 x = (1-t^2)/(1+t^2), y = 2t/(1+t^2), (0<t<1) とおく。
320132人目の素数さん:2012/09/03(月) 01:32:16.89
n=3,4 の時はどう?
321132人目の素数さん:2012/09/03(月) 07:15:41.74
>>320

ぬ〜ん…
 
    /⌒ヽ
  /⌒  ・ >
  E ̄U) ε | 
  E ̄∩) ・ >
゛゛゛゛゛゛゛゛゛゛゛゛゛゛゛゛゛゛゛゛゛゛

           , /    ,
        ,   / /   ,   /  ,
          / '^メ-' ─/- 、   / ,
       ∠r  _,゛_ /  , ヽ/__/ モウ ダメダ…
        ''ヽ'_・.ノ` ' r/、 ヘ /‐’
       ./ " j 厂゙j | レ_`> j__ /
        '  .:‘::'ニ‘.:‐'´─゙.:´一’ 
322132人目の素数さん:2012/09/03(月) 09:44:31.80
lllllllllllllllllllllllllllllllllllllllllllllllllllllllllllllllllllllllllllllllllllllllll
llllllllllllllllllllllllll/ ̄ ̄ヽlllllllllllllllllllllllllllllllllllllllllll
lllllllllllllllllllll /      ヽllllllllllllllllllllllllllllllllllllll
iiiiiiiiiiiiiiiiiiiiii  試 そ あ .iiiiiiiiiiiiiiiiiiiiiiiiiiiiiiiiiiiiii
iiiiiiiiiiiiiiiiiiiii|  合 こ き  |iiiiiiiiiiiiiiiiiiiiiiiiiiiiiiiiiiiii
;;;;;;;;;;;;;;;;;;;;;|  終 で ら  |;;;;;;;;;;;;;;;;;;;;;;;;;;;;;;;;;;;;;
;;;;;;;;;;;;;;;;;;;;;|  了  め  |;;;;;;;;;;;;;;;;;;;;;;;;;;;;;;;;;;;;;
;:;:;:;:;:;:;:;:;:;:;|  だ  .た  |:;:;:;:;:;:;:;:;:;:;:;:;:;:;:;:;:;:;:
;:;:;:;:;:;:;:;:;:;:;|  よ   ら  |:;:;:;:;:;:;:;:;:;:;:;:;:;:;:;:;:;:;:
:.:.:.:.:.:.:.:.:.:.:ヽ、      /.:.:.:.:.:.:.:.:.:.:.:.:.:.:.:.:.:.:.:
:. :. :. :. :. :. :. :. ‐‐--‐‐':. :. :. :. :. :. :. :. :. :. :. :.
: : : : : : : : : : : : : : : : : : : : : : : : ,.‐- 、 : : : :
                  廴ミノ
                 ///¨' 、
                 y':;:;:;:/⌒i!
                J:;:;:;:;};:;:/;},
      ;il||||li'       t`'---‐';:;:;:l
     ,.r'"''、,┘        7;:;:;:;:;:;:;:;「
    ノ4 (⌒i        .}:;:;:;:;:;:;;/
   /..,__彡{, |         `i:;:;:;:;:;}
   (  .ミi!} l、         .」:;:;:丿
  クュ二二`Lっ)        `==='
323132人目の素数さん:2012/09/03(月) 19:31:30.53
n=3くらいは強引に出来ないものか?
324132人目の素数さん:2012/09/04(火) 00:30:39.62
_ノ乙(、ン、)_ モウダメダ…
325132人目の素数さん:2012/09/04(火) 01:20:05.87
>>276

( 農[k:1->n] x_k - 1)( 農[k:1->n] 1/x_k + 1)
= 農[k:1->n] x_k 農[k:1->n] 1/x_k + 農[k:1->n] x_k - 農[k:1->n] 1/x_k -1

ここで、第1項はコーシー・シュワルツの不等式より、
   農[k:1->n] x_k 農[k:1->n] 1/x_k ≧ { 農[k:1->n] 1 }^2 =n^2

また、条件 農[k:1->n] (x_k)^2 = 1 より、
    農[k:1->n] x_k ≦ { 農[k:1->n] (x_k)^2 }^{1/2} {農[k:1->n] 1^2}^{1/2} = √n 
となる。
よって、
326132人目の素数さん:2012/09/04(火) 01:33:45.25
>>325
> よって、

       ヽ|/
     / ̄ ̄ ̄`ヽ、
    /         ヽ
   /  \,, ,,/    |
   | (●) (●)|||  |
   |  / ̄⌒ ̄ヽ U.|   ・・・・・・・・ゴクリ。
   |  | .l~ ̄~ヽ |   |
   |U ヽ  ̄~ ̄ ノ   |
   |    ̄ ̄ ̄    |
327132人目の素数さん:2012/09/05(水) 05:32:05.81
このスレを見て不等式に魅了されましたが>>2の[4]は不等式初心者でも理解できますか?
328328:2012/09/05(水) 06:17:05.80
3=log_(2){8}
329329:2012/09/05(水) 07:47:52.50
3^2=9
330330:2012/09/05(水) 13:15:12.42
3-3=0
331331:2012/09/05(水) 13:49:11.38
3÷3=1
332332:2012/09/05(水) 15:27:55.82
3 ! = 3 × 2
333132人目の素数さん:2012/09/05(水) 15:30:13.82
>>327
最後のほうは厄介だけど前半分は高校生でも頑張れば分かるレベル
334132人目の素数さん:2012/09/05(水) 18:20:14.46
このスレって高校数学のスレだったのか…
335福地 裕:2012/09/05(水) 23:54:30.63
ある人は蝶を集め、ある人は切手を収集し、ある人は不等式を集める

ここにこれを書いたのは俺なんだよ。自慢でもなんでもなく。
ただ、数学板が「数学」になればいいなと思ってさ。
無論元ネタは別だよ。あの本もいいよね。
336132人目の素数さん:2012/09/06(木) 00:08:03.91
>>335
へんな奴が沸いたな
337132人目の素数さん:2012/09/06(木) 00:21:59.50
数学板のあちこちに書き込んでるな
新しい荒らしか?
とりあえずNG登録しておこう
338福地 裕:2012/09/06(木) 00:28:53.46
あらしでもなんでもない。ほんとうの事だ。ななしにあきただけだ。君が
おれのなをNG登録するのは君の自由だ。ここはけいじばんなんだぜ。
339132人目の素数さん:2012/09/06(木) 02:59:13.94
>>332
上手いな

339は簡単
3×3=9
340132人目の素数さん:2012/09/06(木) 03:16:54.53
>>339
他所でやってくれないか?
341132人目の素数さん:2012/09/06(木) 14:33:34.81
>>334
高校生もいるが、主に高校の教師や予備校,塾の講師が多い。
いずれにせよ、受験関係者で、研究者は少ない
342132人目の素数さん:2012/09/06(木) 14:52:54.16
ただの不等式ヲタも居まつ!
343132人目の素数さん:2012/09/06(木) 15:06:47.31
>>342
そんな不等式ヲタに問題

[問題]
実数 a_1 … a_n と正数 x_1 … x_n >0 に対して、次を示せ。

Σ_[i,j=1 … n] (a_i a_j)/ (x_i + x_j) ≧ 0
344132人目の素数さん:2012/09/06(木) 21:24:04.27
   /                i丶   \ 
   /    /   ∧.      l  \   ヽ
  ,' /    !.  / ',     l    ヽ   ',             /
  ! l    l.  /   ',      lヽ、___',.  }            /   \
  | l    | / __ノヘ   /リ!      l  |             ̄ ̄ ̄ヽ
  | |     l ̄      ', //   _  l  |
  j l    l    _  V   〃 ̄ ` !〉 l/             │  │
.  V 〉、   !  /´ ̄`         l∧!             │  │
    !{ _\ l                  j                  ノ
   l ゝ __ ヽl       ___    /l               __
    !   >.      丶__ノ    イ l                /__/ ヽヽ
    l      `> 、  __ . <   |                  /
    |       リ丶____r‐'</)_j_                 /
    |     _/_: : : : : :: >' /`ー 、\
    |  >'´ ..ィ: \_: :/-、/}>ァ'´  \',            ─┼─
    r<_. <: : : : : :./ ヽ  Y  i /   \              │
    l: :\: : : : : : :/  ヽ \〉l´V /     \           ノ
345132人目の素数さん:2012/09/07(金) 04:30:07.80
>>343

与式をn次元空間 (a_1,a_2,・・・・,a_n) における2次函数と考え、f(a) とおく。
a=0 で ∇f = 0 となり停留値である。
さらに2次函数のヘッセ行列Hは定数であり、

 H = det(∇∇f)
   = (x)^2 / Π[i,j=1,・・・・,n] (x_i + x_j)
   > 0  (←題意)

ただし、凵ix) = Π[1≦i<j≦n] (x_j - x_i),  (差積、Vandermonde)

∴ f は半正値で、f(a) ≧ f(0) = 0.


〔補題〕 (Cauchy)
n次正方行列について
 det| 1/(x_i + y_i) | = ± (x) (y)/Π[i,j=1,・・・・,n] (x_i + y_j)
346132人目の素数さん:2012/09/07(金) 10:09:50.10
>>345
素晴らしい!
347132人目の素数さん:2012/09/08(土) 14:06:29.66
>>345 の補足。

n次対称行列 { 1/(x_i+x_j) }_{i,j=1,・・・・,n} をFとおく。

補題により、Fの左上角の主座小行列式(principal minor)はすべて非負。

 det| 1/(x_i+x_j) |_{i,j=1,・・・・,k} ≧ 0, (k=1,・・・・,n)

また、Fは対称(エルミート)行列ゆえ、fは半正値。
348132人目の素数さん:2012/09/08(土) 20:50:56.21
問題
 a, b, c > 0 が abc=1 を満たすとき、
a^2 + b^2 + c^2 ≦ a^3 + b^3 + c^3
を示せ。
349132人目の素数さん:2012/09/08(土) 21:14:03.03
>>348
改造せずにはいられない ( ゚∀゚)プケラッチョ!

abc=1 をみたす正の数 a、b、c に対して、a^3 + b^3 + c^3 ≧ a^2 + b^2 + c^2 ≧ a+b+c

いやいやいや、もっと ( ゚∀゚)プケラッチョ!

abc=1 をみたす正の数 a、b、c と任意の自然数 n に対して、a^n + b^n + c^n ≧ a^(n-1) + b^^(n-1) + c^^(n-1)

(゚∀゚ ) プウ
ノヽノ) =3'A`)ノ ヒャー
  くく へヘノ
350132人目の素数さん:2012/09/08(土) 21:57:07.30
【問題】
A = (a_{ij} ) を n 次正方行列とし、λ_1, , , λ_n をその固有値とするとき、次の不等式を示せ。
Σ_[i=1,,n] |λ_i|^2 ≦ Σ_[i j =1,,n] |a_{ij} |^2
351132人目の素数さん:2012/09/08(土) 21:59:29.44
>>349
さらに改造

abc=1 をみたす正の数 a、b、c と任意の実数 x に対して、a^x + b^x + c^x ≧ a^(x-1) + b^(x-1) + c^(x-1)
352132人目の素数さん:2012/09/08(土) 23:52:10.48
>>351
a=2, b=1, c=1/2, x=0
353132人目の素数さん:2012/09/09(日) 01:50:55.40
>>351
j実数 x≧1 ではなかろうか? ( ゚∀゚)プケラッチョ!
354132人目の素数さん:2012/09/09(日) 02:35:19.14
>>353
改良

abc=1 をみたす正の数 a、b、c >0 と実数 x ≧ y >0 に対して、次を示せ。

 a^x + b^x + c^x ≧ a^y + b^y + c^y
355132人目の素数さん:2012/09/09(日) 07:21:15.85
>>354
 (abc)^(1/3) = G とおく(相乗平均)。
 (x-y)y ≧ 0,

〔解1〕
 (y/x)a^x + {(x-y)/x}(a^y+b^x+c^x)/3
 ≧ (y/x)a^x + {(x-y)/x}G^x  (←相加・相乗平均)
 ≧ a^y・G^(x-y),
循環的にたす。
 (左辺) ≧ (a^y + b^y + c^y)G^(x-y)
     = (右辺)G^(x-y),

〔解2〕
  {a^y, b^y, c^y} と {a^(x-y), b^(x-y), c^(x-y)} とは同順序。

 (左辺) ≧ (右辺){a^(x-y) + b^(x-y) + c^(x-y)}/3 (←チェビシェフ)
     ≧ (右辺)G^(x-y),    (←相加・相乗平均)
356132人目の素数さん:2012/09/09(日) 17:51:53.19
>>354-355
すげー!

解法も鮮やかだが、ここまで一般化できるのか!
357132人目の素数さん:2012/09/09(日) 18:08:15.60
>>354-355
さらに拡張

[問題]
(a_1…a_n) =1 をみたす正の数 a_1、…、a_n > 0 と実数 x ≧ y ≧ 0 に対して、
次の不等式が成立する。

 (a_1)^x + (a_2)^x + … + (a_n)^x ≧ (a_1)^y + (a_2)^y + … + (a_n)^y


[解答]
G= (a_1…a_n)^{1/n} を相乗平均とする。
また、a_1≧a_2≧…≧a_n としてもよいので、
  {(a_1)^y, …, (a_n)^y} と { (a_1)^(x-y), …, (a_n)^(x-y)} とは同順序。

 (左辺) ≧ (右辺){ (a_1)^(x-y) + … + (a_n)^(x-y)}/n (←チェビシェフ)
     ≧ (右辺)G^(x-y),    (←相加・相乗平均)
     =(右辺)(← G=1)
358132人目の素数さん:2012/09/09(日) 20:53:13.73
          __ノ)-'´ ̄ ̄`ー- 、_
        , '´  _. -‐'''"二ニニ=-`ヽ、
      /   /:::::; -‐''"        `ーノ
     /   /:::::/           \
     /    /::::::/          | | |  |
     |   |:::::/ /     |  | | | |  |
      |   |::/ / / |  | ||  | | ,ハ .| ,ハ|
      |   |/ / / /| ,ハノ| /|ノレ,ニ|ル' 
     |   |  | / / レ',二、レ′ ,ィイ|゙/   私は只の数ヲタなんかとは付き合わないわ。
.     |   \ ∠イ  ,イイ|    ,`-' |      頭が良くて数学が出来てかっこいい人。それが必要条件よ。
     |     l^,人|  ` `-'     ゝ  |        さらに Ann.of Math に論文書けば十分条件にもなるわよ。
      |      ` -'\       ー'  人          一番嫌いなのは論文数を増やすためにくだらない論文を書いて
    |        /(l     __/  ヽ、           良い論文の出版を遅らせるお馬鹿な人。
     |       (:::::`‐-、__  |::::`、     ヒニニヽ、         あなたの論文が Ann of Math に accept される確率は?
    |      / `‐-、::::::::::`‐-、::::\   /,ニニ、\            それとも最近は Inv. Math. の方が上かしら?
   |      |::::::::::::::::::|` -、:::::::,ヘ ̄|'、  ヒニ二、 \
.   |      /::::::::::::::::::|::::::::\/:::O`、::\   | '、   \
   |      /:::::::::::::::::::/:::::::::::::::::::::::::::::'、::::\ノ  ヽ、  |
  |      |:::::/:::::::::/:::::::::::::::::::::::::::::::::::'、',::::'、  /:\__/‐、
  |      |/:::::::::::/::::::::::::::::::::::::::::::::::O::| '、::| く::::::::::::: ̄|
   |     /_..-'´ ̄`ー-、:::::::::::::::::::::::::::::::::::|/:/`‐'::\;;;;;;;_|
   |    |/::::::::::::::::::::::\:::::::::::::::::::::::::::::|::/::::|::::/:::::::::::/
    |   /:::::::::::::::::::::::::::::::::|:::::::::::::::::::::O::|::|::::::|:::::::::::::::/
359132人目の素数さん:2012/09/10(月) 02:49:59.73
解法はともかく、不等式自体はムーアヘッドの特殊化だよね
360132人目の素数さん:2012/09/12(水) 01:07:23.11
【問題】
x, y, z >0 が xyz ≧1 を満たすとき、 (1+x)(1+y)(1+z) ≧ 9 を示せ。
361132人目の素数さん:2012/09/12(水) 01:08:53.21
>>360の訂正
【問題】
x, y, z >0 が xyz ≧1 を満たすとき、 (1+x)(1+y)(1+z) ≧ 8 を示せ。
362132人目の素数さん:2012/09/12(水) 02:18:28.15
>>361
 (xyz)^(1/3) = G とおく。

 (左辺) = 1 + (x+y+z) + (xy+yz+zx) + xyz
     ≧1 + 3(xyz)^(1/3) + 3(xyz)^(2/3) + xyz
     = 1 + 3G + 3G^2 + G^3
     = (1+G)^3,
363132人目の素数さん:2012/09/12(水) 02:28:43.55
>>361
相加・相乗平均より、
1 + x ≧ 2 (x)^{1/2}
y, z についても同様なので、
 (左辺) ≧ 2 (x)^{1/2} ・2 (y)^{1/2}・2 (z)^{1/2}
       =8 (xyz)^{1/2} ≧ 8.
364132人目の素数さん:2012/09/12(水) 05:00:22.27
(*゚∀゚)=3 ハァハァ…
365132人目の素数さん:2012/09/12(水) 23:08:02.02
>>361

〔コーシーの拡張〕
m>1, n>1 は自然数, a_k(i) ≧ 0 とする。
 Π[i=1,m]{Σ[k=1,n] a_k(i)} ≧ (Σ[k=1,n] G_k)^m
ここに、G_k = {Π[i=1,m] a_k(i)}^(1/m)

(略証)
I. m=2^L (2ベキ) のときはコーシーをL回使う。

II. 2^(L-1) < m < 2^L のとき
 m < i ≦ 2^L に対して a_k(i) = G_k とおいて I. を使う。
366132人目の素数さん:2012/09/12(水) 23:10:43.78
(*´∀`)=3 ハァハァ…
367132人目の素数さん:2012/09/13(木) 01:39:01.73
[問題]
x, y, z≧0が x+y+z=1 を満たすとき、 x^3 + y^3 + z^3 + 6xyz ≧ 1/4 を証明せよ。
368132人目の素数さん:2012/09/13(木) 08:02:16.87
>>367

 x^3 + y^3 + z^3 +(15/4)xyz -(1/4)(x+y+z)^3
 = (3/4){x(x-y)(x-z) + y(y-z)(y-x) + z(z-x)(z-y)}
 = (3/4)F_1(x,y,z)
 ≧ 0,    {← x,y,z≧0, Schur(n=1)}
369132人目の素数さん:2012/09/13(木) 08:04:27.61
>>368
その係数は どうやって思いつくんだよ
370132人目の素数さん:2012/09/13(木) 14:49:13.49
>>368
>>367の不等式が荒くて、もっと改良できるってこと?
371132人目の素数さん:2012/09/13(木) 19:14:49.98
等号成立条件は?
372132人目の素数さん:2012/09/13(木) 20:54:13.88
>>367
 (左辺) -(1/4)(x+y+z)^3 = (3/4){x(x-y-z)^2 + y(y-z-x)^2 + z(z-x-y)^2},
 等号成立は (x,y,z) = (x,x,0) (x,0,x) (0,y,y)

>>368 は牛刀でござる。
 x=y=z でも等号成立。
373132人目の素数さん:2012/09/13(木) 22:35:38.19
(;´ρ`) ハァハァ…
374132人目の素数さん:2012/09/13(木) 22:39:35.02
>>372
素晴らしい!

ところで、Shur の不等式って4変数以上でも成り立つの?
375132人目の素数さん:2012/09/14(金) 00:30:00.58
a[0] = a > 0
b[0] = b > 0
a[n+1] = (a[n] + b[n])/2
b[n+1] = (a[n]*b[n])^(1/2)
とするとき、lim[n→∞] a[n]、lim[n→∞] b[n] を求めよ
376375:2012/09/14(金) 00:32:09.86
おなら貼り忘れた (不要なのは分かっているが、貼りたい年頃なんだ…、小学生が皆うんち好きなように…)

('A` ) プウ
ノヽノ) =3'A`)ノ ヒャー
  くく へヘノ
377132人目の素数さん:2012/09/14(金) 08:46:02.18
プロ棋士が二歩で負ける瞬間
ttp://www.youtube.com/watch?v=MUdvinVHoNs&feature=related
378132人目の素数さん:2012/09/14(金) 18:09:58.36
>>377
素で吹いた
379132人目の素数さん:2012/09/14(金) 23:47:40.19
380132人目の素数さん:2012/09/14(金) 23:52:01.47
将棋はスレ違いじゃないのかな、夏厨くん
381132人目の素数さん:2012/09/14(金) 23:59:18.47
おいらも冷えぴたを貼って、試験に臨むか
382132人目の素数さん:2012/09/15(土) 01:10:57.14
>>375
n≧1に対して、a[n] ≧ a[n+1]、 a[n] ≧ b[n]、 b[n+1] ≧ b[n]
つまり、a[1] ≧ a[2] ≧ … ≧a[n] ≧a[n+1] ≧ … ≧ b[n+1] ≧ b[n] ≧ … ≧ b[2] ≧ b[1] までは分かった
で、それからどうするんだっけ?

383132人目の素数さん:2012/09/15(土) 01:35:29.62
>>382
有界単調数列は収束するので、極限をα、βとすると、
 α = (α + β)/2
 β = (αβ)^{1/2}
が成立する。
これを解けば、α=β
384132人目の素数さん:2012/09/15(土) 01:53:15.40
なるほど、同じ極限値に収束するノッカー
で、その値はどうやったら得られるのですカニ?
385132人目の素数さん:2012/09/15(土) 02:07:30.32
>>375

算術幾何平均とか云うらしい。(Gauss)

極限値は
 α = β = π(a+b)/{4K(k)}, k = |a-b|/(a+b),
ここに
 K(k) = ∫[0,π/2] 1/√[1 - (k・sinφ)^2] dφ,
 第1種の完全楕円積分

参考文献
 http://ja.wikipedia.org/wiki/算術幾何平均
 http://mathworld.wolfram.com/Arithmetic-GeometricMean.html
 高木:「解析概論」改訂第三版、岩波書店 (1961) p.33 練習問題1の (1)
386132人目の素数さん:2012/09/15(土) 02:22:17.94
相加平均と総乗平均を使って定義された数列の極限を弄ったら、楕円積分が出てきたの巻…
387132人目の素数さん:2012/09/15(土) 02:26:39.33
藪をつついたら蛇が出たの巻
388132人目の素数さん:2012/09/15(土) 02:30:53.73
正の数a、bの相加平均、総乗平均、調和平均をA、G、Hとおくとき、 G = √(AH)
つまり、「総乗平均は、相加平均と調和平均の総乗平均に等しい」

3文字以上のときは、G と √(AH) の大小って簡単に分かるんです蟹?
389132人目の素数さん:2012/09/15(土) 03:17:46.25
k=1 の時とかは入試問題にありそうだな
390132人目の素数さん:2012/09/15(土) 12:53:20.09
>>388
とりあえず↓が有り鱒。

〔シェルピンスキーの不等式〕
n文字のとき、
 {A^(n-1)・H}^(1/n) ≧ G ≧ {A・H^(n-1)}^(1/n),

参考文献[3]、p.79〜80 (1987)
391132人目の素数さん:2012/09/15(土) 13:35:26.22
むむむ…、不等式め、侮れぬ…
392132人目の素数さん:2012/09/15(土) 14:31:04.85
>>390 の補足

n文字のとき、
 (k次の基本対称式)/C[n,k] = P_k とおくと、
 A = P_1, G = (P_n)^(1/n), H = P_n/P_(n-1),

>>390
 (P_1)^(n-1)・{P_n/P_(n-1)} ≧ P_n ≧ P_1・{P_n/P_(n-1)}^(n-1),
すなわち、
 P(n-1)/{(P_1)^(n-1)} = Π[k=1,n-2] (Q_k)^(n-1-k) ≧ 1,
 {P_(n-1)}^(n-1)/{(P_1)(P_n)^(n-2)} = Π[k=2,n-1] (Q_k)^(k-1) ≧ 1,
ここに、Q_k = (P_k)^2/{P_(k-1)・P_(k+1)}^(n-1-k).
よって次の補題に帰着する。

〔補題〕
 (P_k)^2/{P_(k-1)・P_(k+1)} ≧ 1,  (k=1〜n-1)

この補題については
 数セミ増刊「数学の問題」第(1)集、日本評論社 (1977) No.21
 Hardy-Littlewood-Polya: "Inequalities", Cambridge (1934) §2.22, 公式51-55
 Beckenbach-Bellman: "Inequalities", Ergebnisse叢書、Springer-Verlag (1961) p.11
393132人目の素数さん:2012/09/15(土) 19:38:00.21
【問題】
x+y+z=1 を満たす実数 x, y, z ≧ 0 は次を満たすことを示せ。
 x^2 y + y^2 x + z^2 x ≦ 4/27
394132人目の素数さん:2012/09/15(土) 20:01:51.04
不等式って何で不等式っていうんですか。
A=Bが等式なら、不等式はA≠Bの形であるのが負触りい用語のように思うのでウsが

A≧Bなら 例えば 大小式 くらいの用語がふさわしのではないかと
395132人目の素数さん:2012/09/15(土) 20:28:20.25
>>349
数学用語は西洋の物だから

例えば、英語では大小関係のある「不等式」は、
(upper/lower) bound, estimate や majoration などと言い、
 inequality よりは良く用いられる。
396132人目の素数さん:2012/09/15(土) 20:41:22.93
>>395
> inequality よりは良く用いられる。
これ本当かいな?
397132人目の素数さん:2012/09/15(土) 20:43:33.25
いちいち数えたことないけど、体感ではダウト
398132人目の素数さん:2012/09/15(土) 22:22:58.54
          __ノ)-'´ ̄ ̄`ー- 、_
        , '´  _. -‐'''"二ニニ=-`ヽ、
      /   /:::::; -‐''"        `ーノ
     /   /:::::/           \
     /    /::::::/          | | |  |
     |   |:::::/ /     |  | | | |  |
      |   |::/ / / |  | ||  | | ,ハ .| ,ハ|
      |   |/ / / /| ,ハノ| /|ノレ,ニ|ル' 
     |   |  | / / レ',二、レ′ ,ィイ|゙/   私は只の数ヲタなんかとは付き合わないわ。
.     |   \ ∠イ  ,イイ|    ,`-' |      頭が良くて数学が出来てかっこいい人。それが必要条件よ。
     |     l^,人|  ` `-'     ゝ  |        さらに Ann.of Math に論文書けば十分条件にもなるわよ。
      |      ` -'\       ー'  人          一番嫌いなのは論文数を増やすためにくだらない論文を書いて
    |        /(l     __/  ヽ、           良い論文の出版を遅らせるお馬鹿な人。
     |       (:::::`‐-、__  |::::`、     ヒニニヽ、         あなたの論文が Ann of Math に accept される確率は?
    |      / `‐-、::::::::::`‐-、::::\   /,ニニ、\            それとも最近は Inv. Math. の方が上かしら?
   |      |::::::::::::::::::|` -、:::::::,ヘ ̄|'、  ヒニ二、 \
.   |      /::::::::::::::::::|::::::::\/:::O`、::\   | '、   \
   |      /:::::::::::::::::::/:::::::::::::::::::::::::::::'、::::\ノ  ヽ、  |
  |      |:::::/:::::::::/:::::::::::::::::::::::::::::::::::'、',::::'、  /:\__/‐、
  |      |/:::::::::::/::::::::::::::::::::::::::::::::::O::| '、::| く::::::::::::: ̄|
   |     /_..-'´ ̄`ー-、:::::::::::::::::::::::::::::::::::|/:/`‐'::\;;;;;;;_|
   |    |/::::::::::::::::::::::\:::::::::::::::::::::::::::::|::/::::|::::/:::::::::::/
    |   /:::::::::::::::::::::::::::::::::|:::::::::::::::::::::O::|::|::::::|:::::::::::::::/
399132人目の素数さん:2012/09/15(土) 22:35:14.35
>>396
estimate や bound を使うのは解析(偏微分方程式)が多いかな

a priori estimate とは言うけど、a priori inequality とは余り言わんね。
400132人目の素数さん:2012/09/16(日) 07:08:55.11
http://www.amaga.me/image/nz19203341267.png

正数a,b,cが ab+bc+ca+2abc=1 を満たすとき
2abc+Σ[cyc]1/(1+2a)^2≧1

のような問題に対して、
a=x/(y+z)、b=y/(z+x)、c=z/(x+y) とおいて・・・とやる解法って不等式オタにとっては一般的なの?
401132人目の素数さん:2012/09/16(日) 07:20:17.98
いろんな置き換えあるよなぁ
どんな式のときに どんな置き換えが有効なのか知りたい

>>400
このサイトって、どこですか?
過去ログを遡って、不等式を蒐集しまくりたいです
402132人目の素数さん:2012/09/16(日) 07:24:10.56
>>401
>>3にもあるんだけど、www.mathlinks.ro/ だよ。
こういう置き換え、誰かリストアップしてくれないかなあ・・・
403132人目の素数さん:2012/09/16(日) 07:37:46.78
>>402
サンクス、潜入してくるわ
404132人目の素数さん:2012/09/16(日) 07:42:48.90
お取込み中ですが....


>>393
 f(x,y,z) = (4/27)(x+y+z)^3 - (左辺)
      = (1/27)(x+y+z)^3 + g(x-y,y-z,z-x),
とおく。
{x,y,z} を一斉に増加するとき、fも増加する。
∴ min{x,y,z} = m とおくと
 f(x,y,z) ≧ f(x-m,y-m,z-m)
      = f(0,a,b)    (a≧0, b≧0)
      = (1/27)(4a+b)(a-2b)^2
      ≧ 0,
等号成立は (x,y,z) = (0,2,1) (2,1,0) (1,0,2)


>>392 訂正スマソ.
 ここに、Q_k = (P_k)^2/{P_(k-1)・P_(k+1)},
405132人目の素数さん:2012/09/16(日) 07:47:36.67
神乙!

不等式コレクションに分類していたときに、置き換えについて整理したことがあったんだけど、
そのときのTeXファイルがどこかに逝ってしまったの巻… ( ゚∀゚)プケラッチョ!
406132人目の素数さん:2012/09/16(日) 08:00:14.43
>>405
> そのときのTeXファイルがどこかに逝ってしまった
Oh....
407132人目の素数さん:2012/09/16(日) 10:46:44.99
>>404

具体的に書けば
 g(x-y,y-z,z-x) = {(x+y-2z)(x-y)^2 + (y+z-2x)(y-z)^2 + (z+x-2y)(z-x)^2 + 9凩/2,
ここに
 (x,y,z) = (x-y)(y-z)(z-x),
408132人目の素数さん:2012/09/16(日) 11:33:02.61
>>393>>100 (n=2)と同じ....
409132人目の素数さん:2012/09/16(日) 12:40:39.68
          __ノ)-'´ ̄ ̄`ー- 、_
        , '´  _. -‐'''"二ニニ=-`ヽ、
      /   /:::::; -‐''"        `ーノ
     /   /:::::/           \
     /    /::::::/          | | |  |
     |   |:::::/ /     |  | | | |  |
      |   |::/ / / |  | ||  | | ,ハ .| ,ハ|
      |   |/ / / /| ,ハノ| /|ノレ,ニ|ル' 
     |   |  | / / レ',二、レ′ ,ィイ|゙/   私は只の数ヲタなんかとは付き合わないわ。
.     |   \ ∠イ  ,イイ|    ,`-' |      頭が良くて数学が出来てかっこいい人。それが必要条件よ。
     |     l^,人|  ` `-'     ゝ  |        さらに Ann.of Math に論文書けば十分条件にもなるわよ。
      |      ` -'\       ー'  人          一番嫌いなのは論文数を増やすためにくだらない論文を書いて
    |        /(l     __/  ヽ、           良い論文の出版を遅らせるお馬鹿な人。
     |       (:::::`‐-、__  |::::`、     ヒニニヽ、         あなたの論文が Ann of Math に accept される確率は?
    |      / `‐-、::::::::::`‐-、::::\   /,ニニ、\            それとも最近は Inv. Math. の方が上かしら?
   |      |::::::::::::::::::|` -、:::::::,ヘ ̄|'、  ヒニ二、 \
.   |      /::::::::::::::::::|::::::::\/:::O`、::\   | '、   \
   |      /:::::::::::::::::::/:::::::::::::::::::::::::::::'、::::\ノ  ヽ、  |
  |      |:::::/:::::::::/:::::::::::::::::::::::::::::::::::'、',::::'、  /:\__/‐、
  |      |/:::::::::::/::::::::::::::::::::::::::::::::::O::| '、::| く::::::::::::: ̄|
   |     /_..-'´ ̄`ー-、:::::::::::::::::::::::::::::::::::|/:/`‐'::\;;;;;;;_|
   |    |/::::::::::::::::::::::\:::::::::::::::::::::::::::::|::/::::|::::/:::::::::::/
    |   /:::::::::::::::::::::::::::::::::|:::::::::::::::::::::O::|::|::::::|:::::::::::::::/
410132人目の素数さん:2012/09/16(日) 13:44:25.58
>>400
文字を置き換えるのは工房レベル

大学数学ではLagrangeの未定乗数法がほぼ万能
411132人目の素数さん:2012/09/16(日) 14:40:00.38
余裕のない奴だな。もっと楽しめよ
412132人目の素数さん:2012/09/16(日) 15:42:16.93
面白みのない男でつね
413132人目の素数さん:2012/09/18(火) 01:38:30.23
>>411-412
コニャロ、ティクショウメ!最低の褒め言葉ダゼィ!
414132人目の素数さん:2012/09/19(水) 04:53:55.86
x, y, z>0, xyz=64/27 ⇒ √(1+x^2)+√(1+y^2)+√(1+z^2)≧1+x+y+z
415132人目の素数さん:2012/09/19(水) 05:08:53.98
          __ノ)-'´ ̄ ̄`ー- 、_
        , '´  _. -‐'''"二ニニ=-`ヽ、
      /   /:::::; -‐''"        `ーノ
     /   /:::::/           \
     /    /::::::/          | | |  |
     |   |:::::/ /     |  | | | |  |
      |   |::/ / / |  | ||  | | ,ハ .| ,ハ|
      |   |/ / / /| ,ハノ| /|ノレ,ニ|ル' 
     |   |  | / / レ',二、レ′ ,ィイ|゙/   私は只の数ヲタなんかとは付き合わないわ。
.     |   \ ∠イ  ,イイ|    ,`-' |      頭が良くて数学が出来てかっこいい人。それが必要条件よ。
     |     l^,人|  ` `-'     ゝ  |        さらに Ann.of Math に論文書けば十分条件にもなるわよ。
      |      ` -'\       ー'  人          一番嫌いなのは論文数を増やすためにくだらない論文を書いて
    |        /(l     __/  ヽ、           良い論文の出版を遅らせるお馬鹿な人。
     |       (:::::`‐-、__  |::::`、     ヒニニヽ、         あなたの論文が Ann of Math に accept される確率は?
    |      / `‐-、::::::::::`‐-、::::\   /,ニニ、\            それとも最近は Inv. Math. の方が上かしら?
   |      |::::::::::::::::::|` -、:::::::,ヘ ̄|'、  ヒニ二、 \
.   |      /::::::::::::::::::|::::::::\/:::O`、::\   | '、   \
   |      /:::::::::::::::::::/:::::::::::::::::::::::::::::'、::::\ノ  ヽ、  |
  |      |:::::/:::::::::/:::::::::::::::::::::::::::::::::::'、',::::'、  /:\__/‐、
  |      |/:::::::::::/::::::::::::::::::::::::::::::::::O::| '、::| く::::::::::::: ̄|
   |     /_..-'´ ̄`ー-、:::::::::::::::::::::::::::::::::::|/:/`‐'::\;;;;;;;_|
   |    |/::::::::::::::::::::::\:::::::::::::::::::::::::::::|::/::::|::::/:::::::::::/
    |   /:::::::::::::::::::::::::::::::::|:::::::::::::::::::::O::|::|::::::|:::::::::::::::/
416132人目の素数さん:2012/09/20(木) 01:13:57.32
〔補題〕
= (a-b)(b-c)(c-a) とおくとき
 |處 ≦ (2/√27)(s^2 -3t)^(3/2),
ここに、s=a+b+c, t=ab+bc+ca, u=abc.

(略証1)
 bはaとcの中間にあるとしてよい。|a-b| =x, |b-c|=y とおく。
 s^2 -3t = {x^2 + y^2 + (x+y)^2}/2 ≧ (3/4)(x+y)^2,
 |處 = xy(x+y) ≦ (1/4)(x+y)^3 ≦ (2/√27)(s^2 -3t)^(3/2)

(略証2)
 27竸2 = 4(s^2 -3t)^3 - (2s^3 -9st +27u)^2,
 楕円曲線論に出てくる公式。
417132人目の素数さん:2012/09/20(木) 06:06:33.19
割り込んですいませんが、「不等式」の本できました。
九大の学会の書籍展示で先行販売していますので、ちょと眺めてみて下さい。
418132人目の素数さん:2012/09/20(木) 07:02:43.22

              l三`ー 、_;:;:;:;:;:;:j;:;:;:;:;:;:_;:;:;_;:-三三三三三l
               l三  r=ミ''‐--‐';二,_ ̄    ,三三三彡彡l_
              lミ′   ̄    ー-'"    '=ミニ彡彡/‐、ヽ
                  l;l  ,_-‐ 、    __,,.. - 、       彡彡彳、.//  
_______∧,、_‖ `之ヽ、, i l´ _,ィ辷ァ-、、   彡彡'r ノ/_ ______
 ̄ ̄ ̄ ̄ ̄ ̄ ̄'`'` ̄ 1     ̄フ/l l::. ヽこ~ ̄     彡彳~´/  ̄ ̄ ̄ ̄ ̄ ̄
                 ヽ   ´ :l .l:::.         彡ィ-‐'′
                ゝ、  / :.  :r-、        彡′
              / ィ:ヘ  `ヽ:__,ィ='´        彡;ヽ、
          _,,..-‐'7 /:::::::ヽ   _: :_    ヽ      ィ´.}::ヽ ヽ、
      _,-‐'´    {  ヽ:::::::::ヘ `'ー===ー-- '   /ノ /::::::ヘ, ヽー、
419132人目の素数さん:2012/09/20(木) 15:26:12.25
不等式よりABC予想の時代
420132人目の素数さん:2012/09/20(木) 22:51:02.85
知恵袋での拾いもん、大したもんじゃないかもしれんが

正の数a,b,c,dが1/a+1/b+1/c+1/d=1を満たすとき、ab+bc+cd+da+ac+bd≧96
421132人目の素数さん:2012/09/20(木) 23:32:54.60
>>420
コレクションにあるかな〜と思って探したけど見つからなかったの巻…
422132人目の素数さん:2012/09/20(木) 23:35:06.07
>>417
http://comingbook.honzuki.jp/index.php?detail=9784903342702

前スレで予告されていた本ですね
さっそく予約してきました ( ゚∀゚) ワクワク…
423132人目の素数さん:2012/09/21(金) 00:12:33.39
>>422
情報通乙!

               _.. ..‐::´/
             _/::::::::::::/
           _/:::::::::::::/ ____
         ,..::::´::::::::::::::::::::: ̄:::::::::::._/
       /:::::::::::::::::| ヽ、:::::;::::::::::::/
       /:::::::::::::::::::::|´|ヽ   |/_:::.::/
  _ .. -─':::::::::::::::、::|`'   ,   .!::∠
  `'' ‐-.._:::::::;-‐、`(●)  (●) |::::`::-、オッス!オラ、不等式ヲタ
 =ニ二::::::::::::::::|6    \___/、| -──` まだ発売されていないってのに
    ‐=.二;;;;;`‐t    \/  ノ       なんだかすっげえワクワクしてきたぞ!
424132人目の素数さん:2012/09/21(金) 00:27:23.32
>>421
誰かしらから解答来たらそれの載ってたURLはるお
425132人目の素数さん:2012/09/21(金) 02:42:18.50
∀k a[k]≧0 と、∀k p[k]≧0 と、Σp[k]=1 を満たす実数列a[1],...,a[n]とp[1],...,p[n]が与えられたとき
y>x≧1なら Σ{p[k] * (a[k]^x)}^(1/x) ≦ Σ{p[k] * (a[k]^y)}^(1/y)
が成り立ちますか?
426132人目の素数さん:2012/09/21(金) 03:44:40.91
>>420

1 = 1/a+1/b+1/c+1/d ≧ 4・{1/(abcd)}^(1/4) より、 abcd ≧4^4
ab+bc+cd+da+ac+bd ≧ 6・{(abcd)^3}^(1/6) = 6・(abcd)^(1/2) ≧ 6・(4^4)^(1/2) = 6・4^2 = 96

     ,ィ´ ̄ ̄ ̄``ヽ
    /:::::::::::::::::::::::::::::::::::\
  厶 -…ー─‐--、:::::::::::|
∠___,ィ´ ̄ ̄ ̄`ヽ、\_}
   | <●) /、(●>、 ||||    
   |  ,, <、_,> ヽ、,   |    そうか!相乗か!
.    |   mj |=‐ァ'  .::::|   
    \,〈__ノニニ´ .:::/
   /ノ  ノ |||/一´\



427132人目の素数さん:2012/09/21(金) 03:58:08.83
正の数 a_1、…、a_n が Σ(1/a_k) = 1 をみたすとき、Σ_[k<m] (a_k)・(a_m) ≧ nC2・n^2 .

ってことになるの蟹かまぼこ? ( ゚∀゚)プケラッチョ!
428132人目の素数さん:2012/09/21(金) 04:04:10.41
基本対称式を
 S_1 = a_1 + … + a_n
 S_2 = a_1・a_2 + …
などとおくと、逆数和が一定のとき、全部この方法で分かるわけ貝? ( ゚∀゚) ホヘー
429132人目の素数さん:2012/09/21(金) 07:32:13.34
0<x<π/4のとき
∫[0→x]costdt>2∫[0→x]sintdt
430132人目の素数さん:2012/09/21(金) 07:35:56.27
>>426
http://detail.chiebukuro.yahoo.co.jp/qa/question_detail/q1194388471

これの(2)だったんだが、独立して解けたのか…
てっきり(1)が(2)の応用かと思ってた
431431:2012/09/21(金) 11:30:21.50
4-3=1
432132人目の素数さん:2012/09/21(金) 11:35:45.03
>>430
いろんな解き方があって参考になります

     ,ィ´ ̄ ̄ ̄``ヽ
    /:::::::::::::::::::::::::::::::::::\
  厶 -…ー─‐--、:::::::::::|
∠___,ィ´ ̄ ̄ ̄`ヽ、\_}
   | <●) /、(●>、 ||||    また俺っちのコレクションが増えたぜ!
   |  ,, <、_,> ヽ、,   |    ありがとな!
.    |   mj |=‐ァ'  .::::|   
    \,〈__ノニニ´ .:::/
   /ノ  ノ |||/一´\
433132人目の素数さん:2012/09/21(金) 11:51:24.70
>>425
不等号の向きが逆です
434132人目の素数さん:2012/09/21(金) 18:06:56.29
(・3・)くわちくおちえてくだちゃい
435132人目の素数さん:2012/09/21(金) 18:27:32.54
>>417
いま九州大学に行けば手に入るんですか?
436132人目の素数さん:2012/09/21(金) 20:04:33.61
>>417
いつ書店に並ぶんだYO! 風邪引いちまうぢゃないか!
437132人目の素数さん:2012/09/21(金) 20:32:39.09
>>435
落ち着けww
もう少ししたら発売されるだろww
438132人目の素数さん:2012/09/21(金) 20:41:39.03
[4] 不等式入門(数学のかんどころシリーズ9),大関清太,共立出版,2012年
   http://www.kyoritsu-pub.co.jp/bookdetail/9784320019898


これじゃない感が凄かった
439132人目の素数さん:2012/09/21(金) 20:57:02.61
>>436
落ち着け。
全裸で待機しても発売日が早まるわけじゃないから、とりあえずパンツ履け。
440132人目の素数さん:2012/09/21(金) 21:07:17.71
>>438
かんどころシリーズは、One Point双書の21世紀版を目指したらしいが、大失敗だな
素直にOne Point双書をジャンルごとに数冊ずつ纏めて1冊にして発売したほうが売れただろうに…
ゆとりっ子のレベルに合わせたって言い訳しそうだが、実際は編集部の質が落ちているだけだと思う

書店で「かんどころシリーズ」を見たことある?
数学に興味のある中学生向けの内容じゃんか!
その中でも、「かんどころH不等式」は頑張っている方だと思う

このシリーズで出しさえしなければ、前著「不等式への招待」の続編的位置付けになったろうに…
編集部にダメ出しされまくって、著者は泣く泣く証明部分や未紹介の不等式を削ったに違いない
別の出版社から、しっかりした不等式の本を出してほしいと思う吉宗であった
441132人目の素数さん:2012/09/21(金) 21:12:37.16
>>425
∀k a[k]≧0 と、∀k p[k]≧0 と、Σp[k]=1 を満たす実数列a[1],...,a[n]とp[1],...,p[n]が与えられたとき
y>x≧1なら {Σ( p[k] * (a[k]^x) )}^(1/x) ≦ {Σ( p[k] * (a[k]^y) )}^(1/y)
が成り立ちますか?

の間違いでした
>>425だとp[k]^(1/x)を比べるだけになってしまいますねすいません
証明出来ました
442132人目の素数さん:2012/09/21(金) 22:25:11.99
>>441
だから、結果の不等号の向きが逆
443132人目の素数さん:2012/09/21(金) 22:28:28.89
>>440 別の出版社から、しっかりした不等式の本を出してほしいと思う

だから、>>422で新しい不等式の本が出るではないか!
発売日は、9月24日になっている。


不等式 〜21世紀の代数的不等式論 〜
著者 安藤哲哉
出版社 数学書房
本体価格 3500円
ページ数 288p
発売予定日 2012-09-24

内容紹介
不等式を証明するには、式変形の技巧ではなく、
代数・幾何・解析学の諸理論が役に立つことを示した理論的・体系的解説書。
444132人目の素数さん:2012/09/21(金) 22:31:05.51
「不等式への招待」のシリーズの他の本はオンデマンド版で販売されているのに、「不等式の招待」だけ販売無し…
445132人目の素数さん:2012/09/21(金) 23:01:37.34
不等式で卒業論文書きたいなあ
研究するのは厳しい分野なんかな
446132人目の素数さん:2012/09/21(金) 23:07:35.10
>>443
それとは別に、大関の書いた本が読みたいんだyo!
447132人目の素数さん:2012/09/21(金) 23:10:29.90
このスレの神?が書いたテクニカル本とか
448132人目の素数さん:2012/09/21(金) 23:17:23.83
あちこちからかき集めて出題するコレクター悪魔と
それらを片っ端から証明した上に改造もする神の共著ですか?
いや神一人で必要十分ですな
449132人目の素数さん:2012/09/21(金) 23:22:25.91
>>420

 ab+bc+cd+da+ac+bd ≧ 36/(1/ab + 1/bc + 1/cd + 1/da + 1/ac + 1/bd)
          ≧ 36/{(6/16)(1/a+1/b+1/c+1/d)^2}
          = 96/(1/a+1/b+1/c+1/d)^2,
450132人目の素数さん:2012/09/21(金) 23:36:13.30
>>449
最初は相加調和平均、次は?
451132人目の素数さん:2012/09/21(金) 23:46:54.96
>>429

(左辺) = ∫[0→x] cos(t)dt = sin(x) = 2sin(x/2)cos(x/2),

(右辺) = 2∫[0→x] sin(t)dt = 2{1-cos(x)} = 4sin(x/2)^2,

題意より
 (右辺)/(左辺) = 2tan(x/2)
  ≦ 2tan(π/8)
  < 2tan(π/8)/{1-tan(π/8)^2}
  = tan(π/4)
  = 1,
452132人目の素数さん:2012/09/21(金) 23:56:20.01
453132人目の素数さん:2012/09/21(金) 23:59:18.91
>>443
> 不等式を証明するには、式変形の技巧ではなく、

ヲタは要らんということですね
454132人目の素数さん:2012/09/22(土) 00:03:27.51
式変形の技巧に ハァハァ したい年頃なんですがね
数学者に否定されると…
455132人目の素数さん:2012/09/22(土) 00:13:11.29
>>452
面識もないのにどうやって頼むんだyo!


       /  ̄≧ ̄\
     /  \ /  \     我々は不等式ヲタだ!
     |   (●   (●  |      貴殿の本気が見たいのだ!   
     |    (__人__)  |      少ないが取っておき給へ! 
     \    ` ⌒´  /   ☆
    /ヽ、--ー、__,-‐´ \─/
   / >   ヽ▼●▼<\  ||ー、
  / ヽ、   \ i |。| |/  ヽ (ニ、`ヽ
 .l   ヽ     l |。| | r-、y `ニ  ノ \
 l     |    |ー─ |  ̄ l   `~ヽ_ノ____
    / ̄ ̄ ̄ ̄ヽ-'ヽ--'  / 壱万円札 /|
   .| ̄ ̄ ̄ ̄ ̄ ̄|/|    | ̄ ̄ ̄ ̄ ̄ ̄|/| ______
/ ̄壱万円札/|  ̄|__」/_壱万円札 /| ̄|__,」___    /|
| ̄ ̄ ̄ ̄ ̄|/壱万円札 / ̄ ̄ ̄ ̄|/壱万円札 /|  / .|
| ̄ ̄ ̄ ̄ ̄| ̄ ̄ ̄ ̄ ̄|/l ̄ ̄ ̄ ̄| ̄ ̄ ̄ ̄ ̄|/ | /
| ̄ ̄ ̄ ̄ ̄ ̄ ̄ ̄ ̄ ̄ ̄ ̄ ̄ ̄ ̄ ̄ ̄ ̄ ̄ ̄ ̄ ̄ ̄|
456132人目の素数さん:2012/09/22(土) 00:17:13.95
>>445
研究って未解決や未知の問題を明らかにすることだからねえ
不等式の場合、研究すべき未解決の問題を探しにくい所だろうね。
結局、面白い問題を見つけるには、他の数学の深い理論を勉強しないと出てこないし
457132人目の素数さん:2012/09/22(土) 00:22:47.93
「不等式への招待」にも、こんな不等式は必要に迫られてでないと作り出せないとかなんとか…
458132人目の素数さん:2012/09/22(土) 00:26:36.45
>>416

3次式 X^3 -sX^2 +tX -u の根がすべて実数となる条件は

 9st -2s^3 -2(s^2 -3t)^(3/2) ≦ 27u ≦ 9st -2s^3 + 2(s^2 -3t)^(3/2),
459132人目の素数さん:2012/09/22(土) 00:32:58.94
出番やで

919cielblue [2012/09/20(木) 11:26:30]

x, y, z>0, xyz=64/27 ⇒ √(1+x^2)+√(1+y^2)+√(1+z^2)≧1+x+y+z

http://www.casphy.com/bbs/test/read.cgi/highmath/1169210077/l50
460132人目の素数さん:2012/09/22(土) 00:33:02.23
>>457
確かに…
Shapiroの不等式のように、絶妙な問題を出したShapiroは偉いですね。

不等式の場合、問題より予想の方が大事かもね。
461132人目の素数さん:2012/09/22(土) 00:37:20.93
462132人目の素数さん:2012/09/22(土) 00:38:49.75
大関の次は横綱ですね
463132人目の素数さん:2012/09/22(土) 00:39:23.79
>>459
2chに慣れているせいか、このスレを利用したことがなかったな
そういや何度かリンクが貼られていたこともあったが、ずっとヌルーしていたな ('A`)ヴォエァ!
464132人目の素数さん:2012/09/22(土) 00:42:01.76
>>461
              l三`ー 、_;:;:;:;:;:;:j;:;:;:;:;:;:_;:;:;_;:-三三三三三l
               l三  r=ミ''‐--‐';二,_ ̄    ,三三三彡彡l_
              lミ′   ̄    ー-'"    '=ミニ彡彡/‐、ヽ
                  l;l  ,_-‐ 、    __,,.. - 、       彡彡彳、.//  この感じ…、奴が帰ってきたのか!
_______∧,、_‖ `之ヽ、, i l´ _,ィ辷ァ-、、   彡彡'r ノ/_ ______
 ̄ ̄ ̄ ̄ ̄ ̄ ̄'`'` ̄ 1     ̄フ/l l::. ヽこ~ ̄     彡彳~´/  ̄ ̄ ̄ ̄ ̄ ̄
                 ヽ   ´ :l .l:::.         彡ィ-‐'′
                ゝ、  / :.  :r-、        彡′
              / ィ:ヘ  `ヽ:__,ィ='´        彡;ヽ、
          _,,..-‐'7 /:::::::ヽ   _: :_    ヽ      ィ´.}::ヽ ヽ、
      _,-‐'´    {  ヽ:::::::::ヘ `'ー===ー-- '   /ノ /::::::ヘ, ヽー、
465132人目の素数さん:2012/09/22(土) 00:46:07.27
>>461
俺は 不等式,大関信雄・青木雅計,槇書店,1967年(絶版) の方が好きだな
466132人目の素数さん:2012/09/22(土) 00:46:32.04
>>461
オンデマンド (ペーパーバック) とは何ぞや?
ハードカバーが、ふにゃちんカバーになってしまうのかな?

俺の持ってるやつは、書き込んだり消したりした上に、
読みながら寝てしまって、ヨダレとか、顔の油とかで汚れてしまっているから買い直したいな

最も洗脳効果の高かった不等式本ですな
俺の中では神!
467132人目の素数さん:2012/09/22(土) 00:50:32.91
>>461
おお!
って、持っているから特に買うつもりは無いが


>>465
俺はまだその本は見たこと無いんだ。
大関・大関本とどう違うかずっと気になっているんだが
468132人目の素数さん:2012/09/22(土) 00:52:56.32
>>465
これも神!
探しても手に入らなくって、図書館の書庫で紛失していたのを探して借りた
このために、当時3万円くらいの家庭用コピー機(紙は一枚ずつ手差し)を買って、
片面コピーしたのを裏返して差し込んで両面コピーして背表紙に木工用ボンドを塗って製本した
これも復刊してほしい!
469132人目の素数さん:2012/09/22(土) 00:57:26.76
おまいら不等式が好き過ぎだな
問題解決の道具に過ぎないものを道具の方に惚れこんでしまったのか
よいぞよいぞ・・・
470132人目の素数さん:2012/09/22(土) 01:00:09.04
>>468
不等式に賭けるその想い、素晴らしい!!

個人的には、大関本よりも系統立てて書いている渡部本が好き。
こっちは、4,5年前に復刊されたのを買ったんだが、スッキリしていていい。
471132人目の素数さん:2012/09/22(土) 01:03:18.23
>>465
1967年発売なら、もう著作権切れてないか?
472132人目の素数さん:2012/09/22(土) 01:14:01.05
>>470
POD版とやらが復刊されて暫くしてから知って、急いで本屋巡りしたけどどこにもなくて、取り寄せてもらったな
有名不等式を一通り紹介してくれているので、知ってるつもりの知識が整理できてよかった

高校で 「モノグラフ 不等式」 を何気なく買って、
大学生協で色ヤケしていた 「不等式への招待」 をたまたま買って洗脳され、
図書館書庫で発掘した 「不等式(大関・青柳)」 のコピー本を作り、
ずっと後になって、不等式(ハーディ)、不等式の工学への応用、不等式(渡部版)、不等式(かんどころ)を購入

不等式の和書って、少ないですなあ…
473132人目の素数さん:2012/09/22(土) 01:16:56.20
>>471
槙書店はWebサイトないんよな
復刊してくれんかなあ

http://www.hotfrog.jp/%E4%BC%81%E6%A5%AD/%E6%A7%99%E6%9B%B8%E5%BA%97
474132人目の素数さん:2012/09/22(土) 01:18:36.44
>>472
そこに待望の新書>>443の発売で、胸が熱い
もうアマゾンでポッチってしまった…

最近の洋書だと、
 The Cauchy-Schwarz Master Class:
 J. M. Steele,Cambridge Univ. Pr.,2004年
が中々良かったです。
475132人目の素数さん:2012/09/22(土) 01:24:16.62
検索していたら、こんなのを見つけた

復刊ドットコム『不等式(大関信雄・青柳雅計)』 復刊リクエスト投票
http://www.fukkan.com/fk/VoteDetail?no=27792
476132人目の素数さん:2012/09/22(土) 01:33:37.07
>>474
著者の書き込み(>>417)の翌日に予約しました
この不等式ヲタに抜かりはない ( ゚∀゚)プケラッチョ!

The Cauchy-Schwarz Master Class は、何スレか前に紹介されたときに購入しました
本棚に飾ったまま、さっきまで買ったことすら忘れていましたが…

  ,、|,、
 (f⌒i
  U j.|
  UJ
   :
  ‐=‐
477132人目の素数さん:2012/09/22(土) 01:39:12.79
>>469
道具を収集して楽しむなんて、貴族のような人たちですね。
478132人目の素数さん:2012/09/22(土) 01:55:57.70
>>469
すでに>1で完結している

> ある人は蝶を集め、ある人は切手を収集し、ある人は不等式を集める…
>           ___          ----- 参考文献〔3〕 P.65 -----
>     |┃三 ./  ≧ \   
>     |┃   |::::  \ ./ | 
>     |┃ ≡|::::: (● (● |  不等式と聞ゐちゃぁ
> ____.|ミ\_ヽ::::... .ワ......ノ     黙っちゃゐられねゑ!
>     |┃=__    \           ハァハァ…
>     |┃ ≡ )  人 \ ガラッ
479132人目の素数さん:2012/09/22(土) 02:18:27.30
未だにムーアヘッドやマジョライセーション不等式が分からないまま放置プレイしている
480132人目の素数さん:2012/09/22(土) 02:23:57.80
>>477
職人にとって道具は命
481132人目の素数さん:2012/09/22(土) 02:28:27.33
>>462
不覚にも笑った
482132人目の素数さん:2012/09/22(土) 02:29:07.54
埋まっているので再掲

>>459
> 出番やで
>
> 919cielblue [2012/09/20(木) 11:26:30]
>
> x, y, z>0, xyz=64/27 ⇒ √(1+x^2)+√(1+y^2)+√(1+z^2)≧1+x+y+z
>
> http://www.casphy.com/bbs/test/read.cgi/highmath/1169210077/l50
483132人目の素数さん:2012/09/22(土) 02:31:16.64
かんどころで紹介されていた大関がハァハァするほどの洋書が、地元の図書館になかったでござる
484132人目の素数さん:2012/09/22(土) 02:33:39.44
ここは不等式に飢えた狼たちのすくつでつね
485132人目の素数さん:2012/09/22(土) 02:35:55.56
行列(固有値)の不等式も中々面白いぞ

Matrix Inequalities (Lecture Notes in Mathematics,No.1790)
Xingzhi Zhan (著)
ペーパーバック: 132ページ
出版社: Springer; 2版 (2002/10)
http://www.amazon.co.jp/Matrix-Analysis-Graduate-Texts-Mathematics/dp/0387948465/ref=sr_1_10?ie=UTF8&qid=1348248699&sr=8-10

Matrix Analysis (Graduate Texts in Mathematics)
Rajendra Bhatia (著)
ハードカバー: 347ページ
出版社: Springer; 1版 (1996/12)
http://www.amazon.co.jp/Matrix-Analysis-Graduate-Texts-Mathematics/dp/0387948465/ref=sr_1_10?ie=UTF8&qid=1348248699&sr=8-10
486132人目の素数さん:2012/09/22(土) 02:48:58.13
Ky Fan の不等式について(大関信雄) 1973
http://repository.kulib.kyoto-u.ac.jp/dspace/bitstream/2433/107253/1/0191-5.pdf
487132人目の素数さん:2012/09/22(土) 02:53:01.98
>>485
まとめサイトの参考文献の洋書に入れとくか
こんなときに、キバヤシに教わったAmazonのURL短縮法が役立つとは思わなかった

http://www.amazon.co.jp/…/dp/(10桁の数字)/…
…はノイズなので削除する
www.も削除してもいい

  ヽ、.三 ミニ、_ ___ _,. ‐'´//-─=====-、ヾ       /ヽ
        ,.‐'´ `''‐- 、._ヽ   /.i ∠,. -─;==:- 、ゝ‐;----// ヾ.、
       [ |、!  /' ̄r'bゝ}二. {`´ '´__ (_Y_),. |.r-'‐┬‐l l⌒ | }
        ゙l |`} ..:ヽ--゙‐´リ ̄ヽd、 ''''   ̄ ̄  |l   !ニ! !⌒ //
.         i.! l .:::::     ソ;;:..  ヽ、._     _,ノ'     ゞ)ノ./
         ` ー==--‐'´(__,.   ..、  ̄ ̄ ̄      i/‐'/
          i       .:::ト、  ̄ ´            l、_/::|
          !                           |:    |
             ヽ     ー‐==:ニニニ⊃          !::   ト、
おれたちはとんでもない思い違いをしていたようだ。これを見てみろ。
まず「アマゾン」を英字で表記すると 『amazon』
洋書はamazonで注文すればokだから、語尾にokとつける 『amazonok』
これを逆にすると、『konozama』
そしてこれを更に日本語に直すと 『コノザマ』
注文しても発売日に発送すらされないことが多いことを考えれば…
488132人目の素数さん:2012/09/22(土) 02:55:33.65
>>486
達筆すぎて読めねーよ!
489132人目の素数さん:2012/09/22(土) 04:47:46.27
この人がよく不等式出題してる
http://suseum.jp/gd/all_berry_list/3504
490132人目の素数さん:2012/09/22(土) 05:50:29.91
>>489
   ___
 ./  ≧ \
 |::::  \ ./ |
 |::::: (● (● | グッジョブ!
 ヽ::::... .ワ....ノ    n  
 ̄ ̄   \    ( E)
フ     /ヽ ヽ_//
491132人目の素数さん:2012/09/22(土) 05:53:15.63
燃料投下(>>422>>461)で、プチ祭り状態だな
492132人目の素数さん:2012/09/22(土) 09:17:30.66

 
   ___     食欲の秋
 ./  ≧ \ 
 |::::  \ ./ |     不等式の秋    
 |::::: (● (● | 
 ヽ::::... .ワ.....ノ
   ( つ旦O    ,.-、    ,.-、   ,.-、
   と_)_)    (,,■)  (,,■)  (,,■)
493132人目の素数さん:2012/09/22(土) 09:43:07.86
>>483
どの本?
494132人目の素数さん:2012/09/22(土) 11:35:48.97
>>485
洋書を追加

Inequalities: Theorems, Techniques and Selected Problems,
Zdravko Cvetkovski
Springer-Verlag; 2012
http://www.amazon.co.jp/gp/product/3642237916/

Analytic Inequalities,
Dragoslav S., Dr. Mitrinovic
Springer-Verlag;1970
http://www.amazon.co.jp/dp/3642999727/
495132人目の素数さん:2012/09/22(土) 13:12:07.58
   / ノ ≧ ヽ
  _|_____|_    わたしが訓練教官のハートマン先任軍曹であるッ!
   |:::  \ ./ |       話しかけられたとき以外は口を開くな。
   |:::: (● (●|       口でクソたれる前と後に『Sir』と言え!!
   ヽ::::......ワ...ノ        分かったか、ウジ虫どもー!
      人つゝ 人,,             ----- sir, yes sir !
    Yノ人 ノ ノノゞ⌒〜ゞ   ふざけんな! もっと大声をだせー! 
.  ノ /ミ|\、    ノノ ( 彡       ----- SIR, YES SIR !!!
   `⌒  .U~U`ヾ    丿   よーし、それでいい
           ⌒〜⌒      
>>494
追加しますた

>>487
つまり、こういうことですな
http://www.amazon.co.jp/(書籍名・著者名情報)/dp/(10桁の数字)/(アフェリエイトなどのパラメータ)
 ↓
http://.amazon.jp/dp/(10桁の数字)

>>489
実によく訓練された不等式ヲタだ
不等式の埋蔵地のリンクに追加しておこう!

>>493
貴様、それでも不等式ヲタか! とりあえず買って本棚に並べておきたまえ!
http://www.amazon.com/dp/0387048375


     _  ())二) )) 、,r:ニヽ  いいぞ ベイべー!
 @ニ===)二二ニニ)('A` ))  不等式を収集し証明する奴は 数ヲタだ!!
     ^ ̄" フ\''|ノ=ノ-(  )   不等式を改造し拡張する奴は よく訓練された数ヲタだ!!
         _/  \_   L L   ホント不等式はハァハァするぜ! フゥハハハーハァー
496132人目の素数さん:2012/09/22(土) 13:16:54.72
>>493
>>494の2冊目じゃないの?
497132人目の素数さん:2012/09/22(土) 14:15:34.69
理系への数学10月号を立読みしてたら 「対称同次平均」 とかいうのがあったけど、検索しても見つからんな
も一回書店に確認に行ったら、売り切れていたでござる
498132人目の素数さん:2012/09/22(土) 17:39:13.64
                  _______
         /      ∩|   |
        /      / /|不等式
           /  / /  |____|
              / /     | |
  .        / / /∧   ./ /
            / / ´_ゝ`)/  おうい!安藤の新刊書を手に入れたぞー!
          / |      /
            |    /
            |   /⌒l
             ヽ   | /
           / | ゙ー'| L
        /     |  /(_  ヽ
         / / ノ
       /  / /
     /   ( ヽ  
499132人目の素数さん:2012/09/22(土) 17:59:01.60
>>498
今度は投げんなよ!
500132人目の素数さん:2012/09/22(土) 23:58:14.17
                  _______
         /      ∩|   |
        /      / /|不等式
           /  / /  |____|
              / /     | |
  .        / / /∧   ./ /
            / / ´_ゝ`)/  おうい!大関の復刊書を手に入れたぞー!
          / |      /
            |    /
            |   /⌒l
             ヽ   | /
           / | ゙ー'| L
        /     |  /(_  ヽ
         / / ノ
       /  / /
     /   ( ヽ  
501132人目の素数さん:2012/09/23(日) 00:28:49.57
不等式教団がジワジワと勢力を拡げているな
502132人目の素数さん:2012/09/23(日) 02:29:54.22
503132人目の素数さん:2012/09/23(日) 12:31:09.29
                  ___________
         /      ∩|     |
        /      / /|不等式|
           /  / / ..|_________.|
              / /     | |
          / / /∧   ./ /
            / / ´_ゝ`)/  ヒャッハー!
          / |      /   大関・青柳の絶版書を借りてきたぞ〜!
            |    /
            |   /⌒l
             ヽ   | /
           / | ゙ー'| L
        /     |  /(____ ヽ
         / / ノ
       /  / /
     /   ( ヽ
504132人目の素数さん:2012/09/23(日) 13:25:40.76
>>494
> Analytic Inequalities,
> Dragoslav S., Dr. Mitrinovic
> Springer-Verlag;1970
> http://www.amazon.co.jp/dp/3642999727/

Amazonでポチッてみました、あポチッとな!
これが、大関が毎晩ハァハァしているやつなんですね
505132人目の素数さん:2012/09/23(日) 13:45:24.57
アマゾンで洋書買うにはどうしたらいいですか?
506132人目の素数さん:2012/09/23(日) 14:04:56.31
>>486
大関氏の実筆にハァハァ
507132人目の素数さん:2012/09/23(日) 14:27:22.21
>>505
上の洋書みたいに、日本に発送可なら、そのまま注文するのだ!

海外に発送してくれると書いてあっても、購入手続き中に日本は無理って表示が出る場合がある
(英語圏じゃないところには、面倒がって発送してくれないところがあるのだ)
そんなときは、代行業者に依頼して注文すればよい

洋書じゃないが、STAR TREKの模型を買いまくった時期があるが、個人が出品しているものを
代行業者に依頼して落札してもらって、送ってもらった事がある(安いプラモだったので関税0円)
代行業者にはいい加減なところが多いらしいが、俺が頼んだところはいいところだったぜ ( ゚∀゚)プケラッチョ!
508132人目の素数さん:2012/09/23(日) 14:38:41.02
>>505
amazon.co.jp で買えば、通常の和書と同じ

amazon.comだと、アメリカからだから、送料が取られるが、
円高なのでそれでも安い場合がある。
509132人目の素数さん:2012/09/23(日) 14:48:01.72
510132人目の素数さん:2012/09/23(日) 15:01:07.26
>>507>>508
ありがとうございました(´ω`)
511132人目の素数さん:2012/09/23(日) 15:05:02.88
ちゃんと勉強してください
http://up3.viploader.net/pic/src/viploader1242778.jpg
512132人目の素数さん:2012/09/24(月) 10:45:58.09
Wirtinger 型不等式に関する一考察 (高橋・三浦) 2002
http://www.kurims.kyoto-u.ac.jp/~kyodo/kokyuroku/contents/pdf/1253-14.pdf
513132人目の素数さん:2012/09/24(月) 21:45:52.72
>>505
英語がわからないの?

普通に買ってるぞ
514132人目の素数さん:2012/09/24(月) 21:46:28.40
>>512
  ∧_∧
  ( ;´∀`) < おっきしました
  人 Y /
 ( ヽ し
 (_)_)
515132人目の素数さん:2012/09/24(月) 21:57:50.11
>>512
  ___
./  ≧ \
|::::  \ ./ |
|::::: (● (● | グッジョブ!
ヽ::::... .ワ....ノ    n  
⌒`γ´⌒`ヽ( E)
( .人 .人 γ ノ
ミ(こノこノ `ー´
)にノこ(
516132人目の素数さん:2012/09/24(月) 22:09:24.99
517132人目の素数さん:2012/09/24(月) 22:17:55.47
久々に来たら、祭り状態になって興奮したハァハァ
518132人目の素数さん:2012/09/24(月) 22:29:14.68
>>516
発売予定日が早まっていたのは知っていたが、土曜日に手に入ったのはラッキーだった
週末にジックリ…


…読むつもりだったが、雑事が入って碌に睡眠もできていない
で、まだ袋から出してもいないのだ
519132人目の素数さん:2012/09/25(火) 15:36:19.68
>>518
使えねー野郎だな
読んだやつはいないのか?
520132人目の素数さん:2012/09/27(木) 15:07:46.36
                  ___________
         /      ∩|     |
        /      / /|不等式|
           /  / / ..|_________.|
              / /     | |
          / / /∧   ./ /
            / / ´_ゝ`)/  ヒャッハー!
          / |      /   洋書を手に入れたぞ〜!
            |    /
            |   /⌒l ←>>504
             ヽ   | /
           / | ゙ー'| L
        /     |  /(____ ヽ
         / / ノ
       /  / /
     /   ( ヽ
521132人目の素数さん:2012/09/27(木) 18:58:11.99
巡回セールスマン問題やナップサック問題とかは不等式に入る?
522132人目の素数さん:2012/09/27(木) 21:29:54.70
     /: : : : : __: :/: : ::/: : ://: : :/l::|: : :i: :l: : :ヽ: : :丶: : 丶ヾ    ___
     /;,, : : : //::/: : 7l,;:≠-::/: : / .l::|: : :l: :|;,,;!: : :!l: : :i: : : :|: : ::、  /     ヽ
    /ヽヽ: ://: :!:,X~::|: /;,,;,/: :/  リ!: ::/ノ  l`ヽl !: : |: : : :l: :l: リ / そ そ お \
   /: : ヽヾ/: : l/::l |/|||llllヾ,、  / |: :/ , -==、 l\:::|: : : :|i: | /   う う  前  |
.   /: : : //ヾ ; :|!: イ、||ll|||||::||    ノノ  イ|||||||ヾ、 |: ::|!: : イ: ::|/   な 思 が
   /: : ://: : :ヽソ::ヽl |{ i||ll"ン    ´   i| l|||l"l `|: /|: : /'!/l     ん う
 ∠: : : ~: : : : : : : :丶ゝ-―-      ,  ー=z_ソ   |/ ハメ;, :: ::|.   だ ん
   i|::ハ: : : : : : : : : : : 、ヘヘヘヘ     、  ヘヘヘヘヘ /: : : : : \,|.   ろ な
   |!l |: : : : : : : : :、: ::\    、-―-,      / : : :丶;,,;,:ミヽ   う  ら
     丶: :ハ、lヽ: :ヽ: : ::\__  `~ "      /: : ト; lヽ)   ゝ
       レ `| `、l`、>=ニ´        ,  _´ : :} `   /
         ,,、r"^~´"''''"t-`r、 _  -、 ´ヽノ \ノ   /    お ・
       ,;'~  _r-- 、__     ~f、_>'、_         |  で  前 ・
      f~  ,;"     ~"t___    ミ、 ^'t         |  は  ん ・
      ,"  ,~         ヾ~'-、__ ミ_ξ丶     |  な  中 ・
     ;'  ,イ ..          ヽ_   ヾ、0ヽ丶    l         /
     ( ;":: |: :: ..          .`,   ヾ 丶 !    \____/
     ;;;; :: 入:: :: ::      l`ー-、   )l   ヾ 丶
     "~、ソ:: :い:: :     \_  ノ ,    ヾ 丶
523132人目の素数さん:2012/09/28(金) 20:34:32.89
不等式祭りも終わったようだな…








さみしい...
524132人目の素数さん:2012/09/29(土) 11:23:31.69
(a^2 + b^2 + c^2 + d^2 + e^2)*cos(pi/5) ≧ ab + bc + cd + de - ea
525132人目の素数さん:2012/09/29(土) 12:13:30.46
>>523
俺の頭の中では年中祭りなんだが… ( ゚∀゚)
526132人目の素数さん:2012/09/29(土) 12:21:08.27
>>524
なんじゃこりゃ!
527132人目の素数さん:2012/09/29(土) 13:36:29.33
a, b, c≧1, a+b+c+2=abc のとき,

bc√(a^2-1)+ca√(b^2-1)+ab√(c^2-1)≦[(3√3)/2]abc
528132人目の素数さん:2012/09/29(土) 15:35:01.41
>>524 Please edit the problem.
529132人目の素数さん:2012/09/29(土) 21:59:18.18
21世紀の代数的不等式論、get, 一通り読みながした。かなり、専門的な内容のようだ。
全部、理解しているわけではないが、ここに出てくる定理は、何度か、目にしたことがある。
このような定理を使わずして、本当に解けないものなのだろうか。おそらく、そうなのだろう。
そうであっても、不等式の証明は、悪なきロマンの追求だなあ。やめられない。
530132人目の素数さん:2012/10/01(月) 07:51:31.92
>>524
http://www.casphy.com/bbs/test/read.cgi/highmath/1169210077/l50

投稿してるのはいつもの方だと思うけど
連投は違うサイトでは極力控えた方がいいと思うよ
531132人目の素数さん:2012/10/01(月) 12:29:51.95
0≦a, b, c, d≦1⇒

(a+b-ab)(b+c-bc)(c+d-cd)(d+a-da)≦(ac+bd-abcd)^2
532132人目の素数さん:2012/10/01(月) 20:03:21.64
(*゚∀゚)=3 ハァハァ…
533132人目の素数さん:2012/10/01(月) 20:16:03.84
524だが、そのリンク先は俺じゃない
534132人目の素数さん:2012/10/03(水) 04:42:25.27
>>529
大雑把に理解するには、どのくらいの知識量が必要ですか?
535132人目の素数さん:2012/10/03(水) 10:02:28.21
>>534
代数的不等式論ってあるように、代数の知識(有限群論や置換、不変式論)あたりかな。
でも有限の世界だから、不等式自体を証明するには、式変形でほぼ証明できそう。

積分に関する不等式が無いのが物足りない。
こちらは、洋書しかないか>>494
536132人目の素数さん:2012/10/03(水) 10:21:26.51
そういや昔は英語が苦手なので洋書なんて…と思っていたが、
数式を見れば大体分かるので洋書でも気にならなくなった。
でも数式の殆どない洋書は無理だわ
537132人目の素数さん:2012/10/03(水) 11:52:37.31
>>536
洋書の場合、序文が一番難しかったりするからね
数式の殆ど無い洋書は確かに辛いが、和書でもそうなんじゃないかな?

難しい物は洋書だろうが和書だろうが難しい。
言語の問題ではないだろう。
538132人目の素数さん:2012/10/03(水) 17:55:39.85
>>535
この本はこれでいいと思う。望みどおりじゃないか。
似たような本が出てもしょうがないし…
539132人目の素数さん:2012/10/04(木) 20:09:12.11
r≧0, 0≦x,y,z<1 のとき
 (x^r)/(1-yz) + (y^r)/(1-zx) + (z^r)/(1-xy) ≦ (x^r)/(1-x^2) + (y^r)/(1-y^2) + (z^r)/(1-z^2)
540132人目の素数さん:2012/10/04(木) 22:55:37.64
>>539

相加・調和平均で
 1/(1-yz) = 2/(2-2yz) ≦ 2/(2-y^2 -z^2) ≦ {1/(1-y^2) + 1/(1-z^2)}/2,

分子は正かつ単調増加だから、チェビシェフで良さげ・・・
541132人目の素数さん:2012/10/05(金) 22:00:31.44
不等式〜21世紀の代数的不等式論〜を買った

でも難しすぎて全然分からない…涙目
542132人目の素数さん:2012/10/05(金) 22:10:44.30
どんな不等式が載ってました?
543132人目の素数さん:2012/10/05(金) 23:14:31.93
abc 予想
544132人目の素数さん:2012/10/05(金) 23:52:06.88
>>542
不等式──21世紀の代数的不等式論
http://www.sugakushobo.co.jp/903342_70_mae.html

第 l 章 基本的な不等式

1.1 AM-GM不等式
   1.1.1 AM-GM不等式
   1.1.2 Maclaurinの不等式
   1.1.3 Newtonの不等式
1.2 Jensenの不等式
   1.2.1 凸関数
   1.2.2 Jensenの不等式
   1.2.3 重み付きAM-GM不等式
   1.2.4 Karamataの不等式
   1.2.5 Muirheadの不等式
1.3 他の有名な不等式
   1.3.1 Cauchyの不等式
   1.3.2 Chebyshevの不等式
   1.3.3 並べ替え不等式
   1.3.4 Holderの不等式
   1.3.5 Power Mean不等式
   1.3.6 Minkovskiの不等式
   1.3.7 Bernoulliの不等式
545132人目の素数さん:2012/10/05(金) 23:53:02.22
第 2 章 3変数斉次巡回多項式型不等式
2.1 ウォーミングアップ
   2.1.1 対称式と巡回式
   2.1.2 対称不等式
   2.1.3 巡回不等式
   2.1.4 5次以上の対称・巡回不等式
   2.1.5 一般の多項式型不等式
   2.1.6 三角形型斉次対称・巡回不等式
2.2 3次斉次不等式
   2.2.1 基本対称式
   2.2.2 凸錐
   2.2.3 3次巡回不等式の基本定理
2.3 4次斉次不等式
   2.3.1 4次対称不等式の基本定理
   2.3.2 4次巡回不等式の基本定理
   2.3.3 基本定理の応用
2.4 5次斉次不等式
   2.4.1 5次対称不等式の基本定理
   2.4.2 5次巡回不等式
2.5 6次斉次不等式
   2.5.1 6次対称不等式の基本定理
   2.5.2 例題
   2.5.3 6次巡回不等式詳論

第 3 章 3変数有理・無理不等式
3.1 有理式型不等式
   3.1.1 一般論
   3.1.2 分母が1次の斉次巡回有理不等式
   3.1.3 分母が1次の一般有理不等式
   3.1.4 分母が2次の斉次巡回有理不等式
   3.1.5 分母が2次の一般有理不等式
   3.1.6 分母が3次以上の有理不等式
546132人目の素数さん:2012/10/05(金) 23:53:34.17
3.2 無理不等式
   3.2.1 一般論
   3.2.2 斉次巡回無理不等式
   3.2.3 一般無理不等式
3.3 制約条件下の不等式
   3.3.1 一般論
   3.3.2 多様体上の不等式
   3.3.3 領域上の不等式
   3.3.4 無理不等式

第 4 章 凸解析
4.1 基本定理
   4.1.1 Hungの定理
   4.1.2 RCF-定理
   4.1.3 Popoviciu-Cirtoajeの不等式
   4.1.4 EV-定理
   4.1.5 UMV-定理
4.2 例題

第 5 章 巡回和
5.1 やさしい巡回不等式
   5.1.1 Shapiro型の巡回不等式
5.2 Shapiroの巡回不等式
   5.2.1 基本定理
   5.2.2 指数とセグメント
   5.2.3 Bushellの定理
   5.2.4 Bushell-McLeadのセグメント定理
   5.2.5 短いセグメント
   5.2.6 命題P_12とP_23の証明
   5.2.7 n=14, n=25 の場合の考察
   5.2.8 Drinfel'dの定理
547132人目の素数さん:2012/10/05(金) 23:59:31.77
ヲタ向けでなく、不等式の最先端の研究の専門書


不等式の代数的理論は,Muirheadの不等式が 1902年に発見されて以来,
数十年間めぼしい進展がなかった.
そのため,多くの数学者からは,代数 的不等式論は「終わった数学」
のような扱いを受けてきて,あまり省みら れることがなかった.

しかし,1990年ころから,多数の新しい定理が次々 に発見され始めた.

不等式は高校数学の延長のようなものだから,式変形のテクニックさえ
達者であれば,何とか証明できるだろう,などと考えるのは,ちょっと時代錯誤である.

20世紀半ばまでに証明できなかった不等式は,式変形の技巧だけでは,なかなか証明できない.
本格的な理論や道具を工夫することによって,はじ めて証明できる不等式も多いのである.
本書では,従来の不等式の本より, 体系的理論の説明が多く,式変形の技巧による
不等式の証明ではなく,代 数幾何や凸解析を利用した間接的な証明が多く登場する.
548132人目の素数さん:2012/10/06(土) 00:02:25.84

  俺のテクニックは時代遅れだったのか…

           , /    ,
        ,   / /   ,   /  ,
          / '^メ-' ─/- 、   / ,
       ∠r  _,゛_ /  , ヽ/__/  モウ ダメダ…
        ''ヽ'_・.ノ` ' r/、 ヘ /‐’
       ./ " j 厂゙j | レ_`> j__ /
        '  .:‘::'ニ‘.:‐'´─゙.:´一’ 
      ::::::::::::::::::::::::::::::::::::::::::::::::::::::::::::::::
549549:2012/10/06(土) 00:45:55.51
5+4=9
550550:2012/10/06(土) 01:41:53.85
5-5=0
551132人目の素数さん:2012/10/06(土) 02:16:20.43
          __ノ)-'´ ̄ ̄`ー- 、_
        , '´  _. -‐'''"二ニニ=-`ヽ、
      /   /:::::; -‐''"        `ーノ
     /   /:::::/           \
     /    /::::::/          | | |  |
     |   |:::::/ /     |  | | | |  |
      |   |::/ / / |  | ||  | | ,ハ .| ,ハ|
      |   |/ / / /| ,ハノ| /|ノレ,ニ|ル' 
     |   |  | / / レ',二、レ′ ,ィイ|゙/   私は只の数ヲタなんかとは付き合わないわ。
.     |   \ ∠イ  ,イイ|    ,`-' |      頭が良くて数学が出来てかっこいい人。それが必要条件よ。
     |     l^,人|  ` `-'     ゝ  |        さらに Ann.of Math に論文書けば十分条件にもなるわよ。
      |      ` -'\       ー'  人          一番嫌いなのは論文数を増やすためにくだらない論文を書いて
    |        /(l     __/  ヽ、           良い論文の出版を遅らせるお馬鹿な人。
     |       (:::::`‐-、__  |::::`、     ヒニニヽ、         あなたの論文が Ann of Math に accept される確率は?
    |      / `‐-、::::::::::`‐-、::::\   /,ニニ、\            それとも最近は Inv. Math. の方が上かしら?
   |      |::::::::::::::::::|` -、:::::::,ヘ ̄|'、  ヒニ二、 \
.   |      /::::::::::::::::::|::::::::\/:::O`、::\   | '、   \
   |      /:::::::::::::::::::/:::::::::::::::::::::::::::::'、::::\ノ  ヽ、  |
  |      |:::::/:::::::::/:::::::::::::::::::::::::::::::::::'、',::::'、  /:\__/‐、
  |      |/:::::::::::/::::::::::::::::::::::::::::::::::O::| '、::| く::::::::::::: ̄|
   |     /_..-'´ ̄`ー-、:::::::::::::::::::::::::::::::::::|/:/`‐'::\;;;;;;;_|
   |    |/::::::::::::::::::::::\:::::::::::::::::::::::::::::|::/::::|::::/:::::::::::/
    |   /:::::::::::::::::::::::::::::::::|:::::::::::::::::::::O::|::|::::::|:::::::::::::::/
552132人目の素数さん:2012/10/06(土) 02:17:21.84
          __ノ)-'´ ̄ ̄`ー- 、_
        , '´  _. -‐'''"二ニニ=-`ヽ、
      /   /:::::; -‐''"        `ーノ
     /   /:::::/           \
     /    /::::::/          | | |  |
     |   |:::::/ /     |  | | | |  |
      |   |::/ / / |  | ||  | | ,ハ .| ,ハ|
      |   |/ / / /| ,ハノ| /|ノレ,ニ|ル' 
     |   |  | / / レ',二、レ′ ,ィイ|゙/   私は只の数ヲタなんかとは付き合わないわ。
.     |   \ ∠イ  ,イイ|    ,`-' |      頭が良くて数学が出来てかっこいい人。それが必要条件よ。
     |     l^,人|  ` `-'     ゝ  |        さらに Ann.of Math に論文書けば十分条件にもなるわよ。
      |      ` -'\       ー'  人          一番嫌いなのは論文数を増やすためにくだらない論文を書いて
    |        /(l     __/  ヽ、           良い論文の出版を遅らせるお馬鹿な人。
     |       (:::::`‐-、__  |::::`、     ヒニニヽ、         あなたの論文が Ann of Math に accept される確率は?
    |      / `‐-、::::::::::`‐-、::::\   /,ニニ、\            それとも最近は Inv. Math. の方が上かしら?
   |      |::::::::::::::::::|` -、:::::::,ヘ ̄|'、  ヒニ二、 \
.   |      /::::::::::::::::::|::::::::\/:::O`、::\   | '、   \
   |      /:::::::::::::::::::/:::::::::::::::::::::::::::::'、::::\ノ  ヽ、  |
  |      |:::::/:::::::::/:::::::::::::::::::::::::::::::::::'、',::::'、  /:\__/‐、
  |      |/:::::::::::/::::::::::::::::::::::::::::::::::O::| '、::| く::::::::::::: ̄|
   |     /_..-'´ ̄`ー-、:::::::::::::::::::::::::::::::::::|/:/`‐'::\;;;;;;;_|
   |    |/::::::::::::::::::::::\:::::::::::::::::::::::::::::|::/::::|::::/:::::::::::/
    |   /:::::::::::::::::::::::::::::::::|:::::::::::::::::::::O::|::|::::::|:::::::::::::::/
553132人目の素数さん:2012/10/06(土) 02:21:50.69
Jack Steinberger
554132人目の素数さん:2012/10/06(土) 02:26:48.22
第4章のこれらの定理は,2005, 6年あたりから, 世界的には知られている
定理です. でも, あといくつかの定理がこの本では触れられていませんね。
このようなことが影響しているのでしょうか。最近, IMOで不等式の出題が影を
ひそめていました。今年,出題されましたけれども。この問題は,完答しなければ
IMOでは, 戦えないですね。
555132人目の素数さん:2012/10/06(土) 02:27:21.18
          __ノ)-'´ ̄ ̄`ー- 、_
        , '´  _. -‐'''"二ニニ=-`ヽ、
      /   /:::::; -‐''"        `ーノ
     /   /:::::/           \
     /    /::::::/          | | |  |
     |   |:::::/ /     |  | | | |  |
      |   |::/ / / |  | ||  | | ,ハ .| ,ハ|
      |   |/ / / /| ,ハノ| /|ノレ,ニ|ル' 
     |   |  | / / レ',二、レ′ ,ィイ|゙/   私は只の数ヲタなんかとは付き合わないわ。
.     |   \ ∠イ  ,イイ|    ,`-' |      頭が良くて数学が出来てかっこいい人。それが必要条件よ。
     |     l^,人|  ` `-'     ゝ  |        さらに Ann.of Math に論文書けば十分条件にもなるわよ。
      |      ` -'\       ー'  人          一番嫌いなのは論文数を増やすためにくだらない論文を書いて
    |        /(l     __/  ヽ、           良い論文の出版を遅らせるお馬鹿な人。
     |       (:::::`‐-、__  |::::`、     ヒニニヽ、         あなたの論文が Ann of Math に accept される確率は?
    |      / `‐-、::::::::::`‐-、::::\   /,ニニ、\            それとも最近は Inv. Math. の方が上かしら?
   |      |::::::::::::::::::|` -、:::::::,ヘ ̄|'、  ヒニ二、 \
.   |      /::::::::::::::::::|::::::::\/:::O`、::\   | '、   \
   |      /:::::::::::::::::::/:::::::::::::::::::::::::::::'、::::\ノ  ヽ、  |
  |      |:::::/:::::::::/:::::::::::::::::::::::::::::::::::'、',::::'、  /:\__/‐、
  |      |/:::::::::::/::::::::::::::::::::::::::::::::::O::| '、::| く::::::::::::: ̄|
   |     /_..-'´ ̄`ー-、:::::::::::::::::::::::::::::::::::|/:/`‐'::\;;;;;;;_|
   |    |/::::::::::::::::::::::\:::::::::::::::::::::::::::::|::/::::|::::/:::::::::::/
    |   /:::::::::::::::::::::::::::::::::|:::::::::::::::::::::O::|::|::::::|:::::::::::::::/
556132人目の素数さん:2012/10/06(土) 02:28:29.76
 。+ +。。。。。 |l|FFFFFFF|。 。 .。 +|l≡≡≡|ミ|EEEEEEEEEEE|lll| .。+
  * o  o.   |l|FFFFFFF / ̄ ̄ ̄ ̄ ̄ ̄ ̄ ̄ ̄ ̄ ̄ ̄ ̄ ̄ ̄ ̄ ̄ ̄ ̄ ̄
     。。oo  |l|FFFFFF | 今まで俺たちのやっていたことは何だったんだろう…
   /| ̄ ̄ ̄l ::|FFFFFFF \
   |ミ|:」」:」」:」| ::|FFFFFF     ̄|/ ̄ ̄ ̄ ̄ ̄ ̄ ̄ ̄ ̄ ̄ ̄ ̄ ̄ ̄ ̄ ̄ ̄ ̄
   |ミ|:」」:」」:」| 〇 FF.。   ∧_∧  |l≡o + +! + 。 〇   +
   lミl.」」.」〇 ++ +  (´・ω・) o  〇 。 o  +   〇 。 +
  __〇___。_゚__/ ~つと)____o______〇__o___。
  二Il二Il二Il二Il二Il二Il二Il二Il二Il二Il二Il二Il二Il二Il二Il二Il二Il二Il二
557132人目の素数さん:2012/10/06(土) 02:30:45.68
          __ノ)-'´ ̄ ̄`ー- 、_
        , '´  _. -‐'''"二ニニ=-`ヽ、
      /   /:::::; -‐''"        `ーノ
     /   /:::::/           \
     /    /::::::/          | | |  |
     |   |:::::/ /     |  | | | |  |
      |   |::/ / / |  | ||  | | ,ハ .| ,ハ|
      |   |/ / / /| ,ハノ| /|ノレ,ニ|ル' 
     |   |  | / / レ',二、レ′ ,ィイ|゙/   私は只の数ヲタなんかとは付き合わないわ。
.     |   \ ∠イ  ,イイ|    ,`-' |      頭が良くて数学が出来てかっこいい人。それが必要条件よ。
     |     l^,人|  ` `-'     ゝ  |        さらに Ann.of Math に論文書けば十分条件にもなるわよ。
      |      ` -'\       ー'  人          一番嫌いなのは論文数を増やすためにくだらない論文を書いて
    |        /(l     __/  ヽ、           良い論文の出版を遅らせるお馬鹿な人。
     |       (:::::`‐-、__  |::::`、     ヒニニヽ、         あなたの論文が Ann of Math に accept される確率は?
    |      / `‐-、::::::::::`‐-、::::\   /,ニニ、\            それとも最近は Inv. Math. の方が上かしら?
   |      |::::::::::::::::::|` -、:::::::,ヘ ̄|'、  ヒニ二、 \
.   |      /::::::::::::::::::|::::::::\/:::O`、::\   | '、   \
   |      /:::::::::::::::::::/:::::::::::::::::::::::::::::'、::::\ノ  ヽ、  |
  |      |:::::/:::::::::/:::::::::::::::::::::::::::::::::::'、',::::'、  /:\__/‐、
  |      |/:::::::::::/::::::::::::::::::::::::::::::::::O::| '、::| く::::::::::::: ̄|
   |     /_..-'´ ̄`ー-、:::::::::::::::::::::::::::::::::::|/:/`‐'::\;;;;;;;_|
   |    |/::::::::::::::::::::::\:::::::::::::::::::::::::::::|::/::::|::::/:::::::::::/
    |   /:::::::::::::::::::::::::::::::::|:::::::::::::::::::::O::|::|::::::|:::::::::::::::/
558132人目の素数さん:2012/10/06(土) 05:09:25.57
ちょっとナノマシン程度の小問を

a+a > a+ b > a+c > b+b > b+c > a+d > c+c > b+d > c+d > d+d をみたす自然数解を求めよ
559132人目の素数さん:2012/10/06(土) 05:31:36.14
>>547
つまり、山のふもとでハァハァしていたわけですね
560132人目の素数さん:2012/10/06(土) 06:26:35.44
          __ノ)-'´ ̄ ̄`ー- 、_
        , '´  _. -‐'''"二ニニ=-`ヽ、
      /   /:::::; -‐''"        `ーノ
     /   /:::::/           \
     /    /::::::/          | | |  |
     |   |:::::/ /     |  | | | |  |
      |   |::/ / / |  | ||  | | ,ハ .| ,ハ|
      |   |/ / / /| ,ハノ| /|ノレ,ニ|ル' 
     |   |  | / / レ',二、レ′ ,ィイ|゙/   私は只の数ヲタなんかとは付き合わないわ。
.     |   \ ∠イ  ,イイ|    ,`-' |      頭が良くて数学が出来てかっこいい人。それが必要条件よ。
     |     l^,人|  ` `-'     ゝ  |        さらに Ann.of Math に論文書けば十分条件にもなるわよ。
      |      ` -'\       ー'  人          一番嫌いなのは論文数を増やすためにくだらない論文を書いて
    |        /(l     __/  ヽ、           良い論文の出版を遅らせるお馬鹿な人。
     |       (:::::`‐-、__  |::::`、     ヒニニヽ、         あなたの論文が Ann of Math に accept される確率は?
    |      / `‐-、::::::::::`‐-、::::\   /,ニニ、\            それとも最近は Inv. Math. の方が上かしら?
   |      |::::::::::::::::::|` -、:::::::,ヘ ̄|'、  ヒニ二、 \
.   |      /::::::::::::::::::|::::::::\/:::O`、::\   | '、   \
   |      /:::::::::::::::::::/:::::::::::::::::::::::::::::'、::::\ノ  ヽ、  |
  |      |:::::/:::::::::/:::::::::::::::::::::::::::::::::::'、',::::'、  /:\__/‐、
  |      |/:::::::::::/::::::::::::::::::::::::::::::::::O::| '、::| く::::::::::::: ̄|
   |     /_..-'´ ̄`ー-、:::::::::::::::::::::::::::::::::::|/:/`‐'::\;;;;;;;_|
   |    |/::::::::::::::::::::::\:::::::::::::::::::::::::::::|::/::::|::::/:::::::::::/
    |   /:::::::::::::::::::::::::::::::::|:::::::::::::::::::::O::|::|::::::|:::::::::::::::/
561132人目の素数さん:2012/10/06(土) 06:58:45.71
>>559
1990以前の数学か
俺たちは時代の流れに取り残されたのか…
562132人目の素数さん:2012/10/06(土) 07:38:59.88
アンティークを愛でるのも一興
これからは君達を「骨董品コレクター」と呼ばせてもらうよ
563132人目の素数さん:2012/10/06(土) 07:56:29.30
数値評価こそが不等式の存在意義
564132人目の素数さん:2012/10/06(土) 08:18:28.88

通   素材の味を楽しむ          自然本来の味     う     塩でその店のレベルがわかる
は            塩最高     素材の味          な 素材の味                   素材
塩   「塩で」                              ぎ                          の味
           甘ったるいタレで焼き鳥が食えるか      の   タレ厨は味覚障害者
        素材の味                         白.. 異              高い店で食ったことないんだろ?
                    臭い肉をごまかすためのタレ 焼 . .論  素材の味
タレは子供用     素材の味                    き   は                    素
                       素      / ̄ ̄ ̄\       認  タレはタレの味しかしない   材
     最終的にたどり着くのは塩  材    .../.\    /. \      め                     の 
                        の   /  <●>  <●>  \ 本   な  焼き鳥=塩            味
. 「タレ」から「塩」に           味  |    (__人__)    | 当    い
             普通は塩         \    `ー'´    / の   高い店なら塩、安い店ならタレ
     シンプルに塩      ..       /             \ 味  
                素材の味                   覚  たれ(笑)  素材本来の味
 素材の味                                         子供の頃はタレだったが今は塩
                            塩こそ最高の調味料!!
565132人目の素数さん:2012/10/06(土) 20:38:04.41
>>531

〔類題〕
0 ≦ a,b,c,d ≦ 1 ⇒
 (ac+bd-abcd)^2 ≦ (a+b-ab)(b+c-bc)(c+d-cd)(d+a-da) ≦ ac+bd-abcd,
566132人目の素数さん:2012/10/06(土) 20:45:27.60
>>565 (右側)

 {a,b,c,d}のうちの2つが1ならば等号成立。

(右辺) - (左辺) = A(1-b)(1-c)(1-d) + (1-a)B(1-c)(1-d) + (1-a)(1-b)C(1-d) + (1-a)(1-b)(1-c)D + (1-a)(1-b)(1-c)(1-d)E,
ここに、
 A = (a^2)c,
 B = (b^2)d,
 C = (c^2)a,
 D = (d^2)b,
 E = ac(b+d-bd) + bd(a+c-ac) + (ac+bd+abcd),

〔参考〕casphy - 高校数学 - 937〜938
567132人目の素数さん:2012/10/06(土) 21:41:55.21
>>565 (左側)

 ac+bd-abcd = (a+b-ab)(c+d-cd) -a(1-b)(1-c)d -(1-a)bc(1-d)
  ≦ (a+b-ab)(c+d-cd),
 bd+ac-abcd = (b+c-bc)(d+a-da) -ab(1-c)(1-d) -(1-a)(1-b)cd
  ≦ (b+c-bc)(d+a-da),
辺々掛ける。

〔参考〕 casphy - 高校数学 - 939〜940
568132人目の素数さん:2012/10/07(日) 00:59:37.69
リハビリ程度(大学入試)の問題キボンヌ
569132人目の素数さん:2012/10/07(日) 02:26:14.75
570132人目の素数さん:2012/10/07(日) 11:57:45.55
>>569
もっと骨のある問題を下さい
571132人目の素数さん:2012/10/08(月) 22:34:12.34
>>273-282
>>319-324

n=3 のとき >>458 を使って強引に解く。

基本対称式を
 a+b+c = 1+σ, ab+bc+ca = t, abc = u,
とおくと、
 u ≦ {−(1+σ)[2(1+σ)^2 -9t] + 2[(1+σ)^2 -3t]^(3/2)}/27,
572132人目の素数さん:2012/10/08(月) 22:46:00.50
>>319-324 (続き)

題意により、
 (1+σ)^2 - 2t = a^2 + b^2 + c^2 = 1,
 t = σ(2+σ)/2, (0≦σ≦√3 -1 < 7/9)
 (左辺) = σ{σ(2+σ)/(2u) + 1},

ところで、>>571 から
 u ≦ {−(1+σ)(2-5σ-2.5σ^2)+2(1-σ-0.5σ^2)^(3/2)}/27

よって本問は次の補題に帰着する。

〔補題〕
 0≦σ≦√3 -1 のとき、
 {−(1+σ)(2-5σ-2.5σ^2)+2(1-σ-0.5σ^2)^(3/2)}/27
  ≦ σ^2・(2+σ)/{2(4-kσ)},
 k = 4/9 - 4/25 = 2.56/9 = 0.2844444
573132人目の素数さん:2012/10/08(月) 22:54:07.28
>>319-324 (続き)

〔補題〕の略証   >>572
 -(1+σ)(2-5σ-2.5σ^2) = -2 +3σ +7.5σ^2 +2.5σ^3,
また
 (1+σ)^2 -3t = 1 -σ -0.5σ^2,
 √(1-σ -0.5σ^2) ≦ 1 -0.5σ -0.375σ^2,
を辺々掛けて
 2・(1-σ -0.5σ^2)^1.5 ≦ 2 -3σ -0.75σ^2 +1.25σ^3 +0.375σ^4,
以上により
 (左辺) ≦ σ^2・(6.75 +3.75σ +0.375σ^2)/27
  = σ^2・{8 +(40/9)σ +(4/9)σ^2}/32
  < σ^2・(2+σ)(4+kσ)/32  (← *)
  < σ^2・(2+σ)/{2(4-kσ)}
  < (右辺),

*) 8 + (40/9)σ +(4/9)σ^2
 < 8 + (40/9)σ +(4/9)σ^2 + (4/25)σ(7/9 - σ)
 = 8 + (4+2k)σ +kσ^2
 = (2+σ)(4+kσ),

(参考) casphy - 高校数学 - 三次方程式教えてくださいスレ - 16〜19

 華麗なるキャスフィーでござるよ。
574132人目の素数さん:2012/10/08(月) 23:35:25.44
>>571-573
凄い!

n=3の時だけでもこんなに大変なのか!
n≧4の時は絶望的か?
575132人目の素数さん:2012/10/09(火) 20:17:04.58
>>573
> 華麗なるキャスフィーでござるよ。

労力は認めるが、華麗とは思えないなあ
576132人目の素数さん:2012/10/09(火) 20:18:30.53
>>325の続きマダ―

325 自分:132人目の素数さん[sage] 投稿日:2012/09/04(火) 01:20:05.87
>>276

( 農[k:1->n] x_k - 1)( 農[k:1->n] 1/x_k + 1)
= 農[k:1->n] x_k 農[k:1->n] 1/x_k + 農[k:1->n] x_k - 農[k:1->n] 1/x_k -1

ここで、第1項はコーシー・シュワルツの不等式より、
   農[k:1->n] x_k 農[k:1->n] 1/x_k ≧ { 農[k:1->n] 1 }^2 =n^2

また、条件 農[k:1->n] (x_k)^2 = 1 より、
    農[k:1->n] x_k ≦ { 農[k:1->n] (x_k)^2 }^{1/2} {農[k:1->n] 1^2}^{1/2} = √n 
となる。
よって、
577132人目の素数さん:2012/10/09(火) 20:52:07.75
>>576
次が分かればいいんですね

[問題]
農[k:1->n] (x_k)^2 = 1 を満たす x_1, x_2, ,, ,x_n >0 に対して、
農[k:1->n] 1/x_k の最大値を求めよ。
578132人目の素数さん:2012/10/09(火) 21:09:02.08
x_1→0でいくらでも
579132人目の素数さん:2012/10/09(火) 21:36:30.55
グレート・キャスフィー


村上春樹の一番のお気に入りの本。
「この小説に出会わなければ、僕は小説家になっていなかった。」とまで言わせる至上最高の作品。
(もちろん彼にとって)
580132人目の素数さん:2012/10/09(火) 23:13:38.21
>>576-578
2項目と3項目の最小値が問題なのでは?

[問題]
農[k:1->n] (x_k)^2 = 1 を満たす x_1, x_2, ,, ,x_n >0 に対して、
農[k:1->n] x_k - 農[k:1->n] 1/x_k の最小値を求めよ。
581132人目の素数さん:2012/10/10(水) 22:58:10.24
Given a $\triangle ABC$ with the side lengths $BC=a,\ CA=b,\ AB=c$.
Prove that :
$$\displaystyle \frac{a^2+b^2-c^2}{a+b-c}+\frac{b^2+c^2-a^2}{b+c-a}+\frac{c^2+a^2-b^2}{c+a-b}\le a+b+c$$

http://suseum.jp/gq/question/1852
582132人目の素数さん:2012/10/11(木) 00:25:41.03
>>581

BC=a, CA=b, AB=c のとき

(a^2+b^2-c^2)/(a+b-c) + (b^2+c^2-a^2)/(b+c-a) + (c^2+a^2-b^2)/(c+a-b) ≦ a+b+c,
ですね。
 a=y+z, b=z+x, c=x+y (x,y,z>0) とおく。

以下casphyで...
583132人目の素数さん:2012/10/11(木) 00:29:01.70
あまりcasphy荒らすなよ
誰も書かなくなってるじゃん
584132人目の素数さん:2012/10/11(木) 02:21:03.72
a[i]>0(i=1,2,3,...)とする。
Σ[k=1→n]1/a[k]=1のとき、Π[k=1→n](a[k]-1)≧(n-1)^nを示せ
585132人目の素数さん:2012/10/11(木) 08:05:17.13
>>582
casphyぢゃなく、こっちに書いてくだされ!
情報が分散していると整理しづらいでござるよ ( ゚∀゚) ニンニン
586132人目の素数さん:2012/10/11(木) 08:09:36.88
ところで casphy の過去ログとかあるんですか?
スレは>>869
あの一つだけ?
587132人目の素数さん:2012/10/11(木) 08:12:41.79
>>586
変なのが混じった
スレは一つだけ?
588132人目の素数さん:2012/10/11(木) 09:15:52.10
スレ一覧見てると
http://www.casphy.com/bbs/test/subject.cgi?all=highmath

まあマイナー?掲示板だし2007年からのスレだし違う不等式スレは無かったんじゃない?
589132人目の素数さん:2012/10/11(木) 20:52:32.48
>>581 の解答(クロニャンコ/のんぶ)を転載

分母を
 b+c-a = A ・・・・・ (1)
 c+a-b = B ・・・・・ (2)
 a+b-c = C ・・・・・ (3)
とおく。(1) + (2) + (3) から
 a+b+c = A+B+C ・・・・・ (4)
(1),(2),(3),(4) から
 a = (B+C)/2,
 b = (C+A)/2,
 c = (A+B)/2,
分子は
 b^2 +c^2 -a^2 = {A(A+B+C) - BC}/2,
 c^2 +a^2 -b^2 = {B(A+B+C) - CA}/2,
 a^2 +b^2 -c^2 = {C(A+B+C) - AB}/2,
よって
 (左辺) = (3/2)(A+B+C) -(1/2)(BC/A + AC/B + AB/C)
  = A+B+C -(1/2)(BC/A + CA/B + AB/C -A -B -C)
  = A+B+C -{(BC)^2 + (CA)^2 + (AB)^2 -ABC(A+B+C)}/(2ABC)
  = A+B+C -{(A^2)(B-C)^2 + (B^2)(C-A)^2 + (C^2)(A-B)^2}/(4ABC)
  = a+b+c -{(b+c-a)^2・(c-b)^2 + (c+a-b)^2・(a-c)^2 + (a+b-c)^2・(b-a)^2}/(ABC)
  ≦ a+b+c,   (← ABC>0 ← 題意)

等号成立は、a=b=c のとき。 (a-b=c=0 は A=B=0 となる。不適)

※ A/2, B/2, C/2 は各辺を 内接円の接点で分けた線分の長さ。
590132人目の素数さん:2012/10/11(木) 22:11:52.03
【AoPS forum - Inequalities ・ Art of Problem Solving】

http://www.artofproblemsolving.com/Forum/viewforum.php?f=32&

既出?
591132人目の素数さん:2012/10/12(金) 08:11:54.19
>>590
玉石混交で、面白いのを探すのが大変ですね ( ゚∀゚)プケラッチョ!
592132人目の素数さん:2012/10/12(金) 13:43:11.79
玉石混合じゃないんだ・・・
今までずっと(ry
593132人目の素数さん:2012/10/12(金) 13:54:06.15
それはちょっと恥ずかしい加茂
594132人目の素数さん:2012/10/12(金) 21:00:31.69
595132人目の素数さん:2012/10/12(金) 21:57:49.85
>>594

問1 関数f(t)は、つねに f "(t) ≧ 0 であるとする。
  u(x) = ∫[-x,x] f(t)dt
とおく。 x{u '(x) - 2f(0)} ≧ 0, u(x) ≧ 2f(0)x,


問2 関数g(s)は、つねに g "(s) ≧ 0 を満たすとする。a<b のとき、不等式
   ∫[a,b] g(s)ds ≧ (b-a)g((a+b)/2),


(1) x>0 のとき、 log(x+1) - log(x) < 1/x,
(2) x≧1 のとき、 x・log(x) ≧ (x-1)log(x+1),
(3) 自然数n(n≧3)に対して、 (n!)^2 > n^n,
596132人目の素数さん:2012/10/12(金) 22:13:05.31
>>595

問1 
 u(x)は奇関数、u '(x)は偶関数
 u(0) = 0,
 u '(x) = f(x) + f(-x)
 u '(0) = 2f(0),
 u "(x) = f '(x) - f '(-x) = ∫[-x,x] f "(t)dt ≧ 0,
 x{u '(x) - u '(0)} ≧ 0,
 u(x) ≧ u '(0)x,

問2
 {g(s) + g(a+b-s)}/2 ≧ g((a+b)/2),
 (左辺) ≧ g((a+b)/2)∫[a,b] ds = (b-a)g((a+b)/2),
597132人目の素数さん:2012/10/12(金) 23:02:46.46
>>595

(1)
  log(x+1) - log(x) = ∫[x,∞) {1/t - 1/(t+1)}dt
   = ∫[x,∞) 1/{t(t+1)} dt
   < ∫[x,∞) 1/t^2 dt
   = 1/x,

(2)
  log(t+1) - log(t) < 1/t,
 ∫[1,x] {log(t+1) - log(t)}dt < ∫[1,x] 1/t dt,
  (x+1)log(x+1) - x・log(x) -2・log(2) ≦ log(x),

(3) x=1,2,・・・・,n-1 で和をとる。
  n・log(n) -2log(2)(n-1) < log{(n-1)!},
    4^(n-1)・(n-1)! > n^n,
598132人目の素数さん:2012/10/13(土) 00:47:02.94
あんまり綺麗じゃない不等式だが・・・
今年の某大学入試より。

0≦x≦π/2 において、 (sin(x)+cos(x))*((7/4)-sin(x)*cos(x))≦(3/2)^(3/2)
599132人目の素数さん:2012/10/13(土) 00:49:49.88
高校数学はスレ違い

帰れよ!
600132人目の素数さん:2012/10/13(土) 01:47:02.99
>>580
【問題】
農[k:1->n] (x_k)^2 = 1 を満たす x_1, x_2, ,, ,x_n >0 に対して、次の不等式を示せ。
Π_[k:1->n] { x_k - (1/x_k) } ≧ { √n - (1/ √n) }^n
601132人目の素数さん:2012/10/13(土) 01:50:01.18
>>598

{sin(x)+cos(x)}^2 = 1 + 2sin(x)cos(x),
(7/4) -sin(x)cos(x),
(7/4) -sin(x)cos(x),
この3つの相加平均は 3/2 なので....
602132人目の素数さん:2012/10/13(土) 02:07:46.37
>>601
高校数学はスレ違い

帰れよ!
603132人目の素数さん:2012/10/13(土) 02:10:45.85
キチガイ注意報発令
604132人目の素数さん:2012/10/13(土) 02:54:00.21
605132人目の素数さん:2012/10/13(土) 05:20:19.16
  Λ_Λ  \\
  ( ・∀・)   | | ガガッ
 と    )    | |   
   Y /ノ    .人   
    / ) .人 <  >__Λ ∩
  _/し' <  >_Λ∩Д´)/  
 (_フ彡 V`Д´)/   / ←>>599
            / ←>>602
606132人目の素数さん:2012/10/13(土) 12:01:49.89
casphyのコピペは止めろや!


  casphy
  Λ_Λ  \\
  ( ・∀・)   | | ガガッ
 と    )    | |   
   Y /ノ    .人   
    / ) .人 <  >__Λ ∩
  _/し' <  >_Λ∩Д´)/  
 (_フ彡 V`Д´)/   / ←>>603
            / ←>>605


607132人目の素数さん:2012/10/13(土) 12:13:16.56
>>606
ここは不等式回収所だぞ
608132人目の素数さん:2012/10/13(土) 12:15:25.41
  ___ 
./  ≧ \ 仲良くしたまえ!
|::::  \ ./ | 
|::::: (● (● | \\
ヽ::::... .ワ.....ノ  .| | ガガガガッ 人
 と    )    | |    人  <  >__Λ∩
   Y /ノ    .人   <  >__Λ∩Д´)/
    / ) .人 <  >__Λ ∩Д´)/   / ←>>599
  _/し' <  >_Λ∩Д´)/   / ←>>602
 (_フ彡 V`Д´)/   / ←>>605
            / ← >>606
609132人目の素数さん:2012/10/14(日) 01:23:01.19
>>580
 農[k:1→n] (1/x_k − x_k) の最小値。
 f(y) = 1/√y - √y は下に凸だからイェンゼンより
 農[k:1→n] f({x_k}^2) ≧ n・f((1/n)農[k:1→n] {x_k}^2)
   = n・f(1/n)
   = n・(√n - 1/√n)
   = (n-1)√n,

>>598
2sin(x)cos(x) = S とおくと、 |S| ≦ 1
 (3/2)^3 - (左辺)
 = (3/2)^3 - (1+S){(7/4) -S/2}^2
 = (1/16){54 -(1+S)(7-2S)^2}
 = (1/16)(5-S)(1-2S)^2
 ≧ 0,
等号成立は sin(2x)=S=1/2, 2x=π/6, x=π/12.
610132人目の素数さん:2012/10/14(日) 03:18:28.29
Prove that :
$$\tan{A}+\tan{B}+\tan{C}+2(\sin{A}+\sin{B}+\sin{C})>3\pi$$.
http://suseum.jp/gq/question/1829
611132人目の素数さん:2012/10/14(日) 07:16:10.90
有名なe^π>π^eを示す問題っていくつ解法ありますか
612132人目の素数さん:2012/10/14(日) 07:37:54.85
>>610 より転載

スネリウス-ホイヘンスを使う方法(クロニャンコ etc.)が簡単。
他には、
 f(x) = tan(x) + 2sin(x) -3x 
とおくと
 f "(x) = 2sin(x){1-cos(x)^3}/cos(x)^3 ≧ 0,
ゆえ f(x) は下に凸
から Jensen を使う方法(Sayan)や

 tan(x) > x +(1/3)x^3,
 sin(x) > x -(1/6)x^3,
 sin(x) > x -(1/6)x^3,
をたす方法(nicusorz)がある。
613132人目の素数さん:2012/10/14(日) 10:01:15.20
>>612
スネリウス-ホイヘンスって、なんじゃらほい?
614132人目の素数さん:2012/10/14(日) 14:44:15.57
>>611
証明は意外と簡単。
f(x) = logx / x が x ≧e で単調増加であることを言えばOK
615614:2012/10/14(日) 14:45:13.68
>>614
訂正:単調増加→単調減少
616132人目の素数さん:2012/10/14(日) 15:00:34.63
>>611 e^x≧1+xを使えば, 秒殺!
617132人目の素数さん:2012/10/14(日) 15:06:47.73
>>616
ん、どうするの?
618132人目の素数さん:2012/10/14(日) 15:14:46.44
>>614流で秒殺(1分掛からないの意?)なら、まあわからなくもないが
どうするんだろ?
619132人目の素数さん:2012/10/14(日) 15:31:27.22
          __ノ)-'´ ̄ ̄`ー- 、_
        , '´  _. -‐'''"二ニニ=-`ヽ、
      /   /:::::; -‐''"        `ーノ
     /   /:::::/           \
     /    /::::::/          | | |  |
     |   |:::::/ /     |  | | | |  |
      |   |::/ / / |  | ||  | | ,ハ .| ,ハ|
      |   |/ / / /| ,ハノ| /|ノレ,ニ|ル' 
     |   |  | / / レ',二、レ′ ,ィイ|゙/   私は只の数ヲタなんかとは付き合わないわ。
.     |   \ ∠イ  ,イイ|    ,`-' |      頭が良くて数学が出来てかっこいい人。それが必要条件よ。
     |     l^,人|  ` `-'     ゝ  |        さらに Ann.of Math に論文書けば十分条件にもなるわよ。
      |      ` -'\       ー'  人          一番嫌いなのは論文数を増やすためにくだらない論文を書いて
    |        /(l     __/  ヽ、           良い論文の出版を遅らせるお馬鹿な人。
     |       (:::::`‐-、__  |::::`、     ヒニニヽ、         あなたの論文が Ann of Math に accept される確率は?
    |      / `‐-、::::::::::`‐-、::::\   /,ニニ、\            それとも最近は Inv. Math. の方が上かしら?
   |      |::::::::::::::::::|` -、:::::::,ヘ ̄|'、  ヒニ二、 \
.   |      /::::::::::::::::::|::::::::\/:::O`、::\   | '、   \
   |      /:::::::::::::::::::/:::::::::::::::::::::::::::::'、::::\ノ  ヽ、  |
  |      |:::::/:::::::::/:::::::::::::::::::::::::::::::::::'、',::::'、  /:\__/‐、
  |      |/:::::::::::/::::::::::::::::::::::::::::::::::O::| '、::| く::::::::::::: ̄|
   |     /_..-'´ ̄`ー-、:::::::::::::::::::::::::::::::::::|/:/`‐'::\;;;;;;;_|
   |    |/::::::::::::::::::::::\:::::::::::::::::::::::::::::|::/::::|::::/:::::::::::/
    |   /:::::::::::::::::::::::::::::::::|:::::::::::::::::::::O::|::|::::::|:::::::::::::::/
620132人目の素数さん:2012/10/14(日) 16:34:49.95
>>617
> f(x) = logx / x が x ≧e で単調減少であることを言えばOK

これで秒殺と思えないのは、高校数学からやり直せ!
621132人目の素数さん:2012/10/14(日) 16:38:11.14
>>611
f(x) = logx / x が x ≧e で単調減少である(微分すれば分かる)ので、
log e / e > log π / π.
すなわち、
log e^π > log π^e
log は単調増加だから、
e^π > π^e
となる。
622132人目の素数さん:2012/10/14(日) 16:41:36.85
>>620
ん、違う違う。それはわかってるんだ。
俺が言ってるのは>>616

> >>611 e^x≧1+xを使えば, 秒殺!

てどういう解法?ってこと。
623あのこうちやんは始皇帝だった:2012/10/14(日) 19:29:28.40

  オマエたちは、定職に就くのが先決だろがああああああああああああああああああああああ!!!!!!!!!!!!!!!!!!!!!!!!!!!!!

 ニート・無職の、クズどもがあああああああああああああああああああああああああああああああ!!!!!!!!!!!!!!!!!!!!!!!!!!!!!!!!!!!!!!!!!!
624132人目の素数さん:2012/10/14(日) 19:37:18.45
>>622
> >>611 e^x≧1+xを使えば, 秒殺!

x= π/e−1 とおくと、
e^{π/e−1} ≧ π/e
よって、
 e^π ≧π^e
625132人目の素数さん:2012/10/14(日) 19:39:51.36
えっ?
626132人目の素数さん:2012/10/14(日) 19:49:25.50
確かに、まさに秒殺だが…
思いつけと言われても無理です><
627132人目の素数さん:2012/10/14(日) 20:26:39.52
感動が大きいんだが・・・
628132人目の素数さん:2012/10/14(日) 20:35:21.14
>>626
まあ >>621 の解答が一番自然じゃないかな。
e が出てくるんだから log を使うのも自然だし、道筋も王道。

技巧的な証明でも、このレベルじゃ対して価値は無いし
629132人目の素数さん:2012/10/14(日) 20:47:52.57
>このレベルじゃ対して価値は無いし

そんなことはないと思うが。
630132人目の素数さん:2012/10/14(日) 20:57:27.88
技巧的な証明が、そうでない証明と比較してもなお価値があるのは
より初等的な概念だけを使って、少ない道具立てで証明できているときかな?
631132人目の素数さん:2012/10/14(日) 20:58:02.01
もしくは構成的な証明であるとか
632132人目の素数さん:2012/10/14(日) 21:14:54.31
>>630
禿同

>>621>>624じゃあ大差ないわな
まだ理論的な>>621の方が他の問題にも使えるし、価値が高い。
633132人目の素数さん:2012/10/14(日) 21:17:01.50
[拡張問題]
f(x) = e^x -x^e ≧ 0 となる実数 x の範囲を求めよ。
634132人目の素数さん:2012/10/15(月) 03:51:09.33
>>613

947   prime_132 [2012/10/14(日) 18:26:04] sage

>>870
〔Snellius-Huygens の式〕
 θ < (2sinθ + tanθ)/3, (0<θ<π/2)
略証1.
 相加・相乗平均より
 1 < {cos(x) + cos(x) + 1/cos(x)^2}/3,
 xで積分する。 [0→θ]
略証2.
 -cos(x) > -1  (0≦x)
を [0,x'] で3回積分すると、
 sinθ ≧ θ{1 - (1/6)θ^2}, ・・・・ (1)
もう一度積分して
 1 - cosθ ≧ (1/2)θ^2 - (1/24)θ^4,
∴ cosθ ≦ 1 - (1/2)θ^2 + (1/24)θ^4 ≦ {1 - (1/6)θ^2}^3,  ( |θ| < 3)
∴ tanθ ≧ θ/{1 - (1/6)θ^2}^2 ≧ θ{1 + (1/3)θ^2}, ・・・・・ (2)
(1)(2)から相加・相乗平均で
 θ < sinθ/(cosθ)^(1/3) < (2sinθ + tanθ)/3,
635132人目の素数さん:2012/10/15(月) 07:02:37.22
ありがとうございますだ
636132人目の素数さん:2012/10/15(月) 08:39:00.51
Let $0\le x,y,z\le 1$, find the maximum of $\sqrt{|y-z|}+\sqrt{|z-x|}+\sqrt{|x-y|}.$

http://suseum.jp/gq/question/1858
637132人目の素数さん:2012/10/15(月) 13:52:26.21
>>636
この投降者、よく訓練された不等式ヲタですね
638132人目の素数さん:2012/10/15(月) 13:54:49.85
よく訓練されたAAキボンヌ ↓
639132人目の素数さん:2012/10/15(月) 19:56:01.26

     _  ())二) )) 、,r:ニヽ  いいぞ ベイべー!
 @ニ===)二二ニニ)('A` ))  不等式を収集し証明する奴は 数ヲタだ!!
     ^ ̄" フ\''|ノ=ノ-(  )   不等式を改造し拡張する奴は よく訓練された数ヲタだ!!
         _/  \_   L L   ホント不等式はハァハァするぜ! フゥハハハーハァー
640132人目の素数さん:2012/10/15(月) 21:50:24.74
そこまでハァハァされたらしょうがねぇ。。。

>>636
 0≦x,y,z≦1 のとき √|y-z| + √|z-x| + √|x-y| の最大値を求めよ。

(略解)
 min{x,y,z} = m について単調減少で、m=0 のとき最大。
 Max{x,y,z} = M について単調増加で、M=1 のとき最大。
 中間元をxとしても一般性を失わない。
このとき
 (与式) ≦ √(1) + √(1-x) + √(x)
    ≦ 1 + 2√(1/2)  (← √x は上に凸)
    = 1 + √2,
641132人目の素数さん:2012/10/15(月) 22:00:44.28
>>634

sinθ/(cosθ)^(1/3) = f(θ) とおくと、

 f '(θ) = {(cosθ)^2 + (cosθ)^2 + 1}/{3(cosθ)^(4/3)}
     ≧ 1,  (← 相加・相乗平均)
 f(θ) ≧ θ.
でもよい。
642132人目の素数さん:2012/10/15(月) 23:51:55.05
Let x, y are real number, find minimum value of 2x ^{2} +y ^{2} -2x+ \frac{4}{x ^{2} + y ^{2} +1}

http://suseum.jp/gq/question/1859
643132人目の素数さん:2012/10/16(火) 00:26:39.68
なんか、三角関数系で興奮できる不等式教えてください・・・
644132人目の素数さん:2012/10/16(火) 01:14:47.25
一目で殺せる瞬殺問題なので興奮できないが、三角関数系のうんちレベル問題

x>1に対して、sin{1/(x-1)} - sin(1/x) > sin(1/x) - sin{1/(x+1)}
645132人目の素数さん:2012/10/16(火) 09:01:00.82
(2/1)*(4/3)*(6/5)*・・・*{(2n)/(2n-1)} ≧ √(3n)
646132人目の素数さん:2012/10/16(火) 09:53:45.20
a,b,c≧1のとき
2^(a+b+c)≧2^(a+b)+2^(b+c)+2^(c+a)-4を示せ
647132人目の素数さん:2012/10/16(火) 11:08:44.15
3 < ∫[0、π] x・√(1 - sin^3 x) dx < 4 を証明するナリよ、キテレツ
648132人目の素数さん:2012/10/16(火) 20:57:20.82
>>644
目で殺すって…
649132人目の素数さん:2012/10/16(火) 21:02:20.00
>>624
       /.⌒ヽ
      /    .\
    ../      ヽ. \
    (./       ヽ. )
    /        l"
   .ノ          l  甘い痺れがいつまでもとれません!
   l  >   <  ..|
   l   一      |
   ヽ.._____       _,ノ ビクッ
.   丿ノ ノ 丁丁 ̄l\ ぶるぶる
  . く_(__(_(_._」____)ノ

650132人目の素数さん:2012/10/16(火) 21:05:11.31
∫[0、π] x・√(1 - sin^3 x) dx
>∫[0、π] x・√(1 - sin^2 x) dx
=∫[0、π] x・|cos x| dx
651132人目の素数さん:2012/10/16(火) 21:23:31.36
330:大学への名無しさん[]
2012/10/16(火) 07:16:58.78 ID:BrZTwPa50
(1) 任意の実数xに対し、不等式 e^x ≧ 1+x を示せ。
(2) e^{π/e−1} ≧ π/e を示せ。
(3) e^π ≧ π^e を示せ。

332:大学への名無しさん[]
2012/10/16(火) 13:26:36.09 ID:BrZTwPa50
私は結構感動したのですけど向こうではイマイチ不評だったので
コチラの住人的にはどうかな、と思って…

334:大学への名無しさん[]
2012/10/16(火) 13:33:46.87 ID:BrZTwPa50
f(x) = log(x)/x の増減から証明する方法は知ってましたが
よりシンプルな e^x > 1+x から示すのは知りませんでした。
これ有名なんですか。よかったらこれが載ってる参考書とかサイトとかあったら教えて下さい。

335:大学への名無しさん[sage]
2012/10/16(火) 13:34:48.41 ID:BrZTwPa50
「これ」とは勿論「この証明方法」のことです。

【月刊大学への数学】学力コンテスト・宿題14
http://kohada.2ch.net/test/read.cgi/kouri/1345559361/
652132人目の素数さん:2012/10/16(火) 21:40:59.43
(2)がないとE★★★…★★★
(2)があるとA☆
653132人目の素数さん:2012/10/16(火) 21:57:32.48
は?
654132人目の素数さん:2012/10/16(火) 21:59:48.96
>>651
雑誌からのネタだったのか…
最初に考えたやつスゴイな
655132人目の素数さん:2012/10/17(水) 00:24:28.72
>>647
 f(πーx) = f(x) のとき、
 ∫[0,π] x・f(x) dx = ∫[0,π/2] {x + (π-x)}f(x) dx
    = π∫[0,π/2] f(x)dx,

 √{1-sin(x)^3} > √{1-sin(x)^2} = |cos(x)|
より
 ∫[0,π/2」 √{1-sin(x)^3} dx
 > ∫[0,π/2] cos(x)dx
 = [ sin(x) ](x=0,π/2)
 = 1,      >>650

 √{1-sin(x)^3} < 1 -(1/2)sin(x)^3 = 1 -(3/8)sin(x) +(1/8)sin(3x),
より
∫[0,π/2] √{1-sin(x)^3} dx < [ x +(3/8)cos(x) - (1/24)cos(3x) ](x=0,π/2)
  = π/2 -(3/8) +(1/24)
  = π/2 - (1/3),

 π < (与式) < π(π/2 - 1/3) = 1.2374630π
 3.14159265 < (与式) < 3.88760465

なお、(与式) 〜 1.09010166π
656132人目の素数さん:2012/10/17(水) 00:49:44.73
∫[0,π/2] √{1-sin(x)^3} dx
<∫[0,π/2] √{1-sin(x)^4} dx
=∫[0,π/2] cosx√{1+sin(x)^2} dx
<∫[0,π/6] cosx√{1+sin(π/6)^2} dx
+∫[π/6,π/2] cosx√{1+sin(π/2)^2} dx
657132人目の素数さん:2012/10/17(水) 01:26:08.27
>>655
上限はSchwarzの不等式で ( ゚∀゚)ウヒョッ!

                   r、ノVV^ー八
                 、^':::::::::::::::::::::::^vィ       、ヽ l / ,
                 l..:.::::::::::::::::::::::::::::イ      =     =
                    |.:::::::::::::::::::::::::::::: |     ニ= 仙 そ -=
                  |:r¬‐--─勹:::::|     ニ= 道 れ =ニ
                 |:} __ 、._ `}f'〉n_   =- な. で -=
  、、 l | /, ,         ,ヘ}´`'`` `´` |ノ:::|.|  ヽ ニ .ら. も ニ
 .ヽ     ´´,      ,ゝ|、   、,    l|ヽ:ヽヽ  } ´r :   ヽ`
.ヽ し き 仙 ニ.    /|{/ :ヽ -=- ./| |.|:::::| |  |  ´/小ヽ`
=  て っ 道  =ニ /:.:.::ヽ、  \二/ :| |.|:::::| |  /
ニ  く. と な  -= ヽ、:.:::::::ヽ、._、  _,ノ/.:::::| | /|
=  れ.何 ら  -=   ヽ、:::::::::\、__/::.z先.:| |' :|
ニ  る と   =ニ   | |:::::::::::::::::::::::::::::::::::.|'夂.:Y′ト、
/,  : か   ヽ、    | |::::::::::::::::::::::::::::::::::::_土_::|  '゙, .\
 /     ヽ、     | |:::::::::::::::::::::::::::::::::::.|:半:|.ト、    \
  / / 小 \    r¬|ノ::::::::::::::::::::::::::::::::::::::::::::::::| \
658657:2012/10/17(水) 01:28:34.73
不等式ヲタなら、鶏を裂くのに牛刀を用いたいわけでつ ( ゚∀゚)プケラッチョ!
659132人目の素数さん:2012/10/17(水) 01:49:04.12
昔の大数から

四面体の6本の辺の長さの積をL、体積をVとするとき、L^2/V≧72 を示せ
660132人目の素数さん:2012/10/17(水) 06:06:15.86
          __ノ)-'´ ̄ ̄`ー- 、_
        , '´  _. -‐'''"二ニニ=-`ヽ、
      /   /:::::; -‐''"        `ーノ
     /   /:::::/           \
     /    /::::::/          | | |  |
     |   |:::::/ /     |  | | | |  |
      |   |::/ / / |  | ||  | | ,ハ .| ,ハ|
      |   |/ / / /| ,ハノ| /|ノレ,ニ|ル' 
     |   |  | / / レ',二、レ′ ,ィイ|゙/   
.     |   \ ∠イ  ,イイ|    ,`-' |      
     |     l^,人|  ` `-'     ゝ  |        あら、こうちゃん、どうかなったの?
      |      ` -'\       ー'  人           当々○○○○になったの?
    |        /(l     __/  ヽ、           
     |       (:::::`‐-、__  |::::`、     ヒニニヽ、         
    |      / `‐-、::::::::::`‐-、::::\   /,ニニ、\            
   |      |::::::::::::::::::|` -、:::::::,ヘ ̄|'、  ヒニ二、 \
.   |      /::::::::::::::::::|::::::::\/:::O`、::\   | '、   \
   |      /:::::::::::::::::::/:::::::::::::::::::::::::::::'、::::\ノ  ヽ、  |
  |      |:::::/:::::::::/:::::::::::::::::::::::::::::::::::'、',::::'、  /:\__/‐、
  |      |/:::::::::::/::::::::::::::::::::::::::::::::::O::| '、::| く::::::::::::: ̄|
   |     /_..-'´ ̄`ー-、:::::::::::::::::::::::::::::::::::|/:/`‐'::\;;;;;;;_|
   |    |/::::::::::::::::::::::\:::::::::::::::::::::::::::::|::/::::|::::/:::::::::::/
    |   /:::::::::::::::::::::::::::::::::|:::::::::::::::::::::O::|::|::::::|:::::::::::::::/
661132人目の素数さん:2012/10/17(水) 08:13:35.62
>>654
違うお
こっちのスレが先で、感動を分けたくて向こうの板に(問題形式で)転載したんだお
662132人目の素数さん:2012/10/17(水) 09:09:16.38
Let $x,\ y,\ z$ be positive real numbers with $x+y+z=1$.
Prove that : $$(x^2+y^2+z^2)^2\left(\frac{1}{x}+\frac{1}{y}+\frac{1}{z}\right)\geq 1.$$

Let $a,\ b,\ c$ be real numbers such that $\displaystyle \frac{1}{|a+b|}+\frac{1}{|b+c|}+\frac{1}{|c+a|}=3.$
Prove that :
$$\displaystyle |a|+|b|+|c|\geq \frac {3}{2}.$$

Find all non-zero real numbers $x$ such that
\[\min \left\{ 4, x+ \frac 4x \right\} \geq 8 \min \left\{ x,\frac 1x\right\} .\]

For positive real numbers $a,\ b,\ c$ with $a+b+c=2010$, find the minimum value of the expression:
\[\frac{b^2+c^2}{a}+\frac{c^2+a^2}{b}+\frac{a^2+b^2}{c}\]

$a_1 + a_2 + \cdots + a_n = 0$, for some $k$ we have $a_j\leq 0$ for $j\leq k$ and $a_j \geq 0$ for $j > k$. \
If $a_i$ are not all 0, show that $a_1 + 2a_2 + 3a_3 + ... + na_n > 0.$
\begin{flushright}1966 Swedish Mathematical Olympiad\end{flushright}

Let $ x$ and $ y$ positive real numbers such that $ (1+x)(1+y)=2$. \ \\
Prove that?: $$\ \displaystyle xy+\frac{1}{xy}\geq\ 6.$$

Let $a,\ b,\ c$ be positive real numbers such that $a+b+c=1$.\ \\
Find the minimum value of the expression:\ \\
$$\displaystyle a\left(\frac{c}{b}+\frac{a}{c}\right)+b\left(\frac{a}{c}+\frac{b}{a}\right)+c\left(\frac{b}{a}+\frac{c}{b}\right).$$

http://suseum.jp/gd/all_berry_list/3504?page=17
663132人目の素数さん:2012/10/17(水) 13:27:35.10
>>662
すうじあむの不等式ヲタ(←勝手に決めつけている)の出題した問題を全問攻略するんですね、分かります
664132人目の素数さん:2012/10/17(水) 21:59:26.34
>>661
その解法の出典って大数?
だとしたら何年の何月号とか教えて欲しいんだが。
665あのこうちやんは始皇帝だった:2012/10/17(水) 22:04:16.20
          __ノ)-'´ ̄ ̄`ー- 、_
        , '´  _. -‐'''"二ニニ=-`ヽ、
      /   /:::::; -‐''"        `ーノ
     /   /:::::/           \
     /    /::::::/          | | |  |
     |   |:::::/ /     |  | | | |  |
      |   |::/ / / |  | ||  | | ,ハ .| ,ハ|
      |   |/ / / /| ,ハノ| /|ノレ,ニ|ル' 
     |   |  | / / レ',二、レ′ ,ィイ|゙/   
.     |   \ ∠イ  ,イイ|    ,`-' |      
     |     l^,人|  ` `-'     ゝ  |        何時もおんなじ事を書く
      |      ` -'\       ー'  人           馬鹿で無能のこうちゃんは
    |        /(l     __/  ヽ、            やっぱり只の糞キチガイ
     |       (:::::`‐-、__  |::::`、     ヒニニヽ、           ネコも大して変わらない   
    |      / `‐-、::::::::::`‐-、::::\   /,ニニ、\            反論出来ないこうちゃんは
   |      |::::::::::::::::::|` -、:::::::,ヘ ̄|'、  ヒニ二、 \             誰もが認めるクズでカス
.   |      /::::::::::::::::::|::::::::\/:::O`、::\   | '、   \
   |      /:::::::::::::::::::/:::::::::::::::::::::::::::::'、::::\ノ  ヽ、  |
  |      |:::::/:::::::::/:::::::::::::::::::::::::::::::::::'、',::::'、  /:\__/‐、
  |      |/:::::::::::/::::::::::::::::::::::::::::::::::O::| '、::| く::::::::::::: ̄|
   |     /_..-'´ ̄`ー-、:::::::::::::::::::::::::::::::::::|/:/`‐'::\;;;;;;;_|
   |    |/::::::::::::::::::::::\:::::::::::::::::::::::::::::|::/::::|::::/:::::::::::/
    |   /:::::::::::::::::::::::::::::::::|:::::::::::::::::::::O::|::|::::::|:::::::::::::::/
666663:2012/10/18(木) 08:06:16.73
>>664
いやだから出典はこのスレ>>624 。これに感動して向こう(受験板)にも貼ったんだ。
624氏のオリジナルかなあ。
667132人目の素数さん:2012/10/18(木) 08:34:13.16
つまり大数の過去問ではないということですね
分かったかな、664君
668132人目の素数さん:2012/10/18(木) 21:20:26.08
>>666
過去問も何も、こんな証明古くから知られているよ

ていうか、e^x ≧ 1+x を用いるのは、相加・相乗平均の証明で使うだろ。
技術面だけに目を奪われるから、こんなので驚くんだよw
669132人目の素数さん:2012/10/18(木) 21:27:33.39
670132人目の素数さん:2012/10/18(木) 21:36:15.07
>>611

i) f (x) = log x と置いて、これに区間 [1, log π] で平均値の定理を適用すると、
{f (log π) - f (1)}/{(log π) -1} = f '(c) = 1/c,   1 < c, これより
1 > (log log π)/{(log π) - 1},   log (π/e) > log log π,  π/e > logπ,  π > log (πe) より出る。(勿論、 1 < e < π 位は仮定する)

ii) 別解、積分を使う物。
∫[ 1 → π/e] {1 - (1/x)}dx > 0 (被積分関数は積分範囲で端点を除いて正)より、 π/e > logπ を出す
671132人目の素数さん:2012/10/18(木) 21:40:39.68
× π > log (πe)
○ π > log (π^e)
672132人目の素数さん:2012/10/18(木) 22:43:56.80
>>669
        ハァハァ   ∩
               ( ⌒)_   ∩_ _ グッジョブ!!
グッジョブ!! .___  //,. ノ≧ \ .i .,,E)__
     / nCr  \| / /\ ./ |/ / cos \
  _n  .|::::\ ./  |/ /(● (● | ノ\ ./ |
 ( l  |::●) ●) .| /:::... .ワ ....ノ/(● (● |   グッジョブ!!
  \ \ヽ:::::.∀   .ノ      /ヽ:::::... .▽....ノ  n
   ヽ__ ̄   ノ ヽ      |  ̄     \    ( E)
     /    /    \    ヽ フ    / ヽ ヽ_//
673132人目の素数さん:2012/10/18(木) 22:50:09.75
>>662

[1]                >>160 >>170-171
x>0, y>0, z>0 のとき、
 (x^2 +y^2 +z^2)^2・(1/x + 1/y + 1/z) ≧ (x+y+z)^3,

[2]
 (|a|+|b|+|c|)・(1/|a+b| + 1/|b+c| + 1/|c+a|) ≧ 9/2,

[4]
a>0, b>0, c>0, a+b+c=s のとき、
 (b^2 + c^2)/(as) + (c^2 + a^2)/(bs) + (a^2 + b^2)/(cs) の最小値

[5]
 a_1 + a_2 + ・・・・ + a_n = 0,
ある k があって、
 a_j ≦ 0 (j≦k)
 a_j > 0 (j>k)
のとき、
 a_1 + 2・a_2 + ・・・・ + n・a_n > 0,

[6]
x>0, y>0, (1+x)(1+y) = 2 のとき、
 xy ≦ (√2 - 1)^2,
 xy + 1/(xy) ≧ 6,
674132人目の素数さん:2012/10/19(金) 00:12:54.80
>>673

[2] 相乗・調和平均より
 1/|a+b| + 1/|b+c| + 1/|c+a| ≧ 9/(|a+b|+|b+c|+|c+a|)
   ≧ 9/{2(|a|+|b|+|c|)},

[6]
 2 = (1+x)(1+y)
  = 1 +(x+y) + xy
  ≧1 +2√(xy) + xy
  = {1+√(xy)}^2
∴ (√2 -1)^2 ≧ xy,

 1/(xy) -6 +xy = {1 -6xy +(xy)^2}/(xy)
  ≧{(√2 +1)^2 -xy}{(√2 -1)^2 -xy}/(xy)
  ≧ 0,
675あのこうちやんは始皇帝だった:2012/10/19(金) 07:10:18.35
          __ノ)-'´ ̄ ̄`ー- 、_
        , '´  _. -‐'''"二ニニ=-`ヽ、
      /   /:::::; -‐''"        `ーノ
     /   /:::::/           \
     /    /::::::/          | | |  |
     |   |:::::/ /     |  | | | |  |
      |   |::/ / / |  | ||  | | ,ハ .| ,ハ|
      |   |/ / / /| ,ハノ| /|ノレ,ニ|ル' 
     |   |  | / / レ',二、レ′ ,ィイ|゙/   
.     |   \ ∠イ  ,イイ|    ,`-' |      
     |     l^,人|  ` `-'     ゝ  |        何時もおんなじ事を書く
      |      ` -'\       ー'  人           馬鹿で無能のこうちゃんは
    |        /(l     __/  ヽ、            やっぱり只の糞キチガイ
     |       (:::::`‐-、__  |::::`、     ヒニニヽ、           ネコも大して変わらない   
    |      / `‐-、::::::::::`‐-、::::\   /,ニニ、\            反論出来ないこうちゃんは
   |      |::::::::::::::::::|` -、:::::::,ヘ ̄|'、  ヒニ二、 \             誰もが認めるクズでカス
.   |      /::::::::::::::::::|::::::::\/:::O`、::\   | '、   \
   |      /:::::::::::::::::::/:::::::::::::::::::::::::::::'、::::\ノ  ヽ、  |
  |      |:::::/:::::::::/:::::::::::::::::::::::::::::::::::'、',::::'、  /:\__/‐、
  |      |/:::::::::::/::::::::::::::::::::::::::::::::::O::| '、::| く::::::::::::: ̄|
   |     /_..-'´ ̄`ー-、:::::::::::::::::::::::::::::::::::|/:/`‐'::\;;;;;;;_|
   |    |/::::::::::::::::::::::\:::::::::::::::::::::::::::::|::/::::|::::/:::::::::::/
    |   /:::::::::::::::::::::::::::::::::|:::::::::::::::::::::O::|::|::::::|:::::::::::::::/
676132人目の素数さん:2012/10/19(金) 08:04:47.21
>>668
じゃあ
この証明を紹介している本とか教えて下さい。
677132人目の素数さん:2012/10/19(金) 09:46:38.07
おそらく、mathematical excursion

秒殺の解法のヒント

肩の荷を降ろそう
678132人目の素数さん:2012/10/19(金) 13:27:33.67
>>677
何ですか、それは?
679132人目の素数さん:2012/10/19(金) 15:07:59.28
a_{n+1}=2(a_n)^2 (n≧1) は, どう解くかね?
680132人目の素数さん:2012/10/19(金) 15:15:16.25
不等式じゃ無い
681132人目の素数さん:2012/10/19(金) 16:13:39.86
要するに、対数を取れってことだよ。
682132人目の素数さん:2012/10/19(金) 16:50:00.47
このスレも馬鹿が多いな
683132人目の素数さん:2012/10/19(金) 17:08:34.17
おま
684132人目の素数さん:2012/10/19(金) 17:20:18.64
>>624 e^x≧1+xを使えば, 秒殺!

x= π/e−1 とおくと、
e^{π/e−1} ≧ π/e
よって、
 e^π ≧π^e

これって, かなり有名ですわ。

http://www.wilmott.com/messageview.cfm?catid=26&threadid=38623

結局, 671の方のやり方ですわ。
685132人目の素数さん:2012/10/19(金) 18:35:32.59
>>679>>681
等式と不等式の区別も付かない阿呆ども
686132人目の素数さん:2012/10/19(金) 18:50:01.10
>>679
スレ違いだろ馬鹿
687132人目の素数さん:2012/10/19(金) 18:54:12.54
( ;゚д)ザワ(;゚д゚;)ザワ(д゚; )
688あのこうちやんは始皇帝だった:2012/10/19(金) 19:29:02.31

 60代の、無職の、女性恐怖症の、頭デッカチの虚弱児・ひ弱の、関西の、ゴミ・クズ・カス・無能・虫けらのクソガキ!

 死ね!!!!!!!!!!!!!!!!!!!!!!!!!!!!!!!!!!!!!!!!!!!!!!!!!!!!!!!!!!!!!
689あのこうちやんは始皇帝だった:2012/10/19(金) 20:30:26.28
          __ノ)-'´ ̄ ̄`ー- 、_
        , '´  _. -‐'''"二ニニ=-`ヽ、
      /   /:::::; -‐''"        `ーノ
     /   /:::::/           \
     /    /::::::/          | | |  |
     |   |:::::/ /     |  | | | |  |
      |   |::/ / / |  | ||  | | ,ハ .| ,ハ|
      |   |/ / / /| ,ハノ| /|ノレ,ニ|ル' 
     |   |  | / / レ',二、レ′ ,ィイ|゙/   
.     |   \ ∠イ  ,イイ|    ,`-' |      
     |     l^,人|  ` `-'     ゝ  |        何時もおんなじ事を書く
      |      ` -'\       ー'  人           馬鹿で無能のこうちゃんは
    |        /(l     __/  ヽ、            やっぱり只の糞キチガイ
     |       (:::::`‐-、__  |::::`、     ヒニニヽ、           ネコも大して変わらない   
    |      / `‐-、::::::::::`‐-、::::\   /,ニニ、\            反論出来ないこうちゃんは
   |      |::::::::::::::::::|` -、:::::::,ヘ ̄|'、  ヒニ二、 \             誰もが認めるクズでカス
.   |      /::::::::::::::::::|::::::::\/:::O`、::\   | '、   \
   |      /:::::::::::::::::::/:::::::::::::::::::::::::::::'、::::\ノ  ヽ、  |
  |      |:::::/:::::::::/:::::::::::::::::::::::::::::::::::'、',::::'、  /:\__/‐、
  |      |/:::::::::::/::::::::::::::::::::::::::::::::::O::| '、::| く::::::::::::: ̄|
   |     /_..-'´ ̄`ー-、:::::::::::::::::::::::::::::::::::|/:/`‐'::\;;;;;;;_|
   |    |/::::::::::::::::::::::\:::::::::::::::::::::::::::::|::/::::|::::/:::::::::::/
    |   /:::::::::::::::::::::::::::::::::|:::::::::::::::::::::O::|::|::::::|:::::::::::::::/
690132人目の素数さん:2012/10/19(金) 20:52:52.04
Hey!
Do not be angry.
Are you NONKE?
NONKE is very good.
691132人目の素数さん:2012/10/19(金) 21:34:23.67
I am wrong.

NONKI is correct.
692132人目の素数さん:2012/10/19(金) 21:38:32.78
686>> 考え方は同じだろ, がきみたいなこといってんじゃねえ ばーーか
693132人目の素数さん:2012/10/19(金) 23:26:10.62
>>673

[4]
コーシーで
 (2s)s・(与式) = (a+a+b+b+c+c)s・(与式) ≧ (2s)^2
∴ (与式) ≧ 2,

(別解)
r≦0 または r≧1 のとき Jensenの不等式で
 a^r+ b^r + c^r ≧ 3(s/3)^r, >>170-171

これとチェビシェフで
 (与式) ≧ (2/3)(a^2 +b^2 +c^2)(1/a + 1/b + 1/c)(1/s)
     ≧ (2/3)(1/3)s^2 (9/s)(1/s) 
     = 2,

[2] は相加・調和平均だった....orz
694132人目の素数さん:2012/10/19(金) 23:38:13.80
          __ノ)-'´ ̄ ̄`ー- 、_
        , '´  _. -‐'''"二ニニ=-`ヽ、
      /   /:::::; -‐''"        `ーノ
     /   /:::::/           \
     /    /::::::/          | | |  |
     |   |:::::/ /     |  | | | |  |
      |   |::/ / / |  | ||  | | ,ハ .| ,ハ|
      |   |/ / / /| ,ハノ| /|ノレ,ニ|ル' 
     |   |  | / / レ',二、レ′ ,ィイ|゙/   私は只の数ヲタなんかとは付き合わないわ。
.     |   \ ∠イ  ,イイ|    ,`-' |      頭が良くて数学が出来てかっこいい人。それが必要条件よ。
     |     l^,人|  ` `-'     ゝ  |        さらに Ann.of Math に論文書けば十分条件にもなるわよ。
      |      ` -'\       ー'  人          一番嫌いなのは論文数を増やすためにくだらない論文を書いて
    |        /(l     __/  ヽ、           良い論文の出版を遅らせるお馬鹿な人。
     |       (:::::`‐-、__  |::::`、     ヒニニヽ、         あなたの論文が Ann of Math に accept される確率は?
    |      / `‐-、::::::::::`‐-、::::\   /,ニニ、\            それとも最近は Inv. Math. の方が上かしら?
   |      |::::::::::::::::::|` -、:::::::,ヘ ̄|'、  ヒニ二、 \
.   |      /::::::::::::::::::|::::::::\/:::O`、::\   | '、   \
   |      /:::::::::::::::::::/:::::::::::::::::::::::::::::'、::::\ノ  ヽ、  |
  |      |:::::/:::::::::/:::::::::::::::::::::::::::::::::::'、',::::'、  /:\__/‐、
  |      |/:::::::::::/::::::::::::::::::::::::::::::::::O::| '、::| く::::::::::::: ̄|
   |     /_..-'´ ̄`ー-、:::::::::::::::::::::::::::::::::::|/:/`‐'::\;;;;;;;_|
   |    |/::::::::::::::::::::::\:::::::::::::::::::::::::::::|::/::::|::::/:::::::::::/
    |   /:::::::::::::::::::::::::::::::::|:::::::::::::::::::::O::|::|::::::|:::::::::::::::/
695132人目の素数さん:2012/10/20(土) 01:46:52.57
>>645
 a_k = (2k)!!/(2k-1)!!
 b_k = (2k-1)!!/(2k-2)!!
とおくと
 a_k・b_k = 2k,
 a_k・b_(k+1) = 2k+1,
また
 a_k 〜 √{(4k+1)π/4}
 b_k 〜 √{(4k-1)/π}


>>646
 2^a ≧ 2 + A,
 2^b ≧ 2 + B,
 2^c ≧ 2 + C,
とおく。題意より A,B,C ≧0
 (左辺) - (右辺)
 = (2+A)(2+B)(2+C) -(2+A)(2+B) -(2+B)(2+C) -(2+C)(2+A) +4
 = ABC + AB + BC + CA
 ≧ 0,
696あのこうちやんは始皇帝だった:2012/10/20(土) 07:38:21.37
          __ノ)-'´ ̄ ̄`ー- 、_
        , '´  _. -‐'''"二ニニ=-`ヽ、
      /   /:::::; -‐''"        `ーノ
     /   /:::::/           \
     /    /::::::/          | | |  |
     |   |:::::/ /     |  | | | |  |
      |   |::/ / / |  | ||  | | ,ハ .| ,ハ|
      |   |/ / / /| ,ハノ| /|ノレ,ニ|ル' 
     |   |  | / / レ',二、レ′ ,ィイ|゙/   
.     |   \ ∠イ  ,イイ|    ,`-' |      
     |     l^,人|  ` `-'     ゝ  |        何時もおんなじ事を書く
      |      ` -'\       ー'  人           馬鹿で無能のこうちゃんは
    |        /(l     __/  ヽ、            やっぱり只の糞キチガイ
     |       (:::::`‐-、__  |::::`、     ヒニニヽ、           ネコも大して変わらない   
    |      / `‐-、::::::::::`‐-、::::\   /,ニニ、\            反論出来ないこうちゃんは
   |      |::::::::::::::::::|` -、:::::::,ヘ ̄|'、  ヒニ二、 \             誰もが認めるクズでカス
.   |      /::::::::::::::::::|::::::::\/:::O`、::\   | '、   \
   |      /:::::::::::::::::::/:::::::::::::::::::::::::::::'、::::\ノ  ヽ、  |
  |      |:::::/:::::::::/:::::::::::::::::::::::::::::::::::'、',::::'、  /:\__/‐、
  |      |/:::::::::::/::::::::::::::::::::::::::::::::::O::| '、::| く::::::::::::: ̄|
   |     /_..-'´ ̄`ー-、:::::::::::::::::::::::::::::::::::|/:/`‐'::\;;;;;;;_|
   |    |/::::::::::::::::::::::\:::::::::::::::::::::::::::::|::/::::|::::/:::::::::::/
    |   /:::::::::::::::::::::::::::::::::|:::::::::::::::::::::O::|::|::::::|:::::::::::::::/
697132人目の素数さん:2012/10/21(日) 03:09:00.49
>>662
Proof of Inequality 120 が最新だけど、古いやつを見ようとしたら全部消されていた…
こないだまで見れたのにな ('A`)ヴォエァ!
698132人目の素数さん:2012/10/21(日) 03:17:35.94
     //                //
.    / /     ∩          //
   / / ∩ 〜'::""::''ヽ、      / /
  // /r‐、 ___    \    / / うわぁぁあああぁあぁぁぁ
// / ノ●_)(_●つ    ヽ、.. l l
/ / (__ ノ´         } .| | ダメだぁぁああぁ もうダメだぁぁぁ・・
  /      ( _●_) 彡-、  ノ | |
 {    -_二 -‐'' ̄     ) ミ ノ ノ
 ヽ  (    ,r'' " ''‐-‐′ノl//
  彡  `― ''′     ノ//
              //
699132人目の素数さん:2012/10/21(日) 11:29:46.07
あっちの板で出てたけど、
pi^(pi^e)>e^(e^pi)ってどう説明できる?
700132人目の素数さん:2012/10/21(日) 14:54:42.07
>>697
マジかYO!
見つけたときに保存しておくべきだったな
701132人目の素数さん:2012/10/21(日) 22:43:27.68
不等式関係以外残してるの見ると十中八九このスレが原因だな
何が気に入らなかったんだ?
702132人目の素数さん:2012/10/21(日) 22:51:27.30
何でだろうね?
問題の権利を主張したいのかな?
年末年始の時間のあるときに、ジックリ解こうと思って楽しみにしていたのになあ
誰か問題を全部じゃなくてもいいからメモしてる人いないかな?

http://suseum.jp/gd/index/3504
すうじあむ - 投降した人 近谷邦彦さん

たぶん以下の人と同一人物だと思う
http://univ.ena.co.jp/?title=school_guide_ena_kunitachi_h
703132人目の素数さん:2012/10/21(日) 23:14:32.54
>>702
問題解くよりも、問題を作る方が難しいから

研究じゃなくて、単なるゲームになり下がってしまった。
ロールプレイングゲームと同じ、あるいは、それ以下
704132人目の素数さん:2012/10/21(日) 23:19:11.56
不等式120問が消されたなんて、今年最大のショックだな ('A`)ヴォエァ!
705132人目の素数さん:2012/10/21(日) 23:34:01.80
>>703
> >>702
> 問題解くよりも、問題を作る方が難しいから

よく分からん理屈だが、苦労して作ったのを他人に解かれるのが嫌なの?
器が小さいとか言う以前に、ネットで公開した時点で矛盾しているような?
706132人目の素数さん:2012/10/22(月) 00:19:45.88
>>705
そうじゃない
自作の問題を、他人作にされるのが問題なんだろ。
著作権と一緒だよ

707132人目の素数さん:2012/10/22(月) 00:28:43.72
本人が模試の問題とか色々貼ってるし
著作権にあまりうるさくなさそうだけどなあ

あまり不等式に関係ないレスで進行してもしょうがないから控えよ
708132人目の素数さん:2012/10/22(月) 01:14:13.46
エスパー検定3級の俺がリーディングしてみる

 自分が作った or 探してきた不等式を「すうじあむ」で紹介した
   ↓
 自分の知らないところ(2chの不等式スレ)に転載されている
   ↓
 転載するなら一言書いてけよ。なんか面白くねえから消そう

じゃないのかな?
何ヶ月も楽しめる分量だけに、もったいない…
もう消されて見れないんだから、この話は終わりですな
709132人目の素数さん:2012/10/22(月) 01:54:06.09
不等式じゃないけど最大値最小値求める問題は残ってたから

Let $a,\ b,\ c$ be non-negative real numbers such that $a+b+c=9.$
Find the minimum value of $(a^2+4)(b^2+4)(c^2+4).$

Let $a,\ b$ be positive real numbers such that $a^2+b^2=9.$
Find the minimum value of $\displaystyle \frac{2\sqrt{2}}{a}+\frac{1}{b}.$

For $0<x<1,\ 0<y<1$, find the possible maximum value of $\displaystyle \frac{xy(1-x-y)}{(x+y)(1-x)(1-y)}.$

Let $a,b$ be real numbers such that $a^3+b^3=8-6ab$. Find the maximum and minimum value of $a+b$.

Let $a,\ b$ positive real numbers such that $a^3+b^3=5$. Find the range of $a+b$.

Let $a,\ b$ be positive real numbers with $a+b=1$.
Find the minimum value of
$$\displaystyle A=\frac{1}{a^3+b^3}+\frac{1}{ab}.$$

If $a\geq0$,$b\geq0$,$c\geq0$ and $a+2b+c=4$ then find the maximum value of $ab+bc+ca$.

Let $a,\ b,\ c$ be positive real numbers such that $a+b+c=1$.\ \\
Find the minimum value of the expression:\ \\
$$\displaystyle a\left(\frac{c}{b}+\frac{a}{c}\right)+b\left(\frac{a}{c}+\frac{b}{a}\right)+c\left(\frac{b}{a}+\frac{c}{b}\right).$$
710132人目の素数さん:2012/10/22(月) 01:56:38.64
>>709
激しく乙!
711132人目の素数さん:2012/10/22(月) 03:12:25.23
また消された…
712132人目の素数さん:2012/10/22(月) 03:21:52.76
もしかしてproof of inequality復活してる?
713132人目の素数さん:2012/10/22(月) 03:27:03.94
してるよ。

ちなみに例のサイトのファイル保存方法↓
例)
http://suseum.jp/gq/question/abcd
なら、http://suseum.jp/ 以下を /gq/swf_data/abcd としてやればswfファイル単体で切り出せる。

コメント部分は保存できないから、ページとして保存した方がいいのかも。
他のうまいやり方知ってる人がいれば教えてちょ。
714132人目の素数さん:2012/10/22(月) 03:50:16.16
ガラケーからだから俺には無理そうだな
715132人目の素数さん:2012/10/22(月) 06:31:16.07
むむっ、復活していますね
次のようにしたら、コメントごと保存できると思います

ファイル > 名前をつけて保存 > Webアーカイブ、単一のファイル(*.mht) > 保存


┏━━━━━━━━━━━━━━━━━━━━━━━━━━━━━━━━━━┓
┃−−−−−−−−−−−−−−−−−−−−−−−−−−−−−−−−−−┃
┃麻呂用しおり     | 三シ  ヾミ      彡シ  ヾ三 | ピキーン!!               ┃
┃             | 三| -丶、.,_ノ 'i'´(_,,/`_,. i三 |                       ┃
┃_________ト、ニ| <でiンヽ  ;'i"∠でiン |三|._∧,、_________○┃
┃ ̄ ̄ ̄ ̄ ̄ ̄ ̄ ̄ ̄',.iヽ!i ヾ`= ‐' / 、 `ー´ i|シ,イ ̄'`'` ̄ ̄ ̄ ̄ ̄ ̄ ̄ ̄ ̄  ┃
┃              i,ヽリi      ,':  :、     i|f. ノ      復活祭とな !?    ┃
┃−−−−−−−−−−−−−−−−−−−−−−−−−−−−−−−−−−┃
┗━━━━━━━━━━━━━━━━━━━━━━━━━━━━━━━━━━┛
716132人目の素数さん:2012/10/22(月) 07:40:52.75
          __ノ)-'´ ̄ ̄`ー- 、_
        , '´  _. -‐'''"二ニニ=-`ヽ、
      /   /:::::; -‐''"        `ーノ
     /   /:::::/           \
     /    /::::::/          | | |  |
     |   |:::::/ /     |  | | | |  |
      |   |::/ / / |  | ||  | | ,ハ .| ,ハ|
      |   |/ / / /| ,ハノ| /|ノレ,ニ|ル' 
     |   |  | / / レ',二、レ′ ,ィイ|゙/   
.     |   \ ∠イ  ,イイ|    ,`-' |      
     |     l^,人|  ` `-'     ゝ  |        このスレ 馬と鹿と豚ばかりね。
      |      ` -'\       ー'  人          
    |        /(l     __/  ヽ、          
     |       (:::::`‐-、__  |::::`、     ヒニニヽ、         
    |      / `‐-、::::::::::`‐-、::::\   /,ニニ、\            
   |      |::::::::::::::::::|` -、:::::::,ヘ ̄|'、  ヒニ二、 \
.   |      /::::::::::::::::::|::::::::\/:::O`、::\   | '、   \
   |      /:::::::::::::::::::/:::::::::::::::::::::::::::::'、::::\ノ  ヽ、  |
  |      |:::::/:::::::::/:::::::::::::::::::::::::::::::::::'、',::::'、  /:\__/‐、
  |      |/:::::::::::/::::::::::::::::::::::::::::::::::O::| '、::| く::::::::::::: ̄|
   |     /_..-'´ ̄`ー-、:::::::::::::::::::::::::::::::::::|/:/`‐'::\;;;;;;;_|
   |    |/::::::::::::::::::::::\:::::::::::::::::::::::::::::|::/::::|::::/:::::::::::/
    |   /:::::::::::::::::::::::::::::::::|:::::::::::::::::::::O::|::|::::::|:::::::::::::::/
717132人目の素数さん:2012/10/22(月) 07:42:11.81
 ε⌒ ヘ⌒ヽフ
(   (  ・ω・) ブヒ
  しー し─J
718132人目の素数さん:2012/10/22(月) 08:41:05.68
もう保存したか用無し
消していいよ
719132人目の素数さん:2012/10/22(月) 08:48:38.62
>>718
そんなこと言うから消されるんだよ
感謝しろよ!
720132人目の素数さん:2012/10/22(月) 10:22:12.66
721323人目の描 ◆ghclfYsc82 :2012/10/22(月) 12:29:10.39
おお、相変わらずで、馬鹿板が本領を発揮してるがな。

ケケケ描
722132人目の素数さん:2012/10/22(月) 14:26:49.99
元ネタは大学入試問題

n ≧ 3 のとき次の不等式が成り立つことを示せ.( e はネイピアの数 )

Π[k = 0, n-1] { ( nC2 - k ) / nC2} <1/e
723132人目の素数さん:2012/10/22(月) 14:48:39.99
>>720
2012年・第11回中国女子数学オリンピック

正の整数n、非負実数a_1、…、a_nに対して、次の不等式にハァハァせよ
1/(1+a_1) + a_1/{(1+a_1)(1+a_2)} + a_1・a_2…a_n/{(1+a_1)(1+a_2)…(1+a_n)} ≦ 1
724333目の描 ◆ghclfYsc82 :2012/10/22(月) 17:45:35.75


>アホしかいない
>つまり、増田哲也自身がアホ
>
>しかも増田哲也は性犯罪者であり
>アホの中でも最底辺の者にだけ許される ジ・アホの称号を持っている
>
725132人目の素数さん:2012/10/22(月) 22:43:57.08
>>709
〔問題〕

(1) a,b,c は非負実数で、 a+b+c = 9 のとき、
 1360 ≦ (a^2 +4)(b^2 +4)(c^2 +4) ≦ 2352.25 を示せ。

(2) a,b は正の実数で a^2 +b^2 = 9 のとき、
  (2√2)/a + 1/b ≧ √3 を示せ。{等号成立は a=√6, b=√3}.

(3) 0<x<1, 0<y<1 のとき、{xy(1-x-y)}/{(x+y)(1-x)(1-y)} の最大値を求めよ。

(4) a,bは実数で、a^3 +b^3 +(-2)^3 -3ab(-2) = 0 のとき、
  a+b = -4 または 2 を示せ。

(5) a,bは正の実数で、a^3 + b^3 = 5 のとき、
  5^(1/3) < a+b ≦ 20^(1/3) を示せ。

(6) a,bは正の実数で、a+b=s のとき、
  A = s^3/(a^3+b^3) + s^2/ab ≧ (1+√3)^2 を示せ。{等号は ab/ss = 1/(3+√3)}

(7) a≧0, b≧0, c≧0, a+2b+c=4 のとき、
  ab+bc+ca ≦ 4 を示せ。{等号成立は a=c, b=0}

(8) a,b,c は正の実数で、a+b+c=1 のとき、
  a(c/b + a/c) + b(a/c + b/a) + c(b/a +c/b) の最小値を求めよ。
726132人目の素数さん:2012/10/22(月) 22:52:43.46
>>725
 解凍例

(1)
 {0, 0, 9} のとき最小 1360
 {0, 9/2, 9/2} のとき最大 2352.25

(2)
コーシーより
 (a√2 + b)・(与式) ≧ (2+1)^2 = 9,
また
 (a√2 + b)^2 + (a-b√2)^2 = 3(a^2 + b^2) = 3r^2,
 |a√2 + b| ≦ (√3)r,
辺々割って
∴ (与式) ≧ 9/(a√2 +b) ≧ 9/{(√3)r},
題意により、r=3

(4) c=-2 とおくと、
 a^3 + b^3 + c^3 -3abc = (a+b+c){(a-b)^2 + (b-c)^2 + (c-a)^2}/2,
題意より、a+b+c=0 または a=b=c,
727132人目の素数さん:2012/10/22(月) 22:54:32.61
>>725
 解凍例

(5) 
 (1/4)(a+b)^3 ≦ a^3 + b^3 ≦ (a+b)^3,
 {左等号は a=b, 右等号は ab=0}

(6)
 ab/(s^2) = t とおくと、題意より 0<t≦1/4,
 A = (s^3)/(a^3 + b^3) + (s^2)/(ab)
  = (s^2)/(s^2 -3ab) + (s^2)/(ab)
  = 1/(1-3t) + 1/t
  = (1-2t)/{(1-3t)t}
  = (1+√3)^2 + [1-(3+√3)t]^2/{(1-3t)t}
  ≧(1+√3)^2,
 {等号は t = (3-√3)/6, a,b = (1/2) ± √{(√3)/2}・t のとき}

(7)
 (与式) = (1/4){(a+2b+c)^2 -(a-c)^2 -4b^2}
    ≦ (1/4)(a+2b+c)^2,
 {等号は a=c, b=0 のとき}
728132人目の素数さん:2012/10/22(月) 23:07:46.17
>>723

a_1・a_2…a_(k-1)/{(1+a_1)(1+a_2)…(1+a_k)}
= a_1・a_2…a_(k-1)/{(1+a_1)(1+a_2)…[1+a_(k-1)]} - a_1・a_2…a_k/{(1+a_1)(1+a_2)…(1+a_k)},

 telescoping, ハァハァ
729132人目の素数さん:2012/10/22(月) 23:12:37.13
>>647、655

(∫[0、π] √(1 - sin^3 x) dx)^2
 ≦ (∫[0、π] 1^2 dx)・{∫[0、π] (1 - sin^3 x) dx}
 = (π/2)・(π/2 - 2/3)
 = …
 < (1.25)^2

∫[0、π] x・√(1 - sin^3 x) dx
 = π∫[0、π] √(1 - sin^3 x) dx
 < 3.2・1.25
 = 4

( ゚∀゚)プケラッチョ!
730β:2012/10/22(月) 23:43:35.80
簡単すぎ

もっと骨のある問題だしたら?
731333人目の描 ◆ghclfYsc82 :2012/10/23(火) 14:25:55.10


>192 名前:132人目の素数さん :2012/10/23(火) 11:55:56.36
> >>187
> その運営と予算獲得に『すら』関心を示さずに
> 女性の股間にだけ関心を持った猫先生は
> 『研究のアクティビティ』とは無縁だったね。
> 『女性のティクビ』は好きだったんだろうけど。
>
732132人目の素数さん:2012/10/23(火) 14:28:59.17
私は某女子短大で教えているが、女子学生は全員全裸になり、
学生証を安全ピンで乳首に刺して止めておくべきだ。血が出るかも。
やらなければこちらがブスッと刺す。
クリスマスは私と女子学生の乱交パーティーだ

等と云った妄想を毎日している。
733132人目の素数さん:2012/10/23(火) 14:31:56.11
せっかく削除したのに復活とか、バカじゃね?
734132人目の素数さん:2012/10/23(火) 18:06:19.76
>>733
口を慎み給へ、君はラピュタ王の前にいるのだぞ!
735132人目の素数さん:2012/10/23(火) 19:56:55.19
新しいの来てた
121
Let $a,\ b,\ c$ be non-negative real numbers such that $ab+bc+ca=1$.
Prove that :
$$\frac{1}{a+b}+\frac{1}{b+c}+\frac{1}{c+a}\geq \frac 52.$$

http://suseum.jp/gq/question/1863
736132人目の素数さん:2012/10/23(火) 20:27:17.47
さっぱり分からねぇ
737132人目の素数さん:2012/10/23(火) 21:31:47.09
\geq \frac 52. って?
738132人目の素数さん:2012/10/23(火) 21:42:40.21
>>737
TeX で処理すれば
  \geq → ≧
  \frac52 → 5/2
739132人目の素数さん:2012/10/23(火) 21:45:51.32
\frac{5}{2}じゃなくてもいけるのか
ありがとう
740132人目の素数さん:2012/10/23(火) 22:12:49.99
しかし糞みたいな問題ばかりだな
741132人目の素数さん:2012/10/23(火) 22:48:36.83
そうだな
742132人目の素数さん:2012/10/24(水) 00:22:56.69
>>725 (1)

{a,b,c} = {a,a,9-2a} については、   (0≦a≦9/2)

(a^2 +4)(a^2 +4){(9-2a)^2 +4} = (17^3)/4 + (2a-1)^2 {a^4 -8a^3 +21a^2 -49a + 527/4}
 = (17^3)/4 + (2a-1)^2 {(a^2)(a-4)^2 +5a^2 -49a + 527/4}
 = (17^3)/4 + (2a-1)^2 {(a^2)(a-4)^2 +5(a-4.9)^2 + 117/10}
 ≧(17^3)/4
 = 1228.25  ・・・・ 最小値 (a=1/2)
743132人目の素数さん:2012/10/24(水) 21:43:56.92
>>742
蛇足だが.....

 a^4 -8a^3 +21a^2 -49a + 527/4 は a 〜 4+(1/9)√3 付近に極小をもつ。

 a^4 -8a^3 +21a^2 -49a + 527/4
 = 14.85284737 + 0.000303782a + {a-4 -(1/9)√3}^2・{[a+(1/9)√3]^2 +5 +2(1+12√3)/27}
 > 14.85284737 + 0.000303782a

なお、極小値は 14.85412096 @ a=4.19244421181163
744132人目の素数さん:2012/10/25(木) 00:16:07.68
>>725 (8)
 a+b+c=s のとき、
 (与式) = (b+c)b/a + (a+c)c/b + (b+a)a/c
    = s(b/a + c/b + a/c -1)
    ≧ 2s,
745132人目の素数さん:2012/10/25(木) 08:48:08.32

Let $x,\ y,\ z$ be positive real numbers with $x+y+z=1$.
Prove that : $$(x^2+y^2+z^2)^2\left(\frac{1}{x}+\frac{1}{y}+\frac{1}{z}\right)\geq 1.$$

Let $a,\ b,\ c$ be real numbers such that $\displaystyle \frac{1}{|a+b|}+\frac{1}{|b+c|}+\frac{1}{|c+a|}=3.$
Prove that :
$$\displaystyle |a|+|b|+|c|\geq \frac {3}{2}.$$

For positive real numbers $a,\ b,\ c$ with $a+b+c=2010$, find the minimum value of the expression:
\[\frac{b^2+c^2}{a}+\frac{c^2+a^2}{b}+\frac{a^2+b^2}{c}\]

$a_1 + a_2 + \cdots + a_n = 0$, for some $k$ we have $a_j\leq 0$ for $j\leq k$ and $a_j \geq 0$ for $j > k$. \
If $a_i$ are not all 0, show that $a_1 + 2a_2 + 3a_3 + ... + na_n > 0.$

Let $ x$ and $ y$ positive real numbers such that $ (1+x)(1+y)=2$. \ \\
Prove that?: $$\ \displaystyle xy+\frac{1}{xy}\geq\ 6.$$

Let $a,\ b,\ c$ be positive real numbers which are less than 1.
Prove that : $$a+b+c-abc<2.$$

Let $n\geq 2$ and let $0<x_i<1\ (i=1,\ 2,\ \cdots ,\ n).$\ \\
Prove that :$$(1-x_1)(1-x_2)\cdots (1-x_n)+x_1+x_2+\cdots +x_n>1.$$

Let $a,\ b,\ c$ be positive real numbers with $ab+bc+ca=1.$ \ \\
Prove that : $$\frac{1}{a+b}+\frac{1}{b+c}+\frac{1}{c+a}\geq \sqrt{3}+\frac{ab}{a+b}+\frac{bc}{b+c}+\frac{ca}{c+a}.$$

Let $a, b$ and $c$ be the lengths of the sides of a triangle such that $ab+bc+ca=1$. \ \\
Prove that : $$(a+1)(b+1)(c+1) < 4.$$
10
For $x>1,\ y>1$, prove that : $$\frac{x^2}{y-1}+ \frac{y^2}{x-1}\geq 8.$$
746132人目の素数さん:2012/10/25(木) 08:50:51.13
11
Let $a,\ b,\ c$ be positive real numbers.\ \\
Prove that $$\log_2 \frac{(a+b)^{671}(b+c)^{671}(c+a)^{671}}{(a^{671}+b^{671})(b^{671}+c^{671}
)(c^{671}+a^{671})}\leq 2010.$$
12
Find the possible maximum value of $a$ such that:
\[\sqrt{x^2+y^2}\geq x+y+a\sqrt{xy}\]
holds for all positive real numbers $x,\ y$.
13
For positive real numbers $a,\ b,\ c$ with $a+b+c=abc$,
Prove that:
$$\frac{1}{(a+b)(b+c)}+\frac{1}{(b+c)(c+a)}+\frac{1}{(c+a)(a+b)}\leq \frac 14$$
14
Let $a,b,c$ be positive real numbers.
Prove that :
$$\frac{a}{a^4+a^2+1}+\frac{b}{b^4+b^2+1}+\frac{c}{c^4+c^2+1}\leq \frac{1}{3}\left(\frac{a}{bc}+\frac{b}{ca}+\frac{c}{ab}\right)$$.
15
Let $ a,\ b,\ c$ be positive real numbers.
Prove that :
$$ \frac {1}{\sqrt {ab}}+\frac {1}{\sqrt {bc}}+\frac{1}{\sqrt {ca}}+2\sqrt {3}\sqrt {a+b+c}\geq 9.$$
16
Let $a,\ b,\ c$ be positive real numbers.
Prove that :
$$\frac{a^{2}}{a+b}+\frac{b^{2}}{b+c}+\frac{c^{2}}{c+a}\geq \frac{\sqrt{2}}{4}\left(\sqrt{a^{2}+b^{2}} +\sqrt{b^{2}+c^{2}}+\sqrt{c^{2}+a^{2}}\right)$$17
Let $a,b,c,d$ be positive real numbrs such that $a+b+c+d=1$.
Prove that :
$$\frac{1}{4a+3b+c}+\frac{1}{3a+b+4d}+\frac{1}{a+4c+3d}+\frac{1}{4b+3c+d} \ge 2.$$
18
Prove the following inequality.
\[\int_1^e \frac{(\log_e x)^{2009}}{x^2}dx>\frac{1}{2010\cdot 2011\cdot2012}\]
747132人目の素数さん:2012/10/25(木) 08:53:29.41
19
Let $a,\ b,\ c$ be positive constants.
(1) If the sum of positive real numbers $x,\ y$ equals to a constant $h$, then
Prove that : $$\frac{a^2}{x}+\frac{b^2}{y}\geq \frac{(a+b)^2}{h}.$$
(2) If the sum of positive real numbers $x,\ y,\ z$ equals to a constant $k$, then
use the previous result to find the minimum value of $$\frac{a^2}{x}+\frac{b^2}{y}+\frac{c^2}{z}.$$
20
Given real numbers $a,\ b,\ c$ with $abc=1$.
Prove that:
$$\frac {1}{a^{3}\left(b + c\right)} + \frac {1}{b^{3}\left(c + a\right)} + \frac {1}{c^{3}\left(a + b\right)}\geq \frac {3}{2}.$$
21
Given positive real numbers $a, b, c$ with $a^2+b^2+c^2+(a+b+c)^2\leq4$.
Prove that :
$$\frac{ab+1}{(a+b)^2}+\frac{bc+1}{(b+c)^2}+\frac{ca+1}{(c+a)^2}\geq 3.$$
22
Let $a,\ b,\ c$ be positive real numbers. Prove the following inequality.
$$2\left(\frac{a+b}{2}-\sqrt{ab}\right)\leq 3\left(\frac{a+b+c}{3}-\sqrt[3]{abc}\right)$$
23
Given real numbers $x,y,z$ with $x+y+z=0$.
Prove that :
\[\frac{x(x+2)}{2x^2+1}+\frac{y(y+2)}{2y^2+1}+\frac{z(z+2)}{2z^2+1}\geq 0.\]
24
Given real numbers $x_{1} \geq x_{2} \geq \cdots \geq x_{n}$ and $\ y_{1} \geq y_{2} \geq \cdots \geq y_{n}.$
Let $z_{1}, z_{2},\ \cdots, z_{n}$ be a permutation of the numbers $y_{1}, y_{2}, \cdots, y_{n}.$
Prove that :
$$\sum_{i=1}^{n} ( x_{i} -\ y_{i} )^{2} \leq \sum_{i=1}^{n} ( x_{i} - z_{i})^{2}.$$
25
Let $a,\ b,\ c$ be real numbers such that $a^2b^2+b^2c^2+c^2a^2+3(a^2+b^2+c^2)=12.$
Prove that:
$$a+b+c+2(ab+bc+ca)\leq 9.$$
748132人目の素数さん:2012/10/25(木) 08:54:46.26
26
For $a,\ b$ with $0<a<b$, Prove that :
$$\int_a^b (x^2+1)e^{-x^2}dx\geq e^{-a^2}-e^{-b^2}$$
27
Let $a,\ b,\ c,\ d$ be real numbers such that $0<a<1,\ 0<b<1,\ 0<c<1,\ 0<d<1$ and
$a+b+c+d=2\sqrt{3}$.
Prove that :
\[\sqrt{1-a^2}+\sqrt{1-b^2}+\sqrt{1-c^2}+\sqrt{1-d^2}\leq 2\]
28
Let $x,\ y,\ z$ be real numbers such that $\displaystyle 0\leq x<\frac{\pi}{4},\ 0\leq y<\frac{\pi}{4},\ 0\leq z<\frac{\pi}{4}.$
Prove the following inequality.
$$\sqrt[3]{\tan x\tan y\tan z}\leq \tan \frac{x+y+z}{3}\leq \frac 13(\tan x+\tan y+\tan z)$$
29
Let $A,\ B,\ C,\ D\in{(0,\ \pi)}.$ Prove the following inequalities.\ \\
(1) $\displaystyle \sin A+\sin B+\sin C+\sin D\leq 4\sin \frac 14(A+B+C+D)$\ \\
(2) $\displaystyle \sin A+\sin B+\sin C\leq 3\sin \frac 13(A+B+C)$
30
Let $ a,\ b,\ c$ be positive real numbers such that $ a+b+c=1$.
Prove that :
$$ \sqrt{a+bc}+\sqrt{b+ca}+\sqrt{c+ab}\leq 2$$.
31
Given positive real numbers $a,b,c,d$ with $a+b+c+d=1$.
Prove that :
$$6(a^{3}+b^{3}+c^{3}+d^{3})\geq a^{2}+b^{2}+c^{2}+d^{2}+\frac{1}{8}$$
32
Given non-negative real numbers $a,\ b,\ c$ with $a+b+c=1$.
749132人目の素数さん:2012/10/25(木) 10:20:59.97
アニオタきめえええええ
750132人目の素数さん:2012/10/25(木) 15:39:24.17
>>745-748
なんかどれも糞問題ばかりだな

何かもっと骨のある問題や興奮する問題は無いのかよ
751132人目の素数さん:2012/10/25(木) 17:06:45.47
>> 735

なかなかむずかしいなあ
752132人目の素数さん:2012/10/25(木) 17:17:18.08
TeXコマンド解読するのは面倒だから放置
753132人目の素数さん:2012/10/25(木) 17:27:16.78
数式全部$で囲まれてるからコピーして処理すればそのまま読めるぞ
754132人目の素数さん:2012/10/25(木) 17:41:36.16
755132人目の素数さん:2012/10/25(木) 18:28:38.73
>>750
巣に帰れ
756132人目の素数さん:2012/10/25(木) 20:40:18.64
>>752
tex じゃなくて、英語が分かんないんだろw
757132人目の素数さん:2012/10/25(木) 21:03:37.44
多分1回ぐらいTEX触れたことないとキツイだろ
758132人目の素数さん:2012/10/25(木) 23:41:19.66
中学生未満の英語力だが、バッチリ分かるぜ!
考えるな、感じるんだ!
759132人目の素数さん:2012/10/26(金) 20:42:46.97
>>750
>>758
確かにこのレベルの問題なら、数学よりも英語の方が難しいかもな
760132人目の素数さん:2012/10/28(日) 08:21:10.11
33
For $ a>0,\ b>0,\ c>0$, prove that $\displaystyle \frac{a^3+b^3+c^3+6}{ab+bc+ca}\left(\frac{b^2}{a}+\frac{c^2}{b}+\frac{a^2}{c}\right)\geq 9.$
34
Let $ a,\ b,\ c$ be postive real numbers such that $\displaystyle \frac{a^2+b^2}{(a+b)^3}+\frac{b^2+c^2}{(b+c)^3}+\frac{c^2+a^2}{(c+a)^3}=\frac{3}{2}$.
Prove that $\displaystyle a^2+b^2+c^2\geq \frac{3}{4}$.
35
Let $a,\ b,\ c$ be positive real numbers.
Prove that :
\[\left(\frac{1}{a}+b\right)\left(\frac{1}{b}+c\right)\left(\frac{1}{c}+a\right)\left(a+\frac{1}{a}\right)\left(b+\frac{1}{b}\right)\left(c+\frac{1}{c}\right)\geq 64.\]
36
Let $a,\ b,\ c$ be positive real numbers such that $a^ab^bc^c=1$.
Prove that $a+b+c\leq 3$.
37
For $-2\leq a,\ b,\ c\leq 1$ and $a+b+c=0$, prove that $a^2+b^2+c^2\leq 6$.
38
Given positive real numbers $x,\ y,\ z$ with $x+y+z=1$.
Prove that :
\[ x\sqrt[3]{1+y-z}+y\sqrt[3]{1+z-x}+z\sqrt[3]{1+x-y}\leq 1. \]
39
For $\displaystyle 0<x<\frac{\pi}{2}$, prove that $\displaystyle \left (\frac{\sin x}{x} \right )^2 > \cos x >\left( \frac{x}{\tan x}\right)^2.$
40
Let $ a,\ b$ be real numbers with $ 0<a<b$ and $ x,\ y,\ z$ satisfy :
$$a\leq x\leq b,\ a\leq y\leq b,\ a\leq z\leq b$$
If $ x+y+z=a+2b$, then prove that $ xyz\geq ab^2$.
761132人目の素数さん:2012/10/28(日) 08:22:03.88
41
Let $a,b,c$ be real numbers such that $|a+b|+|b+c|+|c+a|=3$.
Prove that $(a+b)^{\frac{2}{3}}+(b+c)^{\frac{2}{3}}+(c+a)^{\frac{2}{3}}\leq 3$.
42
Given positive real numbers $a,\ b,\ c$ with $ab+bc+ca = 1.$
Prove that :
\[ \sqrt[3]{ \frac{1}{a} + 6b} + \sqrt[3]{\frac{1}{b} + 6c} + \sqrt[3]{\frac{1}{c} + 6a } \leq \frac{1}{abc}. \]
43
Given positive real numbers $a,\ b,\ c$ with $ab+bc+ca+abc=4$.
Prove that :
$$a+b+c\geq ab+bc+ca.$$
44
Given real numbers $x, y, z$ such that $x^2 + y^2 + z^2 + 2xyz = 1$.
Prove that :
$$\displaystyle x^2+y^2+z^2\ge \frac{3}{4}.$$
45
Given real numbers $a,b,c$ such that $ab+bc+ca=3abc.$
Prove that :
$$4(a^2+b^2+c^2)+9\ge 21abc$$.
46
Let $a,\ b,\ c,\ x,\ y,\ z$ be positive real numbers.
Prove that :
\[\left(\frac{x}{a^2}+\frac{y}{b^2}+\frac{z}{c^2}\right)(ax+by+cz)^2\geq (x+y+z)^3. \]
47
Let $a,\ b$ be real numbers such that $a>1,\ b>1.$
Prove the following inequality.
\[\int_{-1}^1 \left(\frac{1+b^{|x|}}{1+a^{x}}+\frac{1+a^{|x|}}{1+b^{x}}\right)\ dx<a+b+2\]
48
Let $a,\ b,\ c$ be positive real numbers.
Prove that: \[\frac{a^3}{(a+b)^2}+\frac{b^3}{(b+c)^2}+\frac{c^3}{(c+a)^2}\geq \frac{a+b+c}{4}\]
762132人目の素数さん:2012/10/28(日) 08:22:57.91
49
Given real numbers $a,b,c,d$ with $a+b+c+d=6,\ a^2+b^2+c^2+d^2=12.$
Prove that :
\[36 \leq 4 \left(a^3+b^3+c^3+d^3\right) - \left(a^4+b^4+c^4+d^4 \right) \leq 48.\]
50
Let $a,\ b$ be positive real numbers such that $a+b=1.$
Prove that : $a^{a}b^{b}+a^bb^a\leq 1.$
51
Let $a,\ b$ be positive real numbers.
Prove that :
$$ a^2b^2(a^2+b^2-2) \geq (a+b)(ab-1).$$
52
For $0\leq a\leq 1,\ 0\leq b\leq 1,\ 0\leq c\leq 1$,
Prove that :
$$\frac{a}{bc+1}+\frac{b}{ca+1}+\frac{c}{ab+1}\leq 2.$$
53
For real numbers $a,\ b,\ c$, prove that :
$$a(a+b)^3+b(b+c)^3+c(c+a)^3\ge \frac{8}{27}(a+b+c)^4.$$
54
For any real numbers $a,b,c$, prove that
$5a^2 + 5b^2 + 5c^2 \ge 4ab + 4ac + 4bc $
and determine when equality occurs.
55
For positive real numbers $a,\ b,\ c,\ d$ with $abcd=1.$
Prove that :
$$\displaystyle \frac{1}{a} + \frac{1}{b} + \frac{1}{c} + \frac{1}{d} + \frac{9}{{a + b + c + d}} \geq \frac{{25}}{4}$$.
56
Let $x,\ y,\ z$ be positive real numbers such that $x+y+z=1$.
Prove that :
$$\displaystyle \sqrt{xy(1-z)}+\sqrt{yz(1-x)}+\sqrt{zx(1-y)}\leq \sqrt{\frac {2}{3}}$$
763β:2012/10/28(日) 08:36:01.69
全問暗算で解けた
764132人目の素数さん:2012/10/28(日) 08:38:20.05
>>1のAAがキモい
765132人目の素数さん:2012/10/28(日) 09:01:55.93
a,b, c, d>0⇒a^2/(a^2+(a+b)^2)+b^2/(b^2+(b+c)^2)+c^2/(c^2+(c+d)^2)+d^2/(d^2(d+a)^2)≦4/5
766132人目の素数さん:2012/10/28(日) 09:13:30.23
訂正

a,b, c, d>0⇒a^2/(a^2+(a+b)^2)+b^2/(b^2+(b+c)^2)+c^2/(c^2+(c+d)^2)+d^2/(d^2+(d+a)^2)≦4/5
767132人目の素数さん:2012/10/28(日) 21:38:20.94
また消えてるな
768132人目の素数さん:2012/10/28(日) 22:12:32.43
復活させなくていいよ
769132人目の素数さん:2012/10/28(日) 23:21:00.73
>>768
お前が変なこと言うからだよぅ(・3・)YO!
770132人目の素数さん:2012/10/29(月) 01:22:45.47
>>662 >>745

1. For x>0, y>0, z>0, prove that: (x^2+y^2+z^2)^2・(1/x + 1/y + 1/z) ≧ (x+y+z)^3.

  凸不等式(イェンゼン)。
  x^r + y^r + z^r ≧ 3{(x+y+z)/3}^r, (r≦0 or r≧1)  >>662 [1]

3. For a>0, b>0, c>0, prove that:
  (b^2+c^2)/a + (c^2+a^2)/b + (a^2+b^2)/c ≧ 2(a+b+c),

  a+b+c = s とおく。
  (左辺) = (a^2 +b^2 +c^2)(1/a + 1/b + 1/c) - s
   ≧ (1/3)s^2・(9/s) - s
   = 3s - s
   = 2s.       >>662 [4]

4. a_1 + a_2 + ・・・・ + a_n = 0, with a_i are not all 0. There is such a k that:
   a_j≦ 0 for j≦k, and a_j ≧0 for j>k.
  Show that:
   a_1 + 2a_2 + 3a_3 + ... + n a_n > 0.
  (1966 Swedish Mathematical Olympiad)

  (左辺) - (k + 1/2)(a_1 + a_2 + ・・・・ + a_n)
  = Σ[j=1,k] (k + 1/2 -j)(-a_j) + Σ[j=k+1,n] (j-k -1/2)a_j
  > 0,       >>662 [5]
771132人目の素数さん:2012/10/29(月) 01:37:45.19
>>709 >>725

(1) Let a≧0, b≧0, c≧0, with a+b+c=9. Prove that:
 (17^3)/4 ≦ (a^2 +4)(b^2 +4)(c^2 +4) ≦ (97^2)/4
 1228.25 ≦ (a^2 +4)(b^2 +4)(c^2 +4) ≦ 2352.25

(右)
 min{a,b,c} ≦ 3 だから、a+b≧6 としてよい。
 {(1/4)(a+b)^2 + 4}^2 - (a^2 +4)(b^2 +4) = {(1/16)(a+b)^2 +(1/4)ab -2}(a-b)^2 ≧0,
 よって a=b≧3≧c の場合を考えれば十分。
 (与式) ≦ {(1/4)(9-c)^2 +4}(c^2 +4) ≦ (97^2)/4
772132人目の素数さん:2012/10/29(月) 01:44:31.41
>>745
1〜5 は >>662 >>673-674 に外出であった。

6. Let 0<a,b,c<1. Prove that: (a+b+c) -abc < 2.

  2 -(a+b+c) +abc = a(1-b)(1-c) + (1-a)b(1-c) + (1-a)(1-b)c + 2(1-a)(1-b)(1-c) ≧ 0,

7. Let n≧2 and let 0<x_i<1 (i=1, 2, ・・・・ , n). Prove that:
   (1-x_1)(1-x_2) ・・・・ (1-x_n) + x_1 + x_2 + ・・・・ + x_n > 1.

 nについての帰納法で。
  (1-x_1)(1-x_2) ・・・・ (1-x_n) + x_1 + x_2 + ・・・・ + x_n
 = (1-x_1)(1-x_2) ・・・・ (1-x_{n-1}) + x_1 + x_2 + ・・・・ + x_{n-1}
 + {1−(1-x1)(1-x2)・・・・・(1-x_{n-1})}x_n
 > 1.

8. Let a>0, b>0, c>0, with ab+bc+ca=t. Prove that:
 t/(a+b) + t/(b+c) + t/(c+a) - ab/(a+b) - bc/(b+c) - ca/(c+a) ≧ √(3t).

 (左辺) = (t-ab)/(a+b) + (t-bc)/(b+c) + (t-ca)/(c+a) = a+b+c ≧ √(3t).

10. For x>1, y>1, prove that: (x^2)/(y-1) + (y^2)/(x-1) ≧ 8.

 x-1 ≦ (x^2)/4, y-1 ≦ (y^2)/4 より
 (左辺) ≧ 4(x/y)^2 + 4(y/x)^2 = 4(t + 1/t) > 8.
773132人目の素数さん:2012/10/29(月) 01:56:50.02
>>746

12. Find the possible maximum value of a such that:
  √(x^2+y^2) ≧ x + y + a√(xy), holds for all positive real numbers x, y.

  明らかに a <0.
  x^2 + y^2 ≧ (x^2 + y^2) + {(2+a^2)√xy + 2a(x+y)}√xy,
  -2a(x+y) -(2+a^2)√xy ≧ 0,
  -2a(√x - √y)^2 -(2+4a+a^2)√xy ≧ 0,
  ∴ a<0 かつ 2+4a+a^2 ≦ 0,
  ∴ -2-√2 ≦ a ≦ -2+√2,
  ∴ 最大値は -2+√2,

13. Let a>0, b>0, c>0, with a+b+c=abc. Prove that:
  1/{(a+b)(b+c)} + 1/{(b+c)(c+a)} + 1/{(c+a)(a+b)} ≦ 1/4,

  (左辺) = 2(a+b+c)/{(a+b)(b+c)(c+a)} = 2abc/{(a+b)(b+c)(c+a)} ≦ 1/4,
  ∵ (a+b)(b+c)(c+a) -8abc = a(b-c)^2 + b(c-a)^2 + c(a-b)^2 ≧ 0,

14. Let a>0, b>0, c>0. Prove that:
  a/(a^4+a^2+1) + b/(b^4+b^2+1) + c/(c^4+c^2+1) ≦ (1/3){a/(bc) + b/(ca) + c/(ab)}.

  x/(x^4 +x^2 +1) = x/{3x^2 + (x^2 -1)^2} ≦ x/(3x^2) = 1/(3x),
  (左辺) ≦ 1/(3a) + 1/(3b) + 1/(3c)
     = (ab+bc+ca)/(3abc)
     ≦ (a^2 + b^2 + c^2)/(3abc)
     = (1/3)(a/bc + b/ca + c/ab),
774132人目の素数さん:2012/10/29(月) 01:58:23.79
>>746

15. Let a>0, b>0, c>0. Prove that:
  1/√(ab) + 1/√(bc) + 1/√(ca) + 2√{3(a+b+c)} ≧ 9.

  相加・相乗・調和平均。 a+b+c=s とおくと
  (左辺) ≧ 2/(a+b) + 2/(b+c) + 2/(c+a) + 2√(3s)
  ≧ 9/s + √(3s) + √(3s)
  ≧ 9.

16. Let a>0, b>0, c>0. Prove that:
  (a^2)/(a+b) + (b^2)/(b+c) + (c^2)/(c+a) ≧ {(√2)/4} {√(a^2 + b^2) + √(b^2 + c^2) + √(c^2+a^2)}.

  √(a^2 + b^2) ≧ (a+b)/√2 より
  (a^2)/(a+b) = (a^2 + b^2)/{2(a+b)} + (a-b)/2
        ≧ {(√2)/4}√(a^2 + b^2) + (a-b)/2,
  循環的にたす。

17. Let a>0, b>0, c>0, d>0. Prove that:
  1/(4a+3b+c) + 1/(3a+b+4d) + 1/(a+4c+3d) + 1/(4b+3c+d) ≧ 2/(a+b+c+d).

  コーシー。
  左辺に {(4a+3b+c) + (3a+b+4d) + (a+4c+3d) + (4b+3c+d)} = 8(a+b+c+d) を掛ける。

18. Prove the following inequality.
   ∫[1,e] {log(x)^2009}/(x^2) dx > 1/(2010・2011・2012)

  x=exp(t) とおく。
 ∫[0,1] (t^2009)exp(-t)dt = (1/2010)t^2010 exp(-t) + (1/2010)∫(t^2010)exp(-t)dt
   = {(1/2010)t^2010 + (1/(2010・2011))t^2011 + (1/(2010・2011・2012))t^2012 + ・・・・}exp(-t) ](t=0,1)
   > {1/(2010・2011・2012)}(3/e)
   > 1/(2010・2011・2012)
775132人目の素数さん:2012/10/29(月) 02:05:16.07
>>747

19. Let a>0, b>0, c>0.
 (1) Let x>0, y>0, with x+y=h(positive constant). Prove that: (a^2)/x + (b^2)/y ≧ (a+b)^2 /h.
 (2) Let x>0, y>0, z>0, with x+y+z=k(positive constant). Prove that: (a^2)/x + (b^2)/y + (c^2)/z ≧ (a+b+c)^2 /k.
  コーシー。

21. Given a>0, b>0, c>0, with a^2 + b^2 + c^2 + (a+b+c)^2 ≦ 4. Prove that:
  (ab+1)/(a+b)^2 + (bc+1)/(b+c)^2 + (ca+1)/(c+a)^2 ≧ 3.

  ab + 1 ≧ ab + (1/2){a^2 + b^2 + (b+c)(c+a)}
      = (1/2)(a+b)^2 + (1/2)(b+c)(c+a), etc. より
  (左辺) ≧ 3(1/2) + (1/2){(b+c)(c+a)/(a+b)^2 + cyclic.} ≧ 3(1/2) + 3(1/2) = 3.

22. Let a>0, b>0, c>0. Prove the following inequality.
   2{(a+b)/2 - √(ab)} ≦ 3{(a+b+c)/3 - (abc)^(1/3)}.

  相加・相乗平均で
  (右辺) - (左辺) = √(ab) + √(ab) + c - 3(abc)^(1/3) ≧ 0.

25. Let a, b, c be real numbers such that (ab)^2 + (bc)^2 + (ca)^2 + 3(a^2+b^2+c^2) = 12.
Prove that:  (a+b+c) + 2(ab+bc+ca) ≦ 9.

 a+b+c=s, ab+bc+ca=t とおく。
   (ab)^2 + (bc)^2 + (ca)^2 ≧ (1/3)t^2,
   a^2 +b^2 +c^2 = s^2 -2t,
  よって題意より、
   12 ≧ (1/3)t^2 -6t +3s^2
     ≧ (1/3)t^2 -2t +(5/3)s^2  (s^2≧3t)
     = (1/3)(t-3)^2 +(5/3)s^2 -3,
  ∴ s ≦ 3,
  ∴ t ≦ (1/3)s^2 ≦3 より成立。
776132人目の素数さん:2012/10/29(月) 02:09:31.65
>>748

26. For a<b, prove that:∫[a,b] (x^2 +1) e^(-x^2) dx ≧ e^(-a^2) - e^(-b^2),

  x^2 +1 ≧ 2x を使うと、積分を実行できる。

27. Let -1≦a,b,c,d≦1. Prove that:
  √(1-a^2) + √(1-b^2) + √(1-c^2) + √(1-d^2) ≦ √{16-(a+b+c+d)^2},

  凸不等式(Jensen) 円の上半分だから上に凸。

28. Let 0≦x,y,z<π/4. Prove the following inequality.
  {tan(x)・tan(y)・tan(z)}^(1/3) ≦ tan{(x+y+z)/3} ≦ {tan(x) + tan(y) + tan(z)}/3,

 (左) f(x) = log(tan(x)) とおくと、
  f '(x) = 1/{sin(x)・cos(x)} = 2/sin(2x) は 0<x≦π/4 で単調減少、fは上に凸。
  {f(x)+g(x)+h(x)}/3 ≦ f((x+y+z)/3),

 (右) g(x) = tan(x) とおくと、
  g '(x) = 1/cos(x)^2 は単調増加、gは下に凸。
  g((x+y+z)/3) ≦ {g(x)+g(y)+g(z)}/3,

29. Let A,B,C,D ∈ (0,π). Prove the following inequalities.
  (1) sin A + sin B + sin C + sin D ≦ 4 sin{(A+B+C+D)/4},
  (2) sin A + sin B + sin C ≦ 3 sin{(A+B+C)/3},

  凸不等式(イェンゼン)
777132人目の素数さん:2012/10/29(月) 02:12:40.13
>>748

30. Let a>0, b>0, c>0, with a+b+c=s. Prove that:
  √(a+bc) + √(b+ca) + √(c+ab) ≦ √{s(3+s)},

  凸不等式(イェンゼン)
  (左辺) ≦ √{3(a+b+c)+3(ab+bc+ca)} ≦ √(3s + s^2) = √{s(3+s)}.

31. Given positive real numbers a,b,c,d with a+b+c+d=s. Prove that :
  6(a^3 + b^3 + c^3 + d^3) ≧ (a^2 + b^2 + c^2 + d^2)s + (s/2)^3,

  チェビシェフ。
  6a^3 - sa^2 -(1/8)s^2・a = (4a-s)・a(12a+s)/8,
  (左辺) ≧ {(4a-s)+(4b-s)+(4c-s)+(4d-s)}{a(12a+s) + cyclic}/32 = 0,

  6a^3 - sa^2 -(1/32)s^3 = (4a-s)・(48a^2 +4as +s^2)/32,
  (左辺) ≧ {(4a-s)+(4b-s)+(4c-s)+(4d-s)}{(48a^2+4as+s^2) + cyclic}/128 = 0,

取り急ぎ御回答まで。
778132人目の素数さん:2012/10/29(月) 03:50:36.14
>>760

35. Let a>0, b>0, c>0. Prove that:
  (1/a + b)(1/b + c)(1/c + a)(a + 1/a)(b + 1/b)(c + 1/c) ≧ 64.

  (1/a + b)(1/b + c)(1/c + a) ≧ 2√(b/a)・2√(c/b)・2√(a/c) = 8,
  a + 1/a ≧ 2,
  b + 1/b ≧ 2,
  c + 1/c ≧ 2,
  辺々かける。

37. For -2 ≦ a,b,c ≦1 and a+b+c=0, prove that a^2+b^2+c^2 ≦ 6.

 ab, bc, ca ≧ -2 より
 a^2 + b^2 + c^2 = (a+b+c)^2 -2(ab+bc+ca) ≦ 6.

>>761
44. Given real numbers x,y,z such that x^2 + y^2 + z^2 + 2xyz = 1. Prove that:
  x^2+y^2+z^2 ≧ 3/4.

  x^2+y^2+z^2 = S とおく。
  2xyz ≦ 2{(x^2 +y^2 +z^2)/3}^(3/2) = 2(S/3)^(3/2),
∴ (左辺) ≦ S + 2(S/3)^(3/2) これを f(S) とおく。
  題意により、f(S)≧1,
  一方、f(S) は単調増加で、f(3/4) = 1,
∴ S ≧ 3/4.

46. Let a>0, b>0, c>0, x>0, y>0, z>0. Prove that:
 {x/(a^2) + y/(b^2) + z/(c^2)}(ax+by+cz)^2 ≧ (x+y+z)^3.

 コーシーの拡張。
779132人目の素数さん:2012/10/29(月) 03:51:38.86
>>762

54. Prove that: 5(a^2 + b^2 + c^2) ≧ 4(ab+bc+ca),
 and determine when equality occurs.

 (左辺) - (右辺) = a^2 + b^2 + c^2 + 2(a-b)^2 + 2(b-c)^2 + 2(c-a)^2 ≧ 0,
 等号成立は a=b=c=0 に限る。
780132人目の素数さん:2012/10/29(月) 04:10:56.74
>>735
 casphy - 高校数学 - 不等式 - 950〜953
 シューアを使うもの。

今日はこの辺で。。。
781132人目の素数さん:2012/10/29(月) 04:51:07.34
>>766 凡例

 a,c を固定して b,d→0 とすると...
782132人目の素数さん:2012/10/29(月) 08:08:50.32
また糞スレからのコピペかよw
783132人目の素数さん:2012/10/29(月) 16:50:47.28
>>782
口を慎み給へ
784132人目の素数さん:2012/10/29(月) 16:56:58.55
何言ってんだ、このバカ
785132人目の素数さん:2012/10/29(月) 20:55:12.90
実際糞な問題ばかりだろ
786132人目の素数さん:2012/10/29(月) 22:53:27.21
>>766 判例

 b/a = c/b = d/c = ε として ε→0 とすると....
787132人目の素数さん:2012/10/30(火) 00:26:59.08
>>762

49. Given real numbers a,b,c,d with a+b+c+d=6, a^2+b^2+c^2+d^2=12. Prove that:
 36 ≦ 4(a^3 +b^3 +c^3 +d^3) - (a^4 +b^4 +c^4 +d^4) ≦ 48.

http://www.imo-official.org/problems/IMO2010SL.pdf の Problem A2
Yahoo!掲示板 - 科学 - 数学 - 出題(不等式) - 1500〜1503,

 (中辺) = 6S_2 -4S_1 +4 -(a-1)^4 -(b-1)^4 -(c-1)^4 -(d-1)^4,
 -(S_2 -2S_1 +S_0)^2 ≦ -(a-1)^4 -(b-1)^4 -(c-1)^4 -(d-1)^4 ≦ -(1/4)(S_2 -2S_1 +S_0)^2,
 (a-1)^2 +(b-1)^2 +(c-1)^2 +(d-1)^2 = S_2 -2S_1 +S_0,
 S_0 = 4, S_1 = 6, S_2 = 12.

50. Let a>0, b>0, such that a+b=1. Prove that:
 (a^a)(b^b) + (a^b)(b^a) ≦ 1.

 0<a≦b とする。
 (左辺) = (ab)^a (a^|b-a| + b^|b-a|)
     ≦ (ab)^a 2{(a+b)/2}^|b-a|   *
     = (ab)^a 2^(2a)   (a+b=1)
     = (4ab)^a
     ≦ 1,        (4ab≦1)

*) y = x^|b-a| は上に凸 (0≦|b-a| <1)

54. Prove that: 5(a^2 + b^2 + c^2) ≧ 4(ab+bc+ca),
 and determine when equality occurs.

 (左辺) - (右辺) = a^2 + b^2 + c^2 + 2(a-b)^2 + 2(b-c)^2 + 2(c-a)^2 ≧ 0,
 等号は a=b=c=0 に限る。
788仙石60:2012/10/30(火) 02:25:01.41
>>787 
久方ぶりです。
49. Given real numbers a,b,c,d with a+b+c+d=6, a^2+b^2+c^2+d^2=12. Prove that:
 36 ≦ 4(a^3 +b^3 +c^3 +d^3) - (a^4 +b^4 +c^4 +d^4)

対象式s1、s2、s3、s4
であらわすと
s1=6
s2=12
はあきらか
 
P = 4(a^3 +b^3 +c^3 +d^3) - (a^4 +b^4 +c^4 +d^4)
 = ー12s3+4s4+144
さらに判別式 Q = -27s3^4;432s3^3+1512s4s3^2−1728s3^2
-1152s4^2s3-22464s4s3+256s4^3+8784s4^2+82944s4 >=0

ラグランジュ法により
s3=18、s4=27、λ=−1/20736をえる
 このとき最小値 36を得る。
789β:2012/10/30(火) 02:28:24.46
いきてたか!
 死ね!   仙石60め!
790132人目の素数さん:2012/10/30(火) 05:45:07.04
シュアーの不等式に慣れたいからかなり易しめ〜普通レベルの問題くれ
791132人目の素数さん:2012/10/30(火) 23:03:24.11
57
For positive real numbers $a, b, c$,
Prove that :
\[\frac{a+b}{\sqrt{a^2-ab+b^2}}+\frac{b+c}{\sqrt{3b^2-2bc+3c^2}}+\frac{c+a}{\sqrt{6c^2-3ca+6a^2}}\leq \frac{11}{3} \]
58
Let $a,b,c$ be real numbers such that $a+b+c=3$.
Prove that :
$$\frac{1}{5{{a}^{2}}-4a+11}+\frac{1}{5{{b}^{2}}-4b+11}+\frac{1}{5{{c}^{2}}-4c+11}\le \frac{1}{4}$$
59
For $0\leq x,\ y,\ z\leq 1$, prove that :
$$\displaystyle (2^x+2^y+2^z)(2^{-x}+2^{-y}+2^{-z}) \leq 10.$$
60
For real numbers $a,\ b$, if $ab\geq 1$, then prove that $a^2+b^2\geq a+b.$\ \\
61
For positve real numbers $ a,b,c $,
Prove that :
$$ \frac{(b+c-a)^2}{(b+c)^2+a^2}+\frac{(c+a-b)^2}{(c+a)^2+b^2}+\frac{(a+b-c)^2}{(a+b)^2+c^2}\geq\frac{3}{5} $$
62
Let $a,b$ be real numbers such that $ a(a^2+1)^2+b(b^2+1)^2=8$.
Prove that :
\[\ a+b\leq 2\]
63
For $x,y,z \geq 0$ with $ x+y+z=3 $.
Prove that :
$$ (x^2+y+z)(y^2+z+x)(z^2+x+y) \geq 27 $$
64
Let $a,b,c$ be positive real numbers such that $abc=1$.
Prove that :
$$\displaystyle \frac{a+3}{(a+1)^2}+\frac{b+3}{(b+1)^2}+\frac{c+3}{(c+1)^2}\ge 3.$$
792132人目の素数さん:2012/10/30(火) 23:04:48.40
65
For $0\leq x\leq 1,\ 0\leq y\leq 1,\ 0\leq z\leq 1$,
Prove that :
$$\frac{x+y+z}{3}+\sqrt{x(1-x)+y(1-y)+z(1-z)}\leq \frac 32$$
66
Prove the following inequality.
$$\frac 1{2\cos 1}\int_1^{\frac {\pi}2}\frac {\sin x}x\ dx\, \leq\, \frac {\log_e \pi-\log_e 2}{\pi-2}$$.
67
Let $a,\ b,\ c$ be distinct real numbers.
Prove that :
$$|a-b|+|b-c|+|c-a|+\left|\frac{b-c}{a-b}\right|+\left|\frac{c-a}{b-c}\right|+\left|\frac{a-b}{c-a}\right|+\left|\frac{c-a}{a-b}\right|+\left|\frac{a-b}{b-c}\right|+\left|\frac{b-c}{c-a}\right|$$
$$\geq 2\sqrt{|a-b|}+2\sqrt{|b-c|}+2\sqrt{|c-a|}.$$
68
Let $a,\ b,\ c$ be positive real numbers such that $\displaystyle \frac{1}{a}+\frac{1}{b}+\frac{1}{c}=4.$
Prove that :
$$\frac{1}{2a+b+c}+\frac{1}{a+2b+c}+\frac{1}{a+b+2c}\leq 1$$
69
Let $a,\ b,\ c$ be positive real numbers such that $a+b+c=1$.
Prove that :
$$\sqrt[3]{\left(\frac{1}{ab}-2\right)\left(\frac{1}{bc}-2\right)\left(\frac{1}{ca}-2\right)}\geq 7$$
70
Let $a,b,c$ be non-negative real numbers such that no two of which are 0.
Prove that :
$$\frac {ab}{(a+b)^2}+\frac {bc}{(b+c)^2}+\frac {ca}{(c+a)^2}+\frac 1{4}\ge \frac{ab+bc+ca}{a^2+b^2+c^2}$$
793132人目の素数さん:2012/10/30(火) 23:07:15.24
71
Let $ a$, $ b$ and $ c$ be the side lengths of a triangle. Prove that
\[ a^{2}b(a - b) + b^{2}c(b -c) + c^{2}a(c - a)\ge 0.
\]
72
Let $a,\ b,\ c$ be real numbers such that
$$a<b<c,\ a+b+c=0,\ ab+bc+ca=-3.$$
Prove that :
$$-2<a<-1<b<1<c<2.$$
73
Given positive real numbers $a,\ b,\ c$ with $abc=1$.
Prove that :
\[a^4+b^4+c^4+33\geq12(ab+bc+ca)\]
74
Let $a,b,c$ be positive real numbers.
Prove that :
$$a^{b+c}+b^{c+a}+c^{a+b}\geq 1.$$
75
Prove that $\displaystyle x\cos x\leq \frac{\pi^2}{16}$ for $\displaystyle 0\leq x\leq \frac{\pi}{2}.$
76
Given non-negative real numbers $a,b,c,d$ with $a+b+c+d=4$.
Prove that :
\[\displaystyle \frac{a}{a^3+8}+\frac{b}{b^3+8}+\frac{c}{c^3+8}+\frac{d}{d^3+8}\le\frac{4}{9}\]
794132人目の素数さん:2012/10/30(火) 23:12:44.79
77
For $a,\ b,\ c,\ d>0$ with $a^2+b^2+c^2+d^2=4.$
Prove that :
$$\displaystyle (a+b+c+d-2)\left(\frac{1}{a}+\frac{1}{b}+\frac{1}{c}+\frac{1}{d}+\frac 12\right)\geq 9.$$
78
Given non-negative real numbers $x,y$ with $x^2+y^2=2$.
Prove that :
\[4\leq x^4+y^4+x+y\leq 4+\sqrt2.\]
78’
Given a triangle $ABC$ with side lengths $a,\ b,\ c$.
Prove that :
$$a\left(\frac{1}{B}+\frac{1}{C}\right)+b\left(\frac{1}{C}+\frac{1}{A}\right)+c\left(\frac{1}{A}+\frac{1}{B}\right)\ge2\left(\frac{a}{A}+\frac{b}{B}+\frac{c}{C}\right)$$
79
Let $a,\ b$ be positive real numbers.
Prove that :
$$\frac{1+a}{\sqrt{1+a^2}}+\frac{1+b}{\sqrt{1+b^2}}\geq 2\sqrt{2}\left(\frac{a}{1+a^2}+\frac{b}{1+b^2}\right)$$.
80
Let $a,\ b,\ c$ be postive real numbers with $abc=1$.
Prove that :
$$a^3+b^3+c^3+6\geq (a+b+c)^2.$$
795132人目の素数さん:2012/10/30(火) 23:31:15.18
>>760

33. For a>0, b>0, c>0, prove that: {(a^3+b^3+c^3+6)/(ab+bc+ca)}{(b^2)/a + (c^2)/b + (a^2)/c} ≧ 9.

 (1) a^3 +1 +1 ≧ 3a, b^3 +1 +1 ≧ 3b, c^3 +1 +1 ≧ 3c, (相加・相乗平均)

 (2) (b^2)/a + (c^2)/b + (a^2)/c ≧ a+b+c,
 ∵ コーシー あるいは
  abc・{(b^2)/a + (c^2)/b + (a^2)/c -(a+b+c)} = (b^3)c + (c^3)a + (a^3)b - abc・(a+b+c)
  = (1/7)Σ[cyclic] {4(b^3)c + (c^3)a + 2(a^3)b - 7a(b^2)c} ≧ 0, (相加・相乗平均)

 (3) (a+b+c)^2 - 3(ab+bc+ca) = (1/2){(a-b)^2 + (b-c)^2 + (c-a)^2} ≧ 0,
 以上から出る。

36. Let a>0, b>0, c>0 such that a^a・b^b・c^c=1. Prove that a+b+c ≦ 3.

  f(x) = x・log(x) とおくと、
  f '(x) = log(x) +1 は単調増加。
 f は下に凸。イエンゼンより

 0 = a・log(a) + b・log(b) + c・log(c) ≧ (a+b+c)log{(a+b+c)/3},
 (a+b+c)/3 ≦ 1,

39. For 0<x<π/2, prove that: {sin(x)/x}^2 > cos(x) > {x/tan(x)}^2.

 tan(x/2) = t とおく。(x < 2t)
 (左辺) = {(2t)/(1+t^2)x}^2 ≧ 1/(1+t^2)^2 ≧ (1-t^2)/(1+t^2) = cos(x),
 (右辺) = {x(1-t^2)/(2t)}^2 ≦ (1-t^2)^2 ≦ (1-t^2)/(1+t^2) = cos(x),
796132人目の素数さん:2012/10/31(水) 00:04:56.00
 左辺と右辺の相乗平均が cos(x)


40. Let 0<a<b and a ≦ x ≦ y ≦ z ≦ b, x+y+z = a+2b, then prove that: xyz ≧ ab^2.

 y ≦ b より x+z≧a+b,
 xyz ≧ (x+z-b)yb ≧ abb,

>>761

41. Let a,b,c be real numbers such that |a+b|+|b+c|+|c+a|=3. Prove that:
  (a+b)^(2/3) + (b+c)^(2/3) + (c+a)^(2/3) ≦ 3.

  y=x^(2/3) は上に凸だから、イェンゼンより
  (左辺) ≦ |a+b|^(2/3) + |b+c|^(2/3) + |c+a|^(2/3)
     ≦ 3{(|a+b|+|b+c|+|c+a|)/3}^(2/3)
     = 3,
42. Given a>0, b>0, c>0, with ab+bc+ca=t. Prove that:
  {(t/a) + 6b}^(1/3) + {(t/b) + 6c}^(1/3) + {(t/c) + 6a}^(1/3) ≦ t^(5/3)/(abc).

  (左辺) ≦ 3{(t/3)(1/a + 1/b + 1/c) + 2(a+b+c)}^(1/3)
  ≦ 3{(t^2)/(3abc) + 2(t^2)/(3abc)}^(1/3)
   = 3{(t^2)/(3abc)}^(1/3)
  ≦ t^(5/3)/(abc).   {3(a+b+c)abc≦t^2}

>>762

49. (右)
 4a^3 -a^4 ≦ 4a^2, etc.
循環的にたすと、
 (中辺) ≦ 4S_2,
等号成立は a〜d = 0 or 2.
797132人目の素数さん:2012/11/01(木) 03:50:33.70
>>791

59. For 0≦x,y,z≦1, prove that:
  (2^x+2^y+2^z){2^(-x)+2^(-y)+2^(-z)} ≦ 10.

  yはxとzの中間にあるとしてよい。
  (左辺) = 3 + {2^(x-y)+2^(y-x)} + {2^(y-z)+2^(z-y)} + {2^(z-x)+2^(x-z)}
     ≦ 5 + 2 {2^(z-x) + 2^(x-z)}
     ≦ 5 + 2{2 + 1/2}
     = 10,
   等号は {x,y,z} = {0,0,1} {0,1,1}

60. For real numbers a, b, with ab≧1, prove that a^2+b^2 ≧ a+b.

  (a+b)^2 = (a-b)^2 + 4ab ≧ 4ab ≧ 4,
 ∴ |a+b| ≧ 2,
  |a+b-1| ≧ 1

 (左辺)-(右辺) = (1/2)(a-b)^2 +(1/2)(a+b)^2 - (a+b)
        = (1/2)(a-b)^2 + (1/2)(a+b-1)^2 - (1/2)
        ≧ 0,

62. Let a,b be real numbers, such that a(a^2+1)^2 + b(b^2+1)^2 =8. Prove that:
  a+b ≦ 2,

  (左辺) = (a^5+b^5) +2(a^3+b^3) + (a+b)
      = (a+b){a^4 -(a^3)b +(ab)^2 -a(b^3) +b^4 +2(a^2 -ab +b^2) +1},
  題意より、s = a+b > 0,
  8 = (左辺) ≧ s{(s/2)^2 +1}^2 = f(s),
  f は単調増加だから、s ≦ 2.
798132人目の素数さん:2012/11/01(木) 03:55:30.88
>>792

65. For 0≦x,y,z≦1, prove that:
  (x+y+z)/3 + √{x(1-x)+y(1-y)+z(1-z)} ≦ 3/2,

  チェビシェフより
  x(1-x) + y(1-y) + z(1-z) ≦ (1/3)(x+y+z)(3-x-y-z) = (1/3)s(3-s)
   = {(3/2) - (1/3)s}^2 - {(3/2) - (2/3)s}^2
   ≦ {(3/2) - (1/3)s}^2,

66. Prove the following inequality.
  1/{2cos(1)}∫[1,π/2] sin(x)/x dx ≦ log(π/2)/(π-2).

  チェビシェフより
  {sin(x)/x + sin(y)/y} - {sin(x)/y + sin(y)/x} = {sin(x)-sin(y)}(y-x)/(xy) ≦ 0,
  (左辺) = 1/{2cos(1)}∬_[1,π/2] {sin(x)/x + sin(y)/y} dxdy /(π-2)
     ≦ 1/{2cos(1)}∬_[1,π/2] {sin(x)/y + sin(y)/x} dxdy /(π-2)
     = log(π/2)/(π-2),

67. Let a, b, c be distinct real numbers. Prove that:
 |a-b| + |b-c| + |c-a|
 + (1/2){|(b-c)/(a-b)| + |(a-b)/(b-c)|} + (1/2){|(c-a)/(b-c)| + |(b-c)/(c-a)|} + (1/2){|(a-b)/(c-a)| + |(c-a)/(a-b)|}
 ≧ 2√|a-b| + 2√|b-c| + 2√|c-a|.

  相加・相乗平均で
  (1/2)(|x/y| + |y/x|) ≧ 1,
  |a-b| + 1 ≧ 2√|a-b|,
  循環的にたす。
799132人目の素数さん:2012/11/01(木) 03:57:10.94
>>792

68. Let a>0, b>0, c>0. Prove that:
  1/(2a+b+c) + 1/(a+2b+c) + 1/(a+b+2c) ≦ (1/4)(1/a + 1/b + 1/c).

  相加・調和平均で
   1/(2a+b+c) ≦ (1/16)(1/a + 1/a + 1/b + 1/c).
  循環的にたす。


69. Let a>0, b>0, c>0, such that a+b+c=s. Prove that:
  {(ss/ab -2)(ss/bc -2)(ss/ca -2)}^(1/3) ≧ 7,

 (ss-2ab)(ss-2bc)(ss-2ca) = s^6 -2(ab+bc+ca)s^4 +4abcs^3 -8(abc)^2
    ≧ s^6 -(2/3)s^6 +4abcs^3 -8(abc)^2
    = (1/3)s^6 +4(abc)s^3 -8(abc)^2
    ≧ 243(abc)^2 +108(abc)^2 -8(abc)^2   (s^3 ≧27abc)
    = (7^3)(abc)^2,
800132人目の素数さん:2012/11/01(木) 04:47:50.87
>>794 

77. For a>0, b>0, c>0, d>0 with a^2 +b^2 +c^2 +d^2 =4. Prove that:
 (a+b+c+d-2)(1/a + 1/b + 1/c + 1/d + 1/2) ≧ 9.
 >>136 >>273-277 を参照。


80. Let a>0, b>0, c>0, with abc=G^3. Prove that:
  a^3 +b^3 +c^3 +6abc ≧ G(a+b+c)^2.

  >>18-19 >>42 >>253 >>268-269 も見てね。
801132人目の素数さん:2012/11/01(木) 05:10:03.63
>>800
さすが神! いつもながら乙の極みでござるよ
802132人目の素数さん:2012/11/01(木) 13:46:45.05
きめえ
803132人目の素数さん:2012/11/01(木) 17:04:20.61
またお前か
804132人目の素数さん:2012/11/01(木) 18:05:58.89
いい加減にしろ!

荒らし報告するぞ
805あのこうちやんは始皇帝だった:2012/11/01(木) 19:12:04.94

 20代と60代の、ニート・無職の、女性恐怖症の、頭デッカチの虚弱児・ひ弱の、関西の、ゴミ・クズ・カス・無能・虫けらのクソガキども!

 死ね!!!!!!!!!!!!!!!!!!!!!!!!!!!!!!!!!!!!!!!!!!!!!!!!!!!!!!!!!!!!!
806132人目の素数さん:2012/11/02(金) 00:21:28.67
>>760

34. Let a>0, b>0, c>0 such that (a^2+b^2)/(a+b)^3 + (b^2+c^2)/(b+c)^3 + (c^2+a^2)/(c+a)^3 = 3/2.
  Prove that: a^2+b^2+c^2 ≧ 3/4.

 コーシーで
 {(a+b)+(a+b)+(b+c)+(b+c)+(c+a)+(c+a)}(左辺)≧ {a/(a+b) + b/(a+b) + b/(b+c) + c/(b+c) + c/(c+a) + a/(c+a)}^2,
 4(a+b+c)(左辺)≧ (1+1+1)^2 = 9,
 (a+b+c) ≧ 9/{4(左辺)} = 3/2,
 a^2 + b^2 + c^2 ≧ (1/3)(a+b+c)^2 ≧ 3/4,

37. For -2 ≦ a,b,c ≦1 and a+b+c=0, prove that a^2+b^2+c^2 ≦ 6.

 a+b+c = s とおくと、
 a^2 + b^2 + c^2 = 6 -2(1-a)(1-b) -2(1-c)(2+c) +s(a+b-c-2)
   = 6 -2(1-a)(1-b) -2(1-c)(2+c)  (←s=0)
   ≦ 6,             (← 題意)
807132人目の素数さん:2012/11/03(土) 00:36:22.27
>>761
48. Let a>0, b>0, c>0. Prove that: (a^3)/(a+b)^2 + (b^3)/(b+c)^2 + (c^3)/(c+a)^2 ≧ (a+b+c)/4,

  a+b+c=s とおく。
  s・(左辺) ≧ {(a^2)/(a+b) + (b^2)/(b+c) + (c^2)/(c+a)}^2,
  2s{(a^2)/(a+b) + (b^2)/(b+c) + (c^2)/(c+a)}
  = {(a+b)+(b+c)+(c+a)}{(a^2)/(a+b) + (b^2)/(b+c) + (c^2)/(c+a)} ≧ s^2,
  (a^2)/(a+b) + (b^2)/(b+c) + (c^2)/(c+a) ≧ s/2,
または、コーシーの拡張(4)で一発。
  (a+b+c){(a+b)+(b+c)+(c+a)}^2 (左辺) ≧ (a+b+c)^4,
  4s^3(左辺)≧ s^4,

>>762

51. Let a>0, b>0. Prove that: (ab)^2・(a^2 +b^2 -2) ≧ (a+b)(ab-1).

 √ab = G とおくと、
 (a-b)^2 = (a+b)^2 - (2G)^2 = (a+b-2G)(a+b+2G),
 (左辺) - (右辺) = (G^4){2(G^2 -1) +(a-b)^2} - (a+b)(G-1)
    = 2G(G^2 -1)(G^3 -1) + (a+b-2G){(a+b+2G)G^2 -G^2 +1}
    ≧ 0,

* (a+b+2G)G^2 - G^2 +1 ≧ 4G^3 -G^2 +1 ≧ 1 - 1/108,
808132人目の素数さん:2012/11/03(土) 02:14:09.62
>>762
53. Prove that:
 a(a+b)^3 + b(b+c)^3 + c(c+a)^3 ≧ (2/3)^3・(a+b+c)^4.

19 = 3^2 + 3^2 + 1^2 より
 (左辺) - (右辺) = (1/27){3a^2 -3b^2 +c^2 +(7ab-22bc+11ca)/4}^2 + (3/16){a(b-c)}^2 + cyclic.
または
 (左辺) - (右辺) = (1/27){3a^2 -3b^2 -c^2 +(113ab-131bc+46ca)/28}^2 + (3/16){a(b-c)}^2 + cyclic.
[第5章.494、 504]

55. For a>0, b>0, c>0, d>0, prove that:
 1/a + 1/b + 1/c + 1/d + 9/(a+b+c+d) ≧ 25/(4G),
 where G = (abcd)^(1/4).
 [第5章.498, 508-513]

>>791
59. For 0≦x,y,z≦1, prove that:
  (2^x+2^y+2^z){2^(-x)+2^(-y)+2^(-z)} ≦ 10.

〔補題〕
a≧0, b≧0 のとき
  cosh(a) + cosh(b) ≦ 1 + cosh(a+b),
(略証)
 (右辺) - (左辺) = {cosh(a)-1}{cosh(b)-1} + sinh(a)sinh(b) ≧ 0,
または、cosh は下に凸だから、
  cosh(a) ≦ {b + a・cosh(a+b)}/(a+b),
  cosh(b) ≦ {a + b・cosh(a+b)}/(a+b),
辺々たす。
809132人目の素数さん:2012/11/03(土) 02:41:58.77
>>791

64. Let a>0, b>0, c>0, such that abc=1. Prove that:
  (a+3)/(a+1)^2 + (b+3)/(b+1)^2 + (c+3)/(c+1)^2 ≧ 3,

  (a+3)/(a+1)^2 = 1/(a+1) + 2/(a+1)^2, etc.
a+b+c=s, ab+bc+ca=t, abc=u とおくと、
  1/(a+1) + 1/(b+1) + 1/(c+1) = (3+2s+t)/(1+s+t+u),
  1/(a+1)^2 + 1/(b+1)^2 + 1/(c+1)^2 = {(3-6u) + (4-2u)s + 2s^2 + 2st + t^2}/(1+s+t+u)^2,
よって
 (与式) ={9(1-u) + (13-2u)s +(4+u)t +6s^2 +7st +3t^2}/(1+s+t+u)^2
  = 3 + {3(2-5u-u^2) +(7-8u)s -(2+5u)t +3s^2 +st}/(1+s+t+u)^2
  = 3 + (-12 -s -7t +3s^2 +st)/(1+s+t+u)^2   (← u=1)
  = 3 + {(5s/3 +4 +t)(s-3) +(4/3)(s^2 -3t)}/(1+s+t+u)^2
  ≧ 3,
 等号成立は s=t=3 すなわち a=b=c=1 のとき。
 [第5章.655, 663]
810132人目の素数さん:2012/11/03(土) 03:00:22.57
>>793

73. Given a>0, b>0, c>0. Prove that:
  a^4 + b^4 + c^4 + 33G^4 ≧ 12(ab+bc+ca)G^2,
  where G=(abcd)^(1/4).
  [第5章.699,723-726]

74. Let a>0, b>0, c>0. Prove that:
  a^(b+c) + b^(c+a) + c^(a+b) ≧ 1.
  [第5章.698, 708]

76. Given a≧0, b≧0, c≧0, d≧0. Prove that:
  a/(a^3 +8) + b/(b^3 +8) + c/(c^3 +8) + d/(d^3 +8) ≦ (4/9)(s/4)^(2/3),
  where s=a+b+c+d.

  (x^3 +8)(2x+1) - 27x = (x-1)^2・(2x^2 +5x+8) ≧ 0,
  x/(x^3 +8) ≦ (2x+1)/27,  … x=1 での接線
  x = a,b,c,d についてたす。
または、
  x^3 + 1 + 1 + 1 + 1 + 1 + 1 + 1 + 1 ≧ 9 x^(1/3), (相加・相乗平均)
∴ (左辺) ≦ (1/9){a^(2/3) + b^(2/3) + c^(2/3) + d^(2/3)} ≦ (4/9)(s/4)^(2/3).
  [第5章.727-729]
811132人目の素数さん:2012/11/03(土) 03:16:22.33
51. 訂正スマソ.
 (左辺) - (右辺) = (G^4){2(G^2 -1) +(a-b)^2} - (a+b)(G^2 -1)


73. 訂正スマソ.
 where G=(abc)^(1/3).

今日はこの辺で。
812132人目の素数さん:2012/11/03(土) 23:15:30.88
>>762
51. Let a>0, b>0. Prove that: (ab)^2・(a^2 +b^2 -2) ≧ (a+b)(ab-1).

 √ab = G とおくと、
 (a-b)^2 = (a+b)^2 - (2G)^2 = (a+b-2G)(a+b+2G),
 (左辺) - (右辺) = (G^4){2(G^2 -1) +(a-b)^2} - (a+b)(G^2 -1)
    = 2G(G^2 -1)(G^3 -1) + (a+b-2G){(a+b+2G)G^2 -G^2 +1}
    ≧ 0,
* (a+b+2G)G^2 - G^2 +1 ≧ 4G^3 -G^2 +1 ≧ 1 - 1/108,

>>797
62. (蛇足)
 a+b = s,
 f(x) = x{(x/2)^2 +1}^2, とおくと、
 (左辺) = {f(2a) + f(2b)}/2
   = (a+b){a^4 -(a^3)b +(ab)^2 -a(b^3) +b^4 + 2(a^2 -ab +b^2) +1}
   = f(s) + (1/4)s|a-b|^2・{(5/2)s^2 + (5/4)|a-b|^2 + 6}
   ≧ f(s)    (s≧0)
813132人目の素数さん:2012/11/04(日) 00:39:20.96
>>762

52. For 0≦ a,b,c ≦1, prove that:
  a/(bc+1) + b/(ca+1) + c/(ab+1) ≦ 2.

通分して、
 2(bc+1)(ca+1)(ab+1) -a(ca+1)(ab+1) -b(ab+1)(bc+1) -c(bc+1)(ca+1)
 = abc + a(1-b)(1-c) + (1-a)b(1-c) + (1-a)(1-b)c + 2(1-a)(1-b)(1-c)
 ≧ 0,
等号成立は {a,b,c} = {0,1,1}

56. Let x>0, y>0, z>0. Prove that:
 √{xy(x+y)} + √{yz(y+z)} + √{zx(z+x)} ≦ √{2(x+y+z)(xy+yz+zx)},

 (左辺)^2 = xy(x+y) + yz(y+z) + zx(z+x)
    +2y√{z(x+y)}√{x(y+z)} +2z√{x(y+z)}√{y(z+x)} +2x√{y(z+x)}√{z(x+y)}
   ≦ xy(x+y) +yz(y+z) +zx(z+x) +y{z(x+y)+x(y+z)} +z{x(y+z)+y(z+x)} +x{y(z+x)+z(x+y)}
   = {(x+y)+(y+z)+(z+x)}(xy+yz+zx)
   = 2(x+y+z)(xy+yz+zx),
  等号成立は x=y=z.
814132人目の素数さん:2012/11/04(日) 02:59:34.99
>>791

61. For a>0, b>0, c>0, prove that:
 (b+c-a)^2/{(b+c)^2+a^2} + (c+a-b)^2/{(c+a)^2+b^2} + (a+b-c)^2/{(a+b)^2+c^2} ≧ 3/5.
 (JMO-1997 本選, 第2問)

 3a/(a+b+c)=x 等とおくと、x+y+z=3,
 左辺第1項 = (3-2x)^2 /{(3-x)^2 +x^2}
   = 2 - 9/{(3-x)^2 +x^2}
   ≧ 2 - 9/{5-2(x-1)}
   ≧ 2 - (9/25){5+2(x-1)}
   = (1/5) - (18/25)(x-1),  (x=1における接線)
 循環的にたす。
 [初代スレ.103,111-112]

>>748 >>776

28. Let 0≦x,y,z<π/4. Prove the following inequality.
  {tan(x)・tan(y)・tan(z)}^(1/3) ≦ tan{(x+y+z)/3} ≦ {tan(x) + tan(y) + tan(z)}/3,
  (京大・理、1991前期、4番)

  y = log(tan(x)) は上に凸、y = tan(x) は下に凸。
  [初代スレ.227-229,233-235] [分かスレ172.381]
815132人目の素数さん:2012/11/04(日) 04:01:36.49
>>791

57. For a>0, b>0, c>0, prove that:
  (a+b)/√(a^2-ab+b^2) + (b+c)/√(3b^2-2bc+3c^2) + (c+a)/√(6c^2-3ca+6a^2) ≦ 11/3.

   a^2 - ab + b^2 = (1/4)(a+b)^2 + (3/4)|a-b|^2 ≧ (1/4)(a+b)^2,
  3b^2 -2bc +3c^2 =    (b+c)^2 +  2|b-c|^2 ≧   (b+c)^2,
  6c^2 -3ca +6a^2 = (9/4)(c+a)^2 + (15/4)|c-a|^2 ≧ (9/4)(c+a)^2,
 (左辺) ≦ 2 + 1 + (2/3) = 11/3.

>>814
   ≧ 2 - 9/{5-2(x-1)}
は間違い。
*) {(3-x)^2+x^2}{5+2(x-1)} - 25 = 2(2x+1)(x-1)^2 ≧ 0,

ではまた明日。
816132人目の素数さん:2012/11/04(日) 11:51:08.77
a, b, c, d が実数のとき,

a^4+b^4+c^4+d^4+4abcd+2(a^2+d^2)(b^2+c^2)+2

≧2a^2+2b^2+2c^2+2d^2
817132人目の素数さん:2012/11/04(日) 19:12:11.16
>>816

 a^2 + b^2 + c^2 + d^2 = S とおくと、
 (左辺) = (S-1)^2 + 1 -2(ad-bc)^2
   = (1/2)(S-2)^2 + (1/2)S^2 -2(ad-bc)^2
   = (1/2)(S-2)^2 + (1/2)(S-2ad+2bc)(S+2ad-2bc)
   = (1/2)(S-2)^2 + (1/2){(a-d)^2 +(b+c)^2}{(a+d)^2 +(b-c)^2}
   ≧ 0,
等号成立は (a,b,c,d) = (0,1,1,0) (1,0,0,1) のとき。
ぬるぽ
818132人目の素数さん:2012/11/04(日) 19:48:40.45
>>797

60.
 直角双曲線 xy=1 の外側の点P(a,b) について、 |a+b| ≧ 2,
xy=1 の点D(1,1)
 C(1/2, 1/2) と P(a,b) との距離CP ≧ CD,
∴ √(a^2 + b^2 -a -b +1/2) ≧ √(1/2),
でござるな。
819132人目の素数さん:2012/11/04(日) 22:08:51.28
新しいの来てた

122
Let $a,\ b$ be positive real numbers.
Prove that :
$$\displaystyle \frac{(1+a)(1+b)(1-ab)}{(1+a^2)(1+b^2)}\leq \frac{3\sqrt{3}}{4}$$

http://suseum.jp/gq/question/1870
820132人目の素数さん:2012/11/04(日) 23:49:50.13
>>819

122. Let a>0, b>0. Prove that:
  (1+a)(1+b)(1-ab)/{(1+a^2)(1+b^2)} ≦ (3√3)/4,
  等号成立は a = b = 2-√3 = tan(π/12).

 a=tanα, b=tanβ とおくと、(0 < α,β< π/2)
 (左辺) = (cosα+sinα)(cosβ+sinβ)cos(α+β)
     = 2cos(α -π/4)cos(β -π/4)cos(α+β)  
 だが....
821132人目の素数さん:2012/11/04(日) 23:53:01.33
>>817

a^2 + b^2 + c^2 + d^2 = S とおくと、
 (左辺) = (S-1)^2 + 1 -2(ad-bc)^2

何をやってるかわからん!
822132人目の素数さん:2012/11/05(月) 00:18:37.22
>>817 の訂正を...

(左辺)−(右辺) = {S^2 -2(ad)^2 -2(bc)^2 +4abcd +2}−2S
       = (S-1)^2 +1 -2(ad-bc)^2
       = ・・・・・
ですた。スマソ.
823132人目の素数さん:2012/11/05(月) 02:39:21.76
>>122

a, bを正に限る必要がわからん!
824132人目の素数さん:2012/11/07(水) 00:25:16.23
>>819

 (左辺) = (cosα+sinα)(cosβ+sinβ)cos(α+β)
     = {cos(α-β) + sin(α+β)}cos(α+β),

・|α+β|≦π/2 のとき、cos(α+β) ≧ 0,
 (左辺) ≦ {1 + sin(α+β)}cos(α+β)
   = (1+sinθ)cosθ = f(θ) ≦ f(π/6) = (3√3)/4,
 ∵ f '(θ) = (1+sinθ)(1-2sinθ) より、sinθ=1/2 で極大。

・π/2 ≦ |α+β| ≦π のとき、cos(α+β) ≦ 0,
 (左辺) ≦ {-1 + sin(α+β)}cos(α+β)
   = (1-sinθ)(-cosθ) = g(θ) ≦ g(-π/6) = (3√3)/4,
 ∵ g '(θ) = (1-sinθ)(1+2sinθ) より、sinθ=-1/2, で極大。

あるいは
 |sin(α+β)| ≧ |sin(α+β)cos(α-β)| = |sin(2α)+sin(2β)| /2 = |s|,
より
 (左辺)^2 = {1+sin(2α)}{1+sin(2β)} {1 - |sin(α+β)|^2}
  ≦ {1 + [sin(2α)+sin(2β)]/2}^2・(1-s^2)
  = (1+s)^2・(1-s^2)
  = (27/16) - (1/2 -s)^2・{(3/4)+(1+s)+(1+s)^2}
  ≦ 27/16,
等号成立は s=1/2, (α,β) = (π/12,π/12) かな?
825132人目の素数さん:2012/11/09(金) 06:36:35.96
>>725
 解凍例

(3)
x+y>1 の場合は 1-x-y<0 だから (左辺) <0.
∴ x+y≦1 の場合を考えれば十分。
 1-x-y = z ≧ 0 とおくと、
 (左辺) = xyz/{(y+z)(z+x)(x+y)}
     = xyz/{8xyz + x(y-z)^2 +y(z-x)^2 +z(x-y)^2}
    ≦ 1/8,
  等号成立は x=y-z=1/3.
826132人目の素数さん:2012/11/11(日) 00:23:23.19
>>725
 解凍例

(1)
 z = (2+ia)(2+ib)(2+ic)
  = 8 +4i(a+b+c) -2(ab+bc+ca) -iabc
  = 2(4-ab-bc-ca) +i(4a+4b+4c-abc)
  = 2(4-ab-bc-ca) +i(36-abc),

 |z|^2 の最小値を求める。
827132人目の素数さん:2012/11/11(日) 20:44:12.07
〔問題966〕
a,b,c>0, ab+bc+ca + 2abc = 1 のとき
 a+b+c + 4abc ≧ 2,
を示せ。(じゅー)

 casphy - 高校数学 - 不等式 - 966
828132人目の素数さん:2012/11/12(月) 00:14:55.32
>>793

71. Let a, b & c be the side lengths of a triangle. Prove that:
  a^2・b(a-b) + b^2・c(b-c) + c^2・a(c-a) ≧ 0.

 (b+c-a)/2 =x, (c+a-b)/2 =y, (a+b-c)/2 =z, とおく。(x,y,z>0)
 a=y+z, b=z+x, c=x+y,
 a^2・b(a-b) = (y+z)^2・(z+x)(y-x)
       = (xy^3 +yz^3) + (zy^3 -xz^3) +{2(yz)^2 -(zx)^2 -(xy)^2} +xyz(-2x+y-z),
 (左辺) = 2(xy^3 +yz^3 +zx^3 -xyz(x+y+z)}
    = 2{(2/7)xy^3 +(1/7)yz^3 +(4/7)zx^3 -(xyz)x} + cyclic
    ≧ 0.
829132人目の素数さん:2012/11/12(月) 01:18:13.63
x,y,z≧0, x^2+y^2+z^2+xyz=4 のとき, x+y+z≧xy+yz+zx.
830132人目の素数さん:2012/11/12(月) 01:49:45.89
>>793

Σcyc (1/2)(a-b)^2(a+b-c)≧0
831馬と鹿と豚さん:2012/11/12(月) 17:46:26.77
          __ノ)-'´ ̄ ̄`ー- 、_
        , '´  _. -‐'''"二ニニ=-`ヽ、
      /   /:::::; -‐''"        `ーノ
     /   /:::::/           \
     /    /::::::/          | | |  |
     |   |:::::/ /     |  | | | |  |
      |   |::/ / / |  | ||  | | ,ハ .| ,ハ|
      |   |/ / / /| ,ハノ| /|ノレ,ニ|ル' 
     |   |  | / / レ',二、レ′ ,ィイ|゙/   
.     |   \ ∠イ  ,イイ|    ,`-' |      
     |     l^,人|  ` `-'     ゝ  |        このスレは馬と鹿と豚ばかりね。
      |      ` -'\       ー'  人            
    |        /(l     __/  ヽ、          
     |       (:::::`‐-、__  |::::`、     ヒニニヽ、         
    |      / `‐-、::::::::::`‐-、::::\   /,ニニ、\            
   |      |::::::::::::::::::|` -、:::::::,ヘ ̄|'、  ヒニ二、 \
.   |      /::::::::::::::::::|::::::::\/:::O`、::\   | '、   \
   |      /:::::::::::::::::::/:::::::::::::::::::::::::::::'、::::\ノ  ヽ、  |
  |      |:::::/:::::::::/:::::::::::::::::::::::::::::::::::'、',::::'、  /:\__/‐、
  |      |/:::::::::::/::::::::::::::::::::::::::::::::::O::| '、::| く::::::::::::: ̄|
   |     /_..-'´ ̄`ー-、:::::::::::::::::::::::::::::::::::|/:/`‐'::\;;;;;;;_|
   |    |/::::::::::::::::::::::\:::::::::::::::::::::::::::::|::/::::|::::/:::::::::::/
    |   /:::::::::::::::::::::::::::::::::|:::::::::::::::::::::O::|::|::::::|:::::::::::::::/
832132人目の素数さん:2012/11/12(月) 18:13:12.79
角度を初等的に求める問題って、いつまで経っても解けるようにならんよな
不等式はどうだろう
833132人目の素数さん:2012/11/13(火) 00:15:07.19
キャスフィーの解答から...

>>827
定石どおりに置くのがシュアー
 a = x/(y+z), b = y/(z+x), c = z/(x+y),
 a+b+c + 4abc -2 = F_1(x,y,z)/{(x+y)(y+z)(z+x)} ≧ 0,
ここに
 F_n(x,y,z) = x^n・(x-y)(x-z) + y^n・(y-z)(y-x) + z^n・(z-x)(z-y) ≧ 0.

>>828
71.
 (左辺) = (1/2)Σ_cyc (a-b)^2・(a+b-c)(b+c-a) ≧ 0,
834132人目の素数さん:2012/11/13(火) 22:00:14.27
キャスフィーの解答から...

>>829
(2+x)(2-x-yz) = (y-z)^2 ≧ 0,
 0 ≦ 2-x-yz,
循環的にたすと、
 0 ≦ 6-s-t ≦ 6 -√(3t) -t = (√3 -√t)(2√3 +√t)
∴ 0 ≦ t ≦ 3,
∴ (s+t)(s-t) = s^2 - t^2 ≧ (3-t)t ≧ 0,
∴ s-t ≧ 0.

ここに、s=x+y+z, t=xy+yz+zx.
835132人目の素数さん:2012/11/13(火) 23:39:09.92
a+b+c=0をみたす実数a,b,cに対して、ab+bc+ca≦0
836132人目の素数さん:2012/11/14(水) 00:21:09.22
キャスフィーの解答 ぢゃないが...

>>835
 ab+bc+ca = {(a+b+c)^2 -a^2 -b^2 -c^2}/2
      = -(a^2 +b^2 +c^2)/2
      ≦ 0.
837132人目の素数さん:2012/11/15(木) 09:55:02.56
a+b+c=2 (a, b, c≧0) のとき,
√(a+4bc)+√(b+4ca)+√(c+4ab)≧4√(ab+bc+ca)
838馬と鹿と豚さん:2012/11/15(木) 20:26:13.66
          __ノ)-'´ ̄ ̄`ー- 、_
        , '´  _. -‐'''"二ニニ=-`ヽ、
      /   /:::::; -‐''"        `ーノ
     /   /:::::/           \
     /    /::::::/          | | |  |
     |   |:::::/ /     |  | | | |  |
      |   |::/ / / |  | ||  | | ,ハ .| ,ハ|
      |   |/ / / /| ,ハノ| /|ノレ,ニ|ル' 
     |   |  | / / レ',二、レ′ ,ィイ|゙/   
.     |   \ ∠イ  ,イイ|    ,`-' |      
     |     l^,人|  ` `-'     ゝ  |        このスレには馬と鹿と豚さんしかいないのね。
      |      ` -'\       ー'  人            
    |        /(l     __/  ヽ、          
     |       (:::::`‐-、__  |::::`、     ヒニニヽ、         
    |      / `‐-、::::::::::`‐-、::::\   /,ニニ、\            
   |      |::::::::::::::::::|` -、:::::::,ヘ ̄|'、  ヒニ二、 \
.   |      /::::::::::::::::::|::::::::\/:::O`、::\   | '、   \
   |      /:::::::::::::::::::/:::::::::::::::::::::::::::::'、::::\ノ  ヽ、  |
  |      |:::::/:::::::::/:::::::::::::::::::::::::::::::::::'、',::::'、  /:\__/‐、
  |      |/:::::::::::/::::::::::::::::::::::::::::::::::O::| '、::| く::::::::::::: ̄|
   |     /_..-'´ ̄`ー-、:::::::::::::::::::::::::::::::::::|/:/`‐'::\;;;;;;;_|
   |    |/::::::::::::::::::::::\:::::::::::::::::::::::::::::|::/::::|::::/:::::::::::/
    |   /:::::::::::::::::::::::::::::::::|:::::::::::::::::::::O::|::|::::::|:::::::::::::::/
839132人目の素数さん:2012/11/16(金) 07:06:43.12
Find the largest possible value of S = x^2y + 2x + 3 + 2y + xy^2, given that x^2 + y^2 = 2.

C.1144
http://www.komal.hu/verseny/feladat.cgi?a=honap&h=201211&t=mat&l=en
840β:2012/11/16(金) 07:08:51.74
暗算で解けた
841132人目の素数さん:2012/11/17(土) 00:06:16.57
正の数x,y,zに対して
(x+y)(y+z)(z+x)≦(x^3+1/y)(y^3+1/z)(z^3+1/x)


だれか助けて
842132人目の素数さん:2012/11/17(土) 00:19:50.89
>>839-840

C.1144.
解1.
 x^2 + y^2 = r^2 とおくと、
 1 ≦ xy+2 = (1/2){r^2 -(x-y)^2} +2 ≦ (1/2)r^2 +2,
 |x+y| = √{2r^2 - (x-y)^2} ≦ (√2)r,
等号が両立するのは x=y のとき。

∴ S = (xy+2)(x+y) +3
  ≦ (xy+2)|x+y| +3
  ≦ {(1/2)r^2 +2}(√2)r +3.

解2.
 S = (2-x^2)x + (2-y^2)y +2(x+y) +3
  = {3x−(1+1+x^3)} + {3y−(1+1+y^3)} + x+y +7
  = -(2+x)(1-x)^2 -(2+y)(1-y)^2 + x+y +7
  ≦ x+y +7    (等号はx=y=1)
  ≦ |x+y| +7
  = √{2r^2 -(x-y)^2} +7
  ≦ (√2)r +7,

安産...
843132人目の素数さん:2012/11/17(土) 23:14:50.44
>>837

これ,難しい。ようやく, 解けたが...
844132人目の素数さん:2012/11/17(土) 23:57:52.12
出題、既出か?

a,b,c>0 ⇒ a/(2a+b+c)+b/(a+2b+c)+c/(a+b+2c)≦3/4
845132人目の素数さん:2012/11/18(日) 00:26:22.36
          __ノ)-'´ ̄ ̄`ー- 、_
        , '´  _. -‐'''"二ニニ=-`ヽ、
      /   /:::::; -‐''"        `ーノ
     /   /:::::/           \
     /    /::::::/          | | |  |
     |   |:::::/ /     |  | | | |  |
      |   |::/ / / |  | ||  | | ,ハ .| ,ハ|
      |   |/ / / /| ,ハノ| /|ノレ,ニ|ル' 
     |   |  | / / レ',二、レ′ ,ィイ|゙/   
.     |   \ ∠イ  ,イイ|    ,`-' |      
     |     l^,人|  ` `-'     ゝ  |        このスレ 馬と鹿と豚ばかりね。
      |      ` -'\       ー'  人          
    |        /(l     __/  ヽ、          
     |       (:::::`‐-、__  |::::`、     ヒニニヽ、         
    |      / `‐-、::::::::::`‐-、::::\   /,ニニ、\            
   |      |::::::::::::::::::|` -、:::::::,ヘ ̄|'、  ヒニ二、 \
.   |      /::::::::::::::::::|::::::::\/:::O`、::\   | '、   \
   |      /:::::::::::::::::::/:::::::::::::::::::::::::::::'、::::\ノ  ヽ、  |
  |      |:::::/:::::::::/:::::::::::::::::::::::::::::::::::'、',::::'、  /:\__/‐、
  |      |/:::::::::::/::::::::::::::::::::::::::::::::::O::| '、::| く::::::::::::: ̄|
   |     /_..-'´ ̄`ー-、:::::::::::::::::::::::::::::::::::|/:/`‐'::\;;;;;;;_|
   |    |/::::::::::::::::::::::\:::::::::::::::::::::::::::::|::/::::|::::/:::::::::::/
    |   /:::::::::::::::::::::::::::::::::|:::::::::::::::::::::O::|::|::::::|:::::::::::::::/
846132人目の素数さん:2012/11/18(日) 07:14:18.57
>>844

相加・調和平均(またはコーシー)により
 (左辺) = 3 -(a+b+c){1/(2a+b+c) + 1/(a+2b+c) + 1/(a+b+2c)}
     ≦3 -(a+b+c)*9/{(2a+b+c)+(a+2b+c)+(a+b+2c)}  (相加-調和)
     = 3 -(a+b+c)*9/{4(a+b+c)}
     = 3 - (9/4)
     = 3/4.
847132人目の素数さん:2012/11/18(日) 10:18:12.48
>>846 早い、さすが


自分の解答

a/(2a+b+c)
=a/{(a+b)+(c+a)}
≦a*{1/(a+b)+1/(c+a)}/4(∵相加調和平均)
=(1/4){a/(a+b)+a/(c+a)}

他も同様にして、辺々を加えると目的の不等式を得る って考えた
848132人目の素数さん:2012/11/18(日) 16:55:55.17
他所で出題したやつだけど…

x,y,zが三角形の三辺 ⇒ (x+y-z)/(2x+2y-z)+(y+z-x)/(2y+2z-x)+(z+x-y)/(2z+2x-y)≦1
849132人目の素数さん:2012/11/19(月) 20:19:29.42
出題
a,b,c>0 ⇒ (a^2+ab+bc)/{b(c+2a)}+(b^2+bc+ca)/{c(a+2b)}+(c^2+ca+ab)/{a(b+2c)}≧3

なんか形が汚い;
850132人目の素数さん:2012/11/19(月) 21:46:40.60
>>848
キャスフィーの解答から...

解1.
分子 >0 だから
 (x+y-z)/(2x+2y-z) = 2(x+y-z)/[3(x+y-z)+(x+y+z)]
   ≦ (1/2)(x+y-z){1/[3(x+y-z)] + 1/(x+y+z)}  (調和・相加平均)
   = (1/6) + (x+y-z)/[2(x+y+z)],
循環的にたす。

解2.
条件を緩め、 分母>0 とする。
 (x+y-z)/(2x+2y-z) = {2(2x+2y-z) -(x+y+z)}/[3(2x+2y-z)]
   = (2/3) - (1/3)(x+y+z)/(2x+2y-z),
循環的にたすと
 (左辺) = 2 - (1/3)(x+y+z){1/(2x+2y-z) + 1/(2y+2z-x) + 1/(2z+2x-y)}
   ≦ 2 - 3(x+y+z)/[(2x+2y-z)+(2y+2z-x)+(2z+2x-y)]  (相加・相乗平均)
   = 2 - 1.
851132人目の素数さん:2012/11/20(火) 14:37:34.70
シュアー使う簡単な問題くれ
852132人目の素数さん:2012/11/21(水) 23:50:16.29
>>849

(a^2+ab+bc)/{b(c+2a)}=(a^2/b+a+c)/(c+2a)である。

ところで
(a^2/b+a+c)(b+a+c)≧(a+a+c)^2(∵コーシーシュワルツの不等式)
両辺を(a+b+c)(c+2a)で割って
(a^2/b+a+c)/(c+2a)≧(c+2a)/(a+b+c)

循環的に加えれば、(左辺)≧3
853132人目の素数さん:2012/11/22(木) 21:43:05.82
abc=1 をみたす正の数 a、b、c に対して、(a+b)/(ab+a+b) + (b+c)/(bc+b+c) + (c+a)/(ca+a+c) ≧ 2


やる気スイッチ、ポチッとな ( ゚∀゚)!

 パチン __ノ\
   off |  .ノ\| off
      ̄ ̄ ̄
854132人目の素数さん:2012/11/23(金) 03:21:34.30
>>853
 a=G/A, b=G/B, c=G/C, とおくと、
 ABC = G^3,
 (左辺) = (A+B)/(G+A+B) + (B+C)/(G+B+C) + (C+A)/(G+C+A)
   = 2 + {(A+B+C)(AB+BC+CA)-G^3 -2(A+B+C+G)G^2}/{(G+A+B)(G+B+C)(G+C+A)}
   = 2 + {(A+B+C)(AB+BC+CA-3G^2) +(A+B+C-3G)G^2}/{(G+A+B)(G+B+C)(G+C+A)}
   ≧ 2,
∵ 相加相乗平均より A+B+C-3G≧0, AB+BC+CA-3G^2≧0,


>>852
x = a+c≧0. コーシーより
 (a^2/b +x)(b+x) - (a+x)^2 = (a^2/b + b -2a)x = {(a-b)^2 /b}x ≧ 0,
855132人目の素数さん:2012/11/23(金) 04:36:31.81
>>854

その置き換えに気づきませんが、ヒラメキですか?
理詰めで、こう置けば上手くいくだろうとか、式の形から分かるものなのですか?
ちなみに模範解答は、もっと気づきにくい解法でしたが…
http://www.fen.bilkent.edu.tr/~cvmath/Problem/1209a.pdf
856132人目の素数さん:2012/11/23(金) 05:37:50.77
>>855

分母が2次式のままでは通分したくない。(メチャ面倒)
1次分数式にもってくる為に逆数変換しますた。(定石)

3乗根も使いたくないし....
857132人目の素数さん:2012/11/23(金) 06:35:53.83
なるほど〜、そう言われると必然な気もしますが、自分で気づけと言われてもなかなか…
精進しまつ

 パチン __ノ\
   on |  .ノ\| on
      ̄ ̄ ̄
858132人目の素数さん:2012/11/23(金) 12:07:27.76
AA手抜きwww
859132人目の素数さん:2012/11/23(金) 12:36:05.53
n!*e^n / {n+(1/2)}^{n+(1/2)} > 1
860132人目の素数さん:2012/11/23(金) 19:25:10.44
出題、他所で出しちゃったけど
|a|,|b|≦1,a+b≠0 ⇒ √(1-a^2)+√(1-b^2)+1≦2√(1+1/(a+b)^2)
861狢という野獣 ◆yEy4lYsULH68 :2012/11/23(金) 22:15:58.84
阿呆の書き込みは軽蔑に値するだけ。



>389 名前:粋蕎 ◆C2UdlLHDRI :2012/11/23(金) 20:18:33.20
> 低脳撲滅主義の下では現低脳が絶える時に低脳上限上昇による新低脳が生まれる故の無限淘汰地獄。
> 低脳撲滅主義に於いて低脳認定基準を設けても時代と共に基準は改正されるので無駄な事である。
> つまり猫改め描改め狢は学力的弱肉強食主義である。行き過ぎた撲滅主義は文化衰退を招く。
>
862132人目の素数さん:2012/11/24(土) 00:06:06.58
>>859
n->∞で√(2π/e)か、おもしろい
863狢という野獣 ◆yEy4lYsULH68 :2012/11/24(土) 04:30:27.06
阿呆の書き込みは軽蔑に値するだけ。馬鹿蕎麦が思いっきり晒す低脳。



>389 名前:粋蕎 ◆C2UdlLHDRI :2012/11/23(金) 20:18:33.20
> 低脳撲滅主義の下では現低脳が絶える時に低脳上限上昇による新低脳が生まれる故の無限淘汰地獄。
> 低脳撲滅主義に於いて低脳認定基準を設けても時代と共に基準は改正されるので無駄な事である。
> つまり猫改め描改め狢は学力的弱肉強食主義である。行き過ぎた撲滅主義は文化衰退を招く。
>
864132人目の素数さん:2012/11/26(月) 04:57:23.05
123
Let $a,\ b,\ c$ be positive real numbers such that $abc=1$.
Prove that :
$$a+b+c \geq \frac{2}{a+1} + \frac{2}{b+1} + \frac{2}{c+1}$$.

http://suseum.jp/gq/question/1893
865132人目の素数さん:2012/11/26(月) 06:46:01.16
うほっ、いい不等式
866132人目の素数さん:2012/11/26(月) 23:10:30.24
>>860
 キャスフィーの解答から....

a+b=s とおく。y=√(1-x^2) は上半円だから上に凸。
∴ (左辺) ≦ 2√{1 - (s/2)^2} +1 = √(4-s^2) +1,
∴ a=b=s/2 の場合を考えれば十分。

 (右辺) = 2√(1 + 1/s^2) = R(s) とおく。

 {[R(s)-1]^2 -(4-s^2)}{[R(s)+1]^2 -(4-s^2)}
 = {R(s)^2 -1}^2 -2{R(s)^2 -1}(4-s^2) +(4-s^2)^2 -4(4-s^2)
 = {R(s)^2 -1 -(4-s^2)}^2 -4(4-s^2)
 = (4/s^2 -1 +s^2)^2 -4(4-s^2)
 = (4/s^2 -1 -s^2)^2
 ≧ 0,
 等号成立は s^2 = {(√17)-1}/2 のとき。

∴ [R(s)-1]^2 = (4-s^2) + (4/s^2 -1-s^2)^2/{[R(s)+1]^2 -(4-s^2)}
        ≧ 4-s^2,

∴ √(4-s^2) +1 ≦ R(s),
867132人目の素数さん:2012/11/27(火) 05:43:43.52
私は某女子短大で教えているが、女子学生はキャンパス内では全員例外なく全裸になり、
学生証を安全ピンで乳首に刺して止めておくべきだ。
やらなければこちらがブスッと刺す。血が出るかも。
生理の時は私がタンポンを入れたり抜いたりしてやる。血が付くかも。
云う事聞かない奴は逆さ吊りだ。トイレに行きたくなっても行かせない。
クリスマスは私と女子学生の乱交パーティーだ 。勿論女子学生同士の愛も OK.
女子学生は皆食べ頃だ。参加しない奴には単位を出さない。

等と云った妄想を毎日朝から晩までしている。
授業中もチンコが立ちっぱなしで困る。
868132人目の素数さん:2012/11/27(火) 13:03:25.63
869132人目の素数さん:2012/11/27(火) 13:39:34.32
a[i]=(cosx[i])^2  0≦x[i]≦π/2
で終わり
870132人目の素数さん:2012/11/28(水) 23:23:04.88
>>868
 キャスフィーの解答から....

相乗・相加平均:
 (a1・a2・・・・・an)^(1/n) + {(1-a1)(1-a2)・・・・・(1-an)}^(1/n) ≦ 1.
で終わり
871132人目の素数さん:2012/11/29(木) 02:28:26.33
>>841 その問題の作者だけど解答上げるね,コーシーだけで解けたりする。

(x+y)^2≦(x^2+1)(1+y^2)(∵コーシー)
循環的にかけて、
{(x+y)(y+z)(z+x)}^2≦{(x^2+1)(y^2+1)(z^2+1)}^2…@
また、
(x^2+1)^2=(√(x^3)√x+√(1/y)√y)^2≦(x^3+1/y)(x+y)(∵コーシー)
循環的にかけて、
{(x^2+1)(y^2+1)(z^2+1)}^2
≦(x^3+1/y)(y^3+1/z)(z^3+1/x)(x+y)(y+z)(z+x)…A
@,Aより
{(x+y)(y+z)(z+x)}^2≦(x^3+1/y)(y^3+1/z)(z^3+1/x)(x+y)(y+z)(z+x)
両辺を(x+y)(y+z)(z+x)(>0)で割れば
(x+y)(y+z)(z+x)≦(x^3+1/y)(y^3+1/z)(z^3+1/x)
872132人目の素数さん:2012/11/30(金) 13:57:45.41
今月の大数の宿題

△ABCが鋭角三角形のとき
(sinA+sinB+sinC)/(cosA+cosB+cosC)
のとりうる値の範囲を求めよ
873132人目の素数さん:2012/11/30(金) 14:05:11.20
東工大かどっかで似たようなの見たことある気がする
874132人目の素数さん:2012/11/30(金) 14:43:20.05
875132人目の素数さん:2012/11/30(金) 14:55:11.98
あーそこまで似てないか
876132人目の素数さん:2012/12/01(土) 00:28:39.15
877132人目の素数さん:2012/12/01(土) 09:09:46.55
>>872

 {sin(A)+sin(B)+sin(C)}/{cos(A)+cos(B)+cos(C)} = f(A,B,C)
とおく.
 f(A,B,C) − f((A+B)/2,(A+B)/2,C)
  = 2sin(C-(A+B)/2){1-cos((A-B)/2)}/{[cos(A)+cos(B)+cos(C)][2cos((A+B)/2)+cos(C)]}
  = 2sin((3C-π)/2){1-cos((A-B)/2)}/{[cos(A)+cos(B)+cos(C)][2cos((A+B)/2)+cos(C)]}
    (← 題意)

A≦B≦C としてよい。
題意より、C ≧ π/3、
∴ f(A,B,C) ≧ f((A+B)/2, (A+B)/2, C)
    = f((π-C)/2, (π-C)/2, C)
    = {2cos(C/2) + sin(C)}/{2sin(C/2) + cos(C)}
    = 2cos(C/2){1+sin(C/2)}/{[1+sin(C/2)]^2 −3sin(C/2)^2}
    ≧ √3,
   等号は A=B=C=π/3 のとき。

題意より、A ≦π/3,
∴ f(A,B,C) ≦ f(A,(B+C)/2,(B+C)/2)
    = f(A,(π-A)/2,(π-A)/2)
    = {sin(A) + 2cos(A/2)}/{cos(A) + 2sin(A/2)}
    = 2cos(A/2){1+sin(A/2)}/{[1+sin(A/2)]^2 −3sin(A/2)^2}      
    < 2,
  等号は (A,B,C) = (0,π/2,π/2) のとき。
878132人目の素数さん:2012/12/01(土) 09:39:19.73
879132人目の素数さん:2012/12/01(土) 18:25:32.00
右側の第4問 ( ゚∀゚) ウヒョッ!
http://www.bmoc.maths.org/home/bmo2-2010.pdf
880132人目の素数さん:2012/12/02(日) 09:12:06.13
>877

www
881132人目の素数さん:2012/12/02(日) 11:05:42.24
    ∩___∩        ∧∧    ∧∧
    |       ヽ       ( 。_。)  ( 。_。) <旦那様が
   / ●   ● ヾ     /<▽> /<▽>
   |   ( _●_)   |     |::::::;;;;::/ |::::::;;;;::/    「エサはまだか?」
  彡、 |) |∪| .B`ミ    |:と),__」 .|:と),__」
_/ _‖ ヽノ ‖_\__. |::::::::|  .|::::::::|      と申しております
 (___)   (__ノ  \:::::::|  |:::::::|
882132人目の素数さん:2012/12/02(日) 11:36:43.70
125
Given real numbers $a,\ b,\ c\in{[0,\ 2]}$ with $a+b+c=3$.
Prove that :
$$\sqrt{4-a^2}+\sqrt{4-b^2}+\sqrt{4-c^2}+(\sqrt{3}-1)(|a-1|+|b-1|+|c-1|)\geq 3\sqrt{3}.$$

126
Let $a,\ b,\ c,\ d$ be real numbers such that $a^2+b^2+c^2+d^2=20$.
Prove that :
\[|a+2|+|b+2|+|c+2|+|d+2|+|a|+|b|+|c|+|d|+|a-2|+|b-2|+|c-2|+|d-2|\leq 28.\]

127
Let $a,b,c$ be positive real numbers such that $ab+bc+ca+2abc=1.$
Prove that :
$$a+b+c-8abc\ge\frac{1}{2}.$$
883132人目の素数さん:2012/12/03(月) 00:42:42.47
>>878
125.
0≦x≦2 のとき
 g(x) = √(4-x^2) + (√3 -1)|x-1| ≧ √3 +1 -x,

∵ y=√(4-x^2) は上半円の一部なので、上に凸。
 0≦x≦1 ⇒ g(x) ≧ g(1) + {g(0)-g(1)}(1-x) = √3 +1 -x,
 1≦x≦2 ⇒ g(x) ≧ g(1) + {g(2)-g(1)}(x-1) = √3 +1 -x,

126.
 |x-2|+|x|+|x+2| = Max{4+|x|, 3|x|} ≦ (9+x^2)/2,
  等号成立は |x| = 1, 3 のとき。
∵ (9+x^2)/2 - (4+|x|) = (1/2)(1-|x|)^2 ≧ 0,
  (9+x^2)/2 - 3|x| = (1/2)(3-|x|)^2 ≧ 0,
884132人目の素数さん:2012/12/03(月) 00:50:55.36
>>882
127.定石どおりに
 a=x/(y+z), b=y/(z+x), c=z/(x+y), (x,y,z>0)
とおく。  >>400 >>833
 x+y+z = s,
 xy+yz+zx = t,
 xyz = u,
とおくと、
 (x+y)(y+z)(z+x) = st-u,
 (左辺) = {(x^3 +y^3 +z^3 -3xyz) + (st-5u)}/(st-u)
   = (1/2) + {(x^3 +y^3 +z^3 -3xyz) + (st-9u)/2}/(st-u)
   ≧ 1/2.
∵ 相加相乗平均より
 x^3+y^3+z^3 -3xyz ≧0,
 st-9u = x(y-z)^2 + y(z-x)^2 + z(x-y)^2 ≧ 0,
885132人目の素数さん:2012/12/03(月) 00:54:51.83
>>879
キャスフィーの解答から....

問題4.
 x,y,z>0 ⇒ 4(x+y+z)^3 > 27(xxy+yyz+zzx),

(略解)
 x≦y,z としても一般性を失わない。
 y-x=b>0, z-x=c>0 とおく。

 f(x,y,z) = (x+y+z)^3 - 9(xxy+yyz+zzx),
  = f(0,y-x,z-x)
  = f(0,b,c)

 (左辺) - (右辺) = 4(x+y+z)^3 -27(xxy+yyz+zzx)
  = (x+y+z)^3 + 3f(x,y,z)
  = (3x+b+c)^3 + 3f(0,b,c)
  ≧ 4(b+c)^3 -27・b^2・c
  = (4b+c)(b-2c)^2
  ≧ 0,
 等号成立は b-2c=0, (x,y,z)=(0,2,1) とそのrotation
 >>100 に一般化がある。
886132人目の素数さん:2012/12/03(月) 01:12:40.89
キャスフィーから....

127.の〔類題763〕
a,b,c >0 が ab+bc+ca + 2abc =1, を満たすとき次を示せ。(じゅー)
 a/(1+a^2) + b/(1+b^2) + c/(1+c^2) ≦ 6/5,

 casphy - 高校数学 - 不等式 - 763
887132人目の素数さん:2012/12/03(月) 23:03:02.18
>>879 >>885

 >>393 >>404-408 で解決済....
888132人目の素数さん:2012/12/04(火) 11:31:52.69
正の数 a、b、c が ab+bc+ca=3 をみたすとき、(3a^2+2)(3b^2+2)(3c^2+2)≧125
889132人目の素数さん:2012/12/04(火) 11:55:48.82
272 名前:132人目の素数さん[sage] 投稿日:2012/01/27(金) 00:52:05.16
拝啓
寒さがひとしお身にしみるころとなりましたが如何お過ごしでせうか。
遅ればせながら、的確なる御指摘に慎んで感謝致しまつ。
風邪など召されませぬやう、くれぐれもお気をつけてお過ごしくださいませ。
急ぎ用件のみにて失礼いたしまつ。
敬具


274 返信:132人目の素数さん[sage] 投稿日:2012/01/27(金) 00:58:55.75
>>272
多忙に付きせんだっての件挨拶が遅れてたいへんもうしわけなく思っていました
890132人目の素数さん:2012/12/04(火) 15:34:34.85
0≦a≦1、0≦b≦1、a^2+b^2=1のとき、(a+b)(7/4-ab)≦1.5^1.5
891132人目の素数さん:2012/12/04(火) 16:37:43.78
凄まじい。既成政党を打破とか言ってるけどお前のとこが一番独善的
共産党やナチスより酷い妙ちくりんな集団
前科者も複数紛れて、いかにも大阪らしい

無党派さん[] 投稿日:2012/12/03(月) 21:55:03.05 ID:3W6Dp3GZ [11/13]
http://www.jiji.com/jc/c?g=pol_30&k=2012120300800

維新 比例名簿の登載順位は次の通り。
(丸囲み数字は順位。同一順位の重複立候補者名は省略。前=前職、元=元職、新=新人。敬称略)
 【北海道=4人】(1)重複候補3人 (4)米長知得(新)
 【東北=11人】(1)小熊慎司(新)=福島4区 (2)重複候補2人 (4)重複候補8人
 【北関東=18人】(1)上野宏史(新)=群馬1区 (2)石関貴史(前)=同2区 (3)重複候補14人 (17)植竹哲也(新)(18)仲田大介(新)
 【南関東=20人】(1)小沢鋭仁(前)=山梨1区 (2)松田学(新) (3)重複候補16人 (19)田中甲(元) (20)横田光弘(新)
 【東京=22人】(1)石原慎太郎(元) (2)今村洋史(新) (3)山田宏(元)=東京19区 (4)重複候補18人 〔22〕上村昭徳(新)
 【北信越=10人】(1)中田宏(元) (2)重複候補8人 (10)堀居哲郎(新)
 【東海=14人】(1)藤井孝男(元) (2)今井雅人(前)=岐阜4区 (3)重複候補11人 (14)近藤浩(元)
 【近畿=40人】(1)東国原英夫(新) (2)西村真悟(元) (3)重複候補8人 (11)三宅博(新) (12)重複候補28人 〔40〕喜多義典(新)
 【中国=8人】(1)重複候補6人 (7)藤井昇(新) (8)谷本彰良(新)
 【四国=7人】(1)重複候補4人 (2)重複候補2人 (7)大内淳司(新)
 【九州=19人】(1)松野頼久(前)=熊本1区 (2)重複候補17人 (19)黒仁田典之(新)

マスコミに踊らされる前に、中身見たほうがいいな
892狢は性犯罪者 ◆yEy4lYsULH68 :2012/12/04(火) 17:47:29.81
>>525
数学者になりたかったら:
1.『犯罪に手を染めない事』:★★★重要な追加事項★★★
2.もし出来たら論文でも書きましょうネ。♪

どや、コレでエエのんかァ! お返事してや〜

ケケケ狢

>525 名前:132人目の素数さん :2012/12/02(日) 15:30:43.08
> >>524
> 犯罪に手を染めない事も付け加えとけ、前科者。
>
893132人目の素数さん:2012/12/04(火) 22:30:15.49
>>886
キャスフィーの解答から....

a/(1+a^2) = (1+28a)/{25(1+a)} - {(1+7a)(1-2a)^2}/{25(1+a)(1+a^2)}
  ≦ (1+28a)/{25(1+a)}
  = (28x+y+z)/{25(x+y+z)},  (← a=x/(y+z))
循環的にたす。等号成立は a=b=c=1/2,
894芳雄:2012/12/05(水) 00:49:33.83
>>892
うるせぇ!

芳雄
895狢は性犯罪者 ◆yEy4lYsULH68 :2012/12/05(水) 03:12:38.34
>>525
数学者になりたかったら:
1.『犯罪に手を染めない事』:★★★重要な追加事項★★★
2.もし出来たら論文でも書きましょうネ。♪

どや、コレでエエのんかァ! お返事してや〜

ケケケ狢

>525 名前:132人目の素数さん :2012/12/02(日) 15:30:43.08
> >>524
> 犯罪に手を染めない事も付け加えとけ、前科者。
>
896132人目の素数さん:2012/12/05(水) 03:27:24.16
おら猫、返事しろや
897132人目の素数さん:2012/12/05(水) 16:19:57.19
>>890
降参
898132人目の素数さん:2012/12/05(水) 16:23:19.51
    ∩___∩        ∧∧    ∧∧
    |       ヽ       ( 。_。)  ( 。_。) <旦那様が
   / ●   ● ヾ     /<▽> /<▽>
   |   ( _●_)   |     |::::::;;;;::/ |::::::;;;;::/    「解答はまだか?」
  彡、 |) |∪| .B`ミ    |:と),__」 .|:と),__」
_/ _‖ ヽノ ‖_\__. |::::::::|  .|::::::::|      と申しております
 (___)   (__ノ  \:::::::|  |:::::::|
899132人目の素数さん:2012/12/05(水) 21:37:58.28
>>890
キャスフィーの解答から....

便宜上、√1.5 = t とおく。
 (右辺) - (左辺)
  = t^3 -(a+b){(9/4)-(1/2)(a+b)^2}
  = t^3 - (3/2)(a+b)t^2 + (1/2)(a+b)^3
  = {t + (a+b)/2}(t-a-b)^2
  ≧ 0,
等号成立は a+b=t, ab=1/4 のとき。
 {a,b} = {(√6 -√2)/4, (√6 +√2)/4}
    = {sin(π/12), cos(π/12)}.
900132人目の素数さん:2012/12/05(水) 23:50:11.12
>>890
    ∩___∩        ∧∧    ∧∧
    |       ヽ       ( 。_。)  ( 。_。) <旦那様が
   / ●   ● ヾ     /<▽> /<▽>
   |   ( _●_)   |     |::::::;;;;::/ |::::::;;;;::/    「a=sinθ、b=cosθ、0≦θ≦π/2と置きたい」
  彡、 |) |∪| .B`ミ    |:と),__」 .|:と),__」
_/ _‖ ヽノ ‖_\__. |::::::::|  .|::::::::|      と申しております
 (___)   (__ノ  \:::::::|  |:::::::|
901132人目の素数さん:2012/12/06(木) 23:47:56.44
美しい不等式の世界 ─数学オリンピックの問題を題材として─

A5/272ページ/2013年01月25日
ISBN978-4-254-11137-8 C3041
定価3,990円(税込)
佐藤淳郎 訳

"Inequalities A Mathematical Olympicd Approach"の翻訳。
数学全般で広く使われる有名な不等式や実用的テクニックを系統立てて説明し,
数学オリンピックの問題をふんだんに使って詳しく解説。
多数の演習問題およびその解答も付す。

http://www.asakura.co.jp/books/isbn/978-4-254-11137-8/



きたか…!!

  ( ゚д゚ ) ガタッ
  .r   ヾ
__|_| / ̄ ̄ ̄/_
  \/    /
902132人目の素数さん:2012/12/07(金) 00:17:27.98
>>888

a+b+c=s, ab+bc+ca=t, abc=u とおく。
題意より、s≧3, t=3, u≦1.
便宜上、K > 1.4444847 とする。

(3a^2 +K)(3b^2 +K)(3c^2 +K) - (t+K)^3
 = -(3u + ct/2)c(a-b)^2 -(3u + at/2)a(b-c)^2 -(3u + bt/2)b(c-a)^2
  + 3{cc(a-b)^2 + aa(b-c)^2 + bb(c-a)^2}K
  + 1.5{(a-b)^2 + (b-c)^2 + (c-a)^2}K^2
 =  (-3u -ct/2 +3ccK +1.5KK)(a-b)^2
  + (-3u -at/2 +3aaK +1.5KK)(b-c)^2
  + (-3u -bt/2 +3bbK +1.5KK)(c-a)^2
 ≧ (-3 -3c/2 +3ccK +1.5KK)(a-b)^2
  + (-3 -3a/2 +3aaK +1.5KK)(b-c)^2
  + (-3 -3b/2 +3bbK +1.5KK)(c-a)^2,
また、
 -3 -3c/2 +3ccK +1.5KK
 = (3/K)(1/4 - cK)^2 + (3/2K)(K^3 -2K -1/8)
 ≧ 0, etc.
903β:2012/12/07(金) 00:27:55.56
解法ダサすぎ
904132人目の素数さん:2012/12/07(金) 01:09:35.22
む、むむむ…
905132人目の素数さん:2012/12/07(金) 15:21:44.31
不等式が得意じゃないので、是非勉強したいと思います。
a^2+b^2+c^2≧ab+bc+ca
まずこれから証明を得たいと思います
でも検討がつきません。
よければコツを教えてください
906132人目の素数さん:2012/12/07(金) 15:26:48.22
>>905
それは代表的な参考書に必ず出ている典型問題
まず自分の持っている参考書を全ページ見直し給え
載っていなかったなら本屋に行って載ってる本を購入し直すことを勧める
その不等式をそのままぐぐっても手掛かりが見つかるはず
ヒントだけ言っとくと2乗の和の形に整理する
907132人目の素数さん:2012/12/07(金) 22:58:09.21
1.2変数から3変数の拡張

2. aについての2次不等式とみなす

3. 対称性に着目
908132人目の素数さん:2012/12/07(金) 23:01:13.03
正の実数 a,b,c が a+b+c=3 を満たすとき, (aa+bb+cc)^2≧3abc(aaa+bbb+ccc)
を示してください。
909132人目の素数さん:2012/12/07(金) 23:44:20.08
>>905 のコツ

・差をとって、2乗の和の形に整理する。 >>906

・相加相乗平均で a^2 + b^2 ≧ 2ab、循環的にたす。

・コーシーで (a^2 +b^2 +c^2)(A^2 +B^2 +C^2) ≧ (aA+bB+cC)^2,
  A→b, B→c, C→a.

・チェビシェフで、 箔ッ順序積 ≧ 迫随序積,

・min{a,b,c)=m, {a,b,c}={m,m+x,m+y} とおく。
 (左辺) - (右辺) = x^2 + y^2 - xy = {x^2 + y^2 + (x-y)^2}/2 ≧ 0
910132人目の素数さん:2012/12/08(土) 00:43:39.28
普通に
(a - b)^2 + (b - c)^2 + (c - a)^2 + 2 (a b + b c + c a) =
2 ( a^2 + b^2 + c^2)

∴( a^2 + b^2 + c^2) >= (a b + b c + c a)
でいいんじゃ?
911132人目の素数さん:2012/12/08(土) 07:54:34.32
>>877
デタラメな解答書くな
912132人目の素数さん:2012/12/08(土) 21:42:09.84
>>877

 f(θ, (π-θ)/2, (π-θ)/2)
  = {sinθ + 2cos(θ/2)}/[cosθ + 2sin(θ/2)]
  ≡ g(θ),
とおくと、
 g '(θ) = {sin(3θ/2)-1}/[cosθ + 2sin(θ/2)]^2 ≦ 0,
で単調減少。
 f(A,B,C) ≧ g(C) > g(π/2) = 1 + (1/√2),
 f(A,B,C) ≦ g(A) < g(0) = 2,
と言いたかった?

>>910
 仰せのとおり。
913132人目の素数さん:2012/12/08(土) 22:34:35.08
910>>

それは,解法を知っているからさくっと、変形できるのですな。

a, b, c, d>0 のとき, a^4+b^4+c^4+d^4-4abcd≧0 を正値式の和形による変形してみよ。
914132人目の素数さん:2012/12/08(土) 22:35:52.39
>>877>>912 だろ
915132人目の素数さん:2012/12/09(日) 00:13:50.92
>>913
たとえば
 a^4 + b^4 -2(ab)^2 = (a^2 -b^2)^2,
 c^4 + d^4 -2(cd)^2 = (c^2 -d^2)^2,
 (ab)^2 + (cd)^2 -2abcd = (ab-cd)^2,
 (ab)^2 + (cd)^2 -2abcd = (ab-cd)^2,
辺々たす。
(符号条件は不要かも....)
916132人目の素数さん:2012/12/10(月) 01:24:53.39
>>901
密かに期待している、これ読めば解法が身につくだろうか?
パッと見てどの置き換えでいけばいいか見えてくればいいのだがの
917132人目の素数さん:2012/12/14(金) 19:41:44.94
128
Prove that :
$$\left(1-\frac{1}{n}\right)^{n-1}\left(1+\frac{1}{n}\right)^n \geq 1\ (n=2,\ 3,\ \cdots)$$
without using definition of $e$.
918あのこうちやんは始皇帝だった:2012/12/14(金) 19:52:52.33
 20代と60代の、ニート・無職の、女性恐怖症の、頭デッカチの虚弱児・ひ弱の、ゴミ・クズ・カス・無能・虫けらのクソガキども!

 死ね!!!!!!!!!!!!!!!!!!!!!!
919御令嬢様:2012/12/15(土) 00:18:35.04
          __ノ)-'´ ̄ ̄`ー- 、_
        , '´  _. -‐'''"二ニニ=-`ヽ、
      /   /:::::; -‐''"        `ーノ
     /   /:::::/           \
     /    /::::::/          | | |  |
     |   |:::::/ /     |  | | | |  |
      |   |::/ / / |  | ||  | | ,ハ .| ,ハ|
      |   |/ / / /| ,ハノ| /|ノレ,ニ|ル' 
     |   |  | / / レ',二、レ′ ,ィイ|゙/   私は只の数ヲタなんかとは付き合わないわ。
.     |   \ ∠イ  ,イイ|    ,`-' |      頭が良くて数学が出来てかっこいい人。それが必要条件よ。
     |     l^,人|  ` `-'     ゝ  |        さらに Ann.of Math に論文書けば十分条件にもなるわよ。
      |      ` -'\       ー'  人          一番嫌いなのは論文数を増やすためにくだらない論文を書いて
    |        /(l     __/  ヽ、           良い論文の出版を遅らせるお馬鹿な人。
     |       (:::::`‐-、__  |::::`、     ヒニニヽ、         あなたの論文が Ann of Math に accept される確率は?
    |      / `‐-、::::::::::`‐-、::::\   /,ニニ、\            それとも最近は Inv. Math. の方が上かしら?
   |      |::::::::::::::::::|` -、:::::::,ヘ ̄|'、  ヒニ二、 \
.   |      /::::::::::::::::::|::::::::\/:::O`、::\   | '、   \
   |      /:::::::::::::::::::/:::::::::::::::::::::::::::::'、::::\ノ  ヽ、  |
  |      |:::::/:::::::::/:::::::::::::::::::::::::::::::::::'、',::::'、  /:\__/‐、
  |      |/:::::::::::/::::::::::::::::::::::::::::::::::O::| '、::| く::::::::::::: ̄|
   |     /_..-'´ ̄`ー-、:::::::::::::::::::::::::::::::::::|/:/`‐'::\;;;;;;;_|
   |    |/::::::::::::::::::::::\:::::::::::::::::::::::::::::|::/::::|::::/:::::::::::/
    |   /:::::::::::::::::::::::::::::::::|:::::::::::::::::::::O::|::|::::::|:::::::::::::::/
920132人目の素数さん:2012/12/15(土) 00:57:20.73
>>917

128.
まづ、
 1+x ≦ e^x より、log(1+x) ≦ x,

 log(1 -1/N) = -log(1 + 1/(N-1)) ≧ -1/(N-1),
∴ (N-1)log(1 -1/N) ≧ -1,
N = n^2 とおいて n+1 で割ると、
 (n-1)log(1 -1/n^2) ≧ -1/(n+1),
一方、
 log(1 +1/n) = -log(1 -1/(n+1)) ≧ 1/(n+1),
辺々たすと、

 log(左辺) = (n-1)log(1 -1/n) + n・log(1 +1/n)
 = (n-1)log(1 -1/n^2) + log(1 +1/n)
 ≧ -1/(n+1) + 1/(n+1)
 = 0,
921132人目の素数さん:2012/12/15(土) 13:18:10.28
自然数nに対して
納i=0,n](n-i)*(2nCi)<2^2n

コレって正しいですか?
Cがあるとうまい方法が思い浮かばない
922令嬢:2012/12/15(土) 20:56:33.79
          __ノ)-'´ ̄ ̄`ー- 、_
        , '´  _. -‐'''"二ニニ=-`ヽ、
      /   /:::::; -‐''"        `ーノ
     /   /:::::/           \
     /    /::::::/          | | |  |
     |   |:::::/ /     |  | | | |  |
      |   |::/ / / |  | ||  | | ,ハ .| ,ハ|
      |   |/ / / /| ,ハノ| /|ノレ,ニ|ル' 
     |   |  | / / レ',二、レ′ ,ィイ|゙/   
.     |   \ ∠イ  ,イイ|    ,`-' |      
     |     l^,人|  ` `-'     ゝ  |        このスレは馬と鹿と豚さんばかりね。
      |      ` -'\       ー'  人            
    |        /(l     __/  ヽ、          
     |       (:::::`‐-、__  |::::`、     ヒニニヽ、         
    |      / `‐-、::::::::::`‐-、::::\   /,ニニ、\            
   |      |::::::::::::::::::|` -、:::::::,ヘ ̄|'、  ヒニ二、 \
.   |      /::::::::::::::::::|::::::::\/:::O`、::\   | '、   \
   |      /:::::::::::::::::::/:::::::::::::::::::::::::::::'、::::\ノ  ヽ、  |
  |      |:::::/:::::::::/:::::::::::::::::::::::::::::::::::'、',::::'、  /:\__/‐、
  |      |/:::::::::::/::::::::::::::::::::::::::::::::::O::| '、::| く::::::::::::: ̄|
923132人目の素数さん:2012/12/15(土) 23:05:10.25
>>921

 (n-i)*C[2n,i] = {(2n-i)*C[2n,i] - i*C[2n,i]}/2
        = n*{C[2n-1,i] - C[2n-1,i-1]},  (0<i<2n)
 (n-i)*C[2n,i] = n = n*C[2n-1,i]        (i=0)
よって
 (左辺) = n*C[2n-1,n] = (n/2)*C[2n,n]

答: n≦12 ならば正しいが、n≧13 ならば正しくない。
924132人目の素数さん:2012/12/16(日) 01:32:10.67
正しい不等式ではなかったんですね
それにしても左辺がこんなにきれいな形になるなんて驚きました
教えてくださってありがとうございます
925令嬢:2012/12/16(日) 13:36:42.98
          __ノ)-'´ ̄ ̄`ー- 、_
        , '´  _. -‐'''"二ニニ=-`ヽ、
      /   /:::::; -‐''"        `ーノ
     /   /:::::/           \
     /    /::::::/          | | |  |
     |   |:::::/ /     |  | | | |  |
      |   |::/ / / |  | ||  | | ,ハ .| ,ハ|
      |   |/ / / /| ,ハノ| /|ノレ,ニ|ル' 
     |   |  | / / レ',二、レ′ ,ィイ|゙/   
.     |   \ ∠イ  ,イイ|    ,`-' |      
     |     l^,人|  ` `-'     ゝ  |        このスレは馬と鹿と豚さんばかりね。
      |      ` -'\       ー'  人            
    |        /(l     __/  ヽ、          
     |       (:::::`‐-、__  |::::`、     ヒニニヽ、         
    |      / `‐-、::::::::::`‐-、::::\   /,ニニ、\            
   |      |::::::::::::::::::|` -、:::::::,ヘ ̄|'、  ヒニ二、 \
.   |      /::::::::::::::::::|::::::::\/:::O`、::\   | '、   \
   |      /:::::::::::::::::::/:::::::::::::::::::::::::::::'、::::\ノ  ヽ、  |
  |      |:::::/:::::::::/:::::::::::::::::::::::::::::::::::'、',::::'、  /:\__/‐、
  |      |/:::::::::::/::::::::::::::::::::::::::::::::::O::| '、::| く::::::::::::: ̄|
   |     /_..-'´ ̄`ー-、:::::::::::::::::::::::::::::::::::|/:/`‐'::\;;;;;;;_|
   |    |/::::::::::::::::::::::\:::::::::::::::::::::::::::::|::/::::|::::/:::::::::::/
    |   /:::::::::::::::::::::::::::::::::|:::::::::::::::::::::O::|::|::::::|:::::::::::::::/
926132人目の素数さん:2012/12/17(月) 23:20:58.22
>>917
キャスフィーの解答から....

128.
{n/(n-1), ・・・・, n/(n-1), 1} のn個で相加・相乗する。
 (1 + 1/n)^n > {n/(n-1)}^(n-1),
927132人目の素数さん:2012/12/22(土) 00:58:58.38
正の数 a_i (i=1,2,…,n)、n≧2 に対して、(Σa_i)・{Σ(1/a_i)} ≧ n^2 - 2 + [ {Σ(a_i)^k}・{Σ(1/a_i)^k} - n^2 + 2^k]^(1/k)
928132人目の素数さん:2012/12/22(土) 01:10:03.62
n leq 2, ((n+1)^(n-1))((n+2)^n) > (3^n)(n!)^2
929132人目の素数さん:2012/12/23(日) 22:14:08.20
>>928
キャスフィーの解答から....

・略解1
nについての帰納法による。
n=1 のときは、等号成立。
n≧2 のときは
(n+2)^n/n^(n-2) = {(n+2)/n}^n・n^2
  = (1 + 2/n)^n・n^2
  > 3・n^2   (←2項展開公式)
により成立。

・略解2
 n=1,2 のときは成立。
 n>2 のとき
 (左辺) > 2^(2n-3)・n!・(n+1)! > (3^n)(n!)^2,

〔補題〕
 (n+1)^(n-1) > 2^(n-2)・n!,
(略証)
 (n+1)^2 ≧ 4k(n+1-k), [k=2,3,・・・・,n-1]
辺々掛けて√すると
 (n+1)^(n-2) > 2^(n-2)・(n-1)!
930132人目の素数さん:2012/12/24(月) 00:13:33.49
なるほど、帰納法か

そういや昔、帰納法って書いたら怒られたことがあった。
それは帰納法じゃなくて数学的帰納法だと。
何が起こったのか分からなかったぜ、ありのままに起こったことを話したぜ

  |l、{   j} /,,ィ//|     / ̄ ̄ ̄ ̄ ̄ ̄ ̄ ̄ ̄ ̄ ̄ ̄ ̄ ̄ ̄ ̄ ̄ ̄ ̄ ̄
  i|:!ヾ、_ノ/ u {:}//ヘ     | あ…ありのまま 今 起こった事を話すぜ!
  |リ u' }  ,ノ _,!V,ハ |     < 『おれは奴の前で階段を登っていたと
  fト、_{ル{,ィ'eラ , タ人.    |  思ったらいつのまにか降りていた』
 ヾ|宀| {´,)⌒`/ |<ヽトiゝ   | 催眠術だとか超スピードだとか
  ヽ iLレ  u' | | ヾlトハ〉.   | そんなチャチなもんじゃあ 断じてねえ
   ハ !ニ⊇ '/:}  V:::::ヽ. │ もっと恐ろしいものの片鱗を味わったぜ…
  /:::丶'T'' /u' __ /:::::::/`ヽ \____________________
931132人目の素数さん:2012/12/26(水) 14:58:14.78
>>930
「数学的帰納法」は演繹のテクニックなので決して「帰納法」とは呼べないのだ。
932132人目の素数さん:2012/12/27(木) 04:05:27.52
すうじあむで見つけた不等式、微分してもできんのだけど、不等式ヲタなら瞬殺ですか?
http://suseum.jp/gq/question/1930
933132人目の素数さん:2012/12/28(金) 00:15:05.99
>>902

a+b+c=s, ab+bc+ca=t, abc=u とおく。
便宜上、K≧t/3=1 とする。

(a-b)^2 の係数
 = -3cu + (-t/2 +3K)cc + 1.5KK
 ≧ -3cu + (1/2)cct + (1/6)tt  (← K≧t/3)
 = 3c{-u +(t/3)^(3/2)} + (1/2)t{c-√(t/3)}^2  
 ≧ 0, etc.
∵ (t/3)^(3/2) ≧ u,
934132人目の素数さん:2012/12/28(金) 20:51:06.30
>>888

 a+b+c=s, ab+bc+ca=t, abc=u とおく。
 便宜上、K≧t/3 とする。

(左辺)−(右辺) =
 = -(t^3 -27u^2) + 6K(t^2 -3su) + 3KK(s^2 -3t) ・・・・ (*)
 = 3(sK-3u)^2 - t(t-3K)^2
 ≧ (1/3)(st-9u)^2,   (← K≧t/3)

*) Kについて単調増加。
935132人目の素数さん:2012/12/29(土) 08:10:37.33
936132人目の素数さん:2012/12/30(日) 13:36:52.86
>>932は誰もできないの?
937132人目の素数さん:2012/12/30(日) 13:43:25.19
>>936
君に任せた!
938132人目の素数さん:2012/12/31(月) 11:51:16.15
ヤクザやなりすましを使い成人式を荒らしているのは広告代理店やテレビ局の自作自演です。
反原発デモでチンドンや太鼓を鳴らし、ソントを行っている在日
街宣車に乗り、騒音を撒く朝鮮人

構図は全て同じです
http://mamorenihon.files.wordpress.com/2011/10/zainichi_seijinshiki_1.jpg
http://4.bp.blogspot.com/-1_jEcAWVs1s/Tm4X2mLBhhI/AAAAAAAABok/MObw2nzMyoI/s1600/IMG_9062-714071.jpg
http://wave.ap.teacup.com/renaissancejapan/timg/middle_1228711905.jpg

ソント(声闘)・・朝鮮人は、声が大きければ音が大きければ、主張が通る、どんな凶悪犯罪でも無罪になると思い込む。
だから、朝鮮人は大音量で街宣車を走らせる。
原発サウンドデモ=ソント

在日(もちろんテレビや新聞は通名で報道)が凶悪犯罪を起こしたとき、人権派()弁護士が異様に沢山付くのも
弁護士が多ければ=声が大きければ、無罪になると思い込んでいるから。
939132人目の素数さん:2012/12/31(月) 20:13:47.84
>>937 任されちまった。生姜ねぇなぁ...

〔問題932〕
次を示せ。ただし eは自然対数の底で 2.7<e<2.8 とし、整数乗の計算には計算機を用いてもよい。
 x>0 ならば e^x > 1 + (3/2)x^2,

ヲタなら、まづ相加・相乗するだろうな....
 (2/3)x + (1/3!)x^3 ≧ (2/3)x^2,
 x/12 + x/12 + (1/4!)x^4 ≧ (1/4)・(1/2)^(1/3) x^2 = 0.198425 x^2,
 x/18 + x/18 + x/18 + (1/5!)x^5 ≧ (1/9)・(12/5)^(1/4) x^2 = 0.138296 x^2,
より、
 e^x > Σ[k=0,5] (1/k!)x^k > 1 + 1.503388 x^2 > 1 + (3/2)x^2,  (キャスフィー)

あるいは

 (2/3)x + (1/3!)x^3 ≧ (2/3)x^2,
 x/8 + x/8 + (1/4!)x^4 ≧ (1/8)・3^(2/3) x^2 = 0.260010 x^2,
 x/36 + x/36 + x/36 + (1/5!)x^5 ≧ (1/9)・(3/10)^(1/4) x^2 = 0.082231 x^2,
より、
 e^x > Σ[k=0,5] (1/k!)x^k > 1 + 1.508908 x^2 > 1 + (3/2)x^2,
940132人目の素数さん:2012/12/31(月) 20:23:59.33
>>932 >>936

〔類題〕
次を示せ。ただし eは自然対数の底で 2.7<e<2.8とし、相加・相乗平均を用いてはいけない。
 x>0 ならば e^x > 1 + (e/2)x^2,
941実話です:2013/01/01(火) 16:56:04.29
トゥルルル、トゥルルル・・

女「もしもし、いきなりごめんね。 正月って暇?」

        i
        〔!(^o^)/ キター
           (  )
          | |

男「えッ!?\(^o^)/う、うん暇だよ!\(^o^)/」

        i
        〔!(^o^)/ ウ、ウン、フトウシキ イジッテル ト オモウ ケド ヒマ ダヨ!
          (  )
        / \

女「あはっ、やっぱりーw」 (後ろで大勢の笑い声)

    ヽ\w//
        i
        〔!_(^o^;)
           / )\
           | \

ガチャ、ツー、ツー

        i
        〔!_(゚д゚ )
           / )\
           | \
942132人目の素数さん:2013/01/03(木) 23:46:29.35
>>908
 正の実数 a,b,c に対して
 {(a+b+c)(aa+bb+cc)/3}^2 ≧ 3abc(a^3 +b^3 +c^3),

(略証)
例によって
 min{a,b,c} = m,
 {a,b,c} = {m,m+x,m+y}  (x≧0, y≧0)
とおく。
 a+b+c = 3m + (x+y),
 aa+bb+cc = 3mm + 2m(x+y) + (xx+yy),
 a^3 +b^3 +c^3 = 3m^3 +3mm(x+y) +3m(xx+yy) +(x^3 +y^3),
 abc = m^3 + mm(x+y) + mxy,
より
 (a+b+c)(aa+bb+cc)/3 = 3m^3 + 3mm(x+y) + (1/3)m(5xx+4xy+5yy) + (1/3)(x+y)(xx+yy),

 (左辺) = 9m^6 + 18m^5・(x+y) + m^4・(19xx+26xy+19yy) + 2m^3・{6x^3 +10xy(x+y) +6y^3} + (1/9)mm{43x^4 +76(x^3)y +102xxyy +76x(y^3) +43y^4} +(1/9)m(x+y){10x^4 +8(x^3)y +20xxyy +8x(y^3) +10y^5) +{(x+y)(xx+yy)/3}^2,
 (右辺) = 9m^6 + 18m^5・(x+y) + m^4・(18xx+27xy+18yy) + 2m^3・{6x^3 + 9xy(x+y) +6y^3} + 3mm{x^4 +4(x^3)y +4x(y^3) +y^4} + 3m(x+y){(x^3)y -xxyy +x(y^3)},
辺々引いて
 (左辺) - (右辺) = m^4・(xx-xy+yy) + 2m^3・xy(x+y) + (1/9)mm{16x^4 -32(x^3)y +102xxyy -32x(y^3) +16y^4} +(1/9)m(x+y)(10x^4 -19(x^3)y +47xxyy -19x(y^3) +10y^4} +{(x+y)(xx+yy)/3}^2
         ≧ 0,
943132人目の素数さん:2013/01/07(月) 19:17:44.02
\   |  |   /     /      ヽ-‐ _; ---  _      \        \   |  |   /
_         _   /, -‐==ミ    _,ィニ-‐……ー-: 、`ヽ、   ヽ、     _        _
_  集  も  _    _  ´     ≠:7          ヽ、ヽ |  i  ,    _    集    _
_  め  っ  _  /    -‐ 7´  / ,ハ   ヽ   ゝ-- \ |  !  ,   _    め    _
_  ろ   と  _ /, -‐/    i  /ィ 爪 \ \   \  `ト  !  ′  _    ろ    _
    :       〃  /      | イ ハ | \ 、\  xィ¬ト、 |  .!    ,       :
/        \   /  /    /l |/Гト、      / |_,ノ0 ヽ |  i    ′  /      \
 /  |  |  \   | / /   i i | |ノ0 ト       | ∩  ト |  !     ,   / | | \
             ∨i |   | ヽ| | ∩ |       ! ∪  | |  i    ′              ,ィ /〉
               | |  i  ', |  | ∪ |       |    ! | i     ′            / レ厶 イ
               ヽハ  、 ヽ|  l   |   ,    └―-┘ !  i      ′          ノ   ⊂ニ、'
                い、 \/   ̄ ̄             ',  i       ′     _, - '   ⊂ニ,´
     r 、  _          ヽ 〈        <_  ̄_フ       |  !        ,.-‐T   _,. -‐'´ ̄
   くヾ; U|           | \         ̄       /|  i    _, -‐'   |  /
   r―'   ヽ、          |   \             イ  |  i_,. -‐       |/
   `つ _   ̄ ̄Τ`ー―-- L    `   _  __   〔  |  r┬ '
944132人目の素数さん:2013/01/10(木) 00:49:23.39
945132人目の素数さん:2013/01/10(木) 11:17:17.67
ヤクザやなりすましを使い成人式を荒らしているのは広告代理店やテレビ局の自作自演です。

反原発デモでチンドンや太鼓を鳴らし、ソントを行っている在日
街宣車に乗り、騒音を撒く朝鮮人

構図は全て同じです
http://mamorenihon.files.wordpress.com/2011/10/zainichi_seijinshiki_1.jpg
http://4.bp.blogspot.com/-1_jEcAWVs1s/Tm4X2mLBhhI/AAAAAAAABok/MObw2nzMyoI/s1600/IMG_9062-714071.jpg
http://wave.ap.teacup.com/renaissancejapan/timg/middle_1228711905.jpg

ソント(声闘)・・朝鮮人は、声が大きければ音が大きければ、主張が通る、どんな凶悪犯罪でも無罪になると思い込む。
だから、朝鮮人は大音量で街宣車を走らせる。
原発サウンドデモ=ソント

在日(もちろんテレビや新聞は通名で報道)が凶悪犯罪を起こしたとき、人権派()弁護士が異様に沢山付くのも
弁護士が多ければ=声が大きければ、無罪になると思い込んでいるから。
946132人目の素数さん:2013/01/11(金) 23:09:40.77
 (´Д`;) 解けネーヨ!     
   ∨)       
   (( 
947132人目の素数さん:2013/01/11(金) 23:43:51.38
>>944

Bilkent Univ., Dept. of Math.

〔Problem of the month〕

Term: January 2013

Find the minimum value of the expression
  a/(bbb+27+27) + b/(ccc+27+27) + c/(aaa+27+27),
over all non-negative real numbers a,b,c satisfying a+b+c=1.
948132人目の素数さん:2013/01/11(金) 23:46:08.38
>>944 >>947

(略解)
 最小値は 0.0184826291090075
 (a,b,c) = (0,1-C,C) またはその rotation のとき。
 C = 0.7485464789148035  {C^4 +216C -162 =0 の根}
949132人目の素数さん:2013/01/12(土) 02:29:52.27
Problem 410
http://www.math.ust.hk/excalibur/v17_n3.pdf ( ゚∀゚)ウヒョッ!
950132人目の素数さん:2013/01/14(月) 16:24:27.10
                 _,_. . : : : : . 、
               _,. :'´: : : : : : : : : :`ヽ
            _,.:'´: : : : : : : : : : : : : : : :ハ
          ∠,ヽ: : : : : : : : : : : : : : : : : : :i
        , '´ ̄¨´‐.、 `y : : : : : : : : : : : : : : {     ,.、
      /  .   : :  Y }  ヽ: : : : : : : : : : : |  _,.f´__,\
      .′ :   :    :}v  ,,ゝ: : : : : : : : : : |―¬;.:.:.:.:_:.:.:\
.     l .  :     :. .:!|  ̄ ! ヽ : : : : : : : : ヽj_,.-亠'¨⌒ヽ:_ノ
      ! {  :  :.  |:.:,!r' , 、〉   \∧_: : : :\
     }.:}.  :.l  :.: .!:.i T~^.|        ̄=-'¨
   (_,ノ_;{:...:.:.:!: .:.:. .ハ:{_.`|  i
      f彡Y:.:.}:|:.!:.:!:{:イ'` |  !         / ̄ ̄ ̄ ̄ヽ\
 /\ ⌒j,ィ:.{:!:ハ:|:!{{|   j  !        /          / |
/   l   , '´ ̄,ト! }川,,ー/,.  {,,,.........,,,,,.,,,,,,,,,,.ト .... _ _ - ´   | ,,,,,,
l   |  /   __」 、__ノ―-}n.n r}ー===-=-=- l 、 /_,-、 )    ノ-=''
.、  ` ´  /        ´ ゙           >、_'ー'o_ - ´ 
 ヽ、  /
.   ` ´
951132人目の素数さん:2013/01/15(火) 20:36:55.65
          __ノ)-'´ ̄ ̄`ー- 、_
        , '´  _. -‐'''"二ニニ=-`ヽ、
      /   /:::::; -‐''"        `ーノ
     /   /:::::/           \
     /    /::::::/          | | |  |
     |   |:::::/ /     |  | | | |  |
      |   |::/ / / |  | ||  | | ,ハ .| ,ハ|
      |   |/ / / /| ,ハノ| /|ノレ,ニ|ル' 
     |   |  | / / レ',二、レ′ ,ィイ|゙/   
.     |   \ ∠イ  ,イイ|    ,`-' |      
     |     l^,人|  ` `-'     ゝ  |        このスレは馬と鹿と豚さんばかりね。
      |      ` -'\       ー'  人            
    |        /(l     __/  ヽ、          
     |       (:::::`‐-、__  |::::`、     ヒニニヽ、         
    |      / `‐-、::::::::::`‐-、::::\   /,ニニ、\            
   |      |::::::::::::::::::|` -、:::::::,ヘ ̄|'、  ヒニ二、 \
.   |      /::::::::::::::::::|::::::::\/:::O`、::\   | '、   \
   |      /:::::::::::::::::::/:::::::::::::::::::::::::::::'、::::\ノ  ヽ、  |
  |      |:::::/:::::::::/:::::::::::::::::::::::::::::::::::'、',::::'、  /:\__/‐、
  |      |/:::::::::::/::::::::::::::::::::::::::::::::::O::| '、::| く::::::::::::: ̄|
   |     /_..-'´ ̄`ー-、:::::::::::::::::::::::::::::::::::|/:/`‐'::\;;;;;;;_|
   |    |/::::::::::::::::::::::\:::::::::::::::::::::::::::::|::/::::|::::/:::::::::::/
    |   /:::::::::::::::::::::::::::::::::|:::::::::::::::::::::O::|::|::::::|:::::::::::::::/
952132人目の素数さん:2013/01/17(木) 22:17:57.03
f は C^2 級、f(0) = f(π) = 0 のとき、∫[0 → π] f''(x)^2 dx ≧ ∫[0 → π] f(x)^2 dx
953132人目の素数さん:2013/01/18(金) 10:22:56.29
続けたまえ
954132人目の素数さん:2013/01/19(土) 11:34:10.14
朝鮮人犯罪があまり報道されない、そしてテレビが日常的に嘘を吐く理由。(間借りしているだけなどと言う工作員に注意)
(間借りしているだけだ等と言う社員と工作員には注意)

韓国文化放送(MBC) 〒135-0091 東京都港区台場2-4-8 18F
フジテレビジョン 、、 〒137-8088 東京都港区台場2-4-8 

韓国聯合TVNEWS(YTN) 〒105-0000 東京都港区赤坂5-3-6
TBSテレビ     、 、、 .〒107-8006 東京都港区赤坂5-3-6 

大韓毎日   、、、、、、、、、、、、 〒108-0075 東京都港区港南2-3-13 4F
東京新聞(中日新聞社東京本社) 〒108-8010 東京都港区港南2-3-13

京郷新聞  、、、、、、〒100-0004 東京都千代田区大手町1-7-2
産経新聞東京本社  〒100-8077 東京都千代田区大手町1-7-2
(サンケイスポーツ、夕刊フジ、日本工業新聞社)

朝鮮日報   、、、  〒100-0003 東京都千代田区一ツ橋1-1 4F
毎日新聞東京本社 〒100-8051 東京都千代田区一ツ橋1-1-1

日本放送協会 、、 〒150-8001 東京都渋谷区神南2-2-1
韓国放送公社(KBS) 〒150-0041 東京都渋谷区神南2-2-1NHK東館710-C 
955132人目の素数さん:2013/01/19(土) 19:42:13.88
http://www.asakura.co.jp/books/isbn/978-4-254-11137-8/
                  _______
         /      ∩|   |
        /      / /|不等式
           /  / /  |____|
              / /     | |
  .        / / /∧   ./ /
            / / ´_ゝ`)/  おうい!不等式の新刊書を手に入れたぞー!
          / |      /
            |    /
            |   /⌒l
             ヽ   | /
           / | ゙ー'| L
        /     |  /(_  ヽ
         / / ノ
       /  / /
     /   ( ヽ  
956132人目の素数さん:2013/01/20(日) 01:22:26.92
>>955
ハァハァできますか?
957132人目の素数さん:2013/01/21(月) 00:22:00.37
>>948 の補足

 C^4 + 216C - 162 = (C^2 + 18L)^2 - 36L・(C - 3/L)^2
          = C^4 + 216C - 162・(2/L - 2L^2)^2,
となるように、定数Lを決める。
 2/L - 2L^2 = 1,
補助方程式は
 L^3 +(1/2)L -1 = 0,
これを解くと
 L = {(1/2)[√(55/54) +1]}^(1/3) - {(1/2)[√(55/54) -1]}^(1/3)
  = 0.835122348481366

 C^4 + 216C - 162 = {C^2 +6(√L)・(C -3/L) +18L} {C^2 -6(√L)・(C -3/L) +18L}
          = (C+6.23164851584700)(C-0.748546789148035)(C^2 -5.48310172669896C+34.7290828674771)
958132人目の素数さん:2013/01/21(月) 02:03:01.58
>>956
このスレの過去ログなどで既出のものばかりでしたが、それらのまとめノート的なものだと思えば非常にお得です!
ポチってください
959132人目の素数さん:2013/01/22(火) 01:20:01.90
0<a<1,0<b<1
⇒ 2-√(ab)≧√2{√{a(1-b)}+√{b(1-a)}}
960132人目の素数さん:2013/01/22(火) 01:27:02.95
>>959だけど右辺ちょっとわかりにくいかったなすまん
√2{√{a(1-b)}+√{b(1-a)}}は
(√2)*{√{a(1-b)}+√{b(1-a)}}ね
全部ルートに入ってるように見えてた人ごめん
961132人目の素数さん:2013/01/22(火) 19:32:41.55
          __ノ)-'´ ̄ ̄`ー- 、_
        , '´  _. -‐'''"二ニニ=-`ヽ、
      /   /:::::; -‐''"        `ーノ
     /   /:::::/           \
     /    /::::::/          | | |  |
     |   |:::::/ /     |  | | | |  |
      |   |::/ / / |  | ||  | | ,ハ .| ,ハ|
      |   |/ / / /| ,ハノ| /|ノレ,ニ|ル' 
     |   |  | / / レ',二、レ′ ,ィイ|゙/   
.     |   \ ∠イ  ,イイ|    ,`-' |      
     |     l^,人|  ` `-'     ゝ  |        このスレは馬と鹿と豚さんばかりね。
      |      ` -'\       ー'  人            
    |        /(l     __/  ヽ、          
     |       (:::::`‐-、__  |::::`、     ヒニニヽ、         
    |      / `‐-、::::::::::`‐-、::::\   /,ニニ、\            
   |      |::::::::::::::::::|` -、:::::::,ヘ ̄|'、  ヒニ二、 \
.   |      /::::::::::::::::::|::::::::\/:::O`、::\   | '、   \
   |      /:::::::::::::::::::/:::::::::::::::::::::::::::::'、::::\ノ  ヽ、  |
  |      |:::::/:::::::::/:::::::::::::::::::::::::::::::::::'、',::::'、  /:\__/‐、
  |      |/:::::::::::/::::::::::::::::::::::::::::::::::O::| '、::| く::::::::::::: ̄|
   |     /_..-'´ ̄`ー-、:::::::::::::::::::::::::::::::::::|/:/`‐'::\;;;;;;;_|
   |    |/::::::::::::::::::::::\:::::::::::::::::::::::::::::|::/::::|::::/:::::::::::/
    |   /:::::::::::::::::::::::::::::::::|:::::::::::::::::::::O::|::|::::::|:::::::::::::::/
962132人目の素数さん:2013/01/23(水) 21:27:16.01
>>959-960

 A↑ = (√a, √(1-a))
 B↑ = (√(1-b), √b)
は単位ベクトル。
 (A・B)^2 + |A×B|^2 = |A|^2・|B|^2,
 C↑ = (A・B, |A×B|) = (√[a(1-b)] + √[b(1-a)], √[(1-a)(1-b)] - √(ab))
 |C| = |A|・|B| = 1,
 D↑ = (2(√2)/3, 1/3)
も単位ベクトルだから
 (C・D) ≦ 1.
963132人目の素数さん:2013/01/24(木) 22:10:28.28
おお、ベクトルの解答もきれい
相加相乗を自分は考えてた

2≧√2{√{a(1-b)}+√{b(1-a)}}+√(ab)
1≧(1/√2){√{a(1-b)}+√{b(1-a)}}+√(ab)/2 を示す

右辺
=√(a/2)(1-b)+√(b/2)(1-a)+√(a/2)(b/2)
≧{a/2+(1-b)}/2+{b/2+(1-a)}/2+{(a/2)+(b/2)}/2
=a/4+(1-b)/2+b/4+(1-a)/2+a/4+b/4
=1/2+1/2=1=左辺
964132人目の素数さん:2013/01/25(金) 06:37:07.44
いろいろ手があるんですね
965132人目の素数さん:2013/01/29(火) 16:36:56.98
>>955
美しい不等式の世界: 数学オリンピックの問題を題材として
http://www.amazon.co.jp/dp/4254111371/
価格: ¥ 3,990
出版社: 朝倉書店


翻訳より原著の方が安いなw

Inequalities: A Mathematical Olympiad Approach
http://www.amazon.co.jp/dp/3034600496/
価格: ¥ 3,814
出版社: Birkhauser Basel
966あのこうちやんは始皇帝だった:2013/01/29(火) 19:11:14.47
 テメ〜ら、いいかげんにしねえと、ブッ殺すぞ!

 20代と30代の、ニート・無職の、知的障害の、女性恐怖症の、頭デッカチの虚弱児・ひ弱の、ゴミ・クズ・カス・無能・虫けらのクソガキども!

 死ね!!!!!!!!!!!!!!!!!!!!!!
967132人目の素数さん:2013/01/29(火) 20:07:49.78
          __ノ)-'´ ̄ ̄`ー- 、_
        , '´  _. -‐'''"二ニニ=-`ヽ、
      /   /:::::; -‐''"        `ーノ
     /   /:::::/           \
     /    /::::::/          | | |  |
     |   |:::::/ /     |  | | | |  |
      |   |::/ / / |  | ||  | | ,ハ .| ,ハ|
      |   |/ / / /| ,ハノ| /|ノレ,ニ|ル' 
     |   |  | / / レ',二、レ′ ,ィイ|゙/   私は只の数ヲタなんかとは付き合わないわ。
.     |   \ ∠イ  ,イイ|    ,`-' |      頭が良くて数学が出来てかっこいい人。それが必要条件よ。
     |     l^,人|  ` `-'     ゝ  |        さらに Ann.of Math に論文書けば十分条件にもなるわよ。
      |      ` -'\       ー'  人          一番嫌いなのは論文数を増やすためにくだらない論文を書いて
    |        /(l     __/  ヽ、           良い論文の出版を遅らせるお馬鹿な人。
     |       (:::::`‐-、__  |::::`、     ヒニニヽ、         あなたの論文が Ann of Math に accept される確率は?
    |      / `‐-、::::::::::`‐-、::::\   /,ニニ、\            それとも最近は Inv. Math. の方が上かしら?
   |      |::::::::::::::::::|` -、:::::::,ヘ ̄|'、  ヒニ二、 \
.   |      /::::::::::::::::::|::::::::\/:::O`、::\   | '、   \
   |      /:::::::::::::::::::/:::::::::::::::::::::::::::::'、::::\ノ  ヽ、  |
  |      |:::::/:::::::::/:::::::::::::::::::::::::::::::::::'、',::::'、  /:\__/‐、
  |      |/:::::::::::/::::::::::::::::::::::::::::::::::O::| '、::| く::::::::::::: ̄|
   |     /_..-'´ ̄`ー-、:::::::::::::::::::::::::::::::::::|/:/`‐'::\;;;;;;;_|
   |    |/::::::::::::::::::::::\:::::::::::::::::::::::::::::|::/::::|::::/:::::::::::/
    |   /:::::::::::::::::::::::::::::::::|:::::::::::::::::::::O::|::|::::::|:::::::::::::::/
968132人目の素数さん:2013/02/02(土) 17:55:11.88
■NHKの間抜けなツイッター工作発覚■

NHKの24時のニュースは視聴者からのツイッターを流しているが
自民党や日本企業の話題の時には批判コメントしか流さないなど、
以前からNHKによる意図的な反日工作が指摘されていた。

1月31日深夜のNHK24時ニュースの放送開始の頃にツイッターのサーバーが落ちた。
その時ツイッターがずっと使えない、書き込み自体が出来なかったのは、視聴者が確認している。

にもかかわらず、その時画面では
「地震だったの?」とか「あ、○○さんだー」「都会にこんなのがあるのかー」
と番組の流れに沿ったツイートが流れていた。

番組終了時にNHKは
「ツイッターを『表示するシステム』の不具合があったので、事前に来ていたツイートを流した」
と苦しい説明。

当然視聴者からのツイートは放送できるものを選別しているのだろう。
選別なら別にいいのだが、問題は
視聴者からのリアルタイムのツイートと称して
実際には、NHKが事前に用意したインチキ捏造ツイートを流していたということだ。
969132人目の素数さん:2013/02/04(月) 00:04:26.41
>>940
 (z-1)(e^z - e) ≧ 0,  (← 単調増加)
 e^y - ey = ∫[1,y] (e^z - e)dz ≧ 0,
 e^x -1 -(e/2)x^2 = ∫[0,x] (e^y - ey)dy ≧ 0, (x≧0)


>>952 (Wirtinger)
 f(x) = Σ[k=1,∞) a_k sin(kx),
とフーリエ級数に展開すれば簡単だが....
970132人目の素数さん:2013/02/09(土) 01:46:20.00
正の実数x_1, …, x_n
x_1+x_2+…+x_n≦x_1^3/x_n^2+x_2^3/x_1+x_3^3/x_2^2+…+x_n^3/x_(n-1)^2
大数
971132人目の素数さん:2013/02/10(日) 05:40:45.19
〔問題〕自然数nについて、次を示せ。

 Σ[k=n+1〜∞) 1/(k^2) < 1/(n +1/2),

 Σ[k=n+1〜∞) 1/(k^3) < 1/{2n(n+1)},

 Σ[k=n+1〜∞) 1/(k^4) < 2/{3n(n+1)(2n+1)},

 casphy - 高校数学 - 不等式2 - 013〜015
972132人目の素数さん:2013/02/11(月) 00:26:38.90
>>970
 コーシー不等式の拡張より
 (右辺){x_n+x_1+・・・+x_(n-1)}{x_n+x_1+・・・・x_(n-1)} ≧ (x_1+x_2+・・・・+x_n)^3,
973132人目の素数さん:2013/02/12(火) 22:50:07.63
>>971
ちょっと改良....

〔問題〕自然数nについて、次を示せ。

 1/(n+1) < Σ[k=n+1〜∞) 1/(k^2) < 1/(n +1/2),

 1/{2n(n+1)+1} < Σ[k=n+1〜∞) 1/(k^3) < 1/{2n(n+1)},

 2/{3[n(n+1)+1](2n+1)} < Σ[k=n+1〜∞) 1/(k^4) < 2/{[3n(n+1)+2](2n+1)},
974132人目の素数さん:2013/02/13(水) 23:19:34.61
有名過ぎないかね
975132人目の素数さん:2013/02/13(水) 23:57:26.33
まじ?
976132人目の素数さん:2013/02/16(土) 00:00:15.45
>>970
 キャスフィーの解答から....

 凸不等式でもOK(じゅー)

・f(x) = x^3 は下に凸。
 (右辺) = xn・f(x1/xn) + x1・f(x2/x1) + …… + x(n-1)・f(xn/x(n-1))
  ≧ {xn+x1+ … +x(n-1)}f((x1+x2+ … +xn)/(xn+x1+ … +x(n-1)))
  = {xn+x1+ … +x(n-1)}f(1)
  = {xn+x1+ … +x(n-1)},

・g(x) = 1/x^2 は下に凸。
 (右辺) = x1・g(xn/x1) + x2・g(x1/x2) + …… + xn・f(x(n-1)/xn)
  ≧ (x1+x2+ … +xn)g((xn+x1+ … +x(n-1))/(x1+x2+ … +xn))
  = (x1+x2+ … +xn)g(1)
  = (x1+x2+ … +xn),
977132人目の素数さん:2013/02/20(水) 00:22:53.93
>>971 >>973

m,n:自然数のとき
 Σ[k=n+1〜∞) 1/k^(m+1) < 1/{m・(n + 1/2)^m},
978132人目の素数さん:2013/02/24(日) 15:15:44.50
A New Proof of Shapiro Inequality (by T. Ando)
http://www.math.s.chiba-u.ac.jp/~ando/Shapiro.pdf
979132人目の素数さん:2013/02/24(日) 21:25:21.65
>>978
むかし読んだことがあるようなハロゲンガス
980132人目の素数さん:2013/02/26(火) 23:49:45.87
>>944 >>947
附帯条件が変更されますた。 a+b+c = 9/2.
回答期限も延期されますた(2月末)。といっても2日しかないが。

http://www.fen.bilkent.edu.tr/ → Mathematics → Problem of the month → 2013 → February 2013
http://www.fen.bilkent.edu.tr/~cvmath/Problem/1301q.pdf

>>946
 全くごもっとも。
981132人目の素数さん:2013/03/01(金) 08:04:21.17
982132人目の素数さん:2013/03/01(金) 20:56:39.96
>>981
フルボッキした!
983132人目の素数さん:2013/03/01(金) 21:39:34.06
>>981
単なるベータ関数じゃんw
984132人目の素数さん:2013/03/01(金) 21:49:16.65
ベータ関数とガンマ関数の性質で瞬殺だな
B(x,y)= Γ(x)Γ(y)/ Γ(x+y)
Γ(x+1) =xΓ(x)
Γ(n+1)= n!
985132人目の素数さん:2013/03/02(土) 21:13:31.13
986132人目の素数さん:2013/03/03(日) 00:58:23.53
三百三十九日。
987132人目の素数さん:2013/03/04(月) 00:58:23.20
三百四十日。
988132人目の素数さん:2013/03/04(月) 22:36:56.88
>>949

〔Problem 410.〕
(略証)
(x+y)√{(y+z)(z+x)} -x(y+z) -y(z+x) = x√(y+z)・[√(z+x)-√(y+z)] + y√(z+x)・[√(y+z) - √(z+x)]
 = [x√(y+z) - y√(z+x)]・[√(z+x) - √(y+z)]
 = {t(x-y)/[x√(y+z) + y√(z+x)]}(x-y)・{(x-y)/[√(z+x) + √(y+z)]}
 = {t/[(x+y)√{(y+z)(z+x)} +xy +t]}(x-y)^2
 ≧ {t/[(x+y)(x+y+2z)/2 + xy +t]}(x-y)^2 (← 相乗・相加平均)
 ≧ {t/(x^2 +y^2 +z^2 +2t)}(x-y)^2
 = {t/(x+y+z)^2}(x-y)^2,
循環的にたす。 ここに、t = xy+yz+zx.

 /excalibur/v17_n4.pdf より。
989132人目の素数さん:2013/03/05(火) 00:58:23.33
三百四十一日。
990132人目の素数さん:2013/03/05(火) 08:10:41.22
9-9=0
991132人目の素数さん:2013/03/06(水) 00:58:23.38
三百四十二日。
992132人目の素数さん:2013/03/06(水) 11:48:33.34
00:58:23.
993132人目の素数さん:2013/03/06(水) 12:59:38.91
9=9÷3
994132人目の素数さん:2013/03/07(木) 11:30:34.19
a
995132人目の素数さん
三百四十四日。